You are on page 1of 152

CORPORATION LAW

CASE DIGESTS

COMMERCIAL LAW REVIEW

FEU JD4401
2ND Semester, S.Y. 2017-2018
TABLE OF CONTENTS

1. GENERAL CONCEPTS .......................................................................................................................... 1

(1) SME Bank, Inc. vs. De Guzman ................................................................................................... 1

(2) Philippine National Bank v. Aznar ................................................................................................. 3

(3) Halley vs. Printwell, Inc.............................................................................................................. 5

a. Doctrine of Piercing the Veil of Corporate Fiction .................................................................................. 6

(4) Kukan International Corporation vs. Hon. Amor Reyes ..................................................................... 6

(5) Heirs of Fe Tan Uy v. International Exchange Bank ......................................................................... 8

(6) Lanuza, Jr. and Olbes vs BF Corporation ..................................................................................... 10

(7) Guillermo v. Uson ................................................................................................................... 11

(8) International Academy of Management and Economics vs. Litton and Company, INC. ......................... 13

b. Nationality, Citizenship and Foreign Equity (Place of Incorporation Test, Control Test & Grandfather Rule) ... 15

(9) Gamboa v. Teves .................................................................................................................... 15

(10) Narra Nickel Mining and Development Corp. vs Redmont Consolidated Mines Corporation .................. 16

(11) Roy III v. Herbosa ................................................................................................................. 18

c. Classes of Corporations ................................................................................................................ 21

(12) Boys Scouts of the Philippines vs Commission on Audit ................................................................ 21

(13) Republic vs. City of Parañaque................................................................................................. 23

(14) Torres vs. De Leon ............................................................................................................... 25

2. INCORPORATION AND ORGANIZATION................................................................................................ 26

(15) GSIS Family Bank – Thrift Bank vs. BPI Family Bank ................................................................... 26

(16) Indian Chamber of Commerce Phils., Inc. v. Filipino Indian Chamber of Commerce in the Philippines Inc.
................................................................................................................................................ 27

(17) Paz vs. New International Environmental University .................................................................... 29

(18) Alfredo Montelibano v. Bacolod-Murcia Milling Co., Inc. ................................................................ 30

(19) FILIPINAS PORT SERVICES, INC., represented by its stockholders v. VICTORIANO S. GO, et al. .......... 32

(20) Rebern Development Corporation vs. People's Landless Association ............................................... 34

(21) Heirs of Ignacio v. Home Bankers Savings and Trust Company ...................................................... 35

(22) Philippine Stock Exchange, Inc. vs. Litonjua ............................................................................... 37

(23) Ching vs. Subic Bay and Country Club, Inc. ............................................................................... 38

(24) Lopez Realty, Inc., v. Spouses Tanjangco ................................................................................. 39

(25) Ricafort vs Dicdican ............................................................................................................... 40

(26) Estate of Juvencio Ortanez v. Lee............................................................................................. 41

(27) GRACE BORGONA INSIGNE vs. ABRA VALLEY COLLEGES, INC. G.R. No. 204089 July 29, 2015 By:
Grande, Jonicocel ......................................................................................................................... 43

(28) Multinational Village Homeowners Association v. Gacutan ............................................................. 45

(29) Valle Verde Country Club, Inc. v. Africa ..................................................................................... 47

(30) Bernas, et. al. v. Cinco ........................................................................................................... 49

(31) Marc II Marketing, Inc. vs Alfredo M. Joson ................................................................................ 50

(32) Georg vs. Holy Trinity College, Inc. .......................................................................................... 52


(33) Heirs of Fe Tan Uy vs. International Exchange Bank .................................................................... 53

(34) Arnold and Bradley Hall vs. Judge Piccio, Brown and Capuciong ..................................................... 56

(35) Pioneer Insurance v. Court of Appeals....................................................................................... 57

3. BY-LAWS ....................................................................................................................................... 59

(36) PMI Colleges vs. NLRC ........................................................................................................... 59

(37) Government of the Philippine Islands v. El Hogar Filipino .............................................................. 60

(38) CHINA BANKING CORPORATION v. COURT OF APPEALS and VALLEY GOLF and COUNTRY CLUB, INC. .. 64

4. POWERS OF CORPORATIONS. ............................................................................................................ 66

(39) Sy vs. Gutierrez .................................................................................................................... 66

(40) Violeta Banate v. Philippine Countryside Rural Bank .................................................................... 68

(41) Riosa vs. Tabaco La Suerte Corporation .................................................................................... 71

(42) Lanuza Jr. vs. BF Corporation .................................................................................................. 72

(43) Metropolitan Bank & Trust Co., v. Centro Development Corp ......................................................... 73

(44) Y-I Leisure Philippines, Inc., vs James Yu .................................................................................. 74

(45) Balinghasay v. Castillo ........................................................................................................... 76

5. STOCKS AND STOCKHOLDERS ........................................................................................................... 78

(46) GRACE BORGONA INSIGNE vs. ABRA VALLEY COLLEGES, INC. ...................................................... 78

(47) Turner v. Lorenzo Shipping ..................................................................................................... 79

(48) Lu v Lu Ym Sr ....................................................................................................................... 80

(49) Interport Resources Corporation v. Securities Specialist, Inc. ........................................................ 81

(50) Joselito Musni Puno vs Puno Enterprises, Inc. ............................................................................. 82

(51) Reyes vs. Regional Trial Court of Makati, Br. 142 ........................................................................ 84

(52) Forest Hills Golf & Country Club v. Vertex Sales and Trading, Inc., ................................................. 85

(53) F&S Velasco Company vs. Madrid ............................................................................................. 87

(54) Anna Teng v. Securities and Exchange Commission ..................................................................... 89

(55) Ferro Chemicals vs. Garcia...................................................................................................... 91

(56) Joseph Omar O. Andaya vs. Rural Bank of Cabadbaran, Inc. ......................................................... 92

6. RIGHTS OF SHAREHOLDERS AND CORPORATE BOOKS AND RECORDS ...................................................... 93

(57) SANTIAGO CUA, JR., SOLOMON S. CUA and EXEQUIEL D. ROBLES, in their capacity as Directors of
PHILIPPINE RACING CLUB, INC. v. MIGUEL OCAMPO TAN, JEMIE U. TAN and ATTY. BRIGIDO J. DULAY ...... 93

(58) Legaspi Towers 300, Inc. vs. Muer ......................................................................................... 100

(59) Ching v. Subic Bay Golf and Country Club, Inc. ........................................................................ 102

(60) Villamor, Jr. v. Umale .......................................................................................................... 104

(61) Lorell Lim Po vs. DOJ ........................................................................................................... 105

(62) Yujuico v. Quiambao ............................................................................................................ 106

(63) Terelay Investment and Development Corp vs Yulo ................................................................... 108

7. MERGER AND CONSOLIDATION........................................................................................................ 109

(64) Bank of Philippine Islands v. BPI Employees Union .................................................................... 109

(65) BPI vs. LEE ........................................................................................................................ 111

(66) Bank of Commerce v. Radio Philippines ................................................................................... 112


(67) Commission of Internal Revenue vs. Pilipinas Shell.................................................................... 113

(68) Commissioner of Internal Revenue v. La Tondena Distillers, Inc. ................................................. 114

8. NON-STOCK CORPORATIONS........................................................................................................... 115

(69) Cebu Country Club, Inc. v. Elizagaque .................................................................................... 115

(70) Co, Sr. vs. Philippine Canine Club, Inc. ................................................................................... 117

(71) The Orchard Golf and Country Club vs Yu ................................................................................ 118

(72) Ching vs. Quezon City Sports Club, Inc. .................................................................................. 119

9. DISSOLUTION AND LIQUIDATION .................................................................................................... 120

(73) Alabang Development Corporation v. Alabang Hills Village Association .......................................... 120

(74) Viguilla, et. al. v. Philippine College of Criminology, Inc., ............................................................ 122

(75) Ramon E. Reyes and Clara R. Pastor vs. Bancom Development Corp. ........................................... 123

9. FOREIGN CORPORATIONS ............................................................................................................... 125

(76) TUNA PROCESSING, INC. v. PHILIPPINE KINGFORD, INC. .......................................................... 125

(77) Philippine Deposit Insurance Corp. vs. Citibank, N.A. ................................................................. 127

(78) Steelcase, Inc. v. Design International Selections, Inc., ............................................................. 129

10. PENAL PROVISIONS ..................................................................................................................... 131

(79) Yuico v. Quiambao .............................................................................................................. 131

(80) James Lent vs. Tullett Philippines, Inc., G.R. No. 189158, January 11, 2017 .................................. 132

11. CASES UNDER THE SECURITIES REGULATIONS CODE ........................................................................ 134

(81) Power Homes Unlimited Corp. v. SEC...................................................................................... 134

(82) SEC vs Prosperity.com, Inc ................................................................................................... 135

(83) Securities and Exchange Commission v. Santos ........................................................................ 137

(84) SEC vs. CJH Development..................................................................................................... 139

(85) Abacus Securities Corp. v. Ruben Ampil .................................................................................. 141

(86) Securities and Exchange Commission vs. Interport .................................................................... 143

(87) La Bugal-B’Laan Tribal Association, Inc. v. Ramos ..................................................................... 144

(88) CEMCO Holdings, Inc. v. National Life ..................................................................................... 146

(89) Abacus Securities Corporation vs. Ampil .................................................................................. 148


CORPORATION LAW CASE DIGESTS |1
FEU JD4401 | 2nd SEMESTER, S.Y. 2017-2018

1. GENERAL CONCEPTS

(1) SME Bank, Inc. vs. De Guzman


G.R. No. 184517. October 8, 2013
By: Alba, Ma. Angela

Doctrine: Notwithstanding the stock sale, the corporation continues to be the employer of its people and continues
to be liable for the payment of their just claims. The corporation or its new majority shareholders are not entitled
to lawfully dismiss corporate employees absent a just or authorized cause.

The ruling in Manlimos is expressly reversed insofar as it upheld that, in a stock sale, the buyer in good faith has
no obligation to retain the employees of the selling corporation; and that the dismissal of the affected employees is
lawful, even absent a just or authorized cause.

Facts:
Respondent employees were employees of Small and Medium Enterprise Bank, Incorporated (SME Bank).
Originally, the principal shareholders and corporate directors of the bank were Eduardo M. Agustin, Jr. and Peregrin
de Guzman, Jr. Due to financial difficulties, Agustin and De Guzman sold 86.365% of the shares of stock of SME
Bank to spouses Abelardo and Olga Samson. Spouses Samson then became the principal shareholders of SME
Bank, while Aurelio Villaflor, Jr. was appointed bank president.

As a preconditions for the sale of of SME Bank’s shares of stock, the Letters Agreement sent by Samson to Agustin
and De Guzman, and which the latter accepted, contained the following conditions:

“4. You shall guarantee the peaceful turn over of all assets as well as the peaceful transition of management of the
bank and shall terminate/retire the employees we mutually agree upon, upon transfer of shares in favor of our
group’s nominees; x x x x”

In fulfillment of the abovementioned provision, respondent employees were persuaded by the general manager of
SME to tender their resignations, with the promise that they would be rehired upon reapplication. Relying on this
representation, respondent employees tendered their resignations. As it turned out, respondent employees, except
for Simeon, Jr., were not rehired. Thus, respondent-employees demanded the payment of their respective
separation pays, but their requests were denied.

Aggrieved, respondent employees filed a Complaint before the NLRC and sued SME Bank, spouses Abelardo and
Olga Samson and Aurelio Villaflor (the Samson Group), and Agustin and De Guzman for unfair labor practice, etc.

The labor arbiter ruled that the buyer of an enterprise is not bound to absorb its employees, unless there is an
express stipulation to the contrary. He also found that respondent employees were illegally dismissed. On appeal,
the NLRC found that there was only a mere transfer of shares — and therefore, a mere change of management —
from Agustin and De Guzman to the Samson Group. As the change of management was not a valid ground to
terminate respondent bank employees, the NLRC ruled that they had indeed been illegally dismissed. It further
ruled that Agustin, De Guzman and the Samson Group should be held jointly and severally liable for the employees’
separation pay and backwages. The CA rendered a Decision affirming that of the NLRC. The CA denied the Motions
for Reconsideration, hence this petition.

Issue: Whether the employees were validly dismissed, there being a valid transfer or sale of the corporation from
Agustin and De Guzman to Spouses Samson.

Held: No, the employees were illegally dismissed. There was no transfer of the business establishment to speak of,
but merely a change in the new majority shareholders of the corporation.

There are two types of corporate acquisitions: asset sales and stock sales. In asset sales, the corporate entity sells
all or substantially all of its assets to another entity. In stock sales, the individual or corporate shareholders sell a
controlling block of stock to new or existing shareholders.

In asset sales, the rule is that the seller in good faith is authorized to dismiss the affected employees, but is liable
for the payment of separation pay under the law. The buyer in good faith, on the other hand, is not obliged to
absorb the employees affected by the sale, nor is it liable for the payment of their claims. The most that it may do,
for reasons of public policy and social justice, is to give preference to the qualified separated personnel of the
selling firm.

In contrast with asset sales, in which the assets of the selling corporation are transferred to another entity, the
transaction in stock sales takes place at the shareholder level. Because the corporation possesses a personality
separate and distinct from that of its shareholders, a shift in the composition of its shareholders will not affect its
existence and continuity. Thus, notwithstanding the stock sale, the corporation continues to be the employer of its
CORPORATION LAW CASE DIGESTS |2
FEU JD4401 | 2nd SEMESTER, S.Y. 2017-2018

people and continues to be liable for the payment of their just claims. Furthermore, the corporation or its new
majority shareholders are not entitled to lawfully dismiss corporate employees absent a just or authorized cause

In the case at bar, the Letter Agreements show that their main object is the acquisition by the Samson Group of
86.365% of the shares of stock of SME Bank. Hence, this case involves a stock sale, whereby the transferee
acquires the controlling shares of stock of the corporation. Thus, following the rule in stock sales, respondent
employees may not be dismissed except for just or authorized causes under the Labor Code.

The rule should be different in Manlimos, as this case involves a stock sale. It is error to even discuss transfer of
ownership of the business, as the business did not actually change hands. The transfer only involved a change in
the equity composition of the corporation. To reiterate, the employees are not transferred to a new
employer, but remain with the original corporate employer, notwithstanding an equity shift in its
majority shareholders. This being so, the employment status of the employees should not have been affected by
the stock sale. A change in the equity composition of the corporate shareholders should not result in the automatic
termination of the employment of the corporation’s employees. Neither should it give the new majority
shareholders the right to legally dismiss the corporation’s employees, absent a just or authorized cause.

We therefore see it fit to expressly reverse our ruling in Manlimos insofar as it upheld that, in a stock sale, the
buyer in good faith has no obligation to retain the employees of the selling corporation; and that the dismissal of
the affected employees is lawful, even absent a just or authorized cause.

The fact that there was a change in the composition of its shareholders did not affect the employer-employee
relationship between the employees and the corporation, because an equity transfer affects neither the existence
nor the liabilities of a corporation. Thus, SME Bank continued to be the employer of respondent employees
notwithstanding the equity change in the corporation.

This outcome is in line with the rule that a corporation has a personality separate and distinct from that of its
individual shareholders or members, such that a change in the composition of its shareholders or members would
not affect its corporate liabilities. Therefore, we conclude that, as the employer of the illegally dismissed employees
before and after the equity transfer, petitioner SME Bank is liable for the satisfaction of their claims.
CORPORATION LAW CASE DIGESTS |3
FEU JD4401 | 2nd SEMESTER, S.Y. 2017-2018

(2) Philippine National Bank v. Aznar


G.R No. 171805, May 30, 2011
By: Arid, Hannah Mhae G.

Topic: Separate and Distinct Personality of a Corporation from its stockholders

Doctrine: A corporation has a personality separate and distinct from those of its stockholders and other
corporations to which it may be connected. While a share of stock represents a proportionate or aliquot interest in
the property of the corporation, it does not vest the owner thereof with any legal right or title to any of the
property, his interest in the corporate property being equitable or beneficial in nature. Shareholders are in no legal
sense the owners of corporate property, which is owned by the corporation as a distinct legal person.

Facts: In 1958, RISCO ceased operation due to business reverses. In plaintiffs' desire to rehabilitate RISCO, they
contributed a total amount of P212,720.00 which was used in the purchase of the three (3) parcels of land. After
the purchase of the said lots, titles were issued in the name of RISCO. The amount contributed by plaintiffs
constituted as liens and encumbrances on the aforementioned properties as annotated in the titles of said lots.
Such annotation was made pursuant to the Minutes of the Special Meeting of the Board of Directors of RISCO
(hereinafter referred to as the "Minutes") on March 14, 1961. The President then explained that in a special
meeting of the stockholders previously called for the purpose of putting up certain amount of P212,720.00 for the
rehabilitation of the Company. Thereafter, various subsequent annotations were made on the same titles, including
the Notice of Attachment and Writ of Execution both dated August 3, 1962 in favor of herein defendant PNB. As a
result, a Certificate of Sale was issued in favor of Philippine National Bank, being the lone and highest bidder of the
three (3) parcels of land known as Lot Nos. 3597 and 7380, covered by T.C.T. Nos. 8921 and 8922, respectively,
both situated at Talisay, Cebu, and Lot No. 1328-C covered by T.C.T. No. 24576 situated at Cebu City, for the
amount of Thirty-One Thousand Four Hundred Thirty Pesos (P31,430.00). Subsequently, titles were issued in the
name of PNB which prompted the said stockholders to file a complaint for quieting of title.

The trial court ruled in favor of the plaintiffs on the basis that there was an express trust. The decision was,
however, set aside by the Court of Appeals on the ground that the contributions was a loan secured by a lien rather
than an express trust.

Issue: a.) Are the contributions made by the stockholders of RISCO merely a loan secured by their lien over the
properties, subject to reimbursement or refund, rather than an express trust.

b.) Does the right of respondents Aznar et. al. to refund or repayment of their contributions had not prescribed
and/or that the minutes of the board of directors of RISCO constituted as an effective adverse claim.

Held: a.) YES, there is no express trust. Trust is the right to the beneficial enjoyment of property, the legal title
to which is vested in another. It is a fiduciary relationship that obliges the trustee to deal with the property for the
benefit of the beneficiary. Trust relations between parties may either be express or implied. An express trust is
created by the intention of the trustor or of the parties. An implied trust comes into being by operation of law. The
creation of an express trust must be manifested with reasonable certainty and cannot be inferred from loose and
vague declarations or from ambiguous circumstances susceptible of other interpretations.

The Supreme Court in ruling that there was no express trust created in this case explained that there was no such
reasonable certitude in the creation of an express trust obtains in the case at bar. In fact, a careful scrutiny of the
plain and ordinary meaning of the terms used in the Minutes does not offer any indication that the parties thereto
intended that Aznar, et al., become beneficiaries under an express trust and that RISCO serve as trustor.

The SC said that Aznar, et al., have no right to ask for the quieting of title of the properties at issue because they
have no legal and/or equitable rights over the properties that are derived from the previous registered owner which
is RISCO, the pertinent provision of the law is Section 2 of the Corporation Code (Batas Pambansa Blg. 68), which
states that [a] corporation is an artificial being created by operation of law, having the right of succession and the
powers, attributes and properties expressly authorized by law or incident to its existence.

As a consequence thereof, a corporation has a personality separate and distinct from those of its stockholders and
other corporations to which it may be connected. The SC cited the case of Magsaysay-Labrador v. Court of Appeals
wherein it was held that the interest of the stockholders over the properties of the corporation is merely inchoate
and therefore does not entitle them to intervene in litigation involving corporate property, to wit:
Here, the interest, if it exists at all, of petitioners-movants is indirect, contingent,
remote, conjectural, consequential and collateral. At the very least, their interest is purely
inchoate, or in sheer expectancy of a right in the management of the corporation and to share in
the profits thereof and in the properties and assets thereof on dissolution, after payment of the
corporate debts and obligations.
CORPORATION LAW CASE DIGESTS |4
FEU JD4401 | 2nd SEMESTER, S.Y. 2017-2018

While a share of stock represents a proportionate or aliquot interest in the property of


the corporation, it does not vest the owner thereof with any legal right or title to any of the
property, his interest in the corporate property being equitable or beneficial in nature.
Shareholders are in no legal sense the owners of corporate property, which is owned by the
corporation as a distinct legal person.

In the case at bar, there is no allegation, much less any proof, that the corporate existence of RISCO has ceased
and the corporate property has been liquidated and distributed to the stockholders. The records only indicate that,
as per Securities and Exchange Commission (SEC) Certification dated June 18, 1997, the SEC merely suspended
RISCOs Certificate of Registration beginning on September 5, 1988 due to its non-submission of SEC required
reports and its failure to operate for a continuous period of at least five years.

Verily, Aznar, et al., who are stockholders of RISCO, cannot claim ownership over the properties at issue in this
case on the strength of the Minutes which, at most, is merely evidence of a loan agreement between them and the
company. There is no indication or even a suggestion that the ownership of said properties were transferred to
them which would require no less that the said properties be registered under their names. For this reason, the
complaint should be dismissed since Aznar, et al., have no cause to seek a quieting of title over the subject
properties. At most, what Aznar, et al., had was merely a right to be repaid the amount loaned to RISCO.

b.) The right of the respondents Aznar et. al to be repaid for their contributions had already prescribed.
The pertinent Civil Code provision on prescription which is applicable to the issue at hand is Article 1144(1), to wit:
The following actions must be brought within ten years from the time the right of action accrues a. upon a
written contract; b. upon an obligation created by law; c. upon a judgment.

Applied to the case at bar, the Minutes which was approved on March 14, 1961 is considered as a written contract
between Aznar, et al., and RISCO for the reimbursement of the contributions of the former. As such, the former
had a period of ten (10) years from 1961 within which to enforce the said written contract. However, it does not
appear that Aznar, et al., filed any action for reimbursement or refund of their contributions against RISCO or even
against PNB. Instead the suit that Aznar, et al., brought before the trial court only on January 28, 1998 was one to
quiet title over the properties purchased by RISCO with their contributions. It is unmistakable that their right of
action to claim for refund or payment of their contributions had long prescribed. Thus, it was reversible error for
the Court of Appeals to order PNB to pay Aznar, et al., the amount of their liens based on the Minutes with legal
interests from the time of PNBs acquisition of the subject properties.
CORPORATION LAW CASE DIGESTS |5
FEU JD4401 | 2nd SEMESTER, S.Y. 2017-2018

(3) Halley vs. Printwell, Inc.


G.R. No. 157549. May 30, 2011
By: Bernardo, Gerard

Doctrine: The corporate personality may be disregarded, and the individuals composing the corporation will be
treated as individuals, if the corporate entity is being used as a cloak or cover for fraud or illegality; as a
justification for a wrong; as an alter ego, an adjunct, or a business conduit for the sole benefit of the stockholders.

Facts: BMPI (Business Media Philippines Inc.) is a corporation under the control of its stockholders, including
Donnina Halley. In the course of its business, BMPI commissioned PRINTWELL to print Philippines, Inc. (a magazine
published and distributed by BMPI). PRINTWELL extended 30-day credit accommodation in favor of BMPI and in a
period of 9 mos. BMPI placed several orders amounting to 316,342.76. However, only 25,000 was paid hence a
balance of 291,342.76.

PRINTWELL sued BMPI for collection of the unpaid balance and later on impleaded BMPI’s original stockholders and
incorporators to recover on their unpaid subscriptions.

It appears that BMPI has an authorized capital stock of 3M divided into 300,000 shares with P10 par value. Only
75,000 shares worth P750,000 were originally subscribed of which P187,500 were paid up capital. Halley
subscribed to 35,000 shares worth P350,000 but only paid P87,500. Halley contends that BMPI had separate and
distinct personality.

Issue: Whether or not petitioner Donnina Halley is personally liable.

Held: Yes. Settled is the rule that when the veil of corporate fiction is used as a means of perpetrating fraud or an
illegal act or as a vehicle for the evasion of an existing obligation, the circumvention of statutes, the achievements
or perfection of monopoly or generally the perpetration of knavery or crime, the veil with which the law covers and
isolates the corporation from the members or stockholders who compose it will be lifted to allow for its
consideration merely as an aggregation of individuals (First Philippine International (Bank vs. Court of Appeals, 252
SCRA 259). Moreover, under this doctrine, the corporate existence may be disregarded where the entity is formed
or used for non-legitimate purposes, such as to evade a just and due obligations or to justify wrong (Claparols vs.
CIR, 65 SCRA 613).

Although a corporation has a personality separate and distinct from those of its stockholders, directors, or officers,
such separate and distinct personality is merely a fiction created by law for the sake of convenience and to
promote the ends of justice.

The corporate personality may be disregarded, and the individuals composing the corporation will be treated as
individuals, if the corporate entity is being used as a cloak or cover for fraud or illegality; as a justification for a
wrong; as an alter ego, an adjunct, or a business conduit for the sole benefit of the stockholders. As a general rule,
a corporation is looked upon as a legal entity, unless and until sufficient reason to the contrary appears. Thus, the
courts always presume good faith, and for that reason accord prime importance to the separate personality of the
corporation, disregarding the corporate personality only after the wrongdoing is first clearly and convincingly
established. It thus behooves the courts to be careful in assessing the milieu where the piercing of the corporate
veil shall be done.

Although nowhere in Printwell’s amended complaint or in the testimonies Printwell offered can it be read or inferred
from that the petitioner was instrumental in persuading BMPI to renege on its obligation to pay; or that she
induced Printwell to extend the credit accommodation by misrepresenting the solvency of BMPI to Printwell, her
personal liability, together with that of her co-defendants, remained because the CA found her and the other
defendant stockholders to be in charge of the operations of BMPI at the time the unpaid obligation was transacted
and incurred, to wit:

In the case at bench, it is undisputed that BMPI made several orders on credit from appellee PRINTWELL involving
the printing of business magazines, wrappers and subscription cards, in the total amount of P291,342.76 which
facts were never denied by appellants’ stockholders that they owe(d) appellee the amount of P291,342.76. The
said goods were delivered to and received by BMPI but it failed to pay its overdue account to appellee as well as
the interest thereon, at the rate of 20% per annum until fully paid. It was also during this time that appellants
stockholders were in charge of the operation of BMPI despite the fact that they were not able to pay their unpaid
subscriptions to BMPI yet greatly benefited from said transactions.

In view of the unpaid subscriptions, BMPI failed to pay appellee of its liability, hence appellee in order to protect its
right, can collect from the appellants stockholders regarding their unpaid subscriptions. To deny appellee from
recovering from appellants would place appellee in a limbo on where to assert their right to collect from BMPI since
the stockholders who are appellants herein are availing the defense of corporate fiction to evade payment of its
obligations.
CORPORATION LAW CASE DIGESTS |6
FEU JD4401 | 2nd SEMESTER, S.Y. 2017-2018

a. Doctrine of Piercing the Veil of Corporate Fiction

(4) Kukan International Corporation vs. Hon. Amor Reyes


G.R. No. 182729, September 29, 2010
By: Donna Bigornia

Doctrine: To justify the piercing of the veil of corporate fiction, it must be shown by clear and convincing proof
that the separate and distinct personality of the corporation was purposefully employed to evade a legitimate and
binding commitment and perpetuate a fraud or like wrongdoings.

Facts: Sometime in March 1998, Kukan, Inc. conducted a bidding worth Php 5M (reduced to PhP 3,388,502) for
the supply and installation of signages in a building being constructed in Makati City which was won by Morales.

Despite his compliance, Morales was only paid the amount of PhP 1,976,371.07, leaving a balance of PhP
1,412,130.93, which Kukan, Inc. refused to pay despite demands.
Morales filed a Complaint with the RTC against Kukan, Inc. for a sum of money. However, starting November 2000,
Kukan, Inc. no longer appeared and participated in the proceedings before the trial court, prompting the RTC to
declare Kukan, Inc. in default and paving the way for Morales to present his evidence ex parte.

On November 28, 2002, the RTC rendered a Decision finding for Morales and against Kukan, Inc.

After the above decision became final and executory, Morales moved for and secured a writ of execution against
Kukan, Inc. The sheriff then levied upon various personal properties found at what was supposed to be Kukan,
Inc.’s office at Unit 2205, 88 Corporate Center, Salcedo Village, Makati City. Alleging that it owned the properties
thus levied and that it was a different corporation from Kukan, Inc., Kukan International Corporation (KIC) filed an
Affidavit of Third-Party Claim. Notably, KIC was incorporated in August 2000, or shortly after Kukan, Inc. had
stopped participating in Civil Case No. 99-93173.
In reaction to KIC’s claim, Morales interposed an Omnibus Motion dated April 30, 2003, praying, and applying the
principle of piercing the veil of corporate fiction, that an order be issued for the satisfaction of the judgment debt of
Kukan, Inc. with the properties under the name or in the possession of KIC, it being alleged that both corporations
are but one and the same entity. KIC opposed Morales’ motion. The court denied the omibus motion.

In a bid to establish the link between KIC and Kukan, Inc., Morales filed a Motion for Examination of Judgment
Debtors dated May 4, 2005 which sought that subponae be issued against the primary stockholders of Kukan, Inc.,
among them Michael Chan, a.k.a. Chan Kai Kit. This too was denied by the court.
Morales then sought the inhibition of the presiding judge, Eduardo B. Peralta, Jr., who eventually granted the
motion. The case was re-raffled to Branch 21, presided by public respondent Judge Amor Reyes.
Before the Manila RTC, Branch 21, Morales filed a Motion to Pierce the Veil of Corporate Fiction to declare KIC as
having no existence separate from Kukan, Inc. This time around, the RTC, by Order dated March 12, 2007, granted
the motion. From the above order, KIC moved but was denied reconsideration in another Order dated June 7,
2007.

KIC went to the CA on a petition for certiorari to nullify the aforesaid March 12 and June 7, 2007 RTC Orders but on
January 23, 2008, the CA denied the petition and affirmed the assailed Orders. The CA later denied KIC’s MR in the
assailed resolution.
Hence, the instant petition for review.

Issue:
A. whether the trial court can, after the judgment against Kukan, Inc. has attained finality, execute it against the
property of KIC;
B. whether the trial court acquired jurisdiction over KIC;
C. whether the trial and appellate courts correctly applied, under the premises, the principle of piercing the veil of
corporate fiction.

Ruling:

A. No.
In Carpio v. Doroja, the Court ruled that the deciding court has supervisory control over the execution of its
judgment:

A case in which an execution has been issued is regarded as still pending so that all proceedings on the execution
are proceedings in the suit. There is no question that the court which rendered the judgment has a general
supervisory control over its process of execution, and this power carries with it the right to determine every
question of fact and law which may be involved in the execution.
CORPORATION LAW CASE DIGESTS |7
FEU JD4401 | 2nd SEMESTER, S.Y. 2017-2018

The court’s supervisory control does not, however, extend as to authorize the alteration or amendment of a final
and executory decision, save for certain recognized exceptions, among which is the correction of clerical errors.
Else, the court violates the principle of finality of judgment and immutability.
As may be noted, the above decision, in unequivocal terms, directed Kukan, Inc. to pay the aforementioned awards
to Morales. Thus, making KIC, thru the medium of a writ of execution, answerable for the above judgment liability
is a clear case of altering a decision, an instance of granting relief not contemplated in the decision sought to be
executed. And the change does not fall under any of the recognized exceptions to the doctrine of finality and
immutability of judgment. It is a settled rule that a writ of execution must conform to the fallo of the judgment; as
an inevitable corollary, a writ beyond the terms of the judgment is a nullity.

Thus, on this ground alone, the instant petition can already be granted. Nonetheless, an examination of the other
issues raised by KIC would be proper.

B. No.
In the instant case, KIC was not made a party-defendant in Civil Case No. 99-93173. Even if it is conceded that it
raised affirmative defenses through its aforementioned pleadings, KIC never abandoned its challenge, however
implicit, to the RTC’s jurisdiction over its person. The challenge was subsumed in KIC’s primary assertion that it
was not the same entity as Kukan, Inc. Pertinently, in its Comment and Opposition to Plaintiff’s Omnibus Motion
dated May 20, 2003, KIC entered its “special but not voluntary appearance” alleging therein that it was a different
entity and has a separate legal personality from Kukan, Inc. And KIC would consistently reiterate this assertion in
all its pleadings, thus effectively resisting all along the RTC’s jurisdiction of its person. It cannot be
overemphasized that KIC could not file before the RTC a motion to dismiss and its attachments in Civil Case No.
99-93173, precisely because KIC was neither impleaded nor served with summons. Consequently, KIC could only
assert and claim through its affidavits, comments, and motions filed by special appearance before the RTC that it is
separate and distinct from Kukan, Inc.

Following La Naval Drug Corporation, KIC cannot be deemed to have waived its objection to the court’s lack of
jurisdiction over its person. It would defy logic to say that KIC unequivocally submitted itself to the jurisdiction of
the RTC when it strongly asserted that it and Kukan, Inc. are different entities. In the scheme of things obtaining,
KIC had no other option but to insist on its separate identity and plead for relief consistent with that position.

C. No.
The principle of piercing the veil of corporate fiction, and the resulting treatment of two related corporations as one
and the same juridical person with respect to a given transaction, is basically applied only to determine established
liability; it is not available to confer on the court a jurisdiction it has not acquired, in the first place, over a party
not impleaded in a case. Elsewise put, a corporation not impleaded in a suit cannot be subject to the court’s
process of piercing the veil of its corporate fiction. In that situation, the court has not acquired jurisdiction over
the corporation and, hence, any proceedings taken against that corporation and its property would infringe on its
right to due process.

Aguedo Agbayani, a recognized authority on Commercial Law, stated as much:


Piercing the veil of corporate entity applies to determination of liability not of jurisdiction.
This is so because the doctrine of piercing the veil of corporate fiction comes to play only during the trial of the
case after the court has already acquired jurisdiction over the corporation. Hence, before this doctrine can be
applied, based on the evidence presented, it is imperative that the court must first have jurisdiction over the
corporation.

The implication of the above comment is twofold: (1) the court must first acquire jurisdiction over the corporation
or corporations involved before its or their separate personalities are disregarded; and (2) the doctrine of piercing
the veil of corporate entity can only be raised during a full-blown trial over a cause of action duly commenced
involving parties duly brought under the authority of the court by way of service of summons or what passes as
such service.
To justify the piercing of the veil of corporate fiction, it must be shown by clear and convincing proof that the
separate and distinct personality of the corporation was purposefully employed to evade a legitimate and binding
commitment and perpetuate a fraud or like wrongdoings. To be sure, the Court has, on numerous occasions,
applied the principle where a corporation is dissolved and its assets are transferred to another to avoid a financial
liability of the first corporation with the result that the second corporation should be considered a continuation and
successor of the first entity.
CORPORATION LAW CASE DIGESTS |8
FEU JD4401 | 2nd SEMESTER, S.Y. 2017-2018

(5) Heirs of Fe Tan Uy v. International Exchange Bank


G.R No. 166282, February 13, 2013
By: Corona, Jose Enrico V.

Doctrine: Before a director or officer of a corporation can be held personally liable for corporate obligations,
however, the following requisites must concur: (1) the complainant must allege in the complaint that the director
or officer assented to patently unlawful acts of the corporation, or that the officer was guilty of gross negligence or
bad faith; and (2) the complainant must clearly and convincingly prove such unlawful acts, negligence or bad faith.

Under a variation of the doctrine of piercing the veil of corporate fiction, when two business enterprises are owned,
conducted and controlled by the same parties, both law and equity will, when necessary to protect the rights of
third parties, disregard the legal fiction that two corporations are distinct entities and treat them as identical or one
and the same.

Facts: Respondent granted several loans to Hammer Garments covered by several promissory notes and deeds of
assignment. Hammer was represented by its President and General Manager Chua. The loans were secured by a
Real Estate Mortgage executed by Goldkey as surety. This surety agreement was signed by Chua and his Wife Uy.

Hammer defaulted in the payment of its loans. The respondent foreclosed the mortgaged property. Hammer failed
to pay the deficiency hence, the bank filed a complaint for sum of money against Hammer, Chua, and Uy.

The RTC held Uy liable because she was an officer and stockholder of Hammer. Goldkey was also held liable for
being a mere alter ego of Hammer.

Issues:
(1) Whether or not Uy shall be held liable for the deficiency?
(2) Whether Goldkey can be held liable for the obligation of Hammer for being a mere alter ego of the latter.

Held:

(1) No. A director, officer or employee of a corporation is generally not held personally liable for obligations
incurred by the corporation. Nevertheless, this legal fiction may be disregarded if it is used as a means to
perpetrate fraud or an illegal act, or as a vehicle for the evasion of an existing obligation, the circumvention of
statutes, or to confuse legitimate issues.

Sec. 31. Liability of directors, trustees or officers. – Directors or trustees who wilfully and knowingly vote for or
assent to patently unlawful acts of the corporation or who are guilty of gross negligence or bad faith in directing
the affairs of the corporation or acquire any personal or pecuniary interest in conflict with their duty as such
directors or trustees shall be liable jointly and severally for all damages resulting therefrom suffered by the
corporation, its stockholders or members and other persons.

Before a director or officer of a corporation can be held personally liable for corporate obligations, however, the
following requisites must concur: (1) the complainant must allege in the complaint that the director or officer
assented to patently unlawful acts of the corporation, or that the officer was guilty of gross negligence or bad faith;
and (2) the complainant must clearly and convincingly prove such unlawful acts, negligence or bad faith.

In this case, petitioners are correct to argue that it was not alleged, much less proven, that Uy committed an act as
an officer of Hammer that would permit the piercing of the corporate veil. A reading of the complaint reveals that
with regard to Uy, iBank did not demand that she be held liable for the obligations of Hammer because she was a
corporate officer who committed bad faith or gross negligence in the performance of her duties such that the lifting
of the corporate mask would be merited. What the complaint simply stated is that she, together with her errant
husband Chua, acted as surety of Hammer, as evidenced by her signature on the Surety Agreement which was
later found by the RTC to have been forged.
CORPORATION LAW CASE DIGESTS |9
FEU JD4401 | 2nd SEMESTER, S.Y. 2017-2018

(2) Yes. Under a variation of the doctrine of piercing the veil of corporate fiction, when two business enterprises
are owned, conducted and controlled by the same parties, both law and equity will, when necessary to protect the
rights of third parties, disregard the legal fiction that two corporations are distinct entities and treat them as
identical or one and the same.

While the conditions for the disregard of the juridical entity may vary, the following are some probative factors of
identity that will justify the application of the doctrine of piercing the corporate veil, as laid down in Concept
Builders, Inc. v NLRC:
(1) Stock ownership by one or common ownership of both corporations;
(2) Identity of directors and officers;
(3) The manner of keeping corporate books and records, and
(4) Methods of conducting the business.

Both corporations are family corporations of defendants Manuel Chua and his wife Fe Tan Uy.
Hammer Garments and Goldkey share the same office and practically transact their business from the same place.
Defendant Manuel Chua is the President and Chief Operating Officer of both corporations. All business transactions
of Goldkey and Hammer are done at the instance of defendant Manuel Chua who is authorized to do so by the
corporations.

The assets of Goldkey and Hammer are co-mingled. The real properties of Goldkey are mortgaged to secure
Hammer’s obligation with creditor banks.
CORPORATION LAW CASE DIGESTS | 10
FEU JD4401 | 2nd SEMESTER, S.Y. 2017-2018

(6) Lanuza, Jr. and Olbes vs BF Corporation


G.R. No. 174938. October 1, 2014
By: Kathrina De Castro

Topic: Piercing the Veil of Corporate Fiction


DOCTRINE: When there are allegations of bad faith or malice against corporate directors or representatives, it
becomes the duty of courts or tribunals to determine if these persons and the corporation should be treated as
one. Without a trial, courts and tribunals have no basis for determining whether the veil of corporate fiction should
be pierced.

BF Corporation entered into agreements with Shangri-La for construction of mall and parking space along EDSA.
Shangri-La defaulted in paying BF Corporation but still the construction was completed due to assurance given by
Shangri-La that they will reimburse the expenses of the construction and that there is delay due to delayed
processing of billing statements. Shangri-La refused to pay which prompted BF Corporation to bring the case before
Arbitration. It also alleged that the directors were in bad faith in directing Shangri-La’s affairs and they should be
held jointly and severally liable with Shangri-La for its obligations. That the directors should be made to join the
Arbitration Proceeding to determine the extent of their liabilities.

Can the petitioners who are directors of Shangri-La be made solidarily liable with Shangri-La

Yes. Under the Corporation Code “a corporation is an artificial entity created by fiction of law. This means that
while it is not a person, naturally, the law gives it a distinct personality and treats it as such. A corporation, in the
legal sense, is an individual with a personality that is distinct and separate from other persons including its
stockholders, officers, directors, representatives, and other juridical entities. Because a corporation’s existence is
only by fiction of law, it can only exercise its rights and powers through its directors, officers, or agents, who are all
natural persons. A corporation cannot sue or enter into contracts without them. A consequence of a corporation’s
separate personality is that consent by a corporation through its representatives is not consent of the
representative, personally. Its obligations, incurred through official acts of its representatives, are its own. A
stockholder, director, or representative does not become a party to a contract just because a corporation executed
a contract through that stockholder, director or representative.

However, there are instances when the distinction between personalities of directors, officers, and representatives,
and of the corporation, are disregarded. It is called piercing the veil of corporate fiction. Piercing the corporate veil
is warranted when “[the separate personality of a corporation] is used as a means to perpetrate fraud or an illegal
act, or as a vehicle for the evasion of an existing obligation, the circumvention of statutes, or to confuse legitimate
issues.” It is also warranted in alter ego cases “where a corporation is merely a farce since it is a mere alter ego or
business conduit of a person, or where the corporation is so organized and controlled and its affairs are so
conducted as to make it merely an instrumentality, agency, conduit or adjunct of another corporation.” When
corporate veil is pierced, the corporation and persons who are normally treated as distinct from the corporation are
treated as one person, such that when the corporation is adjudged liable, these persons, too, become liable as if
they were the corporation. When there are allegations of bad faith or malice against corporate directors or
representatives, it becomes the duty of courts or tribunals to determine if these persons and the corporation should
be treated as one. Without a trial, courts and tribunals have no basis for determining whether the veil of corporate
fiction should be pierced.

In this case, the Arbitral Tribunal rendered a decision, finding that BF Corporation failed to prove the existence of
circumstances that render petitioners and the other directors solidarily liable. It ruled that petitioners and Shangri-
La’s other directors were not liable for the contractual obligations of Shangri-La to BF Corporation.
CORPORATION LAW CASE DIGESTS | 11
FEU JD4401 | 2nd SEMESTER, S.Y. 2017-2018

(7) Guillermo v. Uson


G.R. No. 198967, March 7, 2016
By: Gaite, Rhio Angeline

Doctrine: A corporation is still an artificial being invested by law with a personality separate and distinct from that
of its stockholders and from that of other corporations to which it may be connected. It is not in every instance of
inability to collect from a corporation that the veil of corporate fiction is pierced, and the responsible officials are
made liable. Personal liability attaches only when, as enumerated by the said Section 31 of the Corporation Code,
there is a wilfull and knowing assent to patently unlawful acts of the corporation, there is gross negligence or bad
faith in directing the affairs of the corporation, or there is a conflict of interest resulting in damages to the
corporation.

Facts: Uson began his employment with Royal Class Venture as an accounting clerk. Eventually, he was promoted
to the position of accounting supervisor, , until he was allegedly dismissed from employment on December 20,
2000. Uson then filed with the Sub-Regional Arbitration, Dagupan City, of the NLRC a Complaint for Illegal
Dismissal, with prayers for backwages, reinstatement, salaries and 13th month pay, moral and exemplary damages
and attorney’s fees against Royal Class Venture. Royal Class Venture did not make an appearance in the case
despite its receipt of summons. Thus, Uson filed his Position Paper as complainant. Labor Arbiter De Vera rendered
a Decision in favor of the complainant Uson and ordering therein respondent Royal Class Venture to reinstate him
to his former position and pay his backwages, 13th month pay as well as moral and exemplary damages and
attorney’s fees. Uson filed a Motion for Alias Writ of Execution and to Hold Directors and Officers of Respondent
Liable for Satisfaction of the Decision. The motion quoted from a portion of the Sheriff’s Return, which states:

“x x x the undersigned proceeded at the stated present business office address of the respondent which is at
Minien East, Sta. Barbara, Pangasinan to serve the writ of execution. Upon arrival, I found out that the
establishment erected thereat is not [in] the respondent’s name but JOEL and SONS CORPORATION, a family
corporation owned by the Guillermos of which, Jose Emmanuel F. Guillermo, the General Manager of the
respondent, is one of the stockholders who received the writ using his nickname “Joey,” [and who] concealed his
real identity and pretended that he [was] the brother of Jose, which [was] contrary to the statement of the guard-
on-duty that Jose and Joey [were] one and the same person. The former also informed the undersigned that the
respondent’s (sic) corporation has been dissolved. On the succeeding day, as per [advice] by the [complainant’s]
counsel that the respondent has an account at the BPI Magsaysay Branch, A.B. Fernandez Ave., Dagupan City, the
undersigned immediately served a notice of garnishment, thus, the bank replied on the same day stating that the
respondent [does] not have an account with the branch.”

Labor Arbiter Rimando issued an Order granting the motion filed by Uson. The order held that officers of a
corporation are jointly and severally liable for the obligations of the corporation to the employees and there is no
denial of due process in holding them so even if the said officers were not parties to the case when the judgment in
favor of the employees was rendered. Thus, the Labor Arbiter pierced the veil of corporate fiction of Royal Class
Venture and held herein petitioner Jose Emmanuel Guillermo (Guillermo), in his personal capacity, jointly and
severally liable with the corporation for the enforcement of the claims of Uson.

Issues: (1) Whether an officer of a corporation may be included as judgment obligor in a labor case for the first
time only after the decision of the Labor Arbiter had become final and executory
(2) Whether the twin doctrines of “piercing the veil of corporate fiction” and personal liability of company officers in
labor cases apply
(3) Whether this case involves an intra-corporate controversy on the ground that Uson is a stockholder of Royal
Class Venture

Held: (1) Yes. According to the cases decided by the SC, a corporation is still an artificial being invested by law
with a personality separate and distinct from that of its stockholders and from that of other corporations to which it
may be connected. It is not in every instance of inability to collect from a corporation that the veil of corporate
fiction is pierced, and the responsible officials are made liable. Personal liability attaches only when, as enumerated
by the said Section 31 of the Corporation Code, there is a willful and knowing assent to patently unlawful acts of
the corporation, there is gross negligence or bad faith in directing the affairs of the corporation, or there is a
conflict of interest resulting in damages to the corporation. Further, the doctrine of piercing the corporate veil is
held to apply only in three (3) basic areas, namely: (1) defeat of public convenience as when the corporate fiction
is used as a vehicle for the evasion of an existing obligation; (2) fraud cases or when the corporate entity is used
to justify a wrong, protect fraud, or defend a crime; or (3) alter ego cases, where a corporation is merely a farce
since it is a mere alter ego or business conduit of a person, or where the corporation is so organized and controlled
and its affairs are so conducted as to make it merely an instrumentality, agency, conduit or adjunct of another
corporation. In the absence of malice, bad faith, or a specific provision of law making a corporate officer liable,
such corporate officer cannot be made personally liable for corporate liabilities. Indeed, the conferment of liability
on officers for a corporation’s obligations to labor is held to be an exception to the general doctrine of separate
personality of a corporation.
CORPORATION LAW CASE DIGESTS | 12
FEU JD4401 | 2nd SEMESTER, S.Y. 2017-2018

It also bears emphasis that in cases where personal liability attaches, not even all officers are made accountable.
Rather, only the “responsible officer,” i.e., the person directly responsible for and who “acted in bad faith” in
committing the illegal dismissal or any act violative of the Labor Code, is held solidarily liable, in cases wherein the
corporate veil is pierced. In other instances, such as cases of so-called corporate tort of a close corporation, it is
the person “actively engaged” in the management of the corporation who is held liable. In the absence of a clearly
identifiable officer(s) directly responsible for the legal infraction, the Court considers the president of the
corporation as such officer.

For the case at bar, applying the above criteria, a finding of personal and solidary liability against a corporate
officer like Guillermo must be rooted on a satisfactory showing of fraud, bad faith or malice, or the presence of any
of the justifications for disregarding the corporate fiction. It is our finding that such evidence exists in the record.
Like the
A.C. Ransom, and Naguiat cases, the case at bar involves an apparent family corporation. As in those two cases,
the records of the present case bear allegations and evidence that Guillermo, the officer being held liable, is the
person responsible in the actual running of the company and for the malicious and illegal dismissal of the
complainant; he, likewise, was shown to have a role in dissolving the original obligor company in an obvious
“scheme to avoid liability” which jurisprudence has always looked upon with a suspicious eye in order to protect the
rights of labor. Then, it is also clearly reflected in the records that it was Guillermo himself, as President and
General Manager of the company, who received the summons to the case, and who also subsequently and without
justifiable cause refused to receive all notices and orders of the Labor Arbiter that followed. Finally, the records
likewise bear that Guillermo dissolved Royal Class Venture and helped incorporate a new firm, located in the same
address as the former, wherein he is again a stockholder. This is borne by the Sheriff’s Return. The foregoing
clearly indicate a pattern or scheme to avoid the obligations to Uson and frustrate the execution of the judgment
award, which this Court, in the interest of justice, will not countenance.

(2) Yes. The common thread running among the aforementioned cases, however, is that the veil of corporate
fiction can be pierced, and responsible corporate directors and officers or even a separate but related corporation,
may be impleaded and held answerable solidarily in a labor case, even after final judgment and on execution, so
long as it is established that such persons have deliberately used the corporate vehicle to unjustly evade the
judgment obligation, or have resorted to fraud, bad faith or malice in doing so. When the shield of a separate
corporate identity is used to commit wrongdoing and opprobriously elude responsibility, the courts and the legal
authorities in a labor case have not hesitated to step in and shatter the said shield and deny the usual protections
to the offending party, even after final judgment. The key element is the presence of fraud, malice or bad faith.
Bad faith, in this instance, does not connote bad judgment or negligence but imparts a dishonest purpose or some
moral obliquity and conscious doing of wrong; it means breach of a known duty through some motive or interest or
ill will; it partakes of the nature of fraud. As the foregoing implies, there is no hard and fast rule on when corporate
fiction may be disregarded; instead, each case must be evaluated according to its peculiar circumstances.

(3) No. Although Uson is also a stockholder and director of Royal Class Venture, it is settled in jurisprudence that
not all conflicts between a stockholder and the corporation are intra-corporate; an examination of the complaint
must be made on whether the complainant is involved in his capacity as a stockholder or director, or as an
employee. If the latter is found and the dispute does not meet the test of what qualifies as an intra corporate
controversy, then the case is a labor case cognizable by the NLRC and is not within the jurisdiction of any other
tribunal. In the case at bar, Uson’s allegation was that he was maliciously and illegally dismissed as an Accounting
Supervisor by Guillermo, the Company President and General Manager, an allegation that was not even disputed
by the latter nor by Royal Class Venture. It raised no intra-corporate relationship issues between him and the
corporation or Guillermo; neither did it raise any issue regarding the regulation of the corporation. As correctly
found by the appellate court, Uson’s complaint and redress sought were centered alone on his dismissal as an
employee, and not upon any other relationship he had with the company or with Guillermo. Thus, the matter is
clearly a labor dispute cognizable by the labor tribunals.
CORPORATION LAW CASE DIGESTS | 13
FEU JD4401 | 2nd SEMESTER, S.Y. 2017-2018

(8) International Academy of Management and Economics vs. Litton and Company, INC.
G.R. No. 191525 December 13, 2017
By: Grande, Jonicocel

Topic: piercing the veil


Doctrine: The piercing of the corporate veil may apply to corporations as well as natural persons involved with
corporations. Corporate mask may be lifted and the corporate veil may be pierced when a corporation is just but
the alter ego of a person or of another corporation.

Facts:
Litton filed a complaint for unlawful detainer against Santos. The MeTC ruled in Litton's favour. The sheriff of the
MeTC of Manila levied on a piece of real property. registered in the name of International Academy of Management
and Economics Incorporated (I/AME), in order to execute the judgment against Santos. The annotations on in TCT
indicated that such was "only up to the extent of the share of Emmanuel T. Santos
I/AME filed with Me TC a "Motion to Lift or Remove Annotations Inscribed in TCT. I/ AME claimed that it has a
separate and distinct personality from Santos. Hence, its properties should not be made to answer for the latter's
liabilities.

ISSUES: (1) WON Piercing the Corporate Veil may Apply to Non-stock Corporations.

Held: YES.
In determining the propriety of applicability of piercing the veil of corporate fiction, the Court, in a number of
cases, did not put in issue whether a corporation is a stock or non-stock corporation.

In the United States, from which we have adopted our law on corporations, non-profit corporations are not immune
from the doctrine of piercing the corporate veil. Their courts view piercing of the corporation as an equitable
remedy, which justifies said courts to scrutinize any organization however organized and in whatever manner it
operates. Moreover, control of ownership does not hinge on stock ownership.
The concept of equitable ownership, for stock or non-stock corporations, in piercing of the corporate veil scenarios,
may also be considered. An equitable owner is an individual who is a non-shareholder defendant, who exercises
sufficient control or considerable authority over the corporation to the point of completely disregarding the
corporate form and acting as though its assets are his or her alone to manage and distribute.
Given the foregoing, there is no reason why a non-stock corporation such as I/ AME, may not be scrutinized for
purposes of piercing the corporate veil or fiction.

(2)WON Piercing the Corporate Veil may apply to Natural Persons

Held: YES.
Corporate Veil may apply to natural persons when:

a. The Corporation is the Alter Ego of a Natural Person

The doctrine of alter ego is based upon the misuse of a corporation by an individual for wrongful or inequitable
purposes, and in such case the court merely disregards the corporate entity and holds the individual responsible for
acts knowingly and intentionally done in the name of the corporation." Thus, Santos has done in this case. Santos
formed I/ AME, using the non-stock corporation, to evade paying his judgment creditor, Litton.

The piercing of the corporate veil may apply to corporations as well as natural persons involved with corporations.
The Court has held that the "corporate mask may be lifted and the corporate veil may be pierced when a
corporation is just but the alter ego of a person or of another corporation."

I/ AME is the alter ego of Santos and Santos -the natural person -is the alter ego of II AME. Santos falsely
represented himself as President of II AME in the Deed of Absolute Sale when he bought the Makati real property,
at a time when I/ AME had not yet existed. Uncontroverted facts in this case also reveal the findings of Me TC
showing Santos and I/ AME as being one and the same person:

1) Santos is the conceptualizer and implementer of I/AME;


2) Santos' contribution is Pl,200,000.00 (One Million Two Hundred Thousand Pesos) out of the Pl,500,000.00
(One Million Five Hundred Thousand Pesos), making him the majority contributor of I/AME; and,
3) The building being occupied by I/AME is named after Santos using his known nickname (to date it is called,
the "Noli Santos International Tower").

Hence, I/ AME is the alter ego of the natural person, Santos, which the latter used to evade the execution on the
property, thus frustrating the satisfaction of the judgment won by Litton.
CORPORATION LAW CASE DIGESTS | 14
FEU JD4401 | 2nd SEMESTER, S.Y. 2017-2018

b. Reverse Piercing of the Corporate Veil

"Reverse-piercing flows in the opposite direction (of traditional corporate veil-piercing) and makes the corporation
liable for the debt of the shareholders."

It has two (2) types: outsider reverse piercing and insider reverse piercing. Outsider reverse piercing occurs when
a party with a claim against an individual or corporation attempts to be repaid with assets of a corporation owned
or substantially controlled by the defendant. In contrast, in insider reverse piercing, the controlling members will
attempt to ignore the corporate fiction in order to take advantage of a benefit available to the corporation, such as
an interest in a lawsuit or protection of personal assets.

Outsider reverse veil-piercing is applicable in the instant case. Litton, as judgment creditor, seeks the Court's
intervention to pierce the corporate veil of I/ AME in order to make its Makati real property answer for a judgment
against Santos, who formerly owned and still substantially controls I/ AME.

The American court held that "outsider reverse veil-piercing extends the traditional veil-piercing doctrine to permit
a third-party creditor to pierce the veil to satisfy the debts of an individual out of the corporation's assets."

The Court has pierced the corporate veil in a reverse manner in the instances when the scheme was to avoid
corporate assets to be included in the estate of a decedent as in the Cease case and when the corporation was
used to escape a judgment to pay a debt as in the Arcilla case.

It may be possible for this Court to recommend that Litton run after the other properties of Santos that could
satisfy the money judgment -first personal, then other real properties other than that of the school. However, if we
allow this, we frustrate the decades-old yet valid MeTC judgment which levied on the real property now titled under
the name of the school.

Thus, the reverse piercing of the corporate veil of I/ AME to enforce the levy on execution of the Makati real
property where the school now stands is applied.
CORPORATION LAW CASE DIGESTS | 15
FEU JD4401 | 2nd SEMESTER, S.Y. 2017-2018

b. Nationality, Citizenship and Foreign Equity (Place of Incorporation Test, Control Test & Grandfather
Rule)

(9) Gamboa v. Teves


G.R. No. 176579, Oct. 9, 2012
By: Jovero, John Tristram V.

Topic: Capital

Doctrine: The term “capital” in Section 11, Article XII of the Constitution refers only to shares of stock entitled to
vote in the election of directors of a public utility.

Facts: This is a petition to nullify the sale of shares of stock of Philippine Telecommunications Investment
Corporation (PTIC) by the government of the Republic of the Philippines, acting through the Inter-Agency
Privatization Council (IPC), to Metro Pacific Assets Holdings, Inc. (MPAH), an affiliate of First Pacific Company
Limited (First Pacific), a Hong Kong-based investment management and holding company and a shareholder of the
Philippine Long Distance Telephone Company (PLDT).

The petitioner questioned the sale on the ground that it also involved an indirect sale of 12 million shares (or about
6.3 percent of the outstanding common shares) of PLDT owned by PTIC to First Pacific. With the this sale, First
Pacific’s common shareholdings in PLDT increased from 30.7 percent to 37 percent, thereby increasing the total
common shareholdings of foreigners in PLDT to about 81.47%. This, according to the petitioner, violates Section
11, Article XII of the 1987 Philippine Constitution which limits foreign ownership of the capital of a public utility to
not more than 40%.

Issue: Whether or not the term “capital” includes both common shares and non-voting preferred shares?

Held: The term “capital” in Section 11, Article XII of the Constitution refers only to shares of stock entitled to vote
in the election of directors, and thus in the present case only to common shares, and not to the total outstanding
capital stock comprising both common and non-voting preferred shares [of PLDT].

Indisputably, one of the rights of a stockholder is the right to participate in the control or management of the
corporation. This is exercised through his vote in the election of directors because it is the board of directors that
controls or manages the corporation. In the absence of provisions in the articles of incorporation denying voting
rights to preferred shares, preferred shares have the same voting rights as common shares. However, preferred
shareholders are often excluded from any control, that is, deprived of the right to vote in the election of directors
and on other matters, on the theory that the preferred shareholders are merely investors in the corporation for
income in the same manner as bondholders. xxx.

Considering that common shares have voting rights which translate to control, as opposed to preferred shares
which usually have no voting rights, the term “capital” in Section 11, Article XII of the Constitution refers only to
common shares. However, if the preferred shares also have the right to vote in the election of directors, then the
term “capital” shall include such preferred shares because the right to participate in the control or management of
the corporation is exercised through the right to vote in the election of directors. In short, the term “capital” in
Section 11, Article XII of the Constitution refers only to shares of stock that can vote in the election of directors.

It must be stressed, and respondents do not dispute, that foreigners hold a majority of the common shares of
PLDT. In fact, based on PLDT’s 2010 General Information Sheet (GIS), which is a document required to be
submitted annually to the Securities and Exchange Commission, foreigners hold 120,046,690 common shares of
PLDT whereas Filipinos hold only 66,750,622 common shares. In other words, foreigners hold 64.27% of the total
number of PLDT’s common shares, while Filipinos hold only 35.73%. Since holding a majority of the common
shares equates to control, it is clear that foreigners exercise control over PLDT. Such amount of control
unmistakably exceeds the allowable 40 percent limit on foreign ownership of public utilities expressly mandated in
Section 11, Article XII of the Constitution.

The legal and beneficial ownership of 60 percent of the outstanding capital stock must rest in the hands of Filipinos
in accordance with the constitutional mandate. Full beneficial ownership of 60 percent of the outstanding capital
stock, coupled with 60 percent of the voting rights, is constitutionally required for the State’s grant of authority to
operate a public utility. The undisputed fact that the PLDT preferred shares, 99.44% owned by Filipinos, are non-
voting and earn only 1/70 of the dividends that PLDT common shares earn, grossly violates the constitutional
requirement of 60 percent Filipino control and Filipino beneficial ownership of a public utility.
CORPORATION LAW CASE DIGESTS | 16
FEU JD4401 | 2nd SEMESTER, S.Y. 2017-2018

(10) Narra Nickel Mining and Development Corp. vs Redmont Consolidated Mines Corporation
G.R. No. 195580 April 21, 2014
by: Jesus Ros Lapuz, Jr.

Doctrine: The Grandfather Rule or the second part of the SEC Rule applies only when the 60-40 Filipino-foreign
equity ownership is in doubt — where the joint venture corporation with Filipino and foreign stockholders with less
than 60% Filipino stockholdings [or 59%] invests in other joint venture corporation which is either 60-40% Filipino-
alien or the 59% less Filipino.

Facts: Sometime in December 2006, respondent Redmont Consolidated Mines Corp. (Redmont), a domestic
corporation organized and existing under Philippine laws, took interest in mining and exploring certain areas of the
province of Palawan. After inquiring with the Department of Environment and Natural Resources (DENR), it learned
that the areas where it wanted to undertake exploration and mining activities where already covered by Mineral
Production Sharing Agreement (MPSA) applications of petitioners Narra, Tesoro and McArthur. Petitioner McArthur,
through its predecessor-in-interest Sara Marie Mining, Inc. (SMMI), filed an application for an MPSA and
Exploration Permit (EP) with the Mines and Geo-Sciences Bureau (MGB), Region IV-B, Office of the Department of
Environment and Natural Resources (DENR). Subsequently, SMMI was issued MPSA-AMA-IVB-153 covering an area
of over 1,782 hectares in Barangay Sumbiling, Municipality of Bataraza, Province of Palawan and EPA-IVB-44 which
includes an area of 3,720 hectares in Barangay Malatagao, Bataraza, Palawan. The MPSA and EP were then
transferred to Madridejos Mining Corporation (MMC) and, on November 6, 2006, assigned to petitioner McArthur.
Petitioner Narra acquired its MPSA from Alpha Resources and Development Corporation and Patricia Louise Mining
& Development Corporation (PLMDC) which previously filed an application for an MPSA with the MGB, Region IV-B,
DENR on January 6, 1992. Through the said application, the DENR issued MPSA-IV-1-12 covering an area of 3.277
hectares in barangays Calategas and San Isidro, Municipality of Narra, Palawan. Subsequently, PLMDC conveyed,
transferred and/or assigned its rights and interests over the MPSA application in favor of Narra. Another MPSA
application of SMMI was filed with the DENR Region IV-B, labeled as MPSA-AMA-IVB-154 (formerly EPA-IVB-47)
over 3,402 hectares in Barangays Malinao and Princesa Urduja, Municipality of Narra, Province of Palawan. SMMI
subsequently conveyed, transferred and assigned its rights and interest over the said MPSA application to Tesoro.
On January 2, 2007, Redmont filed before the Panel of Arbitrators (POA) of the DENR three (3) separate petitions
for the denial of petitioners’ applications for MPSA designated as AMA-IVB-153, AMA-IVB-154 and MPSA IV-1-12.
In the petitions, Redmont alleged that at least 60% of the capital stock of McArthur, Tesoro and Narra are owned
and controlled by MBMI Resources, Inc. (MBMI), a 100% Canadian corporation. Redmont reasoned that since MBMI
is a considerable stockholder of petitioners, it was the driving force behind petitioners’ filing of the MPSAs over the
areas covered by applications since it knows that it can only participate in mining activities through corporations
which are deemed Filipino citizens. Redmont argued that given that petitioners’ capital stocks were mostly owned
by MBMI, they were likewise disqualified from engaging in mining activities through MPSAs, which are reserved
only for Filipino citizens.

Issue: Whether or not the petitioner corporations are Filipino and can validly be issued MPSA and EP?

Held: NO.
The SEC Rules provide for the manner of calculating the Filipino interest in a corporation for purposes, among
others, of determining compliance with nationality requirements (the ‘Investee Corporation’). Such manner of
computation is necessary since the shares in the Investee Corporation may be owned both by individual
stockholders (‘Investing Individuals’) and by corporations and partnerships (‘Investing Corporation’). The said rules
thus provide for the determination of nationality depending on the ownership of the Investee Corporation and, in
certain instances, the Investing Corporation.

Under the SEC Rules, there are two cases in determining the nationality of the Investee Corporation. The first case
is the ‘liberal rule’, later coined by the SEC as the Control Test in its 30 May 1990 Opinion, and pertains to the
portion in said Paragraph 7 of the 1967 SEC Rules which states, ‘(s)hares belonging to corporations or partnerships
at least 60% of the capital of which is owned by Filipino citizens shall be considered as of Philippine nationality.’
Under the liberal Control Test, there is no need to further trace the ownership of the 60% (or more) Filipino
stockholdings of the Investing Corporation since a corporation which is at least 60% Filipino-owned is considered as
Filipino.

The second case is the Strict Rule or the Grandfather Rule Proper and pertains to the portion in said Paragraph 7 of
the 1967 SEC Rules which states, “but if the percentage of Filipino ownership in the corporation or partnership is
CORPORATION LAW CASE DIGESTS | 17
FEU JD4401 | 2nd SEMESTER, S.Y. 2017-2018

less than 60%, only the number of shares corresponding to such percentage shall be counted as of Philippine
nationality.” Under the Strict Rule or Grandfather Rule Proper, the combined totals in the Investing Corporation and
the Investee Corporation must be traced (i.e., “grandfathered”) to determine the total percentage of Filipino
ownership. Moreover, the ultimate Filipino ownership of the shares must first be traced to the level of the Investing
Corporation and added to the shares directly owned in the Investee Corporation.

In other words, based on the said SEC Rule and DOJ Opinion, the Grandfather Rule or the second part of the SEC
Rule applies only when the 60-40 Filipino-foreign equity ownership is in doubt (i.e., in cases where the joint
venture corporation with Filipino and foreign stockholders with less than 60% Filipino stockholdings [or 59%]
invests in other joint venture corporation which is either 60-40% Filipino-alien or the 59% less Filipino). Stated
differently, where the 60-40 Filipino- foreign equity ownership is not in doubt, the Grandfather Rule will not apply.
CORPORATION LAW CASE DIGESTS | 18
FEU JD4401 | 2nd SEMESTER, S.Y. 2017-2018

(11) Roy III v. Herbosa


G.R. No. 207246, November 22, 2016
By: Mano, Razna I.

Doctrine: While SEC-MC No. 8 does not expressly mention the Beneficial Ownership Test or full beneficial
ownership of stocks requirement in the Foreign Investments Act, this will not, as it does not, render it invalid
meaning, it does not follow that the SEC will not apply this test in determining whether the shares claimed to be
owned by Philippine nationals are Filipino, i.e., are held by them by mere title or in full beneficial ownership.

On May 20, 2013, the SEC issued SEC-MC No. 8 entitled "Guidelines on Compliance with the Filipino-Foreign
Ownership Requirements Prescribed in the Constitution and/or Existing Laws by Corporations Engaged in
Nationalized and Partly Nationalized Activities." Section 2 of SEC-MC No. 8 provides:

Section 2. All covered corporations shall, at all times, observe the constitutional or statutory ownership
requirement. For purposes of determining compliance therewith, the required percentage of Filipino
ownership shall be applied to BOTH (a) the total number of outstanding shares of stock entitled to vote in
the election of directors; AND (b) the total number of outstanding shares of stock, whether or not entitled
to vote in the election of directors.

Corporations covered by special laws which provide specific citizenship requirements shall comply with the
provisions of said law.

Atty. Roy filed the Petition, assailing the validity of SEC-MC No. 8 for not conforming to the letter and spirit of the
Gamboa Decision and Resolution and for having been issued by the SEC with grave abuse of discretion. Petitioner
Roy seeks to apply the 60-40 Filipino ownership requirement separately to each class of shares of a public utility
corporation, whether common, preferred nonvoting, preferred voting or any other class of shares.

Is SEC-MC No. 8 in conformity with the Gamboa Decision and Resolution?

Yes. The "Final Word" of the Gamboa Resolution put to rest the Court's interpretation of the term "capital" that
full beneficial ownership of stocks, coupled with appropriate voting rights is essential.

Section 2 of SEC-MC No. 8 clearly incorporates the Voting Control Test or the controlling interest requirement. In
fact, Section 2 goes beyond requiring a 60-40 ratio in favor of Filipino nationals in the voting stocks; it moreover
requires the 60-40 percentage ownership in the total number of outstanding shares of stock, whether voting or
not. The SEC formulated SEC-MC No. 8 to adhere to the Court's unambiguous pronouncement that "[f]ull beneficial
ownership of 60 percent of the outstanding capital stock, coupled with 60 percent of the voting rights is required."
Clearly, SEC-MC No. 8 cannot be said to have been issued with grave abuse of discretion.

The pronouncement of the Court in the Gamboa Resolution - the constitutional requirement to "apply uniformly and
across the board to all classes of shares, regardless of nomenclature and category, comprising the capital of a
corporation - is clearly an obiter dictum that cannot override the Court's unequivocal definition of the term "capital"
in both the Gamboa Decision and Resolution.

Nowhere in the discussion of the definition of the term "capital" in Section 11, Article XII of the 1987 Constitution
in the Gamboa Decision did the Court mention the 60% Filipino equity requirement to be applied to each class of
shares. The definition of "Philippine national" in the Foreign Investment Act and expounded in its IRR, which the
Court adopted in its interpretation of the term "capital", does not support such application. In fact, even the Final
Word of the Gamboa Resolution does not even intimate or suggest the need for a clarification or re-interpretation.

To revisit or even clarify the unequivocal definition of the term "capital" as referring "only to shares of stock
entitled to vote in the election of directors" and apply the 60% Filipino ownership requirement to each class of
share is effectively and unwarrantedly amending or changing the Gamboa Decision and Resolution. The Gamboa
Decision and Resolution Doctrine did NOT make any definitive ruling that the 60% Filipino ownership requirement
was intended to apply to each class of share.

The fallo or decretal/dispositive portions of both the Gamboa Decision and Resolution are definite, clear and
unequivocal. While there is a passage in the body of the Gamboa Resolution that might have appeared contrary to
the fallo of the Gamboa Decision – capitalized upon by petitioners to espouse a restrictive re-interpretation of
"capital” - the definiteness and clarity of the fallo of the Gamboa Decision must control over the obiter dictum in
the Gamboa Resolution regarding the application of the 60-40 Filipino-foreign ownership requirement to "each class
of shares, regardless of differences in voting rights, privileges and restrictions."
CORPORATION LAW CASE DIGESTS | 19
FEU JD4401 | 2nd SEMESTER, S.Y. 2017-2018

(11) Roy III v. Herbosa, Resolution


G.R. No. 207246, April 18, 2017
By: Mano, Razna I.

Doctrine: If the Filipino has the "specific stock's" voting power (he can vote the stock or direct another to vote for
him), or the Filipino has the investment power over the "specific stock" (he can dispose of the stock or direct
another to dispose it for him), or he has both (he can vote and dispose of the "specific stock" or direct another to
vote or dispose it for him), then such Filipino is the "beneficial owner" of that "specific stock" and that "specific
stock" is considered (or counted) as part of the 60% Filipino ownership of the corporation.

Before the Court is the Motion for Reconsideration dated January 19, 2017 (the Motion) filed by petitioner Jose M.
Roy III (movant) seeking the reversal and setting aside of the Decision dated November 22, 2016 (the Decision)
which denied the movant's petition, and declared that the Securities and Exchange Commission (SEC) did not
commit grave abuse of discretion in issuing Memorandum Circular No. 8, Series of 2013 (SEC-MC No. 8) as the
same was in compliance with, and in fealty to, the decision of the Court in Gamboa v. Finance Secretary Teves,
(Gamboa Decision) and the resolution denying the Motion for Reconsideration therein (Gamboa Resolution).

Some of the grounds raised by movant are: He did not rely on an obiter dictum; and the Court should have
treated the petition as the appropriate device to explain the Gamboa Decision.

Are there compelling and new arguments to justify the reconsideration of the Roy III v. Herbosa
Decision dated November 22, 2016?

NONE. The Decision has painstakingly explained why it considered as obiter dictum that pronouncement in the
Gamboa Resolution that the constitutional requirement on Filipino ownership should "apply uniformly and across
the board to all classes of shares, regardless of nomenclature and category, comprising the capital of a
corporation." The Court stated that:

[T]he fallo or decretal/dispositive portions of both the Gamboa Decision and Resolution are definite, clear
and unequivocal. While there is a passage in the body of the Gamboa Resolution that might have
appeared contrary to the fallo of the Gamboa Decision x x x the definiteness and clarity of the fallo of the
Gamboa Decision must control over the obiter dictum in the Gamboa Resolution regarding the application
of the 60-40 Filipino-foreign ownership requirement to "each class of shares, regardless of differences in
voting rights, privileges and restrictions."

To the Court's mind and, as exhaustively demonstrated in the Decision, the dispositive portion of the Gamboa
Decision was in no way modified by the Gamboa Resolution.

The heart of the controversy is the interpretation of Section 11, Article XII of the Constitution, which provides: "No
franchise, certificate, or any other form of authorization for the operation of a public utility shall be granted except
to citizens of the Philippines or to corporations or associations organized under the laws of the Philippines at least
sixty per centum of whose capital is owned by such citizens x x x."

The Gamboa Decision already held, in no uncertain terms, that what the Constitution requires is "[fJull [and legal]
beneficial ownership of 60 percent of the outstanding capital stock, coupled with 60 percent of the voting rights x x
x must rest in the hands of Filipino nationals x x x." And, precisely that is what SEC-MC No. 8 provides.

In construing "full beneficial ownership," the Implementing Rules and Regulations of the Foreign Investments Act of
1991 (FIA-IRR) provides:

For stocks to be deemed owned and held by Philippine citizens or Philippine nationals, mere legal title is
not enough to meet the required Filipino equity. Full beneficial ownership of the stocks, coupled with
appropriate voting rights is essential. Thus, stocks, the voting rights of which have been assigned or
transferred to aliens cannot be considered held by Philippine citizens or Philippine nationals.

In turn, "beneficial owner" or "beneficial ownership" is defined in the Implementing Rules and Regulations of the
Securities Regulation Code (SRC-IRR) as:

[A]ny person who, directly or indirectly, through any contract, arrangement, understanding, relationship
or otherwise, has or shares voting power (which includes the power to vote or direct the voting of such
CORPORATION LAW CASE DIGESTS | 20
FEU JD4401 | 2nd SEMESTER, S.Y. 2017-2018

security) and/or investment returns or power (which includes the power to dispose of, or direct the
disposition of such security) x x x.

Thus, the definition of "beneficial owner or beneficial ownership" in the SRC-IRR, which is in consonance with the
concept of "full beneficial ownership" in the FIA-IRR, is, as stressed in the Decision, relevant in resolving only the
question of who is the beneficial owner or has beneficial ownership of each "specific stock" of the public utility
company whose stocks are under review. If the Filipino has the voting power of the "specific stock", i.e., he can
vote the stock or direct another to vote for him, or the Filipino has the investment power over the "specific
stock", i.e., he can dispose of the stock or direct another to dispose of it for him, or both, i.e., he can vote and
dispose of that "specific stock" or direct another to vote or dispose it for him, then such Filipino is the "beneficial
owner" of that "specific stock." Being considered Filipino, that "specific stock" is then to be counted as part of the
60% Filipino ownership requirement under the Constitution. The right to the dividends, jus fruendi - a right
emanating from ownership of that "specific stock" necessarily accrues to its Filipino "beneficial owner."

Once more, this is emphasized anew to disabuse any notion that the dividends accruing to any particular stock are
determinative of that stock's "beneficial ownership." Dividend declaration is dictated by the corporation's
unrestricted retained earnings. On the other hand, the corporation's need of capital for expansion programs and
special reserve for probable contingencies may limit retained earnings available for dividend declaration. It bears
repeating here that the Court in the Gamboa Decision adopted the foregoing definition of the term "capital" in
Section 11, Article XII of the 1987 Constitution in express recognition of the sensitive and vital position of public
utilities both in the national economy and for national security, so that the evident purpose of the citizenship
requirement is to prevent aliens from assuming control of public utilities, which may be inimical to the national
interest. This purpose prescinds from the "benefits"/dividends that are derived from or accorded to the particular
stocks held by Filipinos vis-a-vis the stocks held by aliens. So long as Filipinos have controlling interest of a public
utility corporation, their decision to declare more dividends for a particular stock over other kinds of stock is their
sole prerogative - an act of ownership that would presumably be for the benefit of the public utility corporation
itself.

Since Filipinos own at least 60% of the outstanding shares of stock entitled to vote directors, which is what the
Constitution precisely requires, then the Filipino stockholders control the corporation, i.e., they dictate corporate
actions and decisions, and they have all the rights of ownership including, but not limited to, offering certain
preferred shares that may have greater economic interest to foreign investors - as the need for capital for
corporate pursuits (such as expansion), may be good for the corporation that they own. Surely, these "true
owners" will not allow any dilution of their ownership and control if such move will not be beneficial to them.
CORPORATION LAW CASE DIGESTS | 21
FEU JD4401 | 2nd SEMESTER, S.Y. 2017-2018

c. Classes of Corporations

(12) Boys Scouts of the Philippines vs Commission on Audit


GR No. 177131, June 7, 2011
By: Marasigan, Mariz Angelle R.

Doctrine: The Boy Scouts of the Philippines is a public corporation and its funds are subject to the Commission on
Audit’s audit jurisdiction.

The COA issued Resolution No. 99-0115 with the subject “Defining the Commission’s policy with respect to the
audit of the Boy Scouts of the Philippines.” In its whereas clauses, the COA Resolution stated that the BSP was
created as a public corporation under Commonwealth Act No. 111, as amended by Presidential Decree No. 460 and
Republic Act No. 7278; that in Boy Scouts of the Philippines v. National Labor Relations Commission, the Supreme
Court ruled that the BSP, as constituted under its charter, was a “government-controlled corporation within the
meaning of Article IX(B)(2)(1) of the Constitution”; and that “the BSP is appropriately regarded as a government
instrumentality under the 1987 Administrative Code.” The COA Resolution also cited its constitutional mandate
under Section 2(1), Article IX (D). Finally, the COA Resolution reads:

“NOW THEREFORE, in consideration of the foregoing premises, the COMMISSION PROPER HAS RESOLVED, AS IT
DOES HEREBY RESOLVE, to conduct an annual financial audit of the Boy Scouts of the Philippines in
accordance with generally accepted auditing standards, and express an opinion on whether the financial
statements which include the Balance Sheet, the Income Statement and the Statement of Cash Flows present fairly
its financial position and results of operations.
xxxx
BE IT RESOLVED FURTHERMORE, that for purposes of audit supervision, the Boy Scouts of the Philippines shall
be classified among the government corporations belonging to the Educational, Social, Scientific, Civic
and Research Sector under the Corporate Audit Office I, to be audited, similar to the subsidiary corporations, by
employing the team audit approach.” (Emphases supplied.)

The BSP sought reconsideration of the COA Resolution in a letter signed by the BSP National President Jejomar C.
Binay.

The BSP contends that while it concedes that its functions do relate to those that the government might otherwise
completely assume on its own, it avers that this alone was not determinative of the COA’s audit jurisdiction over it.
The BSP further avers that the Court in Boy Scouts of the Philippines v. National Labor Relations Commission
"simply stated x x x that in respect of functions, the BSP is akin to a public corporation" but this was not
synonymous to holding that the BSP is a government corporation or entity subject to audit by the COA.

The BSP contends that Republic Act No. 7278 introduced crucial amendments to its charter; hence, the findings of
the Court in Boy Scouts of the Philippines v. National Labor Relations Commission are no longer valid as the
government has ceased to play a controlling influence in it. The BSP claims that the pronouncements of the Court
therein must be taken only within the context of that case; that the Court had categorically found that its assets
were acquired from the Boy Scouts of America and not from the Philippine government, and that its operations are
financed chiefly from membership dues of the Boy Scouts themselves as well as from property rentals; and that
"the BSP may correctly be characterized as non-governmental, and hence, beyond the audit jurisdiction of the
COA."

To summarize its other arguments, the BSP contends that it is not a government-owned or controlled corporation;
neither is it an instrumentality, agency, or subdivision of the government.

In its Comment, the COA argues as follows:


1. The BSP is a public corporation created under Commonwealth Act No. 111 dated October 31, 1936, and whose
functions relate to the fostering of public virtues of citizenship and patriotism and the general improvement of the
moral spirit and fiber of the youth. The manner of creation and the purpose for which the BSP was created
indubitably prove that it is a government agency.
2. Being a government agency, the funds and property owned or held in trust by the BSP are subject to the audit
authority of respondent Commission on Audit pursuant to Section 2 (1), Article IX-D of the 1987 Constitution.
CORPORATION LAW CASE DIGESTS | 22
FEU JD4401 | 2nd SEMESTER, S.Y. 2017-2018

3. Republic Act No. 7278 did not change the character of the BSP as a government-owned or controlled corporation
and government instrumentality.
Whether the BSP is a public corporation and thus falls under the COA’s audit jurisdiction

Held: Yes.

There are three classes of juridical persons under Article 44 of the Civil Code and the BSP, as presently constituted
under Republic Act No. 7278, falls under the second classification. Article 44 reads:
Art. 44. The following are juridical persons:
(1) The State and its political subdivisions;
(2) Other corporations, institutions and entities for public interest or purpose created by law; their
personality begins as soon as they have been constituted according to law;
(3) Corporations, partnerships and associations for private interest or purpose to which the law grants a
juridical personality, separate and distinct from that of each shareholder, partner or member. (Emphases supplied.)

The BSP, which is a corporation created for a public interest or purpose, is subject to the law creating it under
Article 45 of the Civil Code, which provides:
Art. 45. Juridical persons mentioned in Nos. 1 and 2 of the preceding article are governed by the laws
creating or recognizing them.

The purpose of the BSP as stated in its amended charter shows that it was created in order to implement a State
policy declared in Article II, Section 13 of the Constitution.

Evidently, the BSP, which was created by a special law to serve a public purpose in pursuit of a constitutional
mandate, comes within the class of "public corporations" defined by paragraph 2, Article 44 of the Civil Code and
governed by the law which creates it, pursuant to Article 45 of the same Code.

The BSP is a public corporation or a government agency or instrumentality with juridical personality, which does
not fall within the constitutional prohibition in Article XII, Section 16, notwithstanding the amendments to its
charter. Not all corporations, which are not government owned or controlled, are ipso facto to be considered
private corporations as there exists another distinct class of corporations or chartered institutions which are
otherwise known as “public corporations.” These corporations are treated by law as agencies or instrumentalities of
the government which are not subject to the tests of ownership or control and economic viability but to different
criteria relating to their public purposes/interests or constitutional policies and objectives and their administrative
relationship to the government or any of its Departments or Offices.
CORPORATION LAW CASE DIGESTS | 23
FEU JD4401 | 2nd SEMESTER, S.Y. 2017-2018

(13) Republic vs. City of Parañaque


G.R. No. 191109. July 18, 2012.
By: Pangilinan, Gene Alexis

Doctrine: Section 3 of the Corporation Code defines a stock corporation as one whose “capital stock is divided into
shares and x x x authorized to distribute to the holders of such shares dividends x x x.” Section 87 thereof defines
a non-stock corporation as “one where no part of its income is distributable as dividends to its members, trustees
or officers.”

Many government instrumentalities are vested with corporate powers but they do not become stock or non-stock
corporations, which is a necessary condition before an agency or instrumentality is deemed a Government-Owned
and Controlled Corporations (GOCC); These government instrumentalities are sometimes loosely called
government corporate entities.

Facts: The Public Estates Authority (PEA) is a government corporation created by virtue of P.D. No. 1084. PEA was
designated as the agency primarily responsible for integrating, directing and coordinating all reclamation projects
for and on behalf of the National Government. President Gloria Macapagal-Arroyo issued E.O. No. 380 transforming
PEA into Philippine Reclamation Authority (PRA), which shall perform all the powers and functions of the PEA
relating to reclamation activities. As such, PRA reclaimed several portions of the foreshore and offshore areas of
Manila Bay, including those located in Parañaque City.

Parañaque City Treasurer issued Warrants of Levy on PRA’s reclaimed properties located in Parañaque City based
on the assessment for delinquent real property taxes made by then Parañaque City Assessor for tax years 2001
and 2002. PRA moved to declare as null and void the assessment for real property taxes with the RTC. The RTC
ruled that petitioner PRA is a GOCC, a taxable entity, and, therefore, not exempt from payment of real property
taxes.

PRA asserts that it is not a GOCC under Section 2(13) of the Introductory Provisions of the Administrative Code.
Neither is it a GOCC under Section 16, Article XII of the 1987 Constitution. Instead, PRA is a government
instrumentality vested with corporate powers and performing an essential public service. Although it has a capital
stock divided into shares, it is not authorized to distribute dividends and allotment of surplus and profits to its
stockholders. Therefore, it may not be classified as a stock corporation because it lacks the second requisite of a
stock corporation which is the distribution of dividends and allotment of surplus and profits to the stockholders.

It insists that it may not be classified as a non-stock corporation because it has no members and it is not organized
for charitable, religious, educational, professional, cultural, recreational, fraternal, literary, scientific, social, civil
service, or similar purposes, like trade, industry, agriculture and like chambers as provided in Section 88 of the
Corporation Code.

Issue: Whether or not petitioner PRA is a GOCC, a taxable entity not exempt from payment of real property taxes.

Held: NO. PRA is not a GOCC because it is neither a stock nor a non-stock corporation. PRA is a government
instrumentality vested with corporate powers and performing an essential public service pursuant to the
Administrative Code. Being an incorporated government instrumentality, it is exempt from payment of real
property tax. Under the Administrative Code, a GOCC must be “organized as a stock or non-stock corporation”
while an instrumentality is vested by law with corporate powers.

When the law vests in a government instrumentality corporate powers, the instrumentality does not necessarily
become a corporation. Unless the government instrumentality is organized as a stock or non-stock corporation, it
remains a government instrumentality exercising not only governmental but also corporate powers.

Section 3 of the Corporation Code defines a stock corporation as one whose “capital stock is divided into shares
and x x x authorized to distribute to the holders of such shares dividends x x x.” Section 87 thereof defines a non-
stock corporation as “one where no part of its income is distributable as dividends to its members, trustees or
officers.” Further, Section 88 provides that non-stock corporations are “organized for charitable, religious,
educational, professional, cultural, recreational, fraternal, literary, scientific, social, civil service, or similar
purposes, like trade, industry, agriculture and like chambers.”
CORPORATION LAW CASE DIGESTS | 24
FEU JD4401 | 2nd SEMESTER, S.Y. 2017-2018

Two requisites must concur before one may be classified as a stock corporation, namely: (1) that it has capital
stock divided into shares; and (2) that it is authorized to distribute dividends and allotments of surplus and profits
to its stockholders. If only one requisite is present, it cannot be properly classified as a stock corporation. As for
non-stock corporations, they must have members and must not distribute any part of their income to said
members.

PRA cannot be considered as a stock corporation because although it has a capital stock divided into no par value
shares as provided in Section 7 of P.D. No. 1084, it is not authorized to distribute dividends, surplus allotments or
profits to stockholders.

PRA cannot be considered a non-stock corporation either because it does not have members. A non-stock
corporation must have members. Moreover, it was not organized for any of the purposes mentioned in Section 88
of the Corporation Code. Specifically, it was created to manage all government reclamation projects.

Also, Section 16, Article XII of the 1987 Constitution authorizes Congress to create GOCCs through special charters
on two conditions: 1) the GOCC must be established for the common good; and 2) the GOCC must meet the test of
economic viability. In this case, PRA may have passed the first condition of common good but failed the second one
—economic viability. Undoubtedly, the purpose behind the creation of PRA was not for economic or commercial
activities. Neither was it created to compete in the market place considering that there were no other competing
reclamation companies being operated by the private sector.
CORPORATION LAW CASE DIGESTS | 25
FEU JD4401 | 2nd SEMESTER, S.Y. 2017-2018

(14) Torres vs. De Leon


By: Radovan, Althea

Doctrine: PNRC is not a GOCC, but it is sui generis in character, thus, requiring this Court to approach
controversies involving the PNRC on a case-to- case basis.

Facts: Torres (Petitioner) was the Chapter Administrator of the Philippine National Red Cross (PNRC). Torres was
charged with grave misconduct for violating PNRC Financial Policies, as based on an audit report by respondent De
Leon, petitioner incurred a “technical shortage” in the amount of P4, 306,574.23.
A 1 month suspension and transfer to the National Headquarters was imposed against Petitioner.
Petitioner appealed to the Civil Service Commission (CSC). The CSC, on April 21, 2008, promulgated a Resolution
dismissing petitioner’s appeal and imposing upon her the penalty of dismissal from service. Petitioner filed a motion
for reconsideration with the CSC, but the same was denied. Hence this petition.

According to petitioner, this Court has decided that PNRC is not a government-owned and-controlled corporation
(GOCC), hence, the CSC has no jurisdiction or authority to review the appeal that she herself filed. As such, she
insists that the CSC committed grave abuse of discretion in modifying the decision of respondent De Leon.

Issue:
1. Whether the PNRC is a GOCC?
2. Whether the CSC has jurisdiction over the PNRC in this case?

Held:
1. PNRC is not a GOCC, but it is sui generis in character, thus, requiring this Court to approach controversies
involving the PNRC on a case-to- case basis.

The PNRC, as a National Society of the International Red Cross and Red Crescent Movement, can neither “be
classified as an instrumentality of the State, so as not to lose its character of neutrality” as well as its
independence, nor strictly as a private corporation since it is regulated by international humanitarian law and is
treated as an auxiliary of the State. Although it is neither a subdivision, agency, or instrumentality of the
government, nor a government-owned or -controlled corporation or a subsidiary thereof, such a conclusion does
not ipso facto imply that the PNRC is a “private corporation” within the contemplation of the provision
of the Constitution, that must be organized under the Corporation Code. As correctly mentioned by
Justice Roberto A. Abad, the sui generis character of PNRC requires us to approach controversies
involving the PNRC on a case-to-case basis.

2. YES. The CSC has jurisdiction over the PNRC because the issue at hand is the enforcement of labor laws and
penal statutes, thus, in this particular matter, the PNRC can be treated as a GOCC and as such, it is within the
ambit of Rule I, Section 1 of the Implementing Rules of Republic Act No. 6713,5 stating that:

Section 1. These Rules shall cover all officials and employees in the government, elective and appointive,
permanent or temporary, whether in the career or noncareer service, including military and police personnel,
whether or not they receive compensation, regardless of amount.

Thus, having jurisdiction over the PNRC, the CSC had authority to modify the penalty and order the dismissal of
petitioner from the service.
CORPORATION LAW CASE DIGESTS | 26
FEU JD4401 | 2nd SEMESTER, S.Y. 2017-2018

2. INCORPORATION AND ORGANIZATION

(15) GSIS Family Bank – Thrift Bank vs. BPI Family Bank
G.R. No. 175278, September 23, 2015
By: Rosario, Patricia Kaye T.

Doctrine: To fall within the prohibition of the law on the right to the exclusive use of a corporate name, two
requisites must concur: (1) that the complainant corporation acquired a prior right over the use of such corporate
name; and (2) the proposed name is either: (a) identical or (b) deceptive or confusingly similar to that of any
existing corporation or to any other name already protected by law; or (c) patently deceptive, confusing or
contrary to existing law. In determining the existence of confusing similarity in corporate names, the test is
whether the similarity is such as to mislead a person using ordinary care and discrimination. And even without such
proof of actual confusion between the two corporate names, it suffices that confusion is probable or likely to occur.

To improve its marketability to the public, petitioner changed its corporate name to "GSIS Family Bank, a Thrift
Bank." When respondent learned that petitioner is using or attempting to use the name "Family Bank," respondent
petitioned the SEC Company Registration and Monitoring Department (SEC CRMD) to disallow or prevent the
registration of the name "GSIS Family Bank" or any other corporate name with the words "Family Bank" in it.
Respondent claimed exclusive ownership to the name "Family Bank" and alleged that it is known as "BPI Family
Bank" or simply "Family Bank" since 1969. As such, it has acquired a reputation and goodwill under the name, not
only with clients here and abroad, but also with correspondent and competitor banks, and the public in general.

Are the names BPI Family Bank and GSIS Family Bank confusingly similar as to require the amendment
of the name of the latter corporation?

YES, they are confusingly similar.

Section 18 of the Corporation Code provides that no corporate name may be allowed by the SEC if the proposed
name is identical or deceptively or confusingly similar to that of any existing corporation or to any other name
already protected by law or is patently deceptive, confusing or contrary to existing laws. In Philips Export B.V. v.
Court of Appeals, this Court ruled that to fall within the prohibition of the law on the right to the exclusive use of a
corporate name, two requisites must be proven, namely:
(1) that the complainant corporation acquired a prior right over the use of such corporate name; and
(2) the proposed name is either:
(a) identical or
(b) deceptive or confusingly similar to that of any existing corporation or to any other name
already protected by law; or
(c) patently deceptive, confusing or contrary to existing law.

These two requisites are present in this case. On the first requisite of a prior right, respondent was incorporated in
1969 as Family Savings Bank and in 1985 as BPI Family Bank. Petitioner was incorporated as GSIS Family – Thrift
Bank only in 2002, or at least 17 years after respondent started using its name.

The second requisite obtains on two points: the proposed name is (a) identical or (b) deceptive or confusingly
similar to that of any existing corporation or to any other name already protected by law. On the first point (a), the
words "Family Bank" present in both petitioner and respondent's corporate name satisfy the requirement that there
be identical names in the existing corporate name and the proposed one. To show contrast with respondent's
corporate name, petitioner used the words "GSIS" and "thrift." But these are not sufficiently distinct words that
differentiate petitioner's corporate name from respondent's.

On the second point (b), there is a deceptive and confusing similarity between petitioner's proposed name and
respondent's corporate name. In determining the existence of confusing similarity in corporate names, the test is
whether the similarity is such as to mislead a person using ordinary care and discrimination. And even without such
proof of actual confusion between the two corporate names, it suffices that confusion is probable or likely to occur.
Petitioner's corporate name is "GSIS Family Bank—A Thrift Bank" and respondent's corporate name is "BPI Family
Bank." The overriding consideration in determining whether a person, using ordinary care and discrimination, might
be misled is the circumstance that both petitioner and respondent are engaged in the same business of banking.
The likelihood of confusion is accentuated in cases where the goods or business of one corporation are the same or
substantially the same to that of another corporation.
CORPORATION LAW CASE DIGESTS | 27
FEU JD4401 | 2nd SEMESTER, S.Y. 2017-2018

(16) Indian Chamber of Commerce Phils., Inc. v. Filipino Indian Chamber of Commerce in the
Philippines Inc.
G.R. No. 184008, August 3, 2016
By: Samson, Maria Johanna Ilyssa

Topic: Corporate Name

Doctrine: Section 18 of the Corporation Code expressly prohibits the use of a corporate name which is identical or
deceptively or confusingly similar to that of any existing corporation. It is settled that to determine the existence of
confusing similarity in corporate names, the test is whether the similarity is such as to mislead a person, using
ordinary care and discrimination.

Filipino-Indian Chamber of Commerce of the Philippines, Inc. (defunct FICCPI) was originally registered with the
SEC as Indian Chamber of Commerce of Manila, Inc., it amended its corporate name into Indian Chamber of
Commerce of the Philippines, Inc., and further amended it into Filipino-Indian Chamber of Commerce of the
Philippines Inc.

SEC Case No. 05-008: Mr. Naresh Mansukhani (Mansukhani) reserved the corporate name “Filipino Indian
Chamber of Commerce in the Philippines, Inc.” (FICCPI), for the period from January 20, 2005 to April 20, 2005,
with the Company Registration and Monitoring Department (CRMD) of the SEC. In an opposition letter, Ram
Sitaldas (Sitaldas), claiming to be a representative of the defunct FICCPI, alleged that the corporate name has
been used by the defunct FICCPI since 1951, and that the reservation by another person who is not its member or
representative is illegal. The CRMD rendered a decision granting Mansukhani’s reservation, holding that he
possesses the better right over the corporate name. Thus, the name “Filipino Indian Chamber of Commerce in the
Philippines, Inc.” is free for appropriation by any party. Sitaldas appealed the decision of the CRMD to the SEC En
Banc. SEC En Banc denied the appeal. Sitaldas appealed the SEC En Banc decision to the CA. The CA affirmed the
decision of the SEC En Banc. It ruled that Mansukhani, reserving the name “Filipino Indian Chamber of Commerce
in the Philippines, Inc.,” has the better right over the corporate name. It ruled that with the expiration corporate
life of the defunct FICCPI, without an extension having been filed and granted, it lost its legal personality as a
corporation. The CA affirmed the SEC En Banc ruling that after the expiration of its term, the defunct FICCPI’s
rights over the name also ended.

SEC Case No. 06-014: Meanwhile, Mr. Pracash Dayacan, who allegedly represented the defunct FICCPI, filed an
application with the CRMD for the reservation of the corporate name “Indian Chamber of Commerce Phils., Inc.”
(ICCPI). Upon knowledge, Mansukhani, in a letter formally opposed the application. Mansukhani cited the SEC En
Banc decision in SEC Case No. 05-008 recognizing him as the one possessing the better right over the corporate
name “Filipino Chamber of Commerce in the Philippines, Inc. CRMD denied Mansukhani’s opposition, it stated that
the name “Indian Chamber of Commerce Phils., Inc.” is not deceptively or confusingly similar to “Filipino Indian
Chamber of Commerce in the Philippines, Inc.” CRMD approved and issued the Certificate of Incorporation of
petitioner ICCPI. Thus, respondent FICCPI, through Mansukhani, appealed the CRMD’s decision to the SEC En Banc.
The SEC En Banc granted the appeal filed by FICCPI, and reversed the CRMD’s decision. Citing Section 18 of the
Corporation Code, the SEC En Banc made a finding that both from the standpoint of their [ICCPI and FICCPI]
corporate names and the purposes for which they were established, there exists a similarity that could inevitably
lead to confusion. ICCPI appealed the SEC En Banc decision to the CA. The CA affirmed the decision of the SEC En
Banc. It held that by simply looking at the corporate names of ICCPI and FICCPI, one may readily notice the
striking similarity between the two. ICCPI now appeals the CA decision before this Court.

Did the Court of Appeals committed serious error when it held that there is similarity between the
petitioner and the respondent’s corporate name that would inevitably lead to confusion?

NO, Section 18 of the Corporation Code expressly prohibits the use of a corporate name which is identical or
deceptively or confusingly similar to that of any existing corporation: “No corporate name may be allowed by the
Securities and Exchange Commission if the proposed name is identical or deceptively or confusingly similar to that
of any existing corporation or to any other name already protected by law or is patently deceptive, confusing or
contrary to existing laws. When a change in the corporate name is approved, the Commission shall issue an
amended certificate of incorporation under the amended name.”

In Philips Export B.V. v. Court of Appeals, this Court ruled that to fall within the prohibition, two requisites must be
proven, to wit: (1) that the complainant corporation acquired a prior right over the use of such corporate name;
and (2) the proposed name is either: (a) identical; or (b) deceptively or confusingly similar to that of any existing
corporation or to any other name already protected by law; or (c) patently deceptive, confusing or contrary to
existing law. These two requisites are present in this case. FICCPI acquired a prior right over the use of the
corporate name. In Industrial Refractories Corporation of the Philippines v. Court of Appeals, the Court applied the
priority of adoption rule to determine prior right, taking into consideration the dates when the parties used their
respective corporate names. In this case, FICCPI was incorporated on March 14, 2006. On the other hand, ICCPI
CORPORATION LAW CASE DIGESTS | 28
FEU JD4401 | 2nd SEMESTER, S.Y. 2017-2018

was incorporated only on April 5, 2006, or a month after FICCPI registered its corporate name. Thus, applying the
principle in the Refractories case, we hold that FICCPI, which was incorporated earlier, acquired a prior right over
the use of the corporate name. When the term of existence of the defunct FICCPI expired on November 24, 2001,
its corporate name cannot be used by other corporations within three years from that date, until November 24,
2004. FICCPI reserved the name “Filipino Indian Chamber of Commerce in the Philippines, Inc.” on January 20,
2005, or beyond the three-year period. Thus, the SEC was correct when it allowed FICCPI to use the reserved
corporate name.

The second requisite in the Philips Export case likewise obtains in two respects: the proposed name is (a) identical
or (b) deceptively or confusingly similar to that of any existing corporation or to any other name already protected
by law. ICCPI argues that the word “Filipino” in FICCPI’s corporate name makes it easily distinguishable from
ICCPI. It adds that confusion and deception are effectively precluded by appending the word “Filipino” to the
phrase “Indian Chamber of Commerce.” Further, ICCPI claims that the corporate name of FICCPI uses the words
“in the Philippines” while ICCPI uses only “Phils., Inc.” ICCPI’s arguments are without merit. These words do not
effectively distinguish the corporate names. On the one hand, the word “Filipino” is merely a description, referring
to a Filipino citizen or one living in the Philippines, to describe the corporation’s members. On the other, the words
“in the Philippines” and “Phils., Inc.” are simply geographical locations of the corporations which, even if appended
to both the corporate names, will not make one distinct from the other. Under the facts of this case, these words
cannot be separated from each other such that each word can be considered to add distinction to the corporate
names. Taken together, the words in the phrase “in the Philippines” and in the phrase “Phils., Inc.” are
synonymous — they both mean the location of the corporation. On the second point, ICCPI’s corporate name is
deceptively or confusingly similar to that of FICCPI. It is settled that to determine the existence of confusing
similarity in corporate names, the test is whether the similarity is such as to mislead a person, using ordinary care
and discrimination. In so doing, the court must examine the record as well as the names themselves.58 Proof of
actual confusion need not be shown. It suffices that confusion is probably or likely to occur. In this case, the
overriding consideration in determining whether a person, using ordinary care and discrimination, might be misled
is the circumstance that both ICCPI and FICCPI have a common primary purpose, that is, the promotion of Filipino-
Indian business in the Philippines.
CORPORATION LAW CASE DIGESTS | 29
FEU JD4401 | 2nd SEMESTER, S.Y. 2017-2018

(17) Paz vs. New International Environmental University


G.R. No. 203993, April 20, 2015
By: Torres, Ma. Roma
Topic: Incorporation and Organization

DOCTRINE: Section 21 of the Corporation Code explicitly provides that one who assumes an obligation to an
ostensible corporation, as such, cannot resist performance thereof on the ground that there was in fact no
corporation

FACTS: Paz, as OIC of the Aircraft Hangar at the Davao International Airport, entered into a MOA with Capt.
Clarke, President of International Environmental University, with the former allowing the latter to use the aircraft
hangar space ”exclusively for company aircraft/helicopter “. Later, Paz complained in a letter that the space was
being used for trucks and equipment, vehicles maintenance and fabrication and manifested therein the termination
of the MOA if said activities will not be stopped. Several letters were sent to the University addressed to Mr. Allan
Clarke, with the last letter demanding the latter to immediately vacate the hangar space and that an application for
electrical disconnection shall be filed to compel the university to desist from continuing the works. The University
filed a complaint for breach of contract against petitioner for cutting its electric and telephone lines, blocking its
employees entrance to the premises, and taking over the hangar space without giving it the requisite 6-month
notice of termination.

In his defense, petitioner alleged, among others, that respondent had no cause of action against him as the MOA
was executed between him and Capt. Clarke in the latter's personal capacity. It was found by the RTC that
respondent was issued the Certificate of Incorporation as New International Environmental Universality, Inc. but
when it amended its Articles of Incorporation, the SEC Extension Office erroneously used the name New
International Environmental University, Inc. The CA ruled that, while there was no corporate entity at the time of
the execution of the MOA when Capt. Clarke signed as "President of International Environmental University,"
petitioner is nonetheless estopped from denying that he had contracted with respondent as a corporation, having
recognized the latter as the "Second Party" in the MOA that "will use the hangar space exclusively for company
aircraft/helicopter." Petitioner was likewise found to have issued checks to respondent, which belied his claim of
contracting with Capt. Clarke in the latter's personal capacity

ISSUE: WON the MOA was entered into by Capt. Clarke in his personal capacity, and thus resulting to a lack of
cause of action of the respondent company to sue Paz for breach of contract

HELD: No, the MOA was not entered by Capt. Clarke in his personal capacity.

From the very language itself of the MOA entered into by petitioner whereby he obligated himself to allow the use
of the hangar space "for company aircraft/helicopter," petitioner cannot deny that he contracted with respondent.
Petitioner further acknowledged this fact in his final letter, where he reiterated and strongly demanded the former
to immediately vacate the hangar space his "company is occupying/utilizing."

Whether or not Capt. Clarke should have been impleaded as an indispensable party was correctly resolved by the
CA which held that the former was merely an agent of respondent. While Capt. Clarke's name and signature
appeared on the MOA, his participation was, nonetheless, limited to being a representative of respondent. As a
mere representative, Capt. Clarke acquired no rights whatsoever, nor did he incur any liabilities, arising from the
contract between petitioner and respondent. Therefore, he was not an indispensable party to the case at bar

Section 21 of the Corporation Code explicitly provides that one who assumes an obligation to an ostensible
corporation, as such, cannot resist performance thereof on the ground that there was in fact no corporation.
Clearly, petitioner is bound by his obligation under the MOA not only on estoppel but by express provision of law.
As aptly raised by respondent in its Comment to the instant petition, it is futile to insist that petitioner issued
the receipts for rental payments in respondent's name and not with Capt. Clarke's, whom petitioner
allegedly contracted in the latter's personal capacity, only because it was upon the instruction of an employee.
Indeed, it is disputably presumed that a person takes ordinary care of his concerns, and that all private
transactions have been fair and regular. Hence, it is assumed that petitioner, who is a pilot, knew what he was
doing with respect to his business with respondent.
CORPORATION LAW CASE DIGESTS | 30
FEU JD4401 | 2nd SEMESTER, S.Y. 2017-2018

(18) Alfredo Montelibano v. Bacolod-Murcia Milling Co., Inc.


G.R. No. L-15092 May 18, 1962
By: Valencia, Emmanuelle Nicole L.

Topic: Exercise of Charter Powers, Probable Losses or Decrease in Profits

Doctrines: The test to be applied is whether the act in question is in direct and immediate furtherance of the
corporation’s business, fairly incident to the express powers and reasonably necessary to their exercise. If so, the
corporation has the power to do it; otherwise, not.

Whether or not a valid and binding resolution passed by the board of directors will cause losses or decrease the
profits of the corporation may not be reviewed by the courts.

Alfredo Montelibano, Alejandro Montelibano and the limited co-partnership Gonzaga and Company are sugar
planters, and have identical milling contracts with Bacolod-Murcia Milling Company, Inc. These contracts were
originally executed in 1919, and were in force for 30 years, beginning with the 1920-21 crop, and provided that the
resulting product should be divided as 45% for the mill, and 55% for the planters.

In 1936, amendments were made to the milling contracts, increasing the planters’ share to 60% of the
manufactured sugar and molasses, other concessions, and extending the operation of the contract to 45 years. A
printed Amended Milling Contract form was drawn up, and on August 20, 1936, Bacolod-Murcia’s Board of Directors
adopted a resolution granting further concessions to the planters, above those contained in the Amended Milling
Contract. Paragraph 9 stated (in Spanish, roughly translated): The Board, in consideration to the planters’ request,
agrees to amend the milling contract as follows: if, during the validity of the Amended Milling Contract, the sugar
mills of Negros Occidental, whose annual production of centrifugal sugar is more than 1/3 of the total production of
all sugar mills in Negros Occidental, grants their planters better conditions than are stipulated in this contract,
those better conditions will be granted to the planters who have signed this Amended Milling Contract.

Montelibano, et. al. signed and executed the printed Amended Milling Contract on September 10, 1936. The copy
of the resolution (dated August 20, 1936) was not attached to the printed contract until April 17, 1937, with the
notation (in Spanish, roughly translated): The amendments transcribed above are part of the Amended Milling
Contract, granted by the Bacolod-Murcia Milling Co., Inc.

In 1953, Montelibano, et. al. initiated an action, contending that three Negros sugar centrals (La Carlota,
Binalbagan-Isabela and San Carlos), with a total annual sugar production exceeding 1/3 of the production of all
sugar central mill sin the province, had already granted increased participation (62.5%) to their planters, and that
in consonance with Paragraph 9 of the August 20 resolution, Bacolod-Murcia was obligated to grant similar
concessions to them.
Bacolod-Murcia, for its part, claimed that the stipulations contained in the resolution were made without
consideration, and therefore, void ab initio, being in effect a donation that was ultra vires and beyond the powers
of the corporate directors to adopt.

Is Bacolod-Murcia obligated to grant the concessions?

Yes, Bacolod-Murcia is obligated to grant the concessions to the planters.

The August 20 resolution was adopted by Bacolod-Murcia as a further amendment of the Amended Milling Contract,
and was approved three weeks prior to the September 10 signing of the Amended Milling Contract by Montelibano,
et. al. Therefore, when the contract was executed, the concessions granted by the resolution had already been
incorporated into its terms.

The resolution formed an integral part of the Amended Milling Contract, and not a separate bargain, evidenced by
the fact that it was simply attached to the printed contract without special negotiations or agreement between the
parties. It follows, therefore, that the August 20 resolution was supported by the same cause or consideration
underlying the main Amended Milling Contract, which was the extension of the operative period to 45 years.

The conduct of the parties indicates that the signing of the September 10 agreement gave rise to a binding
agreement, which existed on the basis of the printed terms of the Amended Milling Contract, as modified by the
CORPORATION LAW CASE DIGESTS | 31
FEU JD4401 | 2nd SEMESTER, S.Y. 2017-2018

August 20 resolution. There is no rational explanation for the company’s assent to the further concessions before
the contracts were signed, except as further inducement for the planters to agree to the extended contract period.
It would be sanctioning a fraud upon the planters to permit Bacolod-Murcia to retract the concessions at this point.

It must be noted that except in the case of statutory forms, or solemn agreements, it is the assent and
concurrence of the parties (the meeting of the minds) that constitutes a binding contract, and not the setting down
of the terms. That a copy of the August 20 resolution was not attached to the printed contract until April 17, 1937
is, therefore, not relevant, especially in light of the fact that the General Manager’s addendum emphasises that the
addition was made so that the terms of the agreement would be full and complete.
There is also no doubt that Bacolod-Murcia’s board of directors had the authority to modify the terms of the
Amended Milling Contract to make them more acceptable to the planters. The test to be applied is whether the act
in question is in direct and immediate furtherance of the corporation’s business, fairly incident to the express
powers and reasonably necessary to their exercise. If so, the corporation has the power to do it; otherwise, not.

The August 20 resolution was passed in good faith by the Board of Directors, and is valid and binding. Whether or
not it will cause losses or decrease the profits of the central, the court has no authority to review them. It is settled
that questions of policy or of management are left solely to the honest decision of officers and directors of a
corporation, and the court is without authority to substitute its judgment of the board of directors; the board is the
business manager of the corporation, and so long as it acts in good faith its orders are not reviewable by the
courts.
Since the sugar centrals of La Carlota, Hawaiian Philippines, San Carlos and Binalbagan, which produce over 1/3 of
the entire sugar production in Negros Occidental, Bacolod-Murcia is, under the terms of the August 20 resolution,
duty bound to grant similar increases to Monteliban, et. al.
CORPORATION LAW CASE DIGESTS | 32
FEU JD4401 | 2nd SEMESTER, S.Y. 2017-2018

(19) FILIPINAS PORT SERVICES, INC., represented by its stockholders v. VICTORIANO S. GO, et al.
G.R. No. 161886 16 March 2007
By: Valencia, Mary Clydeen L.

DOCTRINE: Section 23 of the Corporation Code explicitly provides that unless otherwise provided therein, the
corporate powers of all corporations formed under the Code shall be exercised, all business conducted and all
property of the corporation shall be controlled and held by a board of directors. Thus, with the exception only of
some powers expressly granted by law to stockholders (or members, in case of non-stock corporations), the board
of directors (or trustees, in case of non-stock corporations) has the sole authority to determine policies, enter into
contracts, and conduct the ordinary business of the corporation within the scope of its charter, i.e., its articles of
incorporation, by-laws and relevant provisions of law. / The requisites before a derivative suit can be filed by a
stockholder are: (a) the party bringing suit should be a shareholder as of the time of the act or transaction
complained of, the number of his shares not being material; (b) he has tried to exhaust intra-corporate remedies,
i.e., has made a demand on the board of directors for the appropriate relief but the latter has failed or refused to
heed his plea; and (c) the cause of action actually devolves on the corporation, the wrongdoing or harm having
been, or being caused to the corporation and not to the particular stockholder bringing the suit.

On 4 September 1992, petitioner Eliodoro C. Cruz, a stockholder of Filport - a domestic corporation engaged in
stevedoring services, wrote a letter to the corporation’s Board of Directors questioning its creation of various
positions with a monthly remuneration, and the election thereto of certain members. In his aforesaid letter, Cruz
requested the Board to take necessary action/actions to recover from those elected to the aforementioned
positions the salaries they have received. On 15 September 1992, the Board met and took up Cruz’s letter. The
action/s taken by the board took did not sit well with Cruz.

On 14 June 1993, Cruz, purportedly in representation of Filport and its stockholders, filed with the SEC a petition
which he describes as a derivative suit against the herein respondents who were then the incumbent members of
Filport’s Board, for alleged acts of mismanagement detrimental to the interest of the corporation and its
shareholders at large.

Did Filport’s Board act within its powers in creating the executive committee and the positions of AVPs
for Corporate Planning, Operations, Finance and Administration and those of the Special Assistants to
the President and the Board Chairman, each with corresponding remuneration?

YES. Section 23 of the Corporation Code explicitly provides that unless otherwise provided therein, the corporate
powers of all corporations formed under the Code shall be exercised, all business conducted and all property of the
corporation shall be controlled and held by a board of directors. Thus, with the exception only of some powers
expressly granted by law to stockholders (or members, in case of non-stock corporations), the board of directors
(or trustees, in case of non-stock corporations) has the sole authority to determine policies, enter into contracts,
and conduct the ordinary business of the corporation within the scope of its charter, i.e., its articles of
incorporation, by-laws and relevant provisions of law. Verily, the authority of the board of directors is restricted to
the management of the regular business affairs of the corporation, unless more extensive power is expressly
conferred. In the instant case, the Board’s creation of the positions of Assistant Vice Presidents for Corporate
Planning, Operations, Finance and Administration, and those of the Special Assistants to the President and the
Board Chairman, was in accordance with the regular business operations of Filport as it is authorized to do so by
the corporation’s by-laws, pursuant to the Corporation Code.

Further, based on Section 35 of the Corporation Code, the creation of an executive committee must be provided for
in the bylaws of the corporation. In the instant case, the bylaws of the corporation are actually silent as to the
creation by its board of directors of an executive committee. That notwithstanding, the creation of the executive
committee by the Board cannot be ruled as illegal or unlawful. One reason is the absence of a showing as to the
true nature and functions of said executive committee considering that the "executive committee," referred to in
Section 35 of the Corporation Code which is as powerful as the board of directors and in effect acting for the board
itself, should be distinguished from other committees which are within the competency of the board to create at
anytime and whose actions require ratification and confirmation by the board. Another reason is that the board of
directors has the power to create positions not provided for in Filport’s bylaws since the board is the corporation’s
governing body, clearly upholding the power of its board to exercise its prerogatives in managing the business
affairs of the corporation.
CORPORATION LAW CASE DIGESTS | 33
FEU JD4401 | 2nd SEMESTER, S.Y. 2017-2018

Is the main suit initiated by petitioner Cruz allegedly in representation of and in behalf of its
stockholders derivative in nature?

YES. Under the Corporation Code, where a corporation is an injured party, its power to sue is lodged with its board
of directors or trustees. But an individual stockholder may be permitted to institute a derivative suit in behalf of the
corporation in order to protect or vindicate corporate rights whenever the officials of the corporation refuse to sue,
or when a demand upon them to file the necessary action would be futile because they are the ones to be sued, or
because they hold control of the corporation. In such actions, the corporation is the real party-in-interest while the
suing stockholder, in behalf of the corporation, is only a nominal party. In the instant case, the action instituted is
principally for damages resulting from alleged mismanagement of the affairs of Filport by its directors/officers, it
being alleged that the acts of mismanagement are detrimental to the interests of Filport. Thus, the injury
complained of primarily pertains to the corporation so that the suit for relief should be by the corporation.
However, since the ones to be sued are the directors/officers of the corporation itself, a stockholder, like petitioner
Cruz, may validly institute a "derivative suit" to vindicate the alleged corporate injury, in which case Cruz is only a
nominal party while Filport is the real party-in-interest. For sure, in the prayer portion of petitioners’ petition before
the SEC, the reliefs prayed were asked to be made in favor of Filport.

Besides, the requisites before a derivative suit can be filed by a stockholder are present in this case, to wit:

a) the party bringing suit should be a shareholder as of the time of the act or transaction complained of,
the number of his shares not being material;
b) he has tried to exhaust intra-corporate remedies, i.e., has made a demand on the board of directors for
the appropriate relief but the latter has failed or refused to heed his plea; and
c) the cause of action actually devolves on the corporation, the wrongdoing or harm having been, or being
caused to the corporation and not to the particular stockholder bringing the suit.

Indisputably, petitioner Cruz (1) is a stockholder of Filport; (2) he sought without success to have its board of
directors remedy what he perceived as wrong when he wrote a letter requesting the board to do the necessary
action in his complaint; and (3) the alleged wrong was in truth a wrong against the stockholders of the corporation
generally, and not against Cruz or Minterbro, in particular. In the end, it is Filport, not Cruz which directly stands to
benefit from the suit. And while it is true that the complaining stockholder must show to the satisfaction of the
court that he has exhausted all the means within his reach to attain within the corporation itself the redress for his
grievances, or actions in conformity to his wishes, nonetheless, where the corporation is under the complete
control of the principal defendants, as here, there is no necessity of making a demand upon the directors. The
reason is obvious: a demand upon the board to institute an action and prosecute the same effectively would have
been useless and an exercise in futility. In fine, we rule and so hold that the petition filed with the SEC at the
instance of Cruz, which ultimately found its way to the RTC of Davao City as Civil Case No. 28,552-2001, is a
derivative suit of which Cruz has the necessary legal standing to institute.
CORPORATION LAW CASE DIGESTS | 34
FEU JD4401 | 2nd SEMESTER, S.Y. 2017-2018

(20) Rebern Development Corporation vs. People's Landless Association


G.R. No. 173622 March 11, 2013
By: Alba, Ma. Angela

Doctrine: The corporate nature of the bank confers the power to sell real properties in the higher authorities, not
a mere branch Officer-In-Charge.

Facts: Al-Amanah owned a 2000sqm. lot located in Magtu-od, Davao City. The informal settlers who were
occupying said lot formed an association called People’s Landless Association (PELA). In a letter, PELA offered to
purchase the lot for P 300,000.00, half of which shall be paid as down payment and the remaining half to be paid
within one year. In the lower portion of the said letter, the Officer-In-Charge(OIC) Dalig of Al-Amanah Davao
branch made an annotation stating that “the offer has been “acknowledged/received but processing to take effect
upon putting up of the partial amt. of P150,000.00 on or before April 15, 1993.”

PELA had deposited P150,000.00 as evidenced by four bank receipts. For the first three receipts, the bank labelled
the payments as “Partial deposit on sale of TCT No. 138914”, while it noted the 4th receipt as “Partial/Full payment
on deposit on sale of A/asset TCT No. 138914.” In the meantime, the PELA members remained in the property and
introduced further improvements.

After seven months, Al-Amanah, thru Davao Branch Manager Abraham D. Ututalum-Al Haj, wrote the PELA
President informing him of the Head Office’s disapproval of PELA’s offer to buy the said lot. PELA replied that it had
already reached an agreement with Al-Amanah regarding the sale of the subject lot based on their offered price.

Meanwhile, Al-Amanah acted on the written offer of Robern to buy the subject lot for P 400,000.00 on an “as-is
basis.” Robern was informed of Al-Amanah’s acceptance. It then paid the purchase price of the lot. The Deed of
Sale was executed and a TCT was issued in favor of Robern.

PELA filed a suit for Annulment and Cancellation of Void Deed of Sale against Al-Amanah, its Director Engr. Farouk
Carpizo, OIC Dalig, Robern, and Robern’s President and General Manager, petitioner Rodolfo Bernardo before the
RTC of Davao City. It insisted that as early as March 1993 it has a perfected contract of sale with Al-Amanah.
However, in an apparent act of bad faith and in cahoots with Robern, Al- Amanah proceeded with the sale of the lot
despite the prior sale to PELA.

Issue: Whether the annotation written by OIC Daligin on the letter containing PELA’s purchase offer is
tantamount to consent by Al-Amanah, thus giving birth to a perfected contract of sale.

Held: No. There was no proof of a perfected contract of sale between Al-Amanah and PELA. The parties did not
agree on the price and no consent was given, whether express or implied. Neither can the note written by the bank
that “[s]ubject offer has been acknowledged/received but processing to take effect upon putting up of the partial
amount of P150,000.00 on or before April 15, 1993” be construed as acceptance of PELA’s offer to buy.

Taken at face value, the annotation simply means that the bank merely acknowledged receipt of PELA’s letter-
offer. Furthermore, by ‘processing,’ Al-Amanah only meant that it will ‘act on the offer’, i.e., it still has to evaluate
whether PELA’s offer is acceptable. Until and unless Al-Amanah accepts, there is as yet no perfected contract of
sale. Notably here, the bank never signified its ‘approval’ or ‘acceptance’ of the offer.

PELA even acknowledges, that OIC Dalig made it clear that the acceptance of the offer, notwithstanding the
deposit, is subject to the approval of the Head Office. Recognizing the corporate nature of the bank and that the
power to sell its real properties is lodged in the higher authorities, she never falsely represented to the bidders that
she has authority to sell the bank’s property. And regardless of PELA’s insistence that she execute a written
agreement of the sale, she refused and told PELA to wait for the decision of the Head Office, making it clear that
she has no authority to execute any deed of sale.

In the case at bench, the transaction between Al-Amanah and PELA remained in the negotiation stage. The offer
never materialized into a perfected sale, for no oral or documentary evidence categorically proves that Al-Amanah
expressed amenability to the offered P300,000.00 purchase price.
CORPORATION LAW CASE DIGESTS | 35
FEU JD4401 | 2nd SEMESTER, S.Y. 2017-2018

(21) Heirs of Ignacio v. Home Bankers Savings and Trust Company


G.R. No. 177783, January 23, 2013
By; Arid, Hannah Mhae G.

Topic: Powers of Stockholders; Delegation of such powers

Doctrine: Section 23 of the Corporation Code expressly provides that the corporate powers of all corporations shall
be exercised by the board of directors. Just as a natural person may authorize another to do certain acts in his
behalf, so may the board of directors of a corporation validly delegate some of its functions to individual officers or
agents appointed by it. Thus, contracts or acts of a corporation must be made either by the board of directors or by
a corporate agent duly authorized by the board. Absent such valid delegation/authorization, the rule is that the
declarations of an individual director relating to the affairs of the corporation, but not in the course of, or connected
with, the performance of authorized duties of such director, are held not binding on the corporation.

Facts: The case sprang from a real estate mortgage of two parcels of land in August 1981. Fausto C. Ignacio
mortgaged the properties to Home Bankers Savings and Trust Company (Bank) as security for a loan extended by
the Bank. After Ignacio defaulted in the payment of the loan, the property was foreclosed and subsequently sold to
the Bank in a public auction.Ignacio offered to repurchase the property. Universal Properties Inc. (UPI), the bank’s
collecting agent sent Ignacio a letter on March 22, 1984 which contained the terms of the repurchase. However,
Ignacio annotated in the letter new terms and conditions. He claimed that these were verbal agreements between
himself and the Bank’s collection agent, UPI.No repurchase agreement was finalized between Ignacio and the Bank.
Thereafter the Bank sold the property to third parties. Ignacio then filed an action for specific performance against
the Bank for the reconveyance of the properties after payment of the balance of the purchase price. He argued that
there was implied acceptance of the counter-offer of the sale through the receipt of the terms by representatives of
UPI. The Bank denied that it gave its consent to the counter-offer of Ignacio. It countered that it did not approve
the unilateral amendments placed by Ignacio.

Issue: Are the negotiations between Ignacio and UPI binding on the Bank?

Held: NO. A contract of sale is perfected only when there is consent validly given. There is no consent when a
party merely negotiates a qualified acceptance or a counter-offer. An acceptance must reflect all aspects of the
offer to amount to a meeting of the minds between the parties.In this case, while it is apparent that Ignacio
proposed new terms and conditions to the repurchase agreement, there was no showing that the Bank approved
the modified offer. The negotiations between Ignacio and UPI, the collection agent, were merely preparatory to the
repurchase agreement and, therefore, was not binding on the Bank. Ignacio could not compel the Bank to accede
to the repurchase of the property.

Even assuming that the bank officer or employee whom petitioner claimed he had talked to regarding the March
22, 1984 letter had acceded to his own modified terms for the repurchase, their supposed verbal exchange did not
bind respondent bank in view of its corporate nature. There was no evidence that said Mr. Lazaro or Mr. Fajardo
was authorized by respondent bank's Board of Directors to accept petitioner's counter-proposal to repurchase the
foreclosed properties at the price and terms other than those communicated in the March 22, 1984 letter. As this
Court ruled in AF Realty & Development, Inc. v. Dieselman Freight Services, Co.

Section 23 of the Corporation Code expressly provides that the corporate powers of all corporations shall be
exercised by the board of directors. Just as a natural person may authorize another to do certain acts in his behalf,
so may the board of directors of a corporation validly delegate some of its functions to individual officers or agents
appointed by it. Thus, contracts or acts of a corporation must be made either by the board of directors or by a
corporate agent duly authorized by the board. Absent such valid delegation/authorization, the rule is that the
declarations of an individual director relating to the affairs of the corporation, but not in the course of, or connected
with, the performance of authorized duties of such director, are held not binding on the corporation. Thus, a
corporation can only execute its powers and transact its business through its Board of Directors and through its
officers and agents when authorized by a board resolution or its by-laws.?

In the absence of conformity or acceptance by properly authorized bank officers of petitioner's counter-proposal,
no perfected repurchase contract was born out of the talks or negotiations between petitioner and Mr. Lazaro and
Mr. Fajardo. Petitioner therefore had no legal right to compel respondent bank to accept the P600,000 being
tendered by him as payment for the supposed balance of repurchase price.
A contract of sale is consensual in nature and is perfected upon mere meeting of the minds. When there is merely
an offer by one party without acceptance of the other, there is no contract. When the contract of sale is not
perfected, it cannot, as an independent source of obligation, serve as a binding juridical relation between the
parties.

A corporation may only give valid acceptance of an offer of sale through its authorized officers or agents.
Specifically, a counter-offer to repurchase a property will not bind a corporation by mere acceptance of an agent in
the absence of evidence of authority from the corporation’s board of directors.
CORPORATION LAW CASE DIGESTS | 36
FEU JD4401 | 2nd SEMESTER, S.Y. 2017-2018
CORPORATION LAW CASE DIGESTS | 37
FEU JD4401 | 2nd SEMESTER, S.Y. 2017-2018

(22) Philippine Stock Exchange, Inc. vs. Litonjua


G.R. No. 204014. December 5, 2016
By; Bernardo, Gerard

Doctrine: In corporations, consent is manifested through a board resolution since powers are exercised through its
board of directors; As a general rule, in the absence of authority from the board of directors, no person, not even
its of icers, can validly bind a corporation.

Facts: On 20 April 1999, the Litonjua Group wrote a letter-agreement to Trendline Securities, Inc. (Trendline)
through its President Priscilla D. Zapanta (Zapanta), confirming a previous agreement for the acquisition of the
85% majority equity of Trendline's membership seat in PSE. In a letter-confirmation dated 21 April 1999, the
Litonjua Group undertook to pay the amount of P18,547,643.81 directly to PSE within three working days upon
confirmation that it will be for the full settlement of all claims and outstanding obligations including interest of
Trendline to lift its membership suspension and the resumption to normal trading operation. Further in the letter,
Trendline was obligated to secure the approval and written confirmation of PSE for a new corporation to be
incorporated that will own a seat.

On 29 April 1999, the PSE, through Atty. Ruben L. Almadro (Atty. Almadro), Vice-President for Compliance and
Surveillance Department, sent a letter to Trendline advising the latter that PSE has resolved to accept the amount
of P19,000,000.00 as full and final settlement of its outstanding obligation. In compliance, the Litonjua Group in a
letter dated 12 May 1999, delivered to PSE through Atty. Almadro three check payments, all dated 13 May 1999
and payable to PSE, totaling to an amount of Pl9,000,000.00.

Despite several exchange of letters of conformity and delivery of checks representing payment of full settlement of
Trendline's obligations, PSE failed to lift the suspension imposed on Trendline's seat. On 30 July 2006, the Litonjua
Group, through a letter, requested PSE to reimburse the P19,000,000.00 it had paid with interest, upon knowledge
that the specific performance by PSE of transferring the membership seat under the agreement will no longer be
possible. PSE, however, refused to refund the claimed amount as without any legal basis. As a result, the Litonjua
Group on 10 October 2006 filed a Complaint for Collection of Sum of Money with Damages against PSE before the
RTC of Pasig City.

Declining reimbursement, PSE in its Answer Ad Cautelam raised primarily that it received the amount not from the
Litonjua Group but from Trendline as a settlement of its obligation. It insisted that the cause of action of the
Litonjua Group is against Trendline and not the exchange, the latter being a non-party to the letter agreement.

Issue: Whether or not PSE is considered a party to the letter-agreement.

Held: NO. SE asserts that it is not a party in the letter-agreement due to the absence of any board resolution
authorizing the corporation to be bound by the terms of the contract between Trendline and the Litonjua Group. In
essence, it avers that no consent was given to be bound by the terms of the letter-agreement. We agree.

According to Article 1305 of the Civil Code, "a contract is a meeting of minds between two persons whereby one
binds himself, with respect to the other, to give something or render some service." For a contract to be binding:
there must be consent of the contracting parties; the subject matter of the contract must be certain; and the cause
of the obligation must be established. Admittedly in this case, no board resolution was issued to authorize PSE to
become a party to the letter-agreement. From the foregoing, PSE is not considered as a party to the letter-
agreement.
CORPORATION LAW CASE DIGESTS | 38
FEU JD4401 | 2nd SEMESTER, S.Y. 2017-2018

(23) Ching vs. Subic Bay and Country Club, Inc.


G.R. No. 174353, September 10, 2014
By: Donna Bigornia

Doctrine: The requisite of derivative suit are: (1) He was a stockholder or member at the time the acts or
transactions subject of the action occurred and at the time the action was filed; (2) He exerted all reasonable
efforts, and alleges the same with particularity in the complaint, to exhaust all remedies available under the articles
of incorporation, by-laws, laws or rules governing the corporation or partnership to obtain the relief he desires; (3)
No appraisal rights are available for the act or acts complained of; and (4) The suit is not a nuisance or harassment
suit.

Facts: Petitioners Nestor Ching and Andrew Wellington own stocks of the Subic Bay Golf and Country Club,
Inc.(SBGCCI). On June 27, 1996, Securities and Exchange Commission (SEC) approved amendments to SBGCCI
Articles of Incorporation which the petitioners allege make their shares non-proprietary. Petitioners allege that this
change was made without the appropriate disclosure of SBGCCI to its shareholders. Furthermore, petitioners allege
several instances of fraud committed by SBGCCI’s board of directors in its February 26, 2003 complaint.
Respondent’s answered the complaint by refuting allegations made by petitioners. As a way of defense,
respondents underscored petitioners’ failure to: show that it was authorized by SBGSI to file complaint on said
company’s behalf comply with the requisites for filing a derivative suit and an action for receivership justify their
prayer for injunctive relief since the complaint may be considered a nuisance or harassment suit. Thus,
respondents prayed for dismissal of the complaint. On July 28, 2003, the RTC held that the action is a derivative
suit and issued an order dismissing the complaint. Petitioners elevated the case to the Court of Appeals but the
appellate court affirmed the RTC decision.

Issue: WON the petitioners are proper party in interest WON the complaint is a derivative suit

Ruling: No.

Petitioners did not offer proof that they were authorized to represent SBGSI. The Court ruling in Cua, Jr. v. Tan
elaborated the three (3) types of suit: individual, class or representative, and derivative suit. The reliefs prayed for
by petitioners, to wit: (i) enjoining defendants from acting as officers and Board of Directors of the corporation, (ii)
the appointment of receiver, (iii) damages, clearly show that the complaint was filed to curb the alleged
mismanagement of SBGCCI. The cause of action pleaded by petitioners does not accrue to a single shareholder or
a class of shareholders but to the corporation itself. While there were allegations of fraud in the subscription,
petitioners do not wish to have their subscription rescinded. Instead, the petitioners asked that the respondents be
removed from the management of the corporation. Petitioner’s only possible cause of action as the minority
shareholder against the actions of the board is to file the common law right to file a derivative suit. As minority
shareholders, petitioners do not have any statutory right to override the business judgements of SBGCCI’s officers
and board of directors on the ground of the latter’s alleged lack of qualification to manage a golf course. The legal
standing of the petitioners is not a statutory right, there being no provision in the Corporation Code or related
statutes, but is instead a product of jurisprudence based on equity. However, a derivative suit cannot prosper
without first complying with the legal requisites for its institution: Interim Rules Governing Intra-Corporate
Controversies. Petitioners failed to comply with second requisite: “…exerted all reasonable efforts, and alleges the
same with particularity in the complaint, to exhaust all remedies available under the articles of incorporation, by-
laws, laws or rules governing the corporation or partnership to obtain the relief he desires…”

Thus, a complaint which contained no allegation whatsoever of any effort to avail of intra-corporate remedies
allows the court to dismiss it, even motu proprio. Indeed, even if petitioners thought it was futile to exhaust intra-
corporate remedies, they should have stated the same in the Complaint and specified the reasons for such opinion.
The requirement of this allegation in the Complaint isnot a useless formality which may be disregarded at will.
CORPORATION LAW CASE DIGESTS | 39
FEU JD4401 | 2nd SEMESTER, S.Y. 2017-2018

(24) Lopez Realty, Inc., v. Spouses Tanjangco


G.R. No. 154291, November 12, 2014

Doctrine: The general rule is that a corporation, through its board of directors, should act in the manner and
within the formalities, if any, prescribed by its charter or by the general law. Thus, directors must act as a body in
a meeting called pursuant to the law or the corporation’s bylaws, otherwise, any action taken therein may be
questioned by any objecting director or shareholder.

However, the actions taken in such a meeting by the directors or trustees may be ratified expressly or impliedly.

Facts: Lopez Realty and Jose Tanjangco were the registered owners of 3 parcels of land and a building erected
thereon. Jose’s ½ share was transferred to his son and his daughter-in-law, the here petitioner. 5 out of 7
stockholders of Lopez Realty were members of the Board of Directors. Arturo and Teresita are stockholders but not
members of the Board of Directors. In a stockholder’s meeting, the sale of Lopez Realty’s share in the building was
discussed. Spouses Tangjangco made their offers.

Asuncion, a stockholder and member of the BoD, was given priority to buy the share. However, she failed to
exercise her right. A stockholder meeting was again conducted. Stockholder Arturo was given the power to
negotiate with the Tangjangcos. Arturo then sold the share to Jose. Upon the arrival of Asuncion, she moved for
the repeal of the Board Resolution authorizing Arturo to negotiate with Tangjangcos.

Tangjangcos requested Lopez Realty to execute a deed of sale. Arturo transferred the ½ share of Lopez Realty to
Jose. This share was transferred to the Tangjangcos. Subsequently, this was granted and TCTs were issued in
their name.

Consequently, on November 4, 1981, Lopez Realty and Asuncion (herein petitioners) filed with the then Court of
First Instance of Manila, a Complaint for annulment of sale, cancellation of title, reconveyance and damages.
Asuncion alleged that the resolution was invalid because the meeting was invalid for failing to notify her.

On July 30, 1982, the stockholders of LRI had a meeting where they voted on whether to ratify and confirm the
sale of the subject properties to the spouses Tanjangco.

The RTC ruled in favor of Asuncion. CA reversed and recognized Arturo’s authority to sell the share.

Issues: Was the selling of the share in the building made by Arturo valid?

Held: Yes.
The general rule is that a corporation, through its board of directors, should act in the manner and within the
formalities, if any, prescribed by its charter or by the general law. Thus, directors must act as a body in a meeting
called pursuant to the law or the corporation’s bylaws, otherwise, any action taken therein may be questioned by
any objecting director or shareholder.

However, the actions taken in such a meeting by the directors or trustees may be ratified expressly or impliedly.
"Ratification means that the principal voluntarily adopts, confirms and gives sanction to some unauthorized act of
its agent on its behalf. It is this voluntary choice, knowingly made, which amounts to a ratification of what was
thereto unauthorized and becomes the authorized act of the party so making the ratification. The substance of the
doctrine is confirmation after conduct, amounting to a substitute for a prior authority. Ratification can be made
either expressly or impliedly. Implied ratification may take various forms — like silence or acquiescence, acts
showing approval or adoption of the act, or acceptance and retention of benefits flowing therefrom."

There was a ratification of the resolution authorizing Arturo. This was owing to the subsequent actions taken
therein by the stockholders, including Asuncion herself, as cited by the CA in its decision. On the other hand, the
sale of the property to the spouses Tanjangco was ratified, not because of implied ratification as was the case in
Fontecha but through the passage of the July 30, 1982 Board Resolution.

Upon examination of the July 30, 1982 minutes of the meeting, it can be deduced that the meeting is a joint
stockholders and directors’ meeting. The Court takes into account that majority of the board of directors except for
Asuncion, had already approved of the sale to the spouses Tanjangco prior to this meeting. As a consequence, the
power to ratify the previous resolutions and actions of the board of directors in this case lies inthe stockholders, not
in the board of directors. It would be absurd to require the board of directors to ratify their own acts—acts which
the same directors already approved of beforehand. Hence, Juanito, as the administrator of Teresita’s estate even
though not a director, is entitled to vote on behalf of Teresita’s estate as the administrator thereof.

Despite the lack of all the signatures in the minutes, the ratification was still valid. There is no provision in the
Corporation Code of the Philippines that requires that the minutes of the meeting should be signed by all the
members of the board.
CORPORATION LAW CASE DIGESTS | 40
FEU JD4401 | 2nd SEMESTER, S.Y. 2017-2018

(25) Ricafort vs Dicdican


G.R. No. 202647-50. March 9, 2016
By: Kathrina De Castro

Topic: Annual Stockholder’s Meeting


DOCTRINE: The shorter notice of three (3) days instead of two (2) weeks for stockholders’ regular or special
meeting is clearly allowed under Section 50 of the Corporation Code.
NADECOR is a domestic corporation and a holder of Mining Production Sharing Agreement with the DENER.
Pursuant to Article I Sec 1 of the amended by-laws, its regular annual stockholders’ meeting (ASM) was held to
elect BOD. More than two months after the ASM, petitioners, claiming to be stockholders of record, filed compliant
before the RTC to declare null and void “the ASM of NADECOR, including all proceedings taken thereat, all the
consequences thereof, and all acts carried out pursuant thereto.” They alleged that they have no prior notice of,
and wasn’t able to attend the ASM. Defendants sought dismissal of the case on the grounds that the complaint
involved an election contest, since in effect it sought to nullify the election of the BOD.

A. WON the petition is an election contest The shorter notice of three (3) days instead of two (2)
weeks for stockholders’ regular or special meeting is clearly allowed under Section 50 of the
Corporation Code.

Yes, to nullify the ASM would have had no practical effect except to void the election of the Board of Directors. And
no doubt, this was the trial court’s understanding of the petitioners’ intent when it voided the August 15, 2011 ASM
and all matters taken up thereat. Thus, by declaring as void all “acts, decisions, deeds, incidents, matters taken up
arising from and subsequent to the 2011 [ASM],” things which could only be performed by the newly-elected
Board, and then by directing the issuance of a three-day notice for the holding of a new ASM corresponding to FY
2011-2012, the trial court clearly understood that a new election should be held for Board of Directors of NADECOR
for FY 2011-2012, notwithstanding its express ruling that SEC Case No. 11-164 did not involve an election contest
and therefore. But more importantly, the defendants did not fail to point out to the trial court, as the appellate
court has made copiously clear in its decision, that contrary to the petitioners’ feigned lament that they were
unlawfully deprived of their right as stockholders to participate in the ASM due to late notice, they were in fact
represented by JG Ricafort under an irrevocable proxy which they executed on April 26, 2010.

B. WON petitioners were duly represented in ASM through proxy

Yes, As found by the CA, the petitioners did participate in the stockholders’ meeting through their authorized
representative and proxy, JG Ricafort. In his Affidavit dated November 21, 2011, Gatmaitan, NADECOR Corporate
Secretary, categorically declared under oath that JG Ricafort held a valid irrevocable proxy from the petitioners to
attend and vote their shares at all meetings of the stockholders, and that JG Ricafort signed the attendance sheet
for and in behalf of the plaintiffs as shown by his signatures in the rows in the said attendance sheet for the names
of the plaintiffs who had appointed him as his proxy. During the meeting no one questioned the Irrevocable Proxy.
Thus, JG Ricafort being the real and beneficial owner of the petitioners’ shares, lack of notice to them is
inconsequential because he attended and represented them at the August 15, 2011 ASM. It defies reason, too, that
he could not have informed his wife and children, who live in the same house with him, of the scheduled ASM.
CORPORATION LAW CASE DIGESTS | 41
FEU JD4401 | 2nd SEMESTER, S.Y. 2017-2018

(26) Estate of Juvencio Ortanez v. Lee


G.R. No. 184251, March 9, 2016
By: Gaite, Rhio Angeline

Doctrine: This Court recognizes the significant weight of the Certification issued by the Insurance Commission.
The document certified that Department Order No. 62-87 (5 June 1987), as issued by the Insurance Commission,
required domestic insurance companies to increase their minimum paid-up capital to P10,000,000.00 by the end of
31 December 1987.

Facts: On 6 July 1956, Dr. Ortañez organized and founded the Philippine International Life Insurance Company,
Inc. (Philinterlife). At the time of its incorporation, Dr. Ortañez owned 90% of the subscribed capital stock of
Philinterlife.
Upon his death on 21 July 1980, Dr. Ortañez left behind an estate consisting of, among others, 2,029 shares of
stock in Philinterlife, then representing at least 50.725% of the outstanding capital stock of Philinterlife which was
at 4,000 shares valued at P4,000,000.00. Decedent’s wife, Juliana Ortañez and Special Administrator Jose Ortañez,
sold their shares with right to repurchase in favor of Filipino Loan Assistance Group (FLAG), represented by its
president, Jose C. Lee. Both of them failed to repurchase x x x, thus ownership thereof was consolidated by FLAG
in its name. Also, Juliana Ortañez and her three sons invalidly entered into a Memorandum of Agreement
extrajudicially partitioning the intestate estate among themselves, despite their knowledge that there were other
heirs or claimants to the Estate and before the final settlement of the Estate by the intestate court.

On 30 March 2006, petitioners filed a Complaint for Election Contest before the RTC of Quezon City. The complaint
challenged the lawfulness and validity of the meeting and election conducted by the group of Jose C. Lee
(respondents) on 15 March 2006. During the assailed meeting, Jose C. Lee (Lee), Angel Ong, Benjamin C. Lee,
Carmelita Tan, Ma. Paz C. Lee, John Oliver Pascual, Edwin C. Lee, Conrado C. Cruz, Jr., Brenda Ortañez, Julie Ann
Parado and Gary Jason Santos were elected as members of the Board of Directors of Philinterlife.

Petitioners claimed that before the contested election, they formally informed the respondents that without the
participation of the Estate, no quorum would be constituted in the scheduled annual stockholders’ meeting.
Petitioners averred that in spite of their formal announcement and notice that they were not participating in the
session, the respondents continued, in bad faith, with the illegal meeting. Further, respondents allegedly elected
themselves as directors of Philinterlife and proceeded to elect their own set of officers. Petitioners, who insisted
that they represented at least 51% of the outstanding capital stock of 5,000 shares of Philinterlife, conducted on
the same day and in the same venue but in a different room, their own annual stockholders’ meeting and
proceeded to elect their own set of directors, to wit: Rafael Ortañez, Divina Ortañez-Enderes, Ligaya Novicio, Cesar
Ortañez and Leopoldo Tomas. Petitioners complained that despite being the true and lawful directors, they were
prevented by respondents to enter into the office premises of Philinterlife’s corporate records and assets.

In their backgrounder, petitioners narrated that on 15 April 1989 and 30 October 1991, the 2,029 shares of stock
of the Estate were sold to the group of Lee, through an entity called Filipino Loan Assistance Group (FLAG). By
reason of said sale, respondents took control of the management of the corporation. In the course of their
management, and by voting on the shares that they had illegally acquired, respondents increased the authorized
capital stock of Philinterlife to 5,000 shares. The aforementioned sale of the shares of stock of the Estate was
challenged by some of the heirs (some of the petitioners) before the estate court, which in due course, issued an
order declaring the sale null and void ab initio. The case eventually reached this Court and was docketed as G.R.
No. 146006.

In the Court’s decision in G.R. No. 146006, it affirmed the lower court’s ruling that indeed the sale was null and
void. Furthermore, the Court ruled that all increases in the authorized capital stock of Philinterlife made and
effected by the respondents using the shares that they illegally acquired were null and void as well. Petitioners
submit that as a necessary and logical consequence, majority ownership over Philinterlife was restored to the
Estate, which was the controlling stockholder prior to the unlawful sale of the shares.

Petitioners argued that the valid and lawful capital stock of Philinterlife remained at 5,000 shares of stock. From
this 5,000 shares, petitioner Estate owns 2,029 shares, plus 510 shares which also legally belongs to it by reason
of its preemptive right, or a total of 2,539 shares. These figures indicate that they still represent majority of the
outstanding capital stock of Philinterlife.

Issues: (1) Whether petitioner Estate is the owner of majority of the capital stock of Philinterlife
(2) Whether the election of respondents as directors of Philinterlife was in accordance with the provisions of the
Corporation Code, despite the categorical pronouncement of this Court in G.R. No. 146006 that it is the Estate, and
not the respondents, which own the controlling interest in Philinterlife
CORPORATION LAW CASE DIGESTS | 42
FEU JD4401 | 2nd SEMESTER, S.Y. 2017-2018

Held: (1) No. Upon a closer analysis of our ruling in G.R. No. 146006, however, the Court note that only the 4
March 1982 memorandum of agreement was declared void and as a consequence thereto, the subsequent sale to
FLAG was likewise declared void. With regard to the increases in Philinterlife’s capital stock, the Court only declared
void those increases approved on the vote of petitioners’ nonexistent shareholdings. In other words, only those
increases subsequent to the illegal sales of shares of stock are considered void. The validity of the increases of
stock before 1989 (from 1980 to 1988) has never been questioned before any court. Parenthetically, any question
on the increase of stocks made before the illegal sales should not be raised in the instant election contest case but
should be the subject of a separate proceeding.

The Court give more weight to the Capital Structure of Philinterlife as of 15 December 1980, which shows that the
Estate owned 2,029 shares of the 5,000 total outstanding shares or 40.58%. It is evident, therefore, that as of 15
December 1980, the Estate no longer owned 50.725% of the outstanding capital stock of Philinterlife. In view of
the increase of the capital structure of Philinterlife from 4,000 shares to 5,000 shares in 15 December 1980, the
percentage of shareholdings owned by the Estate was naturally reduced from 50.73% (2,029 shares out of 4,000
shares) to 40.58% (2,029 shares out of 5,000 shares). In other words, the Estate’s 2,029 shares became a
minority shareholder of Philinterlife from 15 December 1980 up to 24 March 1983. The Capital Structure proffered
by the respondents negated the claim of petitioners that they have always been the true and lawful owners of at
least 51% of Philinterlife.

It should be noted that the last valid uncontested outstanding capital stock before the illegal sales was 10,000
shares. Prior to the sales made to FLAG on 15 April 1989 and 30 October 1991, the outstanding capital stock as
reflected in the General Information Sheet dated 16 April 1988,13 is 10,000 shares at P10,000,000.00 and not
5,000 shares as advanced by the petitioners. Therefore, the total number of outstanding shares during the 15
March 2006 annual stockholders’ meeting was definitely not 5,000 shares as petitioners posit. Even before the
illegal sale, the Estate only owned 2,029 shares, not even close to majority of the total outstanding capital stock of
10,000 shares.

Moreover, this Court recognizes the significant weight of the Certification issued by the Insurance Commission. The
document certified that Department Order No. 62-87 (5 June 1987), as issued by the Insurance Commission,
required domestic insurance companies to increase their minimum paid-up capital to P10,000,000.00 by the end of
31 December 1987.

(2) Yes. The Court noted respondents’ submission that in March 1983, Jose S. Ortañez sold certain shares of stocks
which he personally and exclusively owned to Lee and eighteen (18) other stockholders including Divina Ortañez-
Enderes and her family. These shares of stock are separate and distinct from the 2,029 shares of stock belonging
to the Estate. The respondents direct the Court’s attention to the General Information Sheets of Philinterlife from
31 March 1983 to 16 April 1988, where it is shown that even before the alleged illegal sales on 15 April 1980 and
30 October 1996, Lee and the other respondents were stockholders and directors of Philinterlife. Respondents also
claim that as of 27 July 1987, the authorized capital stock of Philinterlife was increased to P10,000,000.00 in
compliance with Ministry Order 2-84; that as of 31 January 1989, the authorized capital stock was still at
P10,000,000.00 and the Estate’s 2,025 shares have minority interest of 20.29% only; that as of 20 February 2003,
90% of the company’s controlling interest approved the increase of capital stock to P50,000,000.00 as mandated
by law. Moreover, respondents allege that the 15 March 2006 annual stockholders’ meeting presided over by Lee
was attended by stockholders representing 98.76% of the 50,000 authorized and fully subscribed capital stock.

The Court agree with the lower courts that the petitioners failed to present credible and convincing evidence that
Philinterlife’s outstanding capital stock during the 15 March 2006 annual stockholders’ meeting was 5,000 and that
they own more than 2,550 shares or 51% thereof. The unrebutted presumption is that respondents, as defendants
below, were duly elected as directors-officers of Philinterlife.
CORPORATION LAW CASE DIGESTS | 43
FEU JD4401 | 2nd SEMESTER, S.Y. 2017-2018

(27) GRACE BORGONA INSIGNE vs. ABRA VALLEY COLLEGES, INC.


G.R. No. 204089 July 29, 2015
By: Grande, Jonicocel

Topic: inspection of stock and transfer book; determination of a shareholder

Doctrine: A stock certificate is prima facie evidence that the holder is a shareholder of the corporation, but the
possession of the certificate is not the sole determining factor of one’s stock ownership. A certificate of stock is
merely the paper representative or tangible evidence of the stock itself and of the various interests therein.

Facts: In his lifetime, Pedro was the founder, president and majority stockholder of respondent (Abra Valley), a
stock corporation. After Pedro’s death, Francis (son with his frist wife) succeeded him as the president of Abra
Valley.
The petitioners (children to his second marriage), filed a complaint against Abra Valley (docketed as praying to
direct Abra Valley to allow them to inspect its corporate books and records, and the minutes of meetings, and to
provide them with its financial statements.

Issue: Whether or not the petitioners could exercise their right to inspect Abra Valley’s corporate books, records
and minutes of meetings, and be furnished with financial statements.

Held: YES.

Petitioners were stockholders of Abra Valley. Their non-production of their stock certificate is not the sole
determining factor of their stock ownership.

A stock certificate is prima facie evidence that the holder is a shareholder of the corporation, but the possession of
the certificate is not the sole determining factor of one’s stock ownership. A certificate of stock is merely the paper
representative or tangible evidence of the stock itself and of the various interests therein. The certificate is not
stock in the corporation but is merely evidence of the holder's interest and status in the corporation, his ownership
of the share represented thereby, but is not in law the equivalent of such ownership. It expresses the contract
between the corporation and the stockholder, but it is not essential to the existence of a share in stock or the
creation of the relation of shareholder to the corporation.

To establish their stock ownership, the petitioners actually turned over to the trial court through their Compliance
and Manifestation the various documents showing their ownership of Abra Valley’s shares, specifically: the official
receipts of their payments for their subscriptions of the shares of Abra Valley; and the copies duly certified by the
Securities and Exchange Commission (SEC) stating that Abra Valley had issued shares in favor of the petitioners,
such as the issuance of part of authorized and unissued capital stock; the letter dated June 17, 1987; the
secretary’s certificate dated June 17, 1987; and the general information sheet.

Also, the petitioners adduced competent proof showing that the respondents had allowed the petitioners to become
members of the Board of Directors.

The petitioners attended the annual meeting as stockholders of Abra Valley, and participated in the election of the
Board of Directors at which some of them were chosen as members.

Section 23 of the Corporation Code requires every director to be the holder of at least one share of capital stock of
the corporation of which he is a director, the respondents would not have then allowed any of the petitioners to be
elected to sit in the Board of Directors as members unless they believed that the petitioners so elected were not
disqualified for lack of stock ownership. Neither did the respondents thereafter assail their acts as Board Directors.
Conformably with the doctrine of estoppel, the respondents could no longer deny the petitioners’ status as
stockholders of Abra Valley.

Hence, the petitioners are stockholders of Abra Valley.


CORPORATION LAW CASE DIGESTS | 44
FEU JD4401 | 2nd SEMESTER, S.Y. 2017-2018

Petitioners were entitled to demand the production of the STB of Abra Valley

A person becomes a stockholder of a corporation by acquiring a share through either purchase or subscription.
Here, the petitioners acquired their shares in Abra Valley: (1) by subscribing to 36 shares each from Abra Valley’s
authorized and unissued capital stock; and (2) by purchasing the shareholdings of existing stockholders, as borne
out by the latter’s indorsement on the stock certificates.

Section 63 of the Corporation Code, which pertinently provides that, Shares of stock so issued are personal
property and may be transferred by delivery of the certificate or certificates indorsed by the owner or his attorney-
in-fact or other person legally authorized to make the transfer. No transfer, however, shall be valid, except as
between the parties, until the transfer is recorded in the books of the corporation showing the names of the parties
to the transaction, the date of the transfer, the number of the certificate or certificates and the number of shares
transferred.

Considering that Abra Valley’s STB was not in the possession of the petitioners, or at their disposal, they could not
be reasonably expected or justly compelled to prove that their stock subscriptions and purchases were recorded
therein. This, more than any other, was precisely why they filed their Motion for Production/Inspection of
Documents to compel the respondents to produce the STB.. Thus, the petitioners are entitled to demand the
production of the STB.
CORPORATION LAW CASE DIGESTS | 45
FEU JD4401 | 2nd SEMESTER, S.Y. 2017-2018

(28) Multinational Village Homeowners Association v. Gacutan


G.R. No. 188307, Aug. 2, 2017
By: Jovero, John Tristram V.

Topic: Hold-over

Doctrine: Term is distinguished from tenure in that an officer's "tenure" represents the term during which the
incumbent actually holds office. The tenure may be shorter (or, in case of holdover, longer) than the term for
reasons within or beyond the power of the incumbent.

Facts: Two days before the scheduled election, or on 21 January 2005, petitioner Jimmy del Mundo sought
injunctive relief from the HLURB-NCRFO because of the alleged lack of transparency in the issuance of proxy forms
and the alleged burning of election records to supposedly prevent verification of the previous elections. On the
same day, the HLURB-NCRFO granted the application and issued a restraining order against, not only the further
issuance of proxy forms, but also proxy voting itself in the forthcoming election.

Majority of the qualified members of petitioner MVHAI allegedly ignored the resolution of the Comelec and
constituted a new Comelec to supervise the election. The village poll proceeded as scheduled and based on the
results, petitioners garnered the highest number of votes. Insisting that petitioners were not authorized under the
association by-laws to call an election, respondents refused to relinquish their posts and declared themselves as
hold-over directors until elections were properly held.

Petitioners then filed an election contest docketed as HLURB Case No. NCRHOA-020105-557 before the HLURB-
NCRFO praying that their election be affirmed and that respondents be permanently enjoined from acting as hold-
over directors of petitioner MVHAI. HLURB-NCRFO dismissed the complaint and nullified the 2005 election for
having been called without authority.

Aggrieved, petitioners appealed to the HLURB-Board of Commissioners (BoC). Reversing the decision of the
HLURB-NCRFO, the HLURB-BoC declared the 2005 election valid on the ground that "the will of the majority of the
members of (petitioner) MVHAI x x x must be respected."

Respondents then filed with the OP a Petition for Review, docketed as O.P. Case No. 05-K-377. In its Decision
dated 16 May 2006, the OP granted the appeal, set aside the decision of the HLURB-BoC and reinstated the earlier
decision of the HLURB-NCRFO.

On 2 April 2007, the OP issued a Clarificatory Resolution in response to the request of the HLURB-BoC. The
dispositive portion reads:
WHEREFORE, in order to give full meaning and equitably enforce the final and executory Decision of this Office
dated May 16, 2006, it is hereby ordered that:

xxx

(3) Pending the conduct of the election and the proclamation of winners, all contracts to be entered into by the
MVHOA shall be held in abeyance, but the 2004 Board of Directors shall manage MVHOA's daily operations; and

xxx

Issue: Is the ruling of the OP extending the 2004 BOD’s management correct?

Held: Yes.

While HLURB Resolution Nos. 770-04 and R-771-04 do not expressly set the maximum period that a director or
officer may serve in a hold-over capacity, the BOD of a homeowners' association cannot unjustifiably refuse to call
and hold an election when mandated by the association by-laws. Section 4 of HLURB Resolution No. R-771-04
expressly authorizes the HLURB-NCRFO to call the election when the circumstances so warrant, as in this case. To
sustain respondents' hold-over positions since 2005 is to make them stay in the BOD for approximately 12 years,
notwithstanding the expiration of their one-year term.

In Valle Verde Country Club, Inc. v. Africa, the Court distinguished term from tenure, thus:

Under the above-quoted Section 29 of the Corporation Code, a vacancy occurring in the board of directors caused
by the expiration of a member's term shall be filled by the corporation's stockholders. Correlating Section 29 with
Section 23 of the same law, VVCC alleges that a member's term shall be for one year and until his successor is
elected and qualified; otherwise stated, a member's term expires only when his successor to the Board is elected
CORPORATION LAW CASE DIGESTS | 46
FEU JD4401 | 2nd SEMESTER, S.Y. 2017-2018

and qualified. Thus, "until such time as [a successor is] elected or qualified in an annual election where a quorum is
present", VVCC contends that "the term of [a member] of the board of directors has yet not expired.”

xxxx

Term is distinguished from tenure in that an officer's "tenure" represents the term during which the incumbent
actually holds office. The tenure may be shorter (or, in case of holdover, longer) than the term for reasons within
or beyond the power of the incumbent.

Based on the above discussion, when Section 23 of the Corporation Code declares that "the board of directors x x x
shall hold office for one (1) year until their successors are elected and qualified", we construe the provision to
mean that the term of the members of the board of directors shall be only for one year; their term expires one
year after election to the office. The holdover period — that time from the lapse of one year from a member's
election to the Board and until his successor's election and qualification — is not part of the director's original term
of office, nor is it a new term; the holdover period, however, constitutes part of his tenure. Corollary, when an
incumbent member of the board of directors continues to serve in a holdover capacity, it implies that the office has
a fixed term, which has expired, and the incumbent is holding the succeeding term.

After the lapse of one year from his election as member of the VVCC Board in 1996, Makalintal's term of office is
deemed to have already expired. That he continued to serve in the VVCC Board in a holdover capacity cannot be
considered as extending his term. To be precise, Makalintal's term of office began in 1996 and expired in 1997,
but, by virtue of the holdover doctrine in Section 23 of the Corporation Code, he continued to hold office until his
resignation on November 10, 1998. This holdover period, however, is not to be considered as part of his term,
which, as declared, had already expired.

Notably, Republic Act No. 9904, or the Magna Carta for Homeowners and Homeowners' Associations, was approved
and became effective in 2010. Section 60 of its Implementing Rules and Regulations expressly sets forth that "(i)n
no case shall the hold-over term of the officers/directors/trustees exceed two (2) years."
CORPORATION LAW CASE DIGESTS | 47
FEU JD4401 | 2nd SEMESTER, S.Y. 2017-2018

(29) Valle Verde Country Club, Inc. v. Africa


G.R. No. 151969, September 4, 2009
by: Jesus Ros Lapuz, Jr.

Doctrine: The business and affairs of a corporation must be governed by a board of directors whose members
have stood for election, and who have actually been elected by the stockholders, on an annual basis.

Facts: On February 27, 1996, Jaime C. Dinglasan (Dinglasan), Eduardo Makalintal (Makalintal), etc. were elected
as BOD during the Annual Stockholders’ Meeting of petitioner Valle Verde Country Club, Inc. (VVCC). From 1997-
2001, the requisite quorum could not be obtained so they continued in a hold-over capacity. By September 1,
1998: Dinglasan resigned, BOD still constituting a quorom elected Eric Roxas (Roxas). On November 10, 1998:
Makalintal resigned; then, on March 6, 2001: Jose Ramirez (Ramirez) was elected by the remaining BOD.
Respondent Africa (Africa), a member of VVCC, questioned the election of Roxas and Ramirez as members of the
VVCC Board with the Securities and Exchange Commission (SEC) and the Regional Trial Court (RTC) as contrary to:

Sec. 23. The board of directors or trustees. - Unless otherwise provided in this Code, the corporate powers of
all corporations formed under this Code shall be exercised, all business conducted and all property of such
corporations controlled and held by the board of directors or trustees to be elected from among the holders of
stocks, or where there is no stock, from among the members of the corporation, who shall hold office for 1 year
until their successors are elected and qualified.

Sec. 29. Vacancies in the office of director or trustee. - Any vacancy occurring in the board of directors or
trustees other than by removal by the stockholders or members or by expiration of term, may be filled by
the vote of at least a majority of the remaining directors or trustees, if still constituting a quorum; otherwise, said
vacancies must be filled by the stockholders in a regular or special meeting called for that purpose. A director or
trustee so elected to fill a vacancy shall be elected only for the unexpired term of his predecessor in office. xxx.

[*NOTE* Roxas’ appointment was not appealed from the SEC thus, it was deemed final. It was only Makalintal’s
case which was appealed]

Makalintal's term should have expired after 1996 there being no unexpired term. The vacancy should have been
filled by the stockholders in a regular or special meeting called for that purpose and not by the election of the
remaining Board of Directors.

VVCC claims that Makalintal had resigned (over the hold-over capacity), and it is not due to the expiration of his
term. Thus, VVCC insists that the board rightfully appointed Ramirez to fill in the vacancy. In connection with Sec.
23 and 29 of the Corporation Code, VVCC contends that “the term of [a member] of the board of directors has yet
not expired until such time as [a successor is] elected or qualified in an annual election where a quorum is
present.”

ISSUES:

A. Whether the powers of the corporation’s board of directors emanate from its stockholders?

YES. The Court stated that VVCC’s construction of Sec. 29 in relation to Sec. 23 of the Corporation Code weakens
the power of the stockholder to participate in the corporate governance by electing their representatives to the
board of directors.

The underlying policy of the Corporation Code is that the business and affairs of a corporation must be governed by
a board of directors whose members have stood for election, and who have actually been elected by the
stockholders, on an annual basis. Only in that way can the directors’ continued accountability to shareholders, and
the legitimacy of their decisions that bind the corporation’s stockholders, be assured. The shareholder vote is
critical to the theory that legitimizes the exercise of power by the directors or officers over properties that they do
not own.

This theory of delegated power of the board of directors similarly explains why, under Section 29 of the Corporation
Code, in cases where the vacancy in the corporation’s board of directors is caused not by the expiration of a
member’s term, the successor “so elected to fill in a vacancy shall be elected only for the unexpired term of his
predecessor in office.” The law has authorized the remaining members of board, it limited the period during which
the successor shall serve only to the “unexpired term of his predecessor in office.”

B. Whether the remaining directors of a corporation’s Board, still constituting a quorum, can elect
another director to fill in a vacancy caused by the resignation of a holdover director?

NO. The Court stated “the vacancy” referred to in Section 29 contemplates a vacancy occurring within the
director’s term of office. When a vacancy is created by the expiration of a term, logically, there is no more
CORPORATION LAW CASE DIGESTS | 48
FEU JD4401 | 2nd SEMESTER, S.Y. 2017-2018

unexpired term to speak of. Hence, Section 29 declares that it shall be the corporation’s stockholders who shall
possess the authority to fill in a vacancy caused by the expiration of a member’s term.
The Court defined “term” as the time during which the officer may claim to hold the office as of right and fixes the
interval after which the several incumbents shall succeed one another. The term of office is not affected by the
holdover. Term is distinguished from tenure in that an officer’s “tenure” represents the term during which the
incumbent actually holds office. The tenure may be shorter (or, in case of holdover, longer) than the term for
reasons within or beyond the power of the incumbent.

The Court construes “the hold office for one (1) year until their successors are elected and qualified,” to mean that
the term of the members of the board of directors shall be only for one year. Their term expires one year after
election to the office. The holdover period—that time from the lapse of one year from a member’s election to the
Board and until his successor’s election and qualification—is not part of the director’s original term of office, nor is
it a new term. The holdover period, however, constitutes part of his tenure.

After the lapse of one year from his election as member of the VVCC Board in 1996, Makalintal’s term of office is
deemed to have already expired. That he continued to serve in the VVCC Board in a holdover capacity cannot be
considered as extending his term. His resignation as a holdover director did not change the nature of the vacancy;
the vacancy due to the expiration of Makalintal’s term had been created long before his resignation.

With the expiration of Makalintal’s term of office, a vacancy resulted which, by the terms of Section 29 of the
Corporation Code, must be filled by the stockholders of VVCC in a regular or special meeting called for the purpose.
CORPORATION LAW CASE DIGESTS | 49
FEU JD4401 | 2nd SEMESTER, S.Y. 2017-2018

(30) Bernas, et. al. v. Cinco


G.R. Nos. 163356-57, July 10, 2015
By: Mano, Razna I.

Doctrine: The Corporation Code laid down the rules on the removal of the Directors of the corporation by
providing, inter alia, the persons authorized to call the meeting and the number of votes required for the purpose
of removal.

As a result of alleged mishandling of corporate funds, stockholders of the Makati Sports Club (MSC) representing at
least 100 shares sought the assistance of the MSC Oversight Committee (MSCOC) in calling for a special
stockholders’ meeting for the purpose of electing a new set of officers, thereby removing the Bernas Group from
the Board of Directors and Officers of the Corporation. The MSCOC thus called a special stockholders’ meeting
wherein the members of the Bernas Group were removed from office and replaced by the Cinco Group. The term of
the Bernas Group was supposed to expire in 1998 or 1999 but the Cinco Group took office after they were elected
on December 17, 1997. In the annual stockholders’ meeting subsequently held on April 20, 1998, at which 2/3 of
stockholders were present, the majority approved and ratified the calling and holding of December 17, 1997 special
stockholders’ meeting, including the removal of the Bernas Group and the election of their replacements. The
Bernas Group filed an action with the SEC claiming that the MSCOC is not vested with the power to call for the
corporate meetings as the authority lies with the corporate secretary.

A. Was the removal of the Bernas Group valid?

No, the removal was not valid.

Section 28 of the Corporation Code provides that any director or trustee of a corporation may be removed from
office by a vote of the stockholders holding or representing at least two-thirds (2/3) of the outstanding capital
stock, or if the corporation is a non-stock corporation, by a vote of at least two-thirds (2/3) of the members
entitled to vote, provided that such removal shall take place either at a regular or special meeting called for the
purpose. Removal may be with or without cause provided such removal may not be used to deprive
minority representation. Such meeting may be called by the (1) secretary on order of the president or
(2) on written demand of the stockholders representing or holding at least a majority of the
outstanding capital stock or if non-stock corporation on written demand of majority of the members.

While directors may be removed with or without cause, however the meeting for the removal of directors must be
done in accordance with the law or the by-laws of the corporation. Neither the Corporation Code nor the MSC by-
laws authorizes MSCOC to exercise the power to call a special meeting for the purpose of removing directors of
MSC. The defect goes into the very authority of the persons who made the call for the meeting.

B. Was the stockholders’ ratification of the removal of the directors valid?

No, the stockholders’ ratification of the removal of the directors is not valid. A distinction should be made between
corporate acts or contracts which are illegal and those which are merely ultra vires. The former contemplates the
doing of an act which is contrary to law, morals or public policy or public duty, and are like similar transactions
between individuals, void. They cannot serve as basis of a court action nor acquire validity by performance,
ratification or estoppel. Mere ultra vires acts, on the other hand, or those which are not illegal or void ab initio, but
are not merely within the scope of the articles of incorporation, are merely voidable and may became binding and
enforceable when ratified by the stockholders. The December 1997 meeting is void ab initio and cannot be
validated. The removal of the Bernas Group is void.
CORPORATION LAW CASE DIGESTS | 50
FEU JD4401 | 2nd SEMESTER, S.Y. 2017-2018

(31) Marc II Marketing, Inc. vs Alfredo M. Joson


GR No. 171993, December 12, 2011
By: Marasigan, Mariz Angelle R.

Doctrine: Not all conflicts between the stockholders and the corporation are classified as intra-corporate. Other
factors such as the status or relationship of the parties and the nature of the question that is the subject of the
controversy must be considered in determining whether the dispute involves corporate matters so as to regard
them as intra-corporate controversies.

Before petitioner Marc II Marketing, Inc. was officially incorporated, Alfredo (respondent) has already been
engaged by petitioner Lucila to work as the General Manager of petitioner corporation. It was formalized through
the execution of a Management Contract.

Pending incorporation of petitioner corporation, respondent was designated as the General Manager of Marc
Marketing, Inc., which was then in the process of winding up its business. For occupying the said position,
respondent was among its corporate officers by the express provision of Section 1, Article IV of its by-laws.

On 15 August 1994, petitioner corporation was officially incorporated and registered with the SEC. Accordingly,
Marc Marketing, Inc. was made non-operational. Respondent continued to discharge his duties as General Manager
but this time under petitioner corporation.

Pursuant to Section 1, Article IV of petitioner corporation’s by-laws, its corporate officers are as follows: Chairman,
President, one or more Vice-President(s), Treasurer and Secretary. Its Board of Directors, however, may, from
time to time, appoint such other officers as it may determine to be necessary or proper.

Per an undated Secretary’s Certificate, petitioner corporations Board of Directors conducted a meeting where
respondent was appointed as one of its corporate officers with the designation or title of General Manager to
function as a managing director with other duties and responsibilities that the Board of Directors may provide and
authorized.

Nevertheless, on 30 June 1997, petitioner corporation decided to stop and cease its operations, as evidenced by an
Affidavit of Non-Operation, due to poor sales collection aggravated by the inefficient management of its affairs. On
the same date, it formally informed respondent of the cessation of its business operation. Concomitantly,
respondent was apprised of the termination of his services as General Manager since his services as such would no
longer be necessary for the winding up of its affairs.

Feeling aggrieved, respondent filed a Complaint for Reinstatement and Money Claim against petitioners before the
Labor Arbiter.

For the parties’ failure to settle the case amicably, the Labor Arbiter required them to submit their respective
position papers. Respondent complied but petitioners opted to file a Motion to Dismiss grounded on the Labor
Arbiter’s lack of jurisdiction as the case involved an intra-corporate controversy, which jurisdiction belongs to the
SEC [now with the Regional Trial Court (RTC)]. Petitioners similarly raised therein the ground of prescription of
respondents monetary claim.

Which between the Labor Arbiter or the RTC, has jurisdiction over respondent’s dismissal as General
Manager of petitioner corporation?

Held: Labor Arbiter.

Under Section 5 of Presidential Decree No. 902-A, intra-corporate controversies are those controversies arising out
of intra-corporate or partnership relations, between and among stockholders, members or associates; between any
or all of them and the corporation, partnership or association of which they are stockholders, members or
associates, respectively; and between such corporation, partnership or association and the State insofar as it
concerns their individual franchise or right to exist as such entity. It also includes controversies in the election or
appointments of directors, trustees, officers or managers of such corporations, partnerships or associations.

Accordingly, in determining whether the SEC (now the RTC) has jurisdiction over the controversy, the status or
relationship of the parties and the nature of the question that is the subject of their controversy must be taken into
consideration.

In Easycall Communications Phils., Inc. v. King, this Court held that in the context of Presidential Decree No. 902-
A, corporate officers are those officers of a corporation who are given that character either by the
Corporation Code or by the corporation’s by-laws. Section 25 of the Corporation Code specifically enumerated
who are these corporate officers, to wit: (1) president; (2) secretary; (3) treasurer; and (4) such other officers
as may be provided for in the by-laws.
CORPORATION LAW CASE DIGESTS | 51
FEU JD4401 | 2nd SEMESTER, S.Y. 2017-2018

A careful perusal of petitioner corporation’s by-laws, particularly paragraph 1, Section 1, Article IV, would explicitly
reveal that its corporate officers are composed only of: (1) Chairman; (2) President; (3) one or more Vice-
President; (4) Treasurer; and (5) Secretary. The position of General Manager was not among those
enumerated.

Paragraph 2, Section 1, Article IV of petitioner corporation’s by-laws, empowered its Board of Directors to appoint
such other officers as it may determine necessary or proper. It is by virtue of this enabling provision that petitioner
corporations Board of Directors allegedly approved a resolution to make the position of General Manager a
corporate office, and, thereafter, appointed respondent thereto making him one of its corporate officers. All of
these acts were done without first amending its by-laws so as to include the General Manager in its roster of
corporate officers.

With the given circumstances and in conformity with Matling Industrial and Commercial Corporation v. Coros, this
Court rules that respondent was not a corporate officer of petitioner corporation because his position as General
Manager was not specifically mentioned in the roster of corporate officers in its corporate by-laws. The enabling
clause in petitioner corporation’s by-laws empowering its Board of Directors to create additional officers, i.e.,
General Manager, and the alleged subsequent passage of a board resolution to that effect cannot make such
position a corporate office. Matling clearly enunciated that the board of directors has no power to create other
corporate offices without first amending the corporate by-laws so as to include therein the newly created corporate
office. Though the board of directors may create appointive positions other than the positions of corporate officers,
the persons occupying such positions cannot be viewed as corporate officers under Section 25 of the Corporation
Code. In view thereof, this Court holds that unless and until petitioner corporations by-laws is amended for the
inclusion of General Manager in the list of its corporate officers, such position cannot be considered as a corporate
office within the realm of Section 25 of the Corporation Code.

That respondent was also a director and a stockholder of petitioner corporation will not automatically make the
case fall within the ambit of intra-corporate controversy and be subjected to RTCs jurisdiction. To reiterate, not all
conflicts between the stockholders and the corporation are classified as intra-corporate. Other factors such as the
status or relationship of the parties and the nature of the question that is the subject of the controversy must be
considered in determining whether the dispute involves corporate matters so as to regard them as intra-corporate
controversies. As previously discussed, respondent was not a corporate officer of petitioner corporation but a mere
employee thereof so there was no intra-corporate relationship between them. With regard to the subject of the
controversy or issue involved herein, i.e., respondent’s dismissal as petitioner corporations General Manager, the
same did not present or relate to an intra-corporate dispute. To note, there was no evidence submitted to show
that respondent’s removal as petitioner corporations General Manager carried with it his removal as its director and
stockholder. Also, petitioners’ allegation that respondents claim of 30% share of petitioner corporations net profit
was by reason of his being its director and stockholder was without basis, thus, self-serving. Such an allegation
was tantamount to a mere speculation for petitioners’ failure to substantiate the same.

With all the foregoing, this Court is fully convinced that, indeed, respondent, though occupying the General
Manager position, was not a corporate officer of petitioner corporation rather he was merely its employee
occupying a high-ranking position.

Accordingly, respondent’s dismissal as petitioner corporations General Manager did not amount to an intra-
corporate controversy. Jurisdiction therefore properly belongs with the Labor Arbiter and not with the RTC.
CORPORATION LAW CASE DIGESTS | 52
FEU JD4401 | 2nd SEMESTER, S.Y. 2017-2018

(32) Georg vs. Holy Trinity College, Inc.


G.R. No. 190408. July 20, 2016
By: Pangilinan, Gene Alexis

Doctrine: The doctrine of apparent authority provides that a corporation will be estopped from denying the agent’s
authority if it knowingly permits one of its officers or any other agent to act within the scope of an apparent
authority, and it holds him out to the public as possessing the power to do those acts.

Facts: The Holy Trinity College Grand Chorale and Dance Company (the Group), composed of students from Holy
Trinity College, was organized by Sr. Medalle, the President of respondent Holy Trinity College. The Group was
slated to perform in Greece, Italy, Spain and Germany. Enriquez, who allegedly represented Sr. Medalle, contacted
petitioner Georg to seek assistance for payment of the Group’s international airplane tickets. A Memorandum of
Agreement with Deed of Assignment (MOA) was executed between petitioner and the Group, represented by Sr.
Medalle. Under the said Agreement, petitioner, through her travel agency, will advance the payment of
international airplane tickets in favor of the Group on the assurance of the Group represented by Sr. Medalle
through Enriquez that there is a confirmed financial allocation from the foundation-grantor, S.C. Roque Foundation.

A Complaint for Sum of Money with Damages was filed before the RTC by the petitioner, claiming that respondent
and the foundation-grantor have not paid and refused to pay their obligation under the MOA. In their Answer with
Counterclaim, respondent argued that the MOA on which petitioner based its cause of action does not state that
respondent is a party. Petitioner claims otherwise because Sr. Medalle in her capacity as President of Holy Trinity
College, affixed her thumbmark in the MOA. The respondent claims that Sr. Medalle was not authorized by the
corporation to enter into any loan agreement thus the MOA executed was null and void for being ultra vires.
Petitioner invokes, as refutation, the doctrine of apparent authority.

The RTC ruled in favor of petitioner. The Court of Appeals noted the absence of respondent’s name in the MOA,
thus it concluded that respondent was clearly not a party to the MOA. The Court of Appeals also ruled that there
was no showing that Sr. Medalle was duly authorized by respondent to enter into the subject MOA.

Issue: Whether or not the respondent is liable under the Memorandum of Agreement with Deed of Assignment.

Held: YES. The respondent is liable under the Memorandum of Agreement with Deed of Assignment. Sr. Medalle,
as President of Holy Trinity, is clothed with sufficient authority to enter into a loan agreement. As held by the trial
court, the Holy Trinity College’s Board of Trustees never contested the standing of the Dance and Chorale Group
and had in fact lent its support in the form of sponsoring uniforms or freely allowed the school premises to be used
by the group for their practice sessions. In addition, petitioner was able to prove that the Board of Trustees, the
administration, as well as the congregation to which they belong have consented or ratified the actions of Sr.
Medalle.

Assuming arguendo that Sr. Medalle was not authorized by the Holy Trinity College Board, the doctrine of apparent
authority applies in this case. The doctrine of apparent authority provides that a corporation will be estopped from
denying the agent’s authority if it knowingly permits one of its officers or any other agent to act within the scope of
an apparent authority, and it holds him out to the public as possessing the power to do those acts.

The existence of apparent authority may be ascertained through (1) the general manner in which the corporation
holds out an officer or agent as having the power to act or, in other words, the apparent authority to act in general,
with which it clothes him; or (2) the acquiescence in his acts of a particular nature, with actual or constructive
knowledge thereof, whether within or beyond the scope of his ordinary powers.

In this case, Sr. Medalle formed and organized the Group. She had been giving financial support to the Group, in
her capacity as President of Holy Trinity College. Sr. Navarro admitted that the Board of Trustees never questioned
the existence and activities of the Group. Thus, any agreement or contract entered into by Sr. Medalle as President
of Holy Trinity College relating to the Group bears the consent and approval of respondent. The petitioner cannot
be faulted for relying on Sr. Medalle’s authority to transact with petitioner.
CORPORATION LAW CASE DIGESTS | 53
FEU JD4401 | 2nd SEMESTER, S.Y. 2017-2018

(33) Heirs of Fe Tan Uy vs. International Exchange Bank


By: Radovan, Althea

Doctrine The piercing of the veil of corporate fiction is frowned upon and can only be done if it has been clearly
established that the separate and distinct personality of the corporation is used to justify a wrong, protect fraud, or
perpetrate a deception.

Facts: Respondent International Exchange Bank (iBank), granted loans to Hammer Garments Corporation
(Hammer), covered by promissory notes and deeds of assignment. The loans were made pursuant to a Letter-
Agreement between iBank and Hammer, represented by its President and General Manager, Manuel Chua (Chua)
a.k.a. Manuel Chua Uy Po Tiong, granting Hammer a P 25 Million-Peso Omnibus Line. The loans were secured by a
P 9 Million-Peso Real Estate Mortgage by Goldkey Development Corporation (Goldkey) over several of its properties
and a P 25 Million-Peso Surety Agreement signed by Chua and his wife, Fe Tan Uy (Uy).

Hammer had an outstanding obligation of P25,420,177.62 to iBank. Hammer defaulted in the payment of its loans
prompting iBank to foreclose on Goldkey’s third-party Real Estate Mortgage. The mortgaged properties were sold
for P 12 million during the foreclosure sale, leaving an unpaid balance of P 13,420,177.62.
Ibank filed a complaint for sum of money against Hammer to recover the deficiency in the latters loan obligation.

Despite service of summons, Chua and Hammer did not file their respective answers and were declared in default.
In her separate answer, Uy claimed that she was not liable to iBank because she never executed a surety
agreement in favor of iBank. Goldkey, on the other hand, also denies liability, averring that it acted only as a third-
party mortgagor and that it was a corporation separate and distinct from Hammer.

RTC: Ruled in favor of iBank. While it made the pronouncement that the signature of Uy on the Surety Agreement
was a forgery, it nevertheless held her liable for the outstanding obligation of Hammer because she was an officer
and stockholder of the said corporation. The RTC agreed with Goldkey that as a third-party mortgagor, its liability
was limited to the properties mortgaged. It came to the conclusion, however, that Goldkey and Hammer were one
and the same entity. As such, the piercing of the veil of corporate fiction was warranted. Uy, as an officer and
stockholder of Hammer and Goldkey, was found liable to iBank together with Chua, Hammer and Goldkey for the
deficiency of P13,420,177.62.

CA: According to the appellate court, iBank was induced to grant the loan because petitioners, with intent to
defraud the bank, submitted a falsified Financial Report for 1996 which incorrectly declared the assets and cashflow
of Hammer. Because petitioners acted maliciously and in bad faith and used the corporate fiction to defraud iBank,
they should be treated as one and the same as Hammer.

Aggrieved, the heirs of Uy and Goldkey (petitioners) elevated the case to the CA. Goldkey contends that it cannot
be held responsible for the obligations of its stockholder, Chua. Moreover, it theorizes that iBank is estopped from
expanding Goldkey’s liability beyond the real estate mortgage. The heirs of Uy argue that the latter could not be
held liable for being merely an officer of Hammer and Goldkey because it was not shown that she had committed
any actionable wrong or that she had participated in the transaction between Hammer and iBank. They further
claim that she had cut all ties with Hammer and her husband long before the execution of the loan.

Issues:
(1) Whether Uy can be held liable to iBank for the loan obligation of Hammer as an officer and stockholder of the
said corporation.
(2) Whether Goldkey can be held liable for the obligation of Hammer for being a mere alter ego of the latter.

Held:
1. Uy is not liable; The piercing of the veil of corporate fiction is not justified
A corporation is a juridical entity which is vested with a legal personality separate and distinct from those
acting for and in its behalf and, in general, from the people comprising it. Following this principle, obligations
incurred by the corporation, acting through its directors, officers and employees, are its sole liabilities. A director,
officer or employee of a corporation is generally not held personally liable for obligations incurred by the
corporation. Nevertheless, this legal fiction may be disregarded if it is used as a means to perpetrate fraud or an
illegal act, or as a vehicle for the evasion of an existing obligation, the circumvention of statutes, or to confuse
legitimate issues. This is consistent with the provisions of the Corporation Code of the Philippines, which states:
Sec. 31. Liability of directors, trustees or officers.―Directors or trustees who wilfully and knowingly vote for or
assent to patently unlawful acts of the corporation or who are guilty of gross negligence or bad faith in directing
the affairs of the corporation or acquire any personal or pecuniary interest in conflict with their duty as such
directors or trustees shall be liable jointly and severally for all damages resulting therefrom suffered by the
corporation, its stockholders or members and other persons.

Solidary liability will then attach to the directors, officers or employees of the corporation in certain
circumstances, such as:
CORPORATION LAW CASE DIGESTS | 54
FEU JD4401 | 2nd SEMESTER, S.Y. 2017-2018

1. When directors and trustees or, in appropriate cases, the officers of a corporation: (a) vote for or assent to
patently unlawful acts of the corporation; (b) act in bad faith or with gross negligence in directing the corporate
affairs; and (c) are guilty of conflict of interest to the prejudice of the corporation, its stockholders or members,
and other persons;
2. When a director or officer has consented to the issuance of watered stocks or who, having knowledge thereof,
did not forthwith file with the corporate secretary his written objection thereto;
3. When a director, trustee or officer has contractually agreed or stipulated to hold himself personally and solidarily
liable with the corporation; or
4. When a director, trustee or officer is made, by specific provision of law, personally liable for his corporate action.

Before a director or officer of a corporation can be held personally liable for corporate obligations,
however, the following requisites must concur:
(1) The complainant must allege in the complaint that the director or officer assented to patently unlawful acts of
the corporation, or that the officer was guilty of gross negligence or bad faith; and
(2) The complainant must clearly and convincingly prove such unlawful acts, negligence or bad faith

In this case, petitioners are correct to argue that it was not alleged, much less proven, that Uy committed
an act as an officer of Hammer that would permit the piercing of the corporate veil. Considering that the only basis
for holding Uy liable for the payment of the loan was proven to be a falsified document, there was no sufficient
justification for the RTC to have ruled that Uy should be held jointly and severally liable to iBank for the unpaid
loan of Hammer.

At most, Uy could have been charged with negligence in the performance of her duties as treasurer of
Hammer by allowing the company to contract a loan despite its precarious financial position. Furthermore, if it was
true, as petitioners claim, that she no longer performed the functions of a treasurer, then she should have formally
resigned as treasurer to isolate herself from any liability that could result from her being an officer of the
corporation. Nonetheless, these shortcomings of Uy are not sufficient to justify the piercing of the corporate veil,
which requires that the negligence of the officer must be so gross that it could amount to bad faith and must be
established by clear and convincing evidence. Gross negligence is one that is characterized by the lack of the
slightest care, acting or failing to act in a situation where there is a duty to act, willfully and intentionally with a
conscious indifference to the consequences insofar as other persons may be affected.
It behooves this Court to emphasize that the piercing of the veil of corporate fiction is frowned upon and
can only be done if it has been clearly established that the separate and distinct personality of the corporation is
used to justify a wrong, protect fraud, or perpetrate a deception.
Indeed, there is no showing that Uy committed gross negligence. And in the absence of any of the aforementioned
requisites for making a corporate officer, director or stockholder personally liable for the obligations of a
corporation, Uy, as a treasurer and stockholder of Hammer, cannot be made to answer for the unpaid debts of the
corporation.

2. YES, Goldkey is a mere alter ego of Hammer, as such they are treated as one and the same entity, making
Goldkey accountable for the debts of Hammer.
Under a variation of the doctrine of piercing the veil of corporate fiction, when two business enterprises are owned,
conducted and controlled by the same parties, both law and equity will, when necessary to protect the rights of
third parties, disregard the legal fiction that two corporations are distinct entities and treat them as identical or one
and the same. While the conditions for the disregard of the juridical entity may vary, the following are some
probative factors of identity that will justify the application of the doctrine of piercing the corporate veil:

(1) Stock ownership by one or common ownership of both corporations;


(2) Identity of directors and officers;
(3) The manner of keeping corporate books and records, and
(4) Methods of conducting the business.

These factors are unquestionably present in the case of Goldkey and Hammer, as follows:
1. Both corporations are family corporations of defendants Manuel Chua and his wife Fe Tan Uy. The other
incorporators and shareholders of the two corporations are the brother and sister of Manuel Chua (Benito Ng Po
Hing and Nenita Chua Tan) and the sister of Fe Tan Uy, Milagros Revilla. The other incorporator/share holder is
Manling Uy, the daughter of Manuel Chua Uy Po Tiong and Fe Tan Uy.
The stockholders of Hammer Garments as of March 23, 1987, aside from spouses Manuel and Fe Tan Uy are:
Benito Chua, brother Manuel Chua, Nenita Chua Tan, sister of Manuel Chua and Tessie See Chua Tan. On March 8,
1988, the shares of Tessie See Chua Uy were assigned to Milagros T. Revilla, thereby consolidating the shares in
the family of Manuel Chua and Fe Tan Uy.
2. Hammer Garments and Goldkey share the same office and practically transact their business from the same
place.
CORPORATION LAW CASE DIGESTS | 55
FEU JD4401 | 2nd SEMESTER, S.Y. 2017-2018

3. Defendant Manuel Chua is the President and Chief Operating Officer of both corporations. All business
transactions of Goldkey and Hammer are done at the instance of defendant Manuel Chua who is authorized to do
so by the corporations.
The promissory notes subject of this complaint are signed by him as Hammer’s President and General Manager.
The third-party real estate mortgage of defendant Goldkey is signed by him for Goldkey to secure the loan
obligation of Hammer Garments with plaintiff “iBank. The other third- party real estate mortgages which Goldkey
executed in favor of the other creditor banks of Hammer are also signed Manuel Chua.
4. The assets of Goldkey and Hammer are co-mingled. The real properties of Goldkey are mortgaged to secure
Hammer’s obligation with creditor banks.
The proceeds of at least two loans which Hammer obtained from plaintiff “iBank”, purportedly to finance its export
to Wal-Mart are instead used to finance the purchase of a manager’s check payable to Goldkey. The defendants’
claim that Goldkey is a creditor of Hammer to justify its receipt of the Manager’s check is not substantiated by
evidence. Despite subpoenas issued by this Court, Goldkey thru its treasurer, defendant Fe Tan Uy and or its
corporate secretary Manling failed to produce the Financial Statement of Goldkey.

5. When defendant Manuel Chua “disappeared”, the defendant Goldkey ceased to operate despite the claim that
the other “officers” and stockholders like Benito Chua, Nenita Chua Tan, Fe Tan Uy, Manling Uy and Milagros T.
Revilla are still around and may be able to continue the business of Goldkey, if it were different or distinct from
Hammer, which suffered financial set back.

Based on the foregoing findings of the RTC, it was apparent that Goldkey was merely an adjunct of Hammer and,
as such, the legal fiction that it has a separate personality from that of Hammer should be brushed aside as they
are, undeniably, one and the same.
CORPORATION LAW CASE DIGESTS | 56
FEU JD4401 | 2nd SEMESTER, S.Y. 2017-2018

(34) Arnold and Bradley Hall vs. Judge Piccio, Brown and Capuciong
G.R. No. L-2598, June 29, 1950
By: Rosario, Patricia Kaye T.

Doctrine: An entity whose certificate of incorporation had not been obtained may be terminated in a private suit
for its dissolution between stockholders, without the intervention of the state. The question as to the right of
minority stockholders to sue for dissolution does not affect the court's jurisdiction, and is a matter for decision by
the judge, subject to review on appeal by the aggrieved party at the proper time.

Petitioners and respondents signed and acknowledged the article of incorporation of the Far Eastern Lumber and
Commercial Co., Inc., organized to engage in a general lumber business to carry on as general contractors,
operators and managers. Respondents Brown filed before the CFI of Leyte civil case No. 381 alleging that the Far
Eastern Lumber and Commercial Co. was an unregistered partnership; and that they wished to have it dissolved
because of bitter dissension among the members, mismanagement and fraud by the managers and heavy financial
losses. Judge Piccio ordered the dissolution of the company. Petitioners countered that the court had no jurisdiction
in civil case No. 381 to decree the dissolution of the company, because it being a de facto corporation, dissolution
thereof may only be ordered in a quo warranto proceeding instituted in accordance with section 19 of the
Corporation Law.

Can the dissolution of a de facto corporation be ordered in a quo warranto proceeding only?

NO, a de facto corporation may be terminated in a private suit for its dissolution between stockholders.

Section 19 of the Corporation law reads as follows:


. . . The due incorporation of any corporations claiming in good faith to be a corporation under this Act
and its right to exercise corporate powers shall not be inquired into collaterally in any private suit to
which the corporation may be a party, but such inquiry may be had at the suit of the Insular Government
on information of the Attorney-General.

There are least two reasons why this section does not govern the situation. First, not having obtained the
certificate of incorporation, the Far Eastern Lumber and Commercial Co. — even its stockholders — may not
probably claim "in good faith" to be a corporation. Second, this is not a suit in which the corporation is a party. This
is a litigation between stockholders of the alleged corporation, for the purpose of obtaining its dissolution. Even the
existence of a de jure corporation may be terminated in a private suit for its dissolution between stockholders,
without the intervention of the state.
CORPORATION LAW CASE DIGESTS | 57
FEU JD4401 | 2nd SEMESTER, S.Y. 2017-2018

(35) Pioneer Insurance v. Court of Appeals


G.R. No. 84197, July 28, 1989
By: Samson, Maria Johanna Ilyssa

Topic: De Facto Partnership

Doctrine: Persons who attempt but fail to form a corporation and who carry on business under the corporate name
occupy the position of partners inter se. Thus, where persons associate themselves together under articles to
purchase property to carry on a business, and their organization is so defective as to come short of creating a
corporation within the statute, they become in legal effect partners inter se, and their rights as members of the
company to the property acquired by the company will be recognized.

Jacob S. Lim (petitioner in G.R. No. 84157) was engaged in the airline business as owner-operator of Southern Air
Lines (SAL) a single proprietorship. At Tokyo, Japan, Japan Domestic Airlines (JDA) and Lim entered into and
executed a sales contract for the sale and purchase of two (2) DC-3A Type aircrafts and one (1) set of necessary
spare parts for the total agreed price of US $109,000.00 to be paid in installments. Pioneer Insurance and Surety
Corporation (Pioneer, petitioner in G.R. No. 84197) as surety executed and issued its Surety Bond No. 6639 in
favor of JDA, in behalf of its principal, Lim, for the balance price of the aircrafts and spare parts. It appears that
Border Machinery and Heavy Equipment Company, Inc. (Bormaheco), Francisco and Modesto Cervantes
(Cervanteses) and Constancio Maglana (respondents in both petitions) contributed some funds used in the
purchase of the above aircrafts and spare parts. The funds were supposed to be their contributions to a new
corporation proposed by Lim to expand his airline business. They executed two (2) separate indemnity agreements
in favor of Pioneer, one signed by Maglana and the other jointly signed by Lim for SAL, Bormaheco and the
Cervanteses. The indemnity agreements stipulated that the indemnitors principally agree and bind themselves
jointly and severally to indemnify and hold and save harmless Pioneer from and against any/all damages, losses,
costs, damages, taxes, penalties, charges and expenses of whatever kind and nature which Pioneer may incur in
consequence of having become surety upon the bond/note and to pay, reimburse and make good to Pioneer, its
successors and assigns, all sums and amounts of money which it or its representatives should or may pay or cause
to be paid or become liable to pay on them of whatever kind and nature. Lim doing business under the name and
style of SAL executed in favor of Pioneer as deed of chattel mortgage as security for the latter’s suretyship in favor
of the former. Lim defaulted on his subsequent instalment payments prompting JDA to request payments from the
surety. Pioneer paid a total sum of P298,626.12. Pioneer then filed a petition for the extrajudicial foreclosure of the
said chattel mortgage. The Cervanteses and Maglana, however, filed a third party claim alleging that they are co-
owners of the aircrafts. Pioneer filed an action for judicial foreclosure with an application for a writ of preliminary
attachment against Lim and respondents, the Cervanteses, Bormaheco and Maglana. After trial on the merits, a
decision was rendered holding Lim liable to pay Pioneer but dismissed Pioneer’s complaint against all other
defendants.

What legal rules govern the relationship among co-investors whose agreement was to do business
through the corporate vehicle but who failed to incorporate the entity in which they had chosen to
invest?

While it has been held that as between themselves the rights of the stockholders in a defectively incorporated
association should be governed by the supposed charter and the laws of the state relating thereto and not by the
rules governing partners (Cannon v. Brush Electric Co.), it is ordinarily held that persons who attempt, but fail, to
form a corporation and who carry on business under the corporate name occupy the position of partners inter se
(Lynch v. Perryman). Thus, where persons associate themselves together under articles to purchase property to
carry on a business, and their organization is so defective as to come short of creating a corporation within the
statute, they become in legal effect partners inter se, and their rights as members of the company to the property
acquired by the company will be recognized.

How are the losses to be treated in situations where their contributions to the intended ‘corporation’
were invested not through the corporate form?

This question is premised on the petitioner’s theory that as a result of the failure of respondents Bormaheco,
Spouses Cervantes, Constancio Maglana and petitioner Lim to incorporate, a de facto partnership among them was
created, and that as a consequence of such relationship all must share in the losses and/or gains of the venture in
proportion to their contribution. The trial court and the appellate court, however, found that the petitioner received
CORPORATION LAW CASE DIGESTS | 58
FEU JD4401 | 2nd SEMESTER, S.Y. 2017-2018

the amount of P151,000.00 representing the participation of Bormaheco and Atty. Constancio B. Maglana in the
ownership of the subject airplanes and spare parts. It is therefore clear that the petitioner never had the intention
to form a corporation with the respondents despite his representations to them. This gives credence to the cross-
claims of the respondents to the effect that they were induced and lured by the petitioner to make contributions to
a proposed corporation which was never formed because the petitioner reneged on their agreement.

However, such a relation does not necessarily exist, for ordinarily persons cannot be made to assume the relation
of partners, as between themselves, when their purpose is that no partnership shall exist (London Assur. Corp. v.
Drennen), and it should be implied only when necessary to do justice between the parties; thus, one who takes no
part except to subscribe for stock in a proposed corporation which is never legally formed does not become a
partner with other subscribers who engage in business under the name of the pretended corporation, so as to be
liable as such in an action for settlement of the alleged partnership and contribution (Ward v. Brigham).

Was there a de facto partnership created among the parties which would entitle the petitioner to a
reimbursement of the supposed losses of the proposed corporation?

There was no de facto partnership created, applying therefore the principles of law earlier cited to the facts of the
case, necessarily, no de facto partnership was created among the parties which would entitle the petitioner to a
reimbursement of the supposed losses of the proposed corporation. The record shows that the petitioner was
acting on his own and not in behalf of his other would-be incorporators in transacting the sale of the airplanes and
spare parts.
CORPORATION LAW CASE DIGESTS | 59
FEU JD4401 | 2nd SEMESTER, S.Y. 2017-2018

3. BY-LAWS

(36) PMI Colleges vs. NLRC


G.R. No. 121466 August 15, 1997
By: Torres, Ma. Roma
Topic: By-laws

DOCTRINE: Since by-laws operate merely as internal rules among the stockholders, they cannot affect or
prejudice third persons who deal with the corporation, unless they have knowledge of the same.

FACTS: PMI Colleges hired Galvan as a contractual instructor for classes in marine engineering. Galvan demanded
unpaid compensation from the school through its president which went unheeded. He demanded his salary for
shipyard and plant visits and the on-going on-the-job training of Class 41 on board MV "Sweet Glory". The school
countered that the classes in the courses offered which complainant claimed to have remained unpaid were not
held or conducted in the school premises of PMI Colleges. Galvan alleged that PMI was fully aware of said shipyard
and plant visits because it even wrote a letter for that purpose; and that basic seaman courses 41 and 42 were
sanctioned by the DECS as shown by the records of the Registrar's Office. private respondent submitted
documentary evidence which were annexed to his complaint, such as the detailed load and schedule of classes with
number of class hours and rate per hour; PMI Colleges Basic Seaman Training Course; the aforementioned letter-
request for payment of salaries by the Acting Director of PMI Colleges ; unpaid load of private respondent; and
vouchers prepared by the accounting department of petitioner but whose amounts indicated therein were actually
never paid to private respondent.

The school puts emphasis on negating Galvan’s claims because of his failure to produce a copy of the contract
pursuant to which he rendered services. Moreover, under petitioner's by-laws only the Chairman is authorized to
sign any contract and that private respondent, in any event, failed to submit documents on the alleged shipyard
and plant visits in Cavite Naval Base.

ISSUE: WON respondent Galvan’s claim for unpaid wages is meritorious despite his failure to produce a contract of
employment duly signed by the Chairman

HELD: Yes. The absence of such copy does not in any manner negate the existence of a contract of employment
since "Contracts shall be obligatory, in whatever form they have been entered into, provided all the essential
requisites for their validity are present." The only exception to this rule is "when the law requires that a contract be
in some form in order that it may be valid or enforceable, or that a contract be proved in a certain way." However,
there is no requirement under the law that the contract of employment of the kind entered into by petitioner with
private respondent should be in any particular form.

Since by-laws operate merely as internal rules among the stockholders, they cannot affect or prejudice third
persons who deal with the corporation, unless they have knowledge of the same." No proof appears on record that
private respondent ever knew anything about the provisions of said by-laws. In fact, petitioner itself merely asserts
the same without even bothering to attach a copy or excerpt thereof to show that there is such a provision. That
this allegation has never been denied by private respondent does not necessarily signify admission of its existence
because technicalities of law and procedure and the rules obtaining in the courts of law do not strictly apply to
proceedings of this nature.
CORPORATION LAW CASE DIGESTS | 60
FEU JD4401 | 2nd SEMESTER, S.Y. 2017-2018

(37) Government of the Philippine Islands v. El Hogar Filipino


GR No. L-26649 July 13, 1927
By: Valencia, Emmanuelle Nicole L.

Topic: Forfeiture of Franchise

Doctrines: The circumstance that one of the provisions contained in the by-laws of a building and loan association
is invalid as conflicting with the express provision of statute is not a misdemeanor on. the part of the corporation
for which the association can be penalized by the forfeiture of its charter.

On March 1, 1906, the Philippine Commission enacted what is known as the Corporation Law (Act. No. 1459),
effective upon April 1, 1906. Sections 171 to 190 are devoted to the subject of building and loan associations,
defining their objects and making various provisions regarding their organisation and administration, and providing
for the exercise of supervision over these entities.

El Hogar Filipino was the first corporation organised in the Philippines under these provisions. The Articles of
Incorporation bear the date December 28, 1910, and its capital stock was subscribed to the amount of PhP
150,000, of which PhP 10,620 was paid in. The law allowed the capitalisation to be up to PhP 3,000,000.

Act No. 2092 was passed on December 23, 1911, and this amended the previous capitalisation cap, increasing it to
PhP 10,000,000. El Hogar took advantage of this, and amended its Articles to provide that the capital should be in
an amount not exceeding the new limit.

By December 31, 1925, El Hogar had 5,826 shareholders, holding 125,750 shares, with a total paid-up value of
PhP 8,703,602.25. It paid to withdrawing stockholders the amount of PhP 7,618,257.72 and distributed dividends
amounting to PhP 7,621,565.81.

Does El Hogar’s owning and holding of a business lot, with a structure thereon, in the financial district
of the City of Manila, and subsequent renting of the same constitute ultra vires acts on the part of the
corporation, with the proper penalty of dissolution? (second cause of action)

No, there is no contravention of the Corporation Law in this case.

On August 28, 1913, the respondent purchased 1,413 square meters of land at the corner of Juan Luna Street and
the Muelle de la Industria, in the City of Manila, immediately adjacent to the building then occupied by the
Hongkong and Shanghai Banking Corporation. At the time the respondent acquired this lot there stood upon it a
building, then nearly fifty years old, which was occupied in part by the offices of an importing firm and in part by
warehouses of the same firm.

In pursuance of a design which had been formed prior to the purchase of the property, the directors of the El Hogar
Filipino caused the old building to be demolished; and they erected thereon a modern reinforced concrete office
building. As at first constructed the new building was three stories high in the main, but in 1920, in order to obtain
greater advantage from the use of the land, an additional story was added to the building, making a structure of
four stories except in one corner where an additional story was placed, making it five stories high over an area of
117.52 square meters.

Since the new building was completed the respondent has used about 324 square meters of floor space for its own
offices and has rented the remainder of the office space in said building, consisting of about 3,175 square meters,
to other persons and entities.

Under subsection 5 of section 13 of the Corporation Law, every corporation has the power to purchase, hold and
lease such real property as the transaction of the lawful business of the corporation may reasonably and
necessarily require. When this property was acquired in 1916, the business of El Hogar Filipino had developed to
such an extent, and its prospects for the future were such as to justify its directors in acquiring a lot in the financial
district of the City of Manila and in constructing thereon a suitable building as the site of its offices; and it cannot
be fairly said that the area of the lot—1,413 square meters —was in excess of its reasonable requirements. The law
expressly declares that corporations may acquire such real estate as is reasonably necessary to enable them to
CORPORATION LAW CASE DIGESTS | 61
FEU JD4401 | 2nd SEMESTER, S.Y. 2017-2018

carry out the purposes for which they were created. The Supreme Court held that the owning of a business lot
upon which to construct and maintain its offices is reasonably necessary to a building and loan association such as
the respondent was at the time this property was acquired. Further, inasmuch as the lot referred to was lawfully
acquired by the respondent, it is entitled to the full beneficial use thereof. It was acting clearly within the exercise
of its corporate right and power.

Is by-law No. 10 null and void for being in conflict with the Corporation Law? If so, does the existence
of this by-law merit the dissolution of the corporation? (fourth cause of action)

Yes, by-law No. 10 is a patent nullity, since it is in direct conflict with the latter part of Sec. 187 of the Corporation
Law.

By-law No. 10 states that “The board of directors of the association, by the vote of an absolute majority of its
members, is empowered to cancel shares and to return to the owner thereof the balance resulting from the
liquidation thereof whenever, by reason of their conduct, or for any other motive, the continuation as members of
the owners of such shares is not desirable.”

Sec. 187 of the Corporation Law expressly declares that the board of directors shall not have the power to force
the surrender and withdrawal of unmatured stock except in case of liquidation of the corporation or of forfeiture of
the stock for delinquency.
No, the existence of this by-law does not justify the dissolution of El Hogar.

The obnoxious by-law, as it stands, is a mere nullity, and could not be enforced even if the directors were to
attempt to do so. There is no provision of law making it a misdemeanor to incorporate an invalid provision in the
by-laws of a corporation; and if there were such, the hazards incident to corporate effort would certainly be largely
increased.

Does the failure of the corporation to hold annual meetings and the filling of vacancies in the
directorate in the manner described constitute misdemeanors on the part of El Hogar? If so, does this
justify the resumption of the franchise by the Government and dissolution of the corporation? (fifth
cause of action)

No, the failure to hold annual meetings, and the procedure adopted for the filling of vacancies in the Board of
Directors does not constitute misdemeanour on the part of El Hogar.

Section 31 of the Corporation Law states that, "at all elections of directors there must be present, either in person
or by representative authorized to act by written proxy, the owners of the majority of the subscribed capital stock
entitled to vote."
Conformably with this requirement, Article 61 of the by-laws of El Hogar Filipino states that, “the attendance in
person or by proxy of shareholders owning one-half plus one of the shareholders shall be necessary to constitute a
quorum for the election of directors.

At the general annual meetings of the El Hogar Filipino held in the years 1911 and 1912, there was a quorum of
shares present or represented at the meetings and directors were duly elected accordingly- As the corporation has
grown, however, it has been found increasingly difficult to get together a quorum of the shareholders, or their
proxies, at the annual meetings; and with the exception of the annual meeting held in 1917, when a new
directorate was elected, the meetings have failed for lack of quorum.

Owing to the failure of a quorum at most of the general meetings, it has been the practice of the directors to fill
vacancies in the directorate by choosing suitable persons from among the stockholders. This custom finds its
sanction in article 71 of the by-laws, which reads as follows: "ART. 71. The directors shall elect from among the
shareholders members to fill the vacancies that may occur in the board of directors until the election at the general
meeting.”

No fault can be imputed to the corporation on account of the failure of the shareholders to attend the annual
meetings; and their non-attendance at such meetings is doubtless to be interpreted in part as expressing their
satisfaction at the way in which things have been conducted. Upon failure of a quorum at any annual meeting the
directorate naturally holds over and continues to function until another directorate is chosen and qualified. Unless
CORPORATION LAW CASE DIGESTS | 62
FEU JD4401 | 2nd SEMESTER, S.Y. 2017-2018

the law or the charter of a corporation expressly provides that an office shall become vacant at the expiration of
the term of office for which the officer was elected, the general rule is to allow the officer to hold over until his
successor is duly qualified. Mere failure of a corporation to elect officers does not terminate the terms of existing
officers nor dissolve the corporation.
Is it valid for El Hogar to prescribe compensation for the members of its Board of Directors? (sixth
cause of action)

Yes, the El Hogar is empowered to prescribe compensation for its Board of Directors. The amount of the
compensation is a matter left to the discretion of the shareholders.

In so far as this court is concerned the question here before us is not one concerning the propriety and wisdom of
the measure of compensation adopted by the respondent but rather the question of the validity of the measure.
The Corporation Law does not undertake to prescribe the rate of compensation for the directors of corporations.
Under Sec. 21 of the Corporation Law, the power to fixed the compensation they shall receive, if any, is left to the
corporation, to be determined in its by-laws.

Pursuant to this authority the compensation for the directors of El Hogar Filipino has been fixed in section 92 of its
by-laws. The justice and propriety of this provision was a proper matter for the shareholders when the by-laws
were framed; and the circumstance that, with the growth of the corporation, the amount paid as compensation to
the directors has increased beyond what would probably be necessary to secure adequate service from them is a
matter that cannot be corrected in this action; nor can it properly be made a basis for depriving the respondent of
its franchise, or even for enjoining it from compliance with the provisions of its own by-laws. If a mistake has been
made, or the rule adopted in the by-laws has been found to work harmful results, the remedy is in the hands of the
stockholders who have power at any lawful meeting to change the rule.

Are by-laws No. 70 and 76 unlawful under the Corporation Law? If so, does the existence of this by-
law merit the dissolution of the corporation? (eighth cause of action)

No, these by-laws are entirely valid, and therefore, do not merit the dissolution of the corporation.

Article 70 of the by-laws in effect requires that persons elected to the board of directors must be holders of shares
of the paid up value of P5,000, which shall be held as security for their action; but it is added that said security
may be put up in the behalf of any director by some other holder of shares in the amount stated. Article 70 is
objectionable in that, under the requirement for security, a poor member, or wage-earner, cannot serve as
director, irrespective of other qualifications, and that as a matter of fact only men of means actually sit on the
board.

Article 76 of the by-laws declares that the directors waive their right as shareholders to receive loans from the
association. Article 76 is criticized on the ground that the provision requiring directors to renounce their right to
loans unreasonably limits their rights and privileges as members.

Section 21 of the Corporation Law expressly gives the power to the corporation to provide in its by-laws for the
qualifications of directors; and the requirement of security from them for the proper discharge of the duties of their
office, in the manner prescribed in article 70, is highly prudent and in conformity with good practice. Article 76,
prohibiting directors from making loans to themselves, is of course designed to prevent the possibility of the
looting of the corporation by unscrupulous directors.

Is it prohibited that the corporation maintain an excessive reserve of funds? Is it unlawful for the
board of directors to settle on paying a straight annual dividend of 10 percent, regardless of losses
suffered and profits made by the corporation? (eleventh and twelfth cause of action)

No, the suggested interpretation of Sec. 188 of the Corporation law is too strict and literal.

The statute provides that profits and losses shall be annually apportioned among the shareholders it is argued that
all earnings should be distributed without carrying anything to the reserve. It will be noted that it is provided in the
same section that the profits and losses shall be determined by the board of directors; and this means that they
shall exercise the usual discretion of good businessmen in allocating a portion of the annual profits to purposes
needful to the welfare of the association. The law contemplates the distribution of earnings and losses after other
CORPORATION LAW CASE DIGESTS | 63
FEU JD4401 | 2nd SEMESTER, S.Y. 2017-2018

legitimate obligations have been met. The Supreme Court’s conclusion is that the respondent has the power to
maintain its reserves; and at any rate, if it be supposed that the reserves referred to have become excessive, the
remedy is in the hands of the Legislature. It is no proper function of the court to arrogate to itself the control of
administrative matters which have been confided to the discretion of the board of directors.

Upon the expiration of the franchise of the association through the effluxion of time, or earlier
liquidation of its business, do the accumulated reserves and other properties accrue to the founder or
his heirs, and then to the board of directors and to those who are ordinary and special shareholders?
(fifteenth cause of action)

It depends on what is stated in the bylaws. The matter is best left to the decision of the directors or to legislative
action if it is deemed expedient to require the gradual suppression of the reserve funds as the time for dissolution
approaches.
In this case, Art. 95 of the bylaws of the corporation states that “the funds obtained by the liquidation of the
association shall be applied in the first place to the repayment of shares and the balance, if any, shall be
distributed in accordance with the system established for the distribution of annual profits.”

Is it permissible that El Hogar made various loans, now outstanding, to corporations and partnerships,
and that these entities have, in some instances, subscribed to shares in El Hogar for the sole purpose of
obtaining those loans? (sixteenth cause of action)

Yes, it is permissible.

Section 173 of the Corporation Law it is declared that "any person" may become a stockholder in building and loan
associations. The word "person" appears to be here used in its general sense, and there is nothing in the context to
indicate that the expression is used in the restricted sense of "natural person." It should therefore be taken to
include both natural and artificial persons. The mere motive with which subscriptions are made, whether to qualify
the stockholders to take a loan or for some other reason, is of no moment in determining whether the subscribers
were competent to make the contracts.

Note: There are seventeen causes of action in this case, but not all relate to corporate law. I have tried to include
only the relevant portions of the case here.
CORPORATION LAW CASE DIGESTS | 64
FEU JD4401 | 2nd SEMESTER, S.Y. 2017-2018

(38) CHINA BANKING CORPORATION v. COURT OF APPEALS and VALLEY GOLF and COUNTRY CLUB,
INC.
G.R. No. 117604 March 26, 1997
By: Valencia, Mary Clydeen L.

DOCTRINE: The better policy in determining which body has jurisdiction over a case would be to consider not only
the status or relationship of the parties but also the nature of the question that is the subject of their controversy. /
The purpose of a by-law is to regulate the conduct and define the duties of the members towards the corporation
and among themselves. They are self-imposed and, although adopted pursuant to statutory authority, have no
status as public law. (Ibid.) Therefore, it is the generally accepted rule that third persons are not bound by by-
laws, except when they have knowledge of the provisions either actually or constructively.

On 21 August 1974, Galicano Calapatia, Jr. (Calapatia, for brevity) a stockholder of private respondent Valley Golf
& Country Club, Inc. (VGCCI, for brevity), pledged his Stock Certificate No. 1219 to petitioner China Banking
Corporation (CBC, for brevity). On 16 September 1974, petitioner wrote VGCCI requesting that the aforementioned
pledge agreement be recorded in its books. In a letter dated 27 September 1974, VGCCI replied that the deed of
pledge executed by Calapatia in petitioner's favor was duly noted in its corporate books.

On 3 August 1983, Calapatia obtained a loan of P20,000.00 from petitioner, payment of which was secured by the
aforestated pledge agreement still existing between Calapatia and petitioner. Due to Calapatia's failure to pay his
obligation, petitioner, on 12 April 1985, filed a petition for extrajudicial foreclosure of the pledged stock before
Notary Public Antonio T. de Vera of Manila. On 14 May 1985, petitioner informed VGCCI of the above-mentioned
foreclosure proceedings and requested that the pledged stock be transferred to its (petitioner's) name and the
same be recorded in the corporate books. However, on 15 July 1985, VGCCI wrote petitioner expressing its
inability to accede to petitioner's request in view of Calapatia's unsettled accounts with the club. Despite the
foregoing, Notary Public de Vera held a public auction on 17 September 1985 and petitioner emerged as the
highest bidder.

On 21 November 1985, VGCCI sent Calapatia a notice demanding full payment of his overdue account in the
amount of P18,783.24. Said notice was followed by a demand letter dated 12 December 1985 for the same
amount9 and another notice dated 22 November 1986 for P23,483.24. On 4 December 1986, VGCCI caused to be
published in the newspaper Daily Express a notice of auction sale of a number of its stock certificates including
Calapatia's share of stock. Through a letter dated 15 December 1986, VGCCI informed Calapatia of the termination
of his membership due to the sale of his share of stock. On 5 May 1989, petitioner advised VGCCI that it is the new
owner of Calapatia's Stock Certificate No. 1219 by virtue of being the highest bidder in the 17 September 1985
auction and requested that a new certificate of stock be issued in its name.

On 9 March 1990, petitioner protested the sale by VGCCI of the subject share of stock and thereafter filed a case
with the Regional Trial Court of Makati for the nullification of the 10 December 1986 auction and for the issuance of
a new stock certificate in its name. On 18 June 1990, the Regional Trial Court of Makati dismissed the complaint for
lack of jurisdiction over the subject matter on the theory that it involves an intra-corporate dispute and on 27
August 1990 denied petitioner's motion for reconsideration.

On 20 September 1990, petitioner filed a complaint with the Securities and Exchange Commission (SEC) for the
nullification of the sale of Calapatia's stock by VGCCI. On 3 January 1992, SEC Hearing Officer Manuel P. Perea
rendered a decision in favor of VGCCI. On 14 April 1992, Hearing Officer Perea denied petitioner's motion for
reconsideration. Petitioner appealed to the SEC en banc and on 4 June 1993, the Commission issued an order
reversing the decision of its hearing officer. VGCCI sought reconsideration, but the smae was denied.

On 15 August 1994, the Court of Appeals rendered its decision nullifying and setting aside the orders of the SEC
and its hearing officer on ground of lack of jurisdiction over the subject matter. Petitioner moved for
reconsideration, but the same was denied by the Court of Appeals in its resolution dated 5 October 1994.

Which body has jurisdiction over the controversy? The regular courts or the SEC?

SEC has jurisdiction over the controversy. P. D. No. 902-A was expounded upon in Viray v. CA and in the recent
cases of Mainland Construction Co., Inc. v. Movilla and Bernardo v. CA, thus: “The better policy in determining
which body has jurisdiction over a case would be to consider not only the status or relationship of the parties but
also the nature of the question that is the subject of their controversy.”

Applying the foregoing principles in the case instant case, to ascertain which tribunal has jurisdiction we have to
determine therefore whether or not petitioner is a stockholder of VGCCI and whether or not the nature of the
controversy between petitioner and private respondent corporation is intra-corporate.

There is no question that the purchase of the subject share or membership certificate at public auction by
petitioner (and the issuance to it of the corresponding Certificate of Sale) transferred ownership of the same to the
CORPORATION LAW CASE DIGESTS | 65
FEU JD4401 | 2nd SEMESTER, S.Y. 2017-2018

latter and thus entitled petitioner to have the said share registered in its name as a member of VGCCI. It is readily
observed that VGCCI did not assail the transfer directly and has in fact, in its letter of 27 September 1974,
expressly recognized the pledge agreement executed by the original owner, Calapatia, in favor of petitioner and
has even noted said agreement in its corporate books. In addition, Calapatia, the original owner of the subject
share, has not contested the said transfer. By virtue of the afore-mentioned sale, petitioner became a bona fide
stockholder of VGCCI and, therefore, the conflict that arose between petitioner and VGCCI aptly exemplifies an
intra-corporate controversy between a corporation and its stockholder under Sec. 5(b) of P.D. 902-A. Moreover, an
important consideration is the nature of the controversy between petitioner and private respondent corporation.
VGCCI claims a prior right over the subject share anchored mainly on Sec. 3, Art VIII of its by-laws which provides
that "after a member shall have been posted as delinquent, the Board may order his/her/its share sold to satisfy
the claims of the Club. . ." It is pursuant to this provision that VGCCI also sold the subject share at public auction,
of which it was the highest bidder. VGCCI caps its argument by asserting that its corporate by-laws should prevail.
The bone of contention, thus, is the proper interpretation and application of VGCCI's aforequoted by-laws, a
subject which irrefutably calls for the special competence of the SEC.

The need for the SEC's technical expertise cannot be over-emphasized involving as it does the meticulous analysis
and correct interpretation of a corporation's by-laws as well as the applicable provisions of the Corporation Code in
order to determine the validity of VGCCI's claims. The SEC, therefore, took proper cognizance of the instant case.

Does VGCCI have the right to sell the share in question in accordance with the express provision found
in its by-laws? Is petitioner bound by the by-laws of respondent corporation?

NO. By-laws signifies the rules and regulations or private laws enacted by the corporation to regulate, govern and
control its own actions, affairs and concerns and its stockholders or members and directors and officers with
relation thereto and among themselves in their relation to it. In other words, by-laws are the relatively permanent
and continuing rules of action adopted by the corporation for its own government and that of the individuals
composing it and having the direction, management and control of its affairs, in whole or in part, in the
management and control of its affairs and activities. (9 Fletcher 4166, 1982 Ed.) The purpose of a by-law is to
regulate the conduct and define the duties of the members towards the corporation and among themselves. They
are self-imposed and, although adopted pursuant to statutory authority, have no status as public law. (Ibid.)
Therefore, it is the generally accepted rule that third persons are not bound by by-laws, except when they have
knowledge of the provisions either actually or constructively.

In the instant case, appellant-petitioner bank as a third party cannot be bound by appellee-respondent's by-laws. It
must be recalled that when appellee-respondent communicated to appellant-petitioner bank that the pledge
agreement was duly noted in the club's books there was no mention of the shareholder-pledgor's unpaid accounts.
The transcript of stenographic notes of the 25 June 1991 Hearing reveals that the pledgor became delinquent only
in 1975. Thus, appellant-petitioner was in good faith when the pledge agreement was contracted. For the exception
to the general accepted rule that third persons are not bound by by-laws to be applicable and binding upon the
pledgee, knowledge of the provisions of the VGCI By-laws must be acquired at the time the pledge agreement was
contracted. Knowledge of said provisions, either actual or constructive, at the time of foreclosure will not affect
pledgee's right over the pledged share.

To sum up, in order to be bound, the third party must have acquired knowledge of the pertinent by-laws at the
time the transaction or agreement between said third party and the shareholder was entered into, at the time the
pledge agreement was executed. VGCCI could have easily informed petitioner of its by-laws when it sent notice
formally recognizing petitioner as pledgee of one of its shares registered in Calapatia's name. Petitioner's belated
notice of said by-laws at the time of foreclosure will not suffice.
CORPORATION LAW CASE DIGESTS | 66
FEU JD4401 | 2nd SEMESTER, S.Y. 2017-2018

4. POWERS OF CORPORATIONS.

(39) Sy vs. Gutierrez


G.R. No. 171579. November 14, 2012
By: Alba, Ma. Angela

Facts: Petitioner Lily Sy claimed that private respondents went to petitioner’s residence at the 10th Floor, Fortune
Wealth, 612 Elcano St., Binondo, Manila and forcibly opened the door, destroyed and dismantled the door lock then
replaced it with a new one, without petitioner’s consent. Glenn and Benito’s act of replacing the door lock appeared
to be authorized by a resolution of Fortune Wealth Mansion Corporation’s Board of Directors, namely, respondents
Glenn, Jennifer, William Sy, Merlyn Sy, and Merry Sy. On that same day, petitioner supposedly saw Benito, Glenn,
Jennifer, Merry and respondent Berthold Lim took from her residence numerous boxes containing her personal
belongings without her consent and, with intent to gain, load them inside a family-owned van/truck named “Wheels
in Motion.”

Private respondents claimed that petitioner’s accusations were brought about by the worsening state of their
personal relationship because of misunderstanding on how to divide the estate of their deceased father. They also
pointed out that the whole condominium building where the alleged residence of petitioner is located, is owned and
registered in the name of the corporation. They explained that the claimed residence was actually the former
residence of their family (including petitioner). After their parents’ death, the corporation allegedly tolerated
petitioner to continuously occupy said unit while they, in turn, stayed in the other vacant units leaving some of
their properties and those of the corporation in their former residence. They further stated that petitioner
transferred to the ground floor because the 10th floor’s electric service was disconnected. They explained that they
changed the unit’s door lock to protect their personal belongings and those of the corporation as petitioner had
initially changed the original lock. They supported their authority to do so with a board resolution duly issued by
the directors.

Assistant City Prosecutor Tating recommended that private respondents be charged with Robbery In An
Uninhabited Place. Upon order of the RTC for reinvestigation, ACP Tating sustained its earlier conclusion. When
elevated before the Secretary of Justice, the ACP’s conclusions were reversed and set aside. The Secretary opined
that the elements of robbery were not present, since there was no violence against or intimidation of persons, or
force upon things, as the replacement of the door lock was authorized by a board resolution. Petitioner’s motion for
reconsideration was denied.

Aggrieved, petitioner went up to the Court of Appeals. The Court granted the petition and, consequently, set aside
the assailed Secretary’s Resolutions and reinstated the OCP’s Resolution. On motion of respondents, the CA
rendered an Amended Decisio setting aside its earlier decision and reinstating the DOJ Secretaries’ Resolutions. It
concluded that as part-owner of the entire building and of the articles allegedly stolen from the subject residential
unit, the very same properties involved in the pending estate proceedings, respondents cannot, as co-owners, steal
what they claim to own and thus cannot be charged with robbery. It continued and held that assuming that the
door was forced open, the same cannot be construed as an element of robbery as such was necessary due to
petitioner’s unjustified refusal to allow the other co-owners to gain access to the premises even for the lawful
purpose of allowing prospective buyers to have a look at the building. Petitioner’s motion for reconsideration was
denied. Hence, this petition.

Issue: Whether respondents could be charged with robbery notwithstanding the fact that there was a board
resolution authorizing them to do the acts.

Held:
No. The evidence presented was not sufficient to support a finding of probable cause.

Respondents were charged with robbery in an uninhabited place, which was later amended to reflect the facts as
alleged in the complaint that the robbery was committed in an inhabited place and that it was committed through
force upon things. To constitute robbery, the following elements must be established:

(1) The subject is personal property belonging to another;


CORPORATION LAW CASE DIGESTS | 67
FEU JD4401 | 2nd SEMESTER, S.Y. 2017-2018

(2) There is unlawful taking of that property;


(3) The taking is with the intent to gain; and
(4) There is violence against or intimidation of any person or use of force upon things.

Admittedly, the subject 10th floor unit is owned by the corporation and served as the family residence prior to the
death of petitioner and respondents’ parents. The 10th floor unit, including the personal properties inside, is the
subject of estate proceedings pending in another court and is, therefore, involved in the disputed claims among the
siblings (petitioner and respondents). Respondents admitted that armed with a Board Resolution authorizing them
to break open the door lock system of said unit and to install a new door lock system, they went up to the subject
unit to implement said resolution. The said corporate action was arrived at because petitioner had allegedly
prevented prospective buyers from conducting ocular inspection.

Petitioner, however, claims that respondents brought out from the unit 34 boxes containing her personal
belongings worth more than P10 million. Assuming that respondents indeed took said boxes containing personal
belongings, said properties were taken under claim of ownership which negates the element of intent to gain.
Taking as an element of robbery means depriving the offended party of ownership of the thing taken with the
character of permanency. The taking should not be under a claim of ownership. Thus, one who takes the property
openly and avowedly under claim of title offered in good faith is not guilty of robbery even though the claim of
ownership is untenable.

In this case, it was shown that respondents believed in good faith that they and the corporation own not only the
subject unit but also the properties found inside. If at all, they took them openly and avowedly under that claim of
ownership.
CORPORATION LAW CASE DIGESTS | 68
FEU JD4401 | 2nd SEMESTER, S.Y. 2017-2018

(40) Violeta Banate v. Philippine Countryside Rural Bank


G.R. No. 163825, July 13, 2010
By; Arid, Hannah Mhae G.

Topic: Powers of Stockholders and its delegation; Doctrine of Apparent Authority

Doctrine: Under the doctrine of apparent authority, acts and contracts of the agent, as are within the apparent
scope of the authority conferred on him, although no actual authority to do such acts or to make such contracts
has been conferred, bind the principal. The principals liability, however, is limited only to third persons who have
been led reasonably to believe by the conduct of the principal that such actual authority exists, although none was
given. In other words, apparent authority is determined only by the acts of the principal and not by the acts of the
agent. It is a settled rule that persons dealing with an agent are bound at their peril, if they would hold the
principal liable, to ascertain not only the fact of agency but also the nature and extent of the agents authority, and
in case either is controverted, the burden of proof is upon them to establish it.

Facts: Petitioner Spouses Maglasang obtained a loan from PCRB for P1,070,000.00. The subject loan was
evidenced by a promissory note and was payable on January 18, 1998. To secure the payment of the subject loan,
the spouses Maglasang executed, in favor of PCRB a real estate mortgage over their property, including the house
constructed thereon owned by petitioners Spouses Cortel, the spouses Maglasangs daughter and son-in-law. Aside
from the subject loan, the spouses Maglasang obtained two other loans from PCRB which were covered by separate
promissory notes and secured by mortgages on their other properties. Before the subject loan became due, the
spouses Maglasang and the spouses Cortel asked PCRBs permission to sell the subject properties. They likewise
requested that the subject properties be released from the mortgage since the two other loans were adequately
secured by the other mortgages. The spouses Maglasang and the spouses Cortel claimed that the PCRB, acting
through its Branch Manager, Pancrasio Mondigo, verbally agreed to their request but required first the full payment
of the subject loan. The spouses Maglasang and the spouses Cortel thereafter sold to petitioner Violeta Banate the
subject properties. The spouses Magsalang and the spouses Cortel used the amount to pay the subject loan with
PCRB. After settling the subject loan, PCRB gave the owners duplicate certificate of title of Lot 12868-H-3-C to
Banate, who was able to secure a new title in her name. The title, however, carried the mortgage lien in favor of
PCRB, prompting the petitioners to request from PCRB a Deed of Release of Mortgage. As PCRB refused to comply
with the petitioners request, the petitioners instituted an action for specific performance before the RTC to compel
PCRB to execute the release deed.

PCRB countered the petitioners allegations by invoking the cross-collateral stipulation in the mortgage deed which
states:
1. That as security for the payment of the loan or advance in principal
sum of one million seventy thousand pesos only (P1,070,000.00) and such other loans
or advances already obtained, or still to be obtained by the MORTGAGOR(s) as
MAKER(s), CO-MAKER(s) or GUARANTOR(s) from the MORTGAGEE plus interest at the
rate of _____ per annum and penalty and litigation charges payable on the dates
mentioned in the corresponding promissory notes, the MORTGAGOR(s) hereby
transfer(s) and convey(s) to MORTGAGEE by way of first mortgage the parcel(s) of land
described hereunder, together with the improvements now existing for which may
hereafter be made thereon, of which MORTGAGOR(s) represent(s) and warrant(s) that
MORTGAGOR(s) is/are the absolute owner(s) and that the same is/are free from all liens
and encumbrances;

Accordingly, PCRB claimed that full payment of the three loans, obtained by the spouses Maglasang, was necessary
before any of the mortgages could be released; the settlement of the subject loan merely constituted partial
payment of the total obligation. Thus, the payment does not authorize the release of the subject properties from
the mortgage lien. PCRB considered Banate as a buyer in bad faith as she was fully aware of the existing mortgage
in its favor when she purchased the subject properties from the spouses Maglasang and the spouses Cortel. It
explained that it allowed the release of the owners duplicate certificate of title to Banate only to enable her to
annotate the sale. PCRB claimed that the release of the title should not indicate the corresponding release of the
subject properties from the mortgage constituted thereon.

RTC: Ruled in favor of the petitioners. It noted that the petitioners, as necessitous men, could not have bargained
on equal footing with PCRB in executing the mortgage, and concluded that it was a contract of adhesion. Therefore,
any obscurity in the mortgage contract should not benefit PCRB.

CA: Reversed the RTC’s decision. The CA did not consider as valid the petitioners new agreement with Mondigo,
which would novate the original mortgage contract containing the cross-collateral stipulation. It ruled that Mondigo
cannot orally amend the mortgage contract between PCRB, and the spouses Maglasang and the spouses Cortel;
therefore, the claimed commitment allowing the release of the mortgage on the subject properties cannot bind
PCRB. Since the cross-collateral stipulation in the mortgage contract (requiring full settlement of all three loans
before the release of any of the mortgages) is clear, the parties must faithfully comply with its terms.
CORPORATION LAW CASE DIGESTS | 69
FEU JD4401 | 2nd SEMESTER, S.Y. 2017-2018

Issue: a.) Did the purported agreement between the petitioners and Mondigo novate the mortgage contract over
the subject properties and is thus binding upon PCRB?

b.) If the first issue is resolved negatively, can Banate demand restitution of the amount paid for the subject
properties on the theory that the new agreement with Mondigo is deemed rescinded?

Held: a.) The purported agreement did not novate the mortgage contract, particularly the cross-
collateral stipulation thereon. As a general rule, a mortgage liability is usually limited to the amount mentioned
in the contract. However, the amounts named as consideration in a contract of mortgage do not limit the amount
for which the mortgage may stand as security if, from the four corners of the instrument, the intent to secure
future and other indebtedness can be gathered. This stipulation is valid and binding between the parties and is
known as the blanket mortgage clause (also known as the dragnet clause). In the present case, the mortgage
contract indisputably provides that the subject properties serve as security, not only for the payment of the subject
loan, but also for such other loans or advances already obtained, or still to be obtained. The cross-collateral
stipulation in the mortgage contract between the parties is thus simply a variety of a dragnet clause. After agreeing
to such stipulation, the petitioners cannot insist that the subject properties be released from mortgage since the
security covers not only the subject loan but the two other loans as well. Novation would have dual functions one
to extinguish an existing obligation, the other to substitute a new one in its place requiring a conflux of four
essential requisites: (1) a previous valid obligation; (2) an agreement of all parties concerned to a new contract;
(3) the extinguishment of the old obligation; and (4) the birth of a valid new obligation. The second requisite is
lacking in this case. Novation presupposes not only the extinguishment or modification of an existing obligation
but, more importantly, the creation of a valid new obligation. For the consequent creation of a new contractual
obligation, consent of both parties is, thus, required. As a general rule, no form of words or writing is necessary to
give effect to a novation. Nevertheless, where either or both parties involved are juridical entities, proof that the
second contract was executed by persons with the proper authority to bind their respective principals is necessary.

Section 23 of the Corporation Code expressly provides that the corporate powers of all corporations
shall be exercised by the board of directors. The power and the responsibility to decide whether the
corporation should enter into a contract that will bind the corporation are lodged in the board, subject
to the articles of incorporation, bylaws, or relevant provisions of law. In the absence of authority from
the board of directors, no person, not even its officers, can validly bind a corporation.However, just as
a natural person may authorize another to do certain acts for and on his behalf, the board of directors
may validly delegate some of its functions and powers to its officers, committees or agents. The
authority of these individuals to bind the corporation is generally derived from law, corporate bylaws
or authorization from the board, either expressly or impliedly by habit, custom or acquiescence in the
general course of business.

The authority of a corporate officer or agent in dealing with third persons may be actual or apparent.
Actual authority is either express or implied. The extent of an agents express authority is to be
measured by the power delegated to him by the corporation, while the extent of his implied authority is
measured by his prior acts which have been ratified or approved, or their benefits accepted by his
principal. The existence of apparent authority may be ascertained through:
1) the general manner in which the corporation holds out an officer or agent as
having the power to act, or in other words, the apparent authority to act in
general, with which it clothes him; or
2) the acquiescence in his acts of a particular nature, with actual or constructive
knowledge thereof, within or beyond the scope of his ordinary powers.

Accordingly, the authority to act for and to bind a corporation may be presumed from acts of
recognition in other instances when the power was exercised without any objection from its board or
shareholders.

Notably, the petitioners action for specific performance is premised on the supposed actual or apparent authority of
the branch manager, Mondigo, to release the subject properties from the mortgage, although the other obligations
remain unpaid. In light of our discussion above, proof of the branch managers authority becomes indispensable to
support the petitioners contention. The petitioners make no claim that Mondigo had actual authority from PCRB,
whether express or implied.

Under the doctrine of apparent authority, acts and contracts of the agent, as are within the apparent
scope of the authority conferred on him, although no actual authority to do such acts or to make such
contracts has been conferred, bind the principal. The principals liability, however, is limited only to
third persons who have been led reasonably to believe by the conduct of the principal that such actual
authority exists, although none was given. In other words, apparent authority is determined only by
the acts of the principal and not by the acts of the agent. There can be no apparent authority of an agent
without acts or conduct on the part of the principal; such acts or conduct must have been known and relied upon in
CORPORATION LAW CASE DIGESTS | 70
FEU JD4401 | 2nd SEMESTER, S.Y. 2017-2018

good faith as a result of the exercise of reasonable prudence by a third party as claimant, and such acts or conduct
must have produced a change of position to the third partys detriment.

In the present case, the decision of the trial court was utterly silent on the manner by which PCRB, as supposed
principal, has clothed or held out its branch manager as having the power to enter into an agreement, as claimed
by petitioners. No proof of the course of business, usages and practices of the bank about, or knowledge that the
board had or is presumed to have of, its responsible officers acts regarding bank branch affairs, was ever adduced
to establish the branch managers apparent authority to verbally alter the terms of mortgage contracts. Neither
was there any allegation, much less proof, that PCRB ratified Mondigos act or is estopped to make a contrary claim.

It is a settled rule that persons dealing with an agent are bound at their peril, if they would hold the
principal liable, to ascertain not only the fact of agency but also the nature and extent of the agents
authority, and in case either is controverted, the burden of proof is upon them to establish it. As parties
to the mortgage contract, the petitioners are expected to abide by its terms. The subsequent purported
agreement is of no moment, and cannot prejudice PCRB, as it is beyond Mondigos actual or apparent
authority, as above discussed.

b.) No, rescission has no legal basis; there can be no restitution of the amount paid. Even if we were to
assume that the purported agreement has been sufficiently established, since it is not binding on the bank for lack
of authority of PCRBs branch manager, then the prayer for restitution of the amount paid would have no legal
basis. Of course, it will be asked: what then is the legal significance of the payment made by Banate? Article 2154
of the Civil Code reads:
Art 2154. If something is received when there is no right to demand it, and it was
unduly delivered through mistake, the obligation to return it arises.

Notwithstanding the payment made by Banate, she is not entitled to recover anything from PCRB under Article
2154. There could not have been any payment by mistake to PCRB, as the check which Banate issued as payment
was to her co-petitioner Mary Melgrid Cortel (the payee), and not to PCRB. The same check was simply endorsed
by the payee to PCRB in payment of the subject loan that the Maglasangs owed PCRB. The mistake, if any, was in
the perception of the authority of Mondigo, as branch manager, to verbally alter the mortgage contract, and not as
to whether the Cortels, as sellers, were entitled to payment. This mistake (on Mondigos lack of authority to alter
the mortgage) did not affect the validity of the payment made to the bank as the existence of the loan was never
disputed. The dispute was merely on the effect of the payment on the security given.

Consequently, no right to recover accrues in Banates favor as PCRB never dealt with her. The borrowers-
mortgagors, on the other hand, merely paid what was really owed. Parenthetically, the subject loan was due on
January 18, 1998, but was paid sometime in November 1997. It appears, however, that at the time the complaint
was filed, the subject loan had already matured. Consequently, recovery of the amount paid, even under a claim of
premature payment, will not prosper.
CORPORATION LAW CASE DIGESTS | 71
FEU JD4401 | 2nd SEMESTER, S.Y. 2017-2018

(41) Riosa vs. Tabaco La Suerte Corporation


G.R. No. 203786. October 23, 2013
By: Bernardo, Gerard

Doctrine: The power to purchase real property is vested in the board of directors or trustees. While a corporation
may appoint agents to negotiate for the purchase of real property needed by the corporation, the final say will
have to be with the board, whose approval will finalize the transaction. A corporation can only exercise its powers
and transact its business through its board of directors and through its officers and agents when authorized by a
board resolution or its by-laws. Absent such valid delegation/authorization, the rule is that the declarations of an
individual director relating to the affairs of the corporation, but not in the course of, or connected with, the
performance of authorized duties of such director, are held not binding on the corporation.

Facts: Petitioner Aquiles Riosa (Aquiles) filed his Complaint for Annulment/Declaration of Nullity of Deed of
Absolute Sale and Transfer Certificate of Title, Reconveyance and Damages against respondent Tabaco La Suerte
Corporation (La Suerte) before the RTC.

Aquiles alleged that he was the owner and in actual possession of a 52-square meter commercial lot situated in
Barangay Quinale, Tabaco City, Albay.

Aquiles claimed that by means of fraud, misrepresentation and deceit employed by Sia Ko Pio, he was made to sign
the document which he thought was a receipt and undertaking to pay the loan, only to find out later that it was a
document of sale. Aquiles averred that he did not appear before the notary public to acknowledge the sale.

La Suerte averred that it was the actual and lawful owner of the commercial property after purchasing it from
Aquiles on December 7, 1990; that it allowed Aquiles to remain in possession of the property to avoid the ire of his
father from whom he had acquired property inter vivos, subject to his obligation to vacate the premises anytime
upon demand.

Issue: Whether or not La Suerte validly purchased the property of Aquiles.

Held: NO. Section 23 of the Corporation Code expressly provides that the corporate powers of all corporations
shall be exercised by the board of directors. Just as a natural person may authorize another to do certain acts in
his behalf, so may the board of directors of a corporation validly delegate some of its functions to individual officers
or agents appointed by it. Thus, contracts or acts of a corporation must be made either by the board of directors or
by a corporate agent duly authorized by the board.

Absent such valid delegation/authorization, the rule is that the declarations of an individual director relating to the
affairs of the corporation, but not in the course of, or connected with, the performance of authorized duties of such
director are held not binding on the corporation.

In the case at bench, Sia Ko Pio, although an officer of La Suerte, had no authority from its Board of Directors to
enter into a contract of sale of Aquiles’ property. It is, thus, clear that the loan obtained by Aquiles from Sia Ko Pio
was a personal loan from the latter, not a transaction between Aquiles and La Suerte. There was no evidence to
show that Sia Ko Pio was clothed with authority to use his personal fund for the benefit of La Suerte. Evidently, La
Suerte was never in the picture and was not the owner of the property in question.
CORPORATION LAW CASE DIGESTS | 72
FEU JD4401 | 2nd SEMESTER, S.Y. 2017-2018

(42) Lanuza Jr. vs. BF Corporation


G.R. No. 174938, October 1, 2014
By: Donna Bigornia

Doctrine: A consequence of a corporation’s separate personality is that consent by a corporation through its
representatives is not consent of the representative, personally. Its obligations, incurred through official acts of its
representatives, are its own. A stockholder, director, or representative does not become a party to a contract just
because a corporation executed a contract through that stockholder, director or representative.

Facts: In 1993, BF Corporation filed a collection complaint with the Regional Trial Court against Shangri-La and the
members of its board of directors: Alfredo C. Ramos, Rufo B.Colayco, Antonio O. Olbes, Gerardo Lanuza, Jr.,
Maximo G. Licauco III, and Benjamin C. Ramos. BF Corporation alleged in its complaint that on December 11, 1989
and May 30, 1991, it entered into agreements with Shangri-La wherein it undertook to construct for Shangri-La a
mall and a multilevel parking structure along EDSA.Shangri-La had been consistent in paying BF Corporation in
accordance with its progress billing statements. However, by October 1991, Shangri-La started defaulting in
payment. BF Corporation alleged that Shangri-La induced BF Corporation to continue with the construction of the
buildings using its own funds and credit despite Shangri-La’s default. According to BF Corporation, Shangri-La
misrepresented that it had funds to pay for its obligations with BF Corporation, and the delay in payment was
simply a matter of delayed processing of BF Corporation’s progress billing statements. BF Corporation eventually
completed the construction of the buildings. Shangri-La allegedly took possession of the buildings while still owing
BF Corporation an outstanding balance. BF Corporation alleged that despite repeated demands, Shangri-La refused
to pay the balance owed to it.It also alleged that the Shangri-La’s directors were in bad faith in directing Shangri-
La’s affairs. Therefore, they should be held jointly and severally liable with Shangri-La for its obligations as well as
for the damages that BF Corporation incurred as a result of Shangri-La’s default. On August 3, 1993, Shangri-La,
Alfredo C. Ramos, Rufo B. Colayco, Maximo G. Licauco III, and Benjamin C. Ramos filed a motion to suspend the
proceedings in view of BF Corporation’s failure to submit its dispute to arbitration, in accordance with the
arbitration clause provided in its contract. Petitioners filed their comment on Shangri-La’s and BF Corporation’s
motions, praying that they be excluded from the arbitration proceedings for being non-parties to Shangri-La’s and
BF Corporation’s agreement.

Issue: Whether or not petitioners as directors of Shangri-La is personally liable for the contractual obligations
entered into by the corporation.

Ruling: No. Because a corporation’s existence is only by fiction of law, it can only exercise its rights and powers
through its directors, officers, or agents, who are all natural persons. A corporation cannot sue or enter into
contracts without them.

A consequence of a corporation’s separate personality is that consent by a corporation through its representatives
is not consent of the representative, personally. Its obligations, incurred through official acts of its representatives,
are its own. A stockholder, director, or representative does not become a party to a contract just because a
corporation executed a contract through that stockholder, director or representative.

Hence, a corporation’s representatives are generally not bound by the terms of the contract executed by the
corporation. They are not personally liable for obligations and liabilities incurred on or in behalf of the corporation.
CORPORATION LAW CASE DIGESTS | 73
FEU JD4401 | 2nd SEMESTER, S.Y. 2017-2018

(43) Metropolitan Bank & Trust Co., v. Centro Development Corp


G.R. No. 1180974, June 13, 2012
By: Corona, Jose Enrico V.

Doctrine: The act of appointing a new trustee of the MTI was a regular business transaction. The appointment
necessitated only a decision of at least a majority of the directors present at the meeting in which there was a
quorum, pursuant to Section 25 of the Corporation Code.

Facts: On 20 March 1990, in a special meeting of the board of directors of respondent Centro Development
Corporation (Centro), its president Go Eng Uy was authorized to mortgage its properties and assets to secure the
medium-term loan of Lucky Two Corporation and Lucky Two Repacking. The properties and assets consisted of a
parcel of land with a building and improvements. This authorization was subsequently approved on the same day
by the stockholders.

A mortgage Trust Indenture with BPI was made. San Carlos was added as a borrower. Meanwhile, respondent
Centro, represented by Go Eng Uy and as authorized under Resolution 005,, approached petitioner Metropolitan
Bank and Trust Company (Metrobank) sometime in 1994 and proposed that the latter assume the role of
successor-trustee of the existing MTI. After petitioner Metrobank agreed to the proposal, the board of directors of
respondent Centro allegedly resolved on 12 August 1994 to constitute petitioner as successor-trustee of BPI.

It was only sometime in 1998 that respondents herein, Chongking Kehyeng, Manuel Co Kehyeng and Quirino
Kehyeng, allegedly discovered that the properties of respondent Centro had been mortgaged, and that the MTI
(Mortgage Trust Indenture) that had been executed appointing petitioner as trustee.

Meanwhile, during the period April 1998 to December 1998, San Carlos obtained loans in the total principal amount
of ₱812,793,513.23 from petitioner Metrobank. San Carlos defaulted causing the extrajudicial foreclosure of the
mortgage.

Before the scheduled foreclosure date, on 3 August 2000, respondents herein filed a Complaint for the annulment
of the 27 September 1994 MTI.

The bone of contention in Civil Case No. 00-942 was that since the mortgaged properties constituted all or
substantially all of the corporate assets, the amendment of the MTI failed to meet the requirements of Section 40
of the Corporation Code on notice and voting requirements. Under this provision, in order for a corporation to
mortgage all or substantially all of its properties and assets, it should be authorized by the vote of its stockholders
representing at least 2/3 of the outstanding capital stock in a meeting held for that purpose. Furthermore, there
must be a written notice of the proposed action and of the time and place of the meeting. Thus, respondents
alleged, the representation of Go Eng Uy that he was authorized by the board of directors and/or stockholders of
Centro was false.

RTC dismissed. CA ruled that the MTI is null and void.

Issues: Whether or not the requirements of Section 40 of the Corporation Code shall be complied with in the
alleged 2nd execution of the MTI.

Held: No.

Reading carefully the Secretary’s Certificate, it is clear that the main purpose of the directors Resolution was to
appoint petitioner as the new trustee of the previously executed and amended MTI. Going through the original and
the revised MTI, we find no substantial amendments to the provisions of the contract. We agree with petitioner that
the act of appointing a new trustee of the MTI was a regular business transaction. The appointment necessitated
only a decision of at least a majority of the directors present at the meeting in which there was a quorum, pursuant
to Section 25 of the Corporation Code.

The second paragraph of the directors Resolution No. 005, s. 1994, which empowered Go Eng Uy to sign the Real
Estate Mortgage and all documents/instruments with the said bank, for and in behalf of the Company which are
necessary and pertinent thereto, must be construed to mean that such power was limited by the conditions of the
existing mortgage, and not that a new mortgage was thereby constituted.

Thus, Section 40 of the Corporation Code finds no application in the present case, as there was no new mortgage
to speak of under the assailed directors Resolution.
CORPORATION LAW CASE DIGESTS | 74
FEU JD4401 | 2nd SEMESTER, S.Y. 2017-2018

(44) Y-I Leisure Philippines, Inc., vs James Yu


G.R. No. 207161. September 8, 2015
By: Kathrina De Castro

Topic: Nell Doctrine


DOCTRINE: The Nell Doctrine states the general rule that the transfer of all the assets of a corporation to another
shall not render the latter liable to the liabilities of the transferor. The exception of the Nell doctrine, which finds its
legal basis under Section 40, provides that the transferee corporation assumes the debts and liabilities of the
transferor corporation because it is merely a continuation of the latter’s business

MADCI was a real estate development corporation. It offers shares of golf and country club located in the vicinity of
Mt. Arayat for sale. Respondent Yu bought 500 golf and 150 country club shares for P650,000 which is paid in
installments with 14 FEBTC checks. Upon full payment, Yu visited the site but such was non-existent. He demanded
recovery of his payment but MADCI recognized that Yu had investment but Yu did not receive any refund. Yu filed
complaint for collection of sum of money against the president of MADCI and MADCI itself. Yu alleged that he dealt
with Sangil, who used MADCI’s corporate personality to defraud him. As defense, Sangil alleged that Yu dealt with
MADCI as a juridical person and that he did not benefit from the sale of shares. He added that the return of Yu’s
money was no longer possible because its approval had been blocked by the new set of officers of MADCI, which
controlled the majority of its board of directors. In its Answer, MADCI claimed that it was Sangil who defrauded YU.
According to MADCI it failed to develop the golf course because its properties were taken over by YIL after he
allegedly violated the MOA. The assets of MADCI had been transferred to YILPI and YICRI.

A. WON petitioners should be held jointly and severally liable to respondent Yu

Respondent Yu bought several golf and country club shares from MADCI. Regrettably, the latter did not develop the
supposed project. Yu then demanded the return of his payment, but MADCI could not return it anymore because all
its assets had been transferred. Through the acts of YIL, MADCI sold all its lands to YILPI and, subsequently to
YICRI. Thus, Yu now claims that the petitioners inherited the obligations of MADCI. On the other hand, the
petitioners counter that they did not assume such liabilities because the transfer of assets was not committed in
fraud of the MADCI’s creditors.

Generally, where one corporation sells or otherwise transfers all of its assets to another corporation, the latter is
not liable for the debts and liabilities of the transferor, except:
1. Where the purchaser expressly or impliedly agrees to assume such debts;
2. Where the transaction amounts to a consolidation or merger of the corporations;
3. Where the purchasing corporation is merely a continuation of the selling corporation; and
4. Where the transaction is entered into fraudulently in order to escape liability for such debts.

The Nell Doctrine states the general rule that the transfer of all the assets of a corporation to another shall not
render the latter liable to the liabilities of the transferor. If any of the above cited exceptions are present, then the
transferee corporation shall assume the liabilities of the transferor.

The first exception under the Nell Doctrine is where the transferee corporation expressly or impliedly agrees to
assume the transferor’s debts. The second exception under the doctrine is the merger and consolidation of
corporations. Another exception of the doctrine is where the sale of all corporate assets is entered into fraudulently
to escape liability for transferor’s debts.

In other words, in this last exception, the transferee purchases not only the assets of the transferor, but also its
business. As a result of the sale, the transferor is merely left with its juridical existence, devoid of its industry and
earning capacity. Fittingly, the proper provision of law that is contemplated by this exception would be Section 40
of the Corporation Code.

Sec. 40. Sale or other disposition of assets.—Subject to the provisions of existing laws on illegal combinations and
monopolies, a corporation may, by a majority vote of its board of directors or trustees, sell, lease, exchange,
mortgage, pledge or otherwise dispose of all or substantially all of its property and assets, including its goodwill
xxx
Section 40 suitably reflects the business-enterprise transfer under the exception of the Nell Doctrine because the
purchasing or transferee corporation necessarily continued the business of the selling or transferor corporation.
CORPORATION LAW CASE DIGESTS | 75
FEU JD4401 | 2nd SEMESTER, S.Y. 2017-2018

Given that the transferee corporation acquired not only the assets but also the business of the transferor
corporation, then the liabilities of the latter are inevitably assigned to the former. It must be clarified, however,
that not every transfer of the entire corporate assets would qualify under Section 40. It does not apply (1) if the
sale of the entire property and assets is necessary in the usual and regular course of business of corporation, or (2)
if the proceeds of the sale or other disposition of such property and assets will be appropriated for the conduct of
its remaining business. Thus, the litmus test to determine the applicability of Section 40 would be the capacity of
the corporation to continue its business after the sale of all or substantially all its assets.

The exception of the Nell doctrine, which finds its legal basis under Section 40, provides that the transferee
corporation assumes the debts and liabilities of the transferor corporation because it is merely a continuation of the
latter’s business. A cursory reading of the exception shows that it does not require the existence of fraud against
the creditors before it takes full force and effect. Indeed, under the Nell Doctrine, the transferee corporation may
inherit the liabilities of the transferor despite the lack of fraud due to the continuity of the latter’s business. Based
on the foregoing, as the exception of the Nell doctrine relates to the protection of the creditors of the
transferor corporation, and does not depend on any deceit committed by the transferee corporation,
then fraud is certainly not an element of the business enterprise doctrine.
CORPORATION LAW CASE DIGESTS | 76
FEU JD4401 | 2nd SEMESTER, S.Y. 2017-2018

(45) Balinghasay v. Castillo


G.R. No. 185664, April 8, 2015
By: Gaite, Rhio Angeline

Doctrine: The main objective of the principle against unjust enrichment is to prevent one from enriching himself at
the expense of another without just cause or consideration. In the case at bar, the ultrasound investors pooled
together the amount of P850,000.00, which was used to purchase the equipment. Because of the MOA’s invalidity,
the ultrasound investors can no longer operate the ultrasound unit within MCP. Nonetheless, it is only fair for the
ultrasound investors to retain ownership of the equipment, which they may use or dispose of independently of
MCPI.

Facts: The MCPI, a domestic corporation organized in 1977, operates the Medical Center Parañaque (MCP).
Castillo, Oscar, Flores, Navarro, and Templo are minority stockholders of MCPI. Each of them holds 25 Class B
shares. On the other hand, nine of the herein petitioners, namely, Balinghasay, Bernabe, Alodia, Jimenez,
Oblepias, Savet, Villamora, Valdez and Villareal, are holders of Class A shares and were Board Directors of MCPI.
The other eight petitioners are holders of Class B shares. The petitioners are part of a group who invested in the
purchase of ultrasound equipment, the operation of and earnings from which gave rise to the instant controversy.

Before 1997, the laboratory, physical therapy, pulmonary and ultrasound services in MCP were provided to patients
by way of concessions granted to independent entities. When the concessions expired in 1997, MCPI decided that it
would provide on its own the said services, except ultrasound. In 1997, the MCPI’s Board of Directors awarded the
operation of the ultrasound unit to a group of investors (ultrasound investors) composed mostly of Obstetrics-
Gynecology (Ob-gyne) doctors. The ultrasound investors held either Class A or Class B shares of MCPI. Among
them were nine of the herein petitioners, who were then, likewise, MCPI Board Directors. The group purchased a
Hitachi model EUB-200 C ultrasound equipment costing P850,000.00 and operated the same. Albeit awarded by
the Board of Directors, the operation was not yet covered by a written contract.

In the meeting of the MCPI’s Board of Directors held in 1998, 7 of the 12 Directors present were part of the
ultrasound investors. The Board Directors made a counter offer anent the operation of the ultrasound unit. Hence,
essentially then, the award of the ultrasound operation still bore no formal stamp of approval.

In 1999, 12 Board Directors attended the Board meeting and 8 of them were among the ultrasound investors. A
Memorandum of Agreement (MOA) was entered into by and between MCPI, represented by its President then,
Bernabe, and the ultrasound investors, represented by Oblepias. Per MOA, the gross income to be derived from the
operation of the ultrasound unit, minus the sonologists’ professional fees, shall be divided between the ultrasound
investors and MCPI, in the proportion of 60% and 40%, respectively. Come April 1, 1999, MCPI’s share would be
45%, while the ultrasound investors would receive 55%. Further, the ownership of the ultrasound machine would
eventually be transferred to MCPI.

Flores wrote MCPI’s counsel a letter challenging the Board of Directors’ approval of the MOA for being prejudicial to
MCPI’s interest. Thereafter, Flores manifested to MCPI’s Board of Directors and President his view regarding the
illegality of the MOA, which, therefore, cannot be validly ratified.

In 2001, the herein respondents filed with the RTC a derivative suit against the petitioners for violation of Section
31 of the Corporation Code. Among the prayers in the Complaint were: (a) the annulment of the MOA and the
accounting of and refund by the petitioners of all profits, income and benefits derived from the said agreement;
and (b) payment of damages and attorney’s fees.

In their Answer with Counterclaim, the petitioners argued that the derivative suit must be dismissed for non-
joinder of MCPI, an indispensable party. The petitioners likewise claimed that under Section 32 of the Corporation
Code, the MOA was merely voidable. Since there was no proof that the subsequent Board of Directors of MCPI
moved to annul the MOA, the same should be considered as having been ratified. Hence, as to which equity
imposes a disability upon him to deal in his own behalf, he shall be liable as a trustee for the corporation and must
account for the profits which otherwise would have accrued to the corporation.

Issues: (1) Whether the MOA into by and between MCPI was valid
(2) Whether the ultrasound investors should retain ownership of the equipment

Held: (1) No. As acknowledged by the petitioners and aptly pointed out by the respondents, the existence of the
circumstances and urgent hospital necessity justifying the purchase and operation of the ultrasound unit by the
investors were not at the outset offered as evidence. Having been belatedly raised, the aforesaid defenses were not
scrutinized during the trial and their truth or falsity was not uncovered. This is fatal to the petitioners’ cause. The
CA thus cannot be faulted for ruling against the petitioners in the face of evidence showing that: (a) there was no
quorum when the Board meetings were held on August 14, 1998 and February 5, 1999; (b) the MOA was not
CORPORATION LAW CASE DIGESTS | 77
FEU JD4401 | 2nd SEMESTER, S.Y. 2017-2018

ratified by a vote of two-thirds of MCPI’s outstanding capital stock; and (c) the Balance Sheets for the years 1996
to 2000 indicated that MCPI was in a financial position to purchase the ultrasound equipment.

The petitioners harp on their lofty purpose, which had supposedly moved them to purchase and operate the
ultrasound unit. Unfortunately, their claims are not evident in the records. Further, even if their claims were to be
assumed as true for argument’s sake, the fact remains that the Board Directors, who approved the MOA, did not
outrightly inform the stockholders about it. The ultrasound equipment was purchased and had been in operation
since 1997, but the matter was only brought up for ratification by the stockholders in the annual meetings held in
the years 2000 to 2003. This circumstance lends no credence to the petitioners’ cause. The Court thus finds the
CA’s ruling anent the invalidity of the MOA as amply supported by both evidence and jurisprudence.

(2) Yes. Article 22 of the New Civil Code provides that “every person who through an act of performance by
another, or any other means, acquires or comes into possession of something at the expense of the latter without
just or legal ground, shall return the same to him.” The main objective of the principle against unjust enrichment is
to prevent one from enriching himself at the expense of another without just cause or consideration.

In the case at bar, the ultrasound investors pooled together the amount of P850,000.00, which was used to
purchase the equipment. Because of the MOA’s invalidity, the ultrasound investors can no longer operate the
ultrasound unit within MCP. Nonetheless, it is only fair for the ultrasound investors to retain ownership of the
equipment, which they may use or dispose of independently of MCPI.
CORPORATION LAW CASE DIGESTS | 78
FEU JD4401 | 2nd SEMESTER, S.Y. 2017-2018

5. STOCKS AND STOCKHOLDERS

(46) GRACE BORGONA INSIGNE vs. ABRA VALLEY COLLEGES, INC.


G.R. No. 204089 July 29, 2015
By: Grande, Jonicocel

Topic: inspection of stock and transfer book; determination of a shareholder

Doctrine: A stock certificate is prima facie evidence that the holder is a shareholder of the corporation, but the
possession of the certificate is not the sole determining factor of one’s stock ownership. A certificate of stock is
merely the paper representative or tangible evidence of the stock itself and of the various interests therein.

Facts: In his lifetime, Pedro was the founder, president and majority stockholder of respondent (Abra Valley), a
stock corporation. After Pedro’s death, Francis (son with his frist wife) succeeded him as the president of Abra
Valley.
The petitioners children to his second marriage, filed a complaint against Abra Valley to allow them to inspect its
corporate books and records, and the minutes of meetings, and to provide them with its financial statements

Issue: Whether or not the petitioners are stockholders of the corporation.

Held: YES.

Petitioners were stockholders of Abra Valley. Their non-production of their stock certificate is not the sole
determining factor of their stock ownership.

A stock certificate is prima facie evidence that the holder is a shareholder of the corporation, but the possession of
the certificate is not the sole determining factor of one’s stock ownership. A certificate of stock is merely the paper
representative or tangible evidence of the stock itself and of the various interests therein. The certificate is not
stock in the corporation but is merely evidence of the holder's interest and status in the corporation, his ownership
of the share represented thereby, but is not in law the equivalent of such ownership. It expresses the contract
between the corporation and the stockholder, but it is not essential to the existence of a share in stock or the
creation of the relation of shareholder to the corporation.

To establish their stock ownership, the petitioners actually turned over to the trial court through their Compliance
and Manifestation the various documents showing their ownership of Abra Valley’s shares, specifically: the official
receipts of their payments for their subscriptions of the shares of Abra Valley; and the copies duly certified by the
Securities and Exchange Commission (SEC) stating that Abra Valley had issued shares in favor of the petitioners,
such as the issuance of part of authorized and unissued capital stock; the letter dated June 17, 1987; the
secretary’s certificate dated June 17, 1987; and the general information sheet.

Also, the petitioners adduced competent proof showing that the respondents had allowed the petitioners to become
members of the Board of Directors.

The petitioners attended the annual meeting as stockholders of Abra Valley and participated in the election of the
Board of Directors at which some of them were chosen as members.

Section 23 of the Corporation Code requires every director to be the holder of at least one share of capital stock of
the corporation of which he is a director, the respondents would not have then allowed any of the petitioners to be
elected to sit in the Board of Directors as members unless they believed that the petitioners so elected were not
disqualified for lack of stock ownership. Neither did the respondents thereafter assail their acts as Board Directors.

Conformably with the doctrine of estoppel, the respondents could no longer deny the petitioners’ status as
stockholders of Abra Valley.

Hence, the petitioners are stockholders of Abra Valley.


CORPORATION LAW CASE DIGESTS | 79
FEU JD4401 | 2nd SEMESTER, S.Y. 2017-2018

(47) Turner v. Lorenzo Shipping


G.R. No. 157479, Nov. 24, 2010
By: Jovero, John Tristram V.

Topic: Right of Appraisal

Doctrine: A corporation can purchase its own shares, provided payment is made out of surplus profits and the
acquisition is for a legitimate corporate purpose.

Facts: The petitioners held 1,010,000 shares of stock of the respondent. Respondent decided to amend its articles
of incorporation to remove the stockholders’ pre-emptive rights to newly issued shares of stock. Feeling that the
corporate move would be prejudicial to their interest as stockholders, the petitioners voted against the amendment
and demanded payment of their shares. The respondent found the fair value of the shares demanded by the
petitioners unacceptable. Respondent insisted that payment could be made only if the respondent had unrestricted
retained earnings in its books to cover the value of the shares, which was not the case.

Upon the respondent’s refusal to pay, the petitioners sued the respondent for collection and damages.

Issue: Whether or not the right of appraisal may be exercised despite having no unrestricted retained earnings?

Held: No. A corporation can purchase its own shares only by using its unrestricted retained earnings.

A stockholder who dissents from certain corporate actions has the right to demand payment of the fair value of his
or her shares. This right, known as the right of appraisal, is expressly recognized in Section 81 of the Corporation
Code.

However, a corporation can purchase its own shares, provided payment is made out of surplus profits and the
acquisition is for a legitimate corporate purpose. In the Philippines, this new rule is embodied in Section 41 of the
Corporation Code. Thus, no payment shall be made to any dissenting stockholder unless the corporation has
unrestricted retained earnings in its books to cover the payment.

The trust fund doctrine backstops the requirement of unrestricted retained earnings to fund the payment of the
shares of stocks of the withdrawing stockholders. Under the doctrine, the capital stock, property, and other assets
of a corporation are regarded as equity in trust for the payment of corporate creditors, who are preferred in the
distribution of corporate assets. The creditors of a corporation have the right to assume that the board of directors
will not use the assets of the corporation to purchase its own stock for as long as the corporation has outstanding
debts and liabilities. There can be no distribution of assets among the stockholders without first paying corporate
debts. Thus, any disposition of corporate funds and assets to the prejudice of creditors is null and void.
CORPORATION LAW CASE DIGESTS | 80
FEU JD4401 | 2nd SEMESTER, S.Y. 2017-2018

(48) Lu v Lu Ym Sr
G.R. No. 153690, Feb. 15, 2011

Doctrine: An intra-corporate controversy always involves a property in litigation, the value of which is always the
basis for computing the applicable filing fees. The latest amendments seem to imply that there can be no case of
intra-corporate controversy where the value of the subject matter cannot be estimated. Even one for a mere
inspection of corporate books.

On August 14, 2000, David Lu, Rosa Go, Silvano Ludo and CL Corporation filed with the Regional Trial Court (RTC)
of Cebu City a complaint against Paterno Lu Ym, Sr., Paterno Lu Ym, Jr., Victor Lu Ym, John Lu Ym, Kelly Lu Ym,
and Ludo & Luym Development Corporation (LLDC) for Declaration of Nullity of Share Issue, Receivership and
Dissolution. The plaintiffs, shareholders of LLDC, claimed that the Lu Ym father and sons, as members of the Board
of Directors, caused the issuance to the latter of 600,000 of the corporations unsubscribed and unissued shares for
less than their actual value. They then prayed for the dissolution of the corporation and the appointment of a
receiver during the pendency of the action. The trial court placed LLDC under receivership.

On March 31, 2003, the plaintiffs therein filed a Motion to Admit Complaint to Conform to the Interim Rules
Governing Intra-Corporate Controversies, which was admitted by the trial court. On January 23, 2004, the Lu Ym
father and sons inquired from the Clerk of Court as to the amount of docket fees paid by David, et al.

On March 1, 2004, the RTC decided the case on the merits. It annulled the issuance of LLDCs 600,000 shares of
stock to the Lu Ym father and sons. It also ordered the dissolution of LLDC and the liquidation of its assets,and
created a management committee to take over LLDC. The Lu Ym father and sons appealed to the CA.

The Lu Ym father and sons filed a motion for reconsideration, wherein they further questioned the sufficiency of the
docket fees paid by David, et al. in the RTC. On December 8, 2005, the CA denied the motion for reconsideration.

On August 26, 2008, this Court rendered judgment as aforesaid. Lu Ym father and sons filed the instant Motion for
Reconsideration. We required David, et al., to submit their Comment thereto. With our directive complied with, we
now resolve the Motion for Reconsideration.

Did the the trial court acquire jurisdiction over the case for the alleged failure of David, et.al. to pay the
correct docket fees?

YES. Aa court acquires jurisdiction over a case only upon the payment of the prescribed fees. Hence, without
payment of the correct docket fees, the trial court did not acquire jurisdiction over the action filed by David, et al.

The new Section 21(k) of Rule 141 of the Rules of Court, as amended by A.M. No. 04-2-04-SC (July 20,
2004), expressly provides that for petitions for insolvency or other cases involving intra-corporate controversies,
the fees prescribed under Section 7(a) shall apply. Said paragraph refers to docket fees for filing actions where the
value of the subject matter cannot be estimated and all other actions not involving property. By referring the
computation of such docket fees to paragraph (a) only, it denotes that an intra-corporate controversy always
involves a property in litigation, the value of which is always the basis for computing the applicable filing fees. The
latest amendments seem to imply that there can be no case of intra-corporate controversy where the value of the
subject matter cannot be estimated. Even one for a mere inspection of corporate books.

The 600,000 shares of stock were, indeed, properties in litigation. They were the subject matter of the complaint,
and the relief prayed for entailed the nullification of the transfer thereof and their return to LLDC. David, et al., are
minority shareholders of the corporation who claim to have been prejudiced by the sale of the shares of stock to
the Lu Ym father and sons. Thus, to the extent of the damage or injury they allegedly have suffered from this sale
of the shares of stock, the action they filed can be characterized as one capable of pecuniary estimation. The
shares of stock have a definite value, which was declared by plaintiffs themselves in their complaint. Accordingly,
the docket fees should have been computed based on this amount.

Further, the Lu Ym father and sons are not estopped from challenging the jurisdiction of the trial court. They raised
the insufficiency of the docket fees before the trial court rendered judgment and continuously maintained their
position even on appeal to the CA. The matter of lack of jurisdiction of the trial court is one that may be raised at
any stage of the proceedings. More importantly, this Court may pass upon this issue motu proprio.

In summary, the trial court did not acquire jurisdiction over the case for failure of David, et.al. to pay the correct
docket fees. Consequently, all interlocutory matters pending before this Court, specifically the incidents subject of
these three consolidated petitions, must be denied for being moot and academic. With the dismissal of the main
action, the ancillary motions have no more leg to stand on.
CORPORATION LAW CASE DIGESTS | 81
FEU JD4401 | 2nd SEMESTER, S.Y. 2017-2018

(49) Interport Resources Corporation v. Securities Specialist, Inc.


G.R. No. 154069, June 06, 2016
By: Mano, Razna I.

Doctrine: The assignment of the subscription agreements is a form of novation by substitution of a new debtor
and which required the consent of or notice to the creditor. In this case, the change of debtor took place when R.C.
Lee assigned the Oceanic shares under Subscription Agreements to SSI so that the latter became obliged to settle
the 75% unpaid balance on the subscription.

Oceanic Inc. entered into a subscription agreement with R.C. Lee Securities, Inc., covering 5,000,000 of its shares.
R.C. Lee paid 25% of the subscription, leaving 75% unpaid. Oceanic merged with Interport, with the latter as the
surviving corporation. Under the terms of the merger, each share of Oceanic was exchanged for a share of
Interport. SSI, a domestic corporation, received Oceanic Subscription Agreements, all outstanding in the name of
R.C. Lee. The Oceanic subscription agreements were duly delivered to SSI through stock assignments indorsed in
blank by R.C. Lee. Later on, R.C. Lee requested Interport for a list of subscription agreements and stock certificates
issued in its name. Interport provided the list. R.C. Lee paid its unpaid subscriptions and was accordingly issued
stock certificates. Thereafter, Interport issued a call for the full payment of subscription receivables. SSI directly
tendered payment to Interport for the balance of the shares covered by the Oceanic subscription agreements.
However, stockbrokers reported to SSI that Interport refused to honor the Oceanic subscriptions. SSI learned that
Interport had issued the shares to R.C. Lee. SSI demanded for the cancellation of the shares issued to R.C. Lee.
SSI filed a case in the SEC to compel the respondent R.C. Lee to deliver the shares on the basis of a purported
assignment of the subscription agreements and to pay damages. SEC ordered Interport to deliver the shares.

Can Interport be held liable to deliver to SSI the Oceanic shares of stock, or the value thereof, under
the Subscription Agreement?

Yes, Interport can be held liable to deliver to SSI the Oceanic shares of stock, or the value thereof, under the
Subscription Agreement.

Novation extinguished an obligation between two parties. Clearly, the effect of the assignment of the subscri ption
agreements to SSI was to extinguish the obligation of R.C. Lee to Oceanic or Interport to settle the unpaid balance
on the subscription. As a result of the assignment, Interport was no longer obliged to accept any payment
from R.C. Lee because the latter had ceased to be privy to Subscription Agreements. On the other hand,
Interport was legally bound to accept SSI's tender of payment for the balance on the subscription price because
SSI had become the new debtor. As such, the issuance of the stock certificates in the name of R.C. Lee had
no legal basis in the absence of a contractual agreement between R. C. Lee and Interport.
CORPORATION LAW CASE DIGESTS | 82
FEU JD4401 | 2nd SEMESTER, S.Y. 2017-2018

(50) Joselito Musni Puno vs Puno Enterprises, Inc.


GR No. 177066, September 11, 2009
By: Marasigan, Mariz Angelle R.

Doctrine: Upon the death of a shareholder, the heirs do not automatically become stockholders of the corporation
and acquire the rights and privileges of the deceased as shareholder of the corporation.

Facts: Carlos L. Puno, who died on June 25, 1963, was an incorporator of respondent Puno Enterprises, Inc. On
March 14, 2003, petitioner Joselito Musni Puno, claiming to be an heir of Carlos L. Puno, initiated a complaint for
specific performance against respondent. Petitioner averred that he is the son of the deceased with the latters
common-law wife, Amelia Puno. As surviving heir, he claimed entitlement to the rights and privileges of his late
father as stockholder of respondent. The complaint thus prayed that respondent allow petitioner to inspect its
corporate book, render an accounting of all the transactions it entered into from 1962, and give petitioner all the
profits, earnings, dividends, or income pertaining to the shares of Carlos L. Puno.

Respondent filed a motion to dismiss on the ground that petitioner did not have the legal personality to sue
because his birth certificate names him as Joselito Musni Muno. Apropos, there was yet a need for a judicial
declaration that Joselito Musni Puno and Joselito Musni Muno were one and the same.

The trial court rendered a Decision allowing the petitioner to inspect the corporate books and records of the
company from 1962 up to the present including the financial statements of the corporation.

On appeal, the CA ordered the dismissal of the complaint. According to the CA, petitioner was not able to establish
the paternity of and his filiation to Carlos L. Puno since his birth certificate was prepared without the intervention of
and the participatory acknowledgment of paternity by Carlos L. Puno. Accordingly, the CA said that petitioner had
no right to demand that he be allowed to examine respondent’s books. Moreover, petitioner was not a stockholder
of the corporation but was merely claiming rights as an heir of Carlos L. Puno, an incorporator of the corporation.
His action for specific performance therefore appeared to be premature; the proper action to be taken was to prove
the paternity of and his filiation to Carlos L. Puno in a petition for the settlement of the estate of the latter.

Is the CA correct in dismissing the complaint?

Held: Yes.

Petitioner failed to establish the right to inspect respondent corporation’s books and receive dividends on the stocks
owned by Carlos L. Puno.

Petitioner anchors his claim on his being an heir of the deceased stockholder. However, we agree with the appellate
court that petitioner was not able to prove satisfactorily his filiation to the deceased stockholder; thus, the former
cannot claim to be an heir of the latter.

In any case, Sections 74 and 75 of the Corporation Code enumerate the persons who are entitled to the inspection
of corporate books, thus

Sec. 74. Books to be kept; stock transfer agent. x x x.

The records of all business transactions of the corporation and the minutes of any meeting shall be open to the
inspection of any director, trustee, stockholder or member of the corporation at reasonable hours on business
days and he may demand, in writing, for a copy of excerpts from said records or minutes, at his expense.

xxxx

Sec. 75. Right to financial statements. Within ten (10) days from receipt of a written request of any stockholder or
member, the corporation shall furnish to him its most recent financial statement, which shall include a balance
sheet as of the end of the last taxable year and a profit or loss of statement for said taxable year, showing in
reasonable detail its assets and liabilities and the result of its operations.

The stockholders right of inspection of the corporation’s books and records is based upon his ownership of shares in
the corporation and the necessity for self-protection. After all, a shareholder has the right to be intelligently
informed about corporate affairs. Such right rests upon the stockholders underlying ownership of the corporation’s
assets and property.

Similarly, only stockholders of record are entitled to receive dividends declared by the corporation, a right inherent
in the ownership of the shares.
CORPORATION LAW CASE DIGESTS | 83
FEU JD4401 | 2nd SEMESTER, S.Y. 2017-2018

Upon the death of a shareholder, the heirs do not automatically become stockholders of the corporation and
acquire the rights and privileges of the deceased as shareholder of the corporation. The stocks must be distributed
first to the heirs in estate proceedings, and the transfer of the stocks must be recorded in the books of the
corporation. Section 63 of the Corporation Code provides that no transfer shall be valid, except as between the
parties, until the transfer is recorded in the books of the corporation. During such interim period, the heirs stand as
the equitable owners of the stocks, the executor or administrator duly appointed by the court being vested with the
legal title to the stock. Until a settlement and division of the estate is effected, the stocks of the decedent are held
by the administrator or executor. Consequently, during such time, it is the administrator or executor who is entitled
to exercise the rights of the deceased as stockholder.

Thus, even if petitioner presents sufficient evidence in this case to establish that he is the son of Carlos L. Puno, he
would still not be allowed to inspect respondent’s books and be entitled to receive dividends from respondent,
absent any showing in its transfer book that some of the shares owned by Carlos L. Puno were transferred to him.
This would only be possible if petitioner has been recognized as an heir and has participated in the settlement of
the estate of the deceased.

Corollary to this is the doctrine that a determination of whether a person, claiming proprietary rights over the
estate of a deceased person, is an heir of the deceased must be ventilated in a special proceeding instituted
precisely for the purpose of settling the estate of the latter. The status of an illegitimate child who claims to be an
heir to a decedent’s estate cannot be adjudicated in an ordinary civil action, as in a case for the recovery of
property. The doctrine applies to the instant case, which is one for specific performance to direct respondent
corporation to allow petitioner to exercise rights that pertain only to the deceased and his representatives.
CORPORATION LAW CASE DIGESTS | 84
FEU JD4401 | 2nd SEMESTER, S.Y. 2017-2018

(51) Reyes vs. Regional Trial Court of Makati, Br. 142


G.R. No. 165744. August 11, 2008
By: Pangilinan, Gene Alexis

Doctrine: The status of heirs as co-owners of shares of stocks prior to the partition of the decedent’s estate does
not immediately and necessarily make them stockholders of the corporation.

Oscar and private respondent Rodrigo C. Reyes (Rodrigo) are two of the four children of the spouses Pedro and
Anastacia Reyes. Pedro, Anastacia, Oscar, and Rodrigo each owned shares of stock of Zenith Insurance Corporation
(Zenith), a domestic corporation established by their family.
As of June 30, 1990, Anastacia owned 136,598 shares of Zenith; Oscar and Rodrigo owned 8,715,637 and 4,250
shares, respectively. Pedro died in 1964, while Anastacia died in 1993. Although Pedro’s estate was judicially
partitioned among his heirs sometime in the 1970s, no similar settlement and partition appear to have been made
with Anastacia’s estate, which included her shareholdings in Zenith.

Zenith and Rodrigo filed a complaint with the SEC against Oscar to obtain an accounting of the funds and assets of
ZENITH and to determine the shares of stock of deceased spouses Pedro and Anastacia Reyes that were arbitrarily
and fraudulently appropriated by Oscar for himself. Oscar denied the charge that he illegally acquired the shares of
Anastacia Reyes and alleged that the case is not a bona fide derivative suit as it partakes of the nature of a petition
for the settlement of estate of the deceased Anastacia that is outside the jurisdiction of a special commercial court.

Issue: Whether or not the complaint is outside the jurisdiction of the RTC acting as a special commercial court.

Held: No. The complaint is outside the jurisdiction of the RTC acting as a special commercial court. P.D. No. 902-A
enumerates the cases over which the SEC (now the RTC acting as a special commercial court) exercises exclusive
jurisdiction:

“SECTION 5. In addition to the regulatory and adjudicative functions of the Securities and Exchange Commission
over corporations, partnership, and other forms of associations registered with it as expressly granted under
existing laws and decrees, it shall have original and exclusive jurisdiction to hear and decide cases involving:

xxx

b) Controversies arising out of intra-corporate or partnership relations, between and among stockholders,
members, or associates; between any or all of them and the corporation, partnership or association of which they
are stockholders, members, or associates, respectively; and between such corporation, partnership or association
and the State insofar as it concerns their individual franchise or right to exist as such entity.”

In this case, while Rodrigo holds shares of stock in Zenith, he holds them in two capacities: in his own right with
respect to the 4,250 shares registered in his name, and as one of the heirs of Anastacia Reyes with respect to the
136,598 shares registered in her name. Article 777 of the Civil Code declares that the successional rights are
transmitted from the moment of death of the decedent. Accordingly, upon Anastacia’s death, her children acquired
legal title to her estate (which title includes her shareholdings in Zenith), and they are, prior to the estate’s
partition, deemed co-owners thereof. This status as co-owners, however, does not immediately and necessarily
make them stockholders of the corporation. Unless and until there is compliance with Section 63 of the Corporation
Code on the manner of transferring shares, the heirs do not become registered stockholders of the corporation.

The transfer of title by means of succession, though effective and valid between the parties involved (i.e., between
the decedent’s estate and her heirs), does not bind the corporation and third parties. The transfer must be
registered in the books of the corporation to make the transferee-heir a stockholder entitled to recognition as such
both by the corporation and by third parties.

Without the settlement of Anastacia’s estate, there can be no definite partition and distribution of the estate to the
heirs. Without the partition and distribution, there can be no registration of the transfer. And without the
registration, the Court cannot consider the transferee-heir a stockholder who may invoke the existence of an intra-
corporate relationship as premise for an intra-corporate controversy within the jurisdiction of a special commercial
court.

In sum, insofar as Anastacia’s shares are concerned—Rodrigo cannot be considered a stockholder of Zenith.
Consequently, it cannot be declared that an intracorporate relationship exists that would serve as basis to bring
this case within the special commercial court’s jurisdiction.
CORPORATION LAW CASE DIGESTS | 85
FEU JD4401 | 2nd SEMESTER, S.Y. 2017-2018

(52) Forest Hills Golf & Country Club v. Vertex Sales and Trading, Inc.,
G.R. No. 202205, March 6, 2013
By: Radovan

Doctrine: the Corporation whose shares of stocks are the subject of a transfer transaction need not be a party to
the transaction. However, to bind the corporation as well as third parties, it is necessary that the transfer be
recorded in the books of the corporation.

Facts: Petitioner Forest Hills Golf & Country Club (Forest Hills) is a domestic non-profit stock corporation that
operates and country club facility in Antipolo City. Forest Hills was created as a result of a joint venture agreement
between Kings Properties Corporation (Kings) and Fil-Estate Golf and Development, Inc. (FEGDI). Accordingly,
Kings and FEGDI owned the shares of stock of Forest Hills, holding 40% and 60% of the shares, respectively.

FEGDI sold to RS Asuncion Construction Corporation (RSACC) one (1) Class “C” common share of Forest Hills for
P1.1 million. Prior to the full payment of the purchase price, RSACC transferred its interests over FEGDI’s Class “C”
common share to respondent Vertex Sales and Trading, Inc. (Vertex).4 RSACC advised FEGDI of the transfer and
FEGDI, in turn, requested Forest Hills to recognize Vertex as a shareholder. Forest Hills acceded to the request, and
Vertex was able to enjoy membership privileges in the golf and country club.

Despite the sale of FEGDI’s Class “C” common share to Vertex, the share remained in the name of FEGDI,
prompting Vertex to demand for the issuance of a stock certificate in its name. As its demand went unheeded,
Vertex filed a complaint6 for rescission with damages against defendants Forest Hills, FEGDI, and Fil-Estate Land,
Inc. (FELI)—the developer of the Forest Hills golf course. Vertex averred that the defendants defaulted in their
obligation as sellers when they failed and refused to issue the stock certificate covering the Class “C” common
share. It prayed for the rescission of the sale and the return of the sums it paid; it also claimed payment of actual
damages for the defendants’ unjustified refusal to issue the stock certificate.

Forest Hills denied transacting business with Vertex and claimed that it was not a party to the sale of the share;
FELI claimed the same defense. While admitting that no stock certificate was issued, FEGDI alleged that Vertex
nonetheless was recognized as a stockholder of Forest Hills and, as such, it exercised rights and privileges of one.
FEGDI added that during the pendency of Vertex’s action for rescission, a stock certificate was issued in Vertex’s
name,7 but Vertex refused to accept it.

RTC: the RTC ruled that the nonissuance of the stock certificate is a mere casual breach that would not entitle
Vertex to rescind the sale.

CA: Reversed the RTC. It declared that “in the sale of shares of stock, physical delivery of a stock certificate is one
of the essential requisites for the transfer of ownership of the stocks purchased. It based its ruling on Section 63 of
the Corporation Code, which requires for a valid transfer of stock—

(1) the delivery of the stock certificate;


(2) the endorsement of the stock certificate by the owner or his attorney-in-fact or other persons legally authorized
to make the transfer; and
(3) to be valid against third parties, the transfer must be recorded in the books of the corporation.
Without the issuance of the stock certificate and despite Vertex’s full payment of the purchase price, the share
cannot be considered as having been validly transferred. Hence, the CA rescinded the sale of the share and ordered
the defendants to return the amount paid by Vertex by reason of the sale.

Hence this petition. Forest Hills argues that rescission should be allowed only for substantial breaches that would
defeat the very object of the parties making the agreement. The delay in the issuance of the stock certificate could
not be considered as a substantial breach, considering that Vertex was recognized as, and enjoyed the privileges
of, a stockholder. Forest Hills also objects to the CA ruling that required it to return the amount paid by Vertex for
the share of stock. It claims that it was not a party to the contract of sale; hence, it did not receive any amount
from Vertex which it would be obliged to return on account of the rescission of the contract.

Issues:

1. Whether recission of the sale should be allowed?- No

2. Whether Forest Hills is liable to return the amount paid by Vertex for the sale?- No.
CORPORATION LAW CASE DIGESTS | 86
FEU JD4401 | 2nd SEMESTER, S.Y. 2017-2018

Held:

1. As correctly pointed out by Forest Hills, it was not a party to the sale even though the subject of the sale was its
share of stock. The corporation whose shares of stock are the subject of a transfer transaction (through sale,
assignment, donation, or any other mode of conveyance) need not be a party to the transaction, as may be
inferred from the terms of Section 63 of the Corporation Code. However, to bind the corporation as well as third
parties, it is necessary that the transfer is recorded in the books of the corporation. In the present case, the parties
to the sale of the share were FEGDI as the seller and Vertex as the buyer (after it succeeded RSACC). As party to
the sale, FEGDI is the one who may appeal the ruling rescinding the sale. The remedy of appeal is available to a
party who has "a present interest in the subject matter of the litigation and is aggrieved or prejudiced by the
judgment. A party, in turn, is deemed aggrieved or prejudiced when his interest, recognized by law in the subject
matter of the lawsuit, is injuriously affected by the judgment, order or decree."

The rescission of the sale does not in any way prejudice Forest Hills in such a manner that its interest in the
subject matter – the share of stock – is injuriously affected. Thus, Forest Hills is in no position to appeal the ruling
rescinding the sale of the share. Since FEGDI, as party to the sale, filed no appeal against its rescission, we
consider as final the CA’s ruling on this matter.

2. A necessary consequence of rescission is restitution: the parties to a rescinded contract must be brought back to
their original situation prior to the inception of the contract; hence, they must return what they received pursuant
to the contract.24 Not being a party to the rescinded contract, however, Forest Hills is under no obligation to return
the amount paid by Vertex by reason of the sale. Indeed, Vertex failed to present sufficient evidence showing that
Forest Hills received the purchase price for the share or any other fee paid on account of the sale (other than the
membership fee which we will deal with after) to make Forest Hills jointly or solidarily liable with FEGDI for
restitution.

Although Forest Hills received ₱150,000.00 from Vertex as membership fee, it should be allowed to retain this
amount. For three years prior to the rescission of the sale, the nominees of Vertex enjoyed membership privileges
and used the golf course and the amenities of Forest Hills. We consider the amount paid as sufficient consideration
for the privileges enjoyed by Vertex's nominees as members of Forest Hills.
CORPORATION LAW CASE DIGESTS | 87
FEU JD4401 | 2nd SEMESTER, S.Y. 2017-2018

(53) F&S Velasco Company vs. Madrid


G.R. No. 208844, November 10, 2015
By: Rosario, Patricia Kaye T.

Doctrine: An owner of shares of stock cannot be accorded the rights pertaining to a stockholder - such as the right
to call for a meeting and the right to vote, or be voted for - if his ownership of such shares is not recorded in the
Stock and Transfer Book.

Petitioner F & S Velasco Company, Inc. (FSVCI) was duly organized and registered as a corporation with Francisco
O. Velasco (Francisco), Simona J. Velasco (Simona), Angela V. Madrid (Angela), herein respondent Dr. Rommel L.
Madrid (Madrid), and petitioner Saturnino O. Velasco (Saturnino) as its incorporators. When Simona and Francisco
died, their daughter, Angela, inherited their shares, thereby giving her control of 70.82%. During her tenure as
Chairman of the Board of Directors of FSVCI, Angela died intestate and without issue. Madrid, as Angela's spouse,
executed an Affidavit of Self-Adjudication covering the latter's estate which includes her 70.82% ownership of
FSVCI's shares of stock. Believing that he is already the controlling stockholder of FSVCI by virtue of such self-
adjudication, Madrid called for a Special Stockholders' and Re-Organizational Meeting to be held on November 18,
2009.

Meanwhile, as Madrid was performing the aforesaid acts, Seva, in his then-capacity as FSVCI corporate secretary,
sent a Notice of an Emergency Meeting to FSVCI's remaining stockholders for the purpose of electing a new
president and vice-president, as well as the opening of a bank account. Such meeting was held on November 6,
2009 which was attended by Saturnino, Seva, and Sunico, during which, Saturnino was recognized as a member of
the FSVCI Board of Directors and thereafter, as FSVCI President, while Scribner was elected FSVCI Vice-President.

Despite the election conducted by the Saturnino Group, the Madrid Group proceeded with the Special Stockholders'
and Re-Organizational Meeting on November 18, 2009, wherein: (a) the current members of FSVCI Board of
Directors (save for Madrid) were ousted and replaced by the members of the Madrid Group; and (b) Madrid,
Danao, Arimado, and Labalan were elected President, Vice-President, Corporate Secretary, and Treaurer,
respectively, of FSVCI (November 18, 2009 Meeting). The Madrid Group likewise applied for the Appointment of a
Management Committee for FSVCI.

A. Is the November 18, 2009 Meeting organized by Madrid legal and valid?

NO, Madrid's inheritance of Angela's shares of stock does not ipso facto afford him the rights accorded to such
majority ownership of FSVCI's shares of stock.

Under Section 63 of the Corporation Code, all transfers of shares of stock must be registered in the Stock and
Transfer Book in order to be binding on the corporation. Further, in Batangas Laguna Tayabas Bus Co., Inc. v.
Bitanga, the Court instructs that an owner of shares of stock cannot be accorded the rights pertaining to a
stockholder - such as the right to call for a meeting and the right to vote, or be voted for - if his ownership of such
shares is not recorded in the Stock and Transfer Book.

In the case at bar, records reveal that at the time Madrid called for the November 18, 2009 Meeting, as well as the
actual conduct thereof, he was already the owner of 74.98% shares of stock of FSVCI as a result of his inheritance
of Angela's 70.82% ownership thereof. However, records are bereft of any showing that the transfer of Angela's
shares of stock to Madrid had been registered in FSVCFs Stock and Transfer Book when he made such call and
when the November 18, 2009 Meeting was held. Madrid could not have made a valid call of the November 18,
2009 Meeting as his stock ownership of FSVCI as registered in the Stock and Transfer Book is only 4.16% in view
of the non-registration of Angela's shares of stock in the FSVCI Stock and Transfer Book in his favor.

Thus, in view of the nullity of the November 6, 2009 Meeting conducted by the Saturnino Group which ruling of the
RTC had already attained finality, as well as the November 18, 2009 Meeting conducted by the Madrid Group - the
FSVCI Board of Directors at the time of Angela's death (i.e. Madrid, Seva, Scribner, and Sunico) should be
reconstituted, and thereafter, fill the vacant seat left by Angela in accordance with Section 29 of the Corporation
Code. Such Board of Directors shall only act in a hold-over capacity until their successors are elected and qualified,
pursuant to Section 23 of the Corporation Code.
CORPORATION LAW CASE DIGESTS | 88
FEU JD4401 | 2nd SEMESTER, S.Y. 2017-2018

B. Should a Management Committee be appointed or constituted to take over the corporate and
business affairs of FSVCI?

NO. Section 1, Rule 9 of the Interim Rules of Procedure Governing Intra-Corporate Controversies provides the
elements needed for the creation of a Management Committee:

SEC. 1. Creation of a management committee. - As an incident to any of the cases filed under these Rules
or the Interim Rules on Corporate Rehabilitation, a party may apply for the appointment of a management
committee for the corporation, partnership or association, when there is imminent danger of:

(1) Dissipation, loss, wastage or destruction of assets or other properties; and

(2) Paralyzation of its business operations which may be prejudicial to the interest of the minority
stockholders, parties-litigants or the general public.

Applicants for the appointment of a management committee need to establish the confluence of these two
requisites because appointed management committees will immediately take over the management of the
corporation and exercise the management powers specified in the law. This may have a negative effect on the
operations and affairs of the corporation with third parties, as persons who are more familiar with its operations are
necessarily dislodged from their positions in favor of appointees who are strangers to the corporation's operations
and affairs.

In the case at bar, the CA merely based its directive of creating a Management Committee for FSVCI on its finding
of "the persisting conflict between the Saturnino and Madrid Groups, the allegations of embezzlement of corporate
funds among the parties, and the uncertainty in the leadership and direction of the corporation had created an
imminent danger of dissipation, loss, and wastage of FSVCI's assets and the paralyzation of its business operations
which may be prejudicial to the minority stockholders, parties-litigants or the general public." However, absent any
actual evidence from the records showing such imminent danger, the CA's findings have no legal or factual basis to
support the appointment/constitution of a Management Committee for FSVCI. Accordingly, the CA erred in ordering
the creation of a Management Committee in this case. Hence, in the event a Management Committee had already
been constituted then it should be immediately dissolved.
CORPORATION LAW CASE DIGESTS | 89
FEU JD4401 | 2nd SEMESTER, S.Y. 2017-2018

(54) Anna Teng v. Securities and Exchange Commission


G.R. No. 184332, February 17, 2016
By: Samson, Maria Johanna Ilyssa

Topic: Certificate of stock and transfer of shares

Doctrine: Section 63 of the Corporation Code provides that shares of stock so issued are personal property and
may be transferred by delivery of the certificate or certificates indorsed by the owner or his attorney-in-fact or
other person legally authorized to make the transfer. No transfer, however, shall be valid, except as between the
parties, until the transfer is recorded in the books of the corporation showing the names of the parties to the
transaction, the date of the transfer, the number of the certificate or certificates and the number of shares
transferred. Under the provision, certain minimum requisites must be complied with for there to be a valid transfer
of stocks, to wit: (a) there must be delivery of the stock certificate; (b) the certificate must be endorsed by the
owner or his attorney-in-fact or other persons legally authorized to make the transfer; and (c) to be valid against
third parties, the transfer must be recorded in the books of the corporation.

This case has its origin in G.R. No. 129777 entitled TCL Sales Corporation and Anna Teng v. Hon. Court of Appeals
and Ting Ping Lay. Herein respondent Ting Ping purchased 480 shares of TCL Sales Corporation (TCL) from Peter
Chiu (Chiu) on February 2, 1979; 1,400 shares on September 22, 1985 from his brother Teng Ching Lay (Teng
Ching), who was also the president and operations manager of TCL; and 1,440 shares from Ismaelita Maluto
(Maluto) on September 2, 1989. Upon Teng Ching’s death in 1989, his son Henry Teng (Henry) took over the
management of TCL. To protect his shareholdings with TCL, Ting Ping requested TCL’s Corporate Secretary, herein
petitioner Teng, to enter the transfer in the Stock and Transfer Book of TCL for the proper recording of his
acquisition. He also demanded the issuance of new certificates of stock in his favor. TCL and Teng, however,
refused despite repeated demands. Because of their refusal, Ting Ping filed a petition for mandamus with the SEC
against TCL and Teng. SEC granted Ting Ping’s petition. TCL and Teng appealed to the SEC En Banc, which, in its
Order, affirmed the SEC decision with modification in that Teng was held solely liable for the payment of moral
damages and attorney’s fees. Not contented, TCL and Teng filed a petition for review with the CA. On January 31,
1997, the CA, however, dismissed the petition for having been filed out of time and for finding no cogent and
justifiable grounds to disturb the findings of the SEC En Banc. This prompted TCL and Teng to come to the Court
via a petition for review on certiorari under Rule 45. After the finality of the Court’s decision, the SEC issued a writ
of execution addressed to the Sheriff of the RTC of Manila. Teng, however, filed a complaint for interpleader with
the RTC of Manila, docketed as Civil Case No. 02-102776, where Teng sought to compel Henry and Ting Ping to
interplead and settle the issue of ownership over the 1,400 shares, which were previously owned by Teng Ching.
Thus, the deputized sheriff held in abeyance the further implementation of the writ of execution pending outcome
of Civil Case No. 02-102776. The RTC of Manila rendered its Decision in Civil Case No. 02-102776, finding Henry to
have a better right to the shares of stock formerly owned by Teng Ching, except as to those covered by Stock
Certificate No. 011 covering 262.5 shares, among others.

Is the surrender of the certificates of stock a requisite before registration of the transfer may be made
in the corporate books and for the issuance of new certificates in its stead?

No. A certificate of stock is a written instrument signed by the proper officer of a corporation stating or
acknowledging that the person named in the document is the owner of a designated number of shares of its stock.
It is prima facie evidence that the holder is a shareholder of a corporation. A certificate, however, is merely a
tangible evidence of ownership of shares of stock. It is not a stock in the corporation and merely expresses the
contract between the corporation and the stockholder. The shares of stock evidenced by said certificates,
meanwhile, are regarded as property and the owner of such shares may, as a general rule, dispose of them as he
sees fit, unless the corporation has been dissolved, or unless the right to do so is properly restricted, or the
owner’s privilege of disposing of his shares has been hampered by his own action.

Section 63 of the Corporation Code prescribes the manner by which a share of stock may be transferred. Section
63 of the Corporation Code provides that shares of stock so issued are personal property and may be transferred
by delivery of the certificate or certificates indorsed by the owner or his attorney-in-fact or other person legally
authorized to make the transfer. No transfer, however, shall be valid, except as between the parties, until the
transfer is recorded in the books of the corporation showing the names of the parties to the transaction, the date of
the transfer, the number of the certificate or certificates and the number of shares transferred. Under the
provision, certain minimum requisites must be complied with for there to be a valid transfer of stocks, to wit: (a)
there must be delivery of the stock certificate; (b) the certificate must be endorsed by the owner or his attorney-
in-fact or other persons legally authorized to make the transfer; and (c) to be valid against third parties, the
transfer must be recorded in the books of the corporation.

It is thus clear that Teng’s position — that Ting Ping must first surrender Chiu’s and Maluto’s respective certificates
of stock before the transfer to Ting Ping may be registered in the books of the corporation — does not have legal
basis. To compel Ting Ping to deliver to the corporation the certificates as a condition for the registration of the
CORPORATION LAW CASE DIGESTS | 90
FEU JD4401 | 2nd SEMESTER, S.Y. 2017-2018

transfer would amount to a restriction on the right of Ting Ping to have the stocks transferred to his name, which is
not sanctioned by law. The only limitation imposed by Section 63 is when the corporation holds any unpaid claim
against the shares intended to be transferred.

It is the delivery of the certificate, coupled with the endorsement by the owner or his duly authorized
representative that is the operative act of transfer of shares from the original owner to the transferee. The delivery
contemplated in Section 63, however, pertains to the delivery of the certificate of shares by the transferor to the
transferee, that is, from the original stockholder named in the certificate to the person or entity the stockholder
was transferring the shares to, whether by sale or some other valid form of absolute conveyance of ownership.

In transferring stock, the secretary of a corporation acts in purely ministerial capacity, and does not try to decide
the question of ownership. If a corporation refuses to make such transfer without good cause, it may, in fact, even
be compelled to do so by mandamus. There are several reasons why registration of the transfer is necessary: one,
to enable the transferee to exercise all the rights of a stockholder; two, to inform the corporation of any change in
share ownership so that it can ascertain the persons entitled to the rights and subject to the liabilities of a
stockholder; and three, to avoid fictitious or fraudulent transfers.

The surrender of the original certificate of stock is necessary before the issuance of a new one so that the old
certificate may be cancelled. A corporation is not bound and cannot be required to issue a new certificate unless
the original certificate is produced and surrendered. Surrender and cancellation of the old certificates serve to
protect not only the corporation but the legitimate shareholder and the public as well, as it ensures that there is
only one document covering a particular share of stock. In the case at bench, Ting Ping manifested from the start
his intention to surrender the subject certificates of stock to facilitate the registration of the transfer and for the
issuance of new certificates in his name. It would be sacrificing substantial justice if the Court were to grant the
petition simply because Ting Ping is yet to surrender the subject certificates for cancellation instead of ordering in
this case such surrender and cancellation, and the issuance of new ones in his name.
CORPORATION LAW CASE DIGESTS | 91
FEU JD4401 | 2nd SEMESTER, S.Y. 2017-2018

(55) Ferro Chemicals vs. Garcia


G.R. Nos. 168134, 168183, October 05, 2016
By: Torres, Ma. Roma

DOCTRINE: A corporation has a personality separate and distinct from that of each stockholder. It has the right of
continuity or perpetual succession, that is, its existence is not extinguished by the transfer of ownership of its
shares of capital stock from one shareholder to another.

FACTS: Ramon Garcia is the president of Ferro Chemicals and brother of respondent Antonio Garcia, Chairman of
the BOD of Chemical Industries. Antonio Garcia and Ferro Chemicals entered into a Deed of Absolute Sale and
Purchase of Shares of Stock over 1,717,678 shares of capital stock of Chemical Industries registered under the
name of Antonio. Ferro Chemicals remitted the amount of 35M to Security Bank in the form of check drawn against
its account with Bank of America, but the check was not accepted by Security Bank because the amount tendered
was not sufficient for the obligation, leaving the obligor with no recourse but to consign the check to the court.

Meanwhile, Antonio Garcia entered into a Compromise Agreement with the Consortium banks (BPI, PISO, PCIB,
RCBC and LBP) pursuant to a surety agreement that Antonio entered with the banks (Consortium case). The RTC
granted a Notice of Garnishment over the same shares of stocks of Antonio. On the ground that only absolute
transfers of shares are required to be on the corporation's stock and transfer books, the Corporate Secretary did
not annotate the banks' claims on Chemical Industries' books. Antonio entered into a Deed of Repurchase
Agreement with Ferro Chemicals but the same was not effected, because there was already an assignment of the
same to Chemphil Export. This prompted Antonio to file a case of specific performance against Ferro Chemicals.
Later, the RTC in the Consortium case issued a writ of execution in favor of the banks, and the sheriff levied on the
subject shares of stocks. Chemphil Export demanded from Ferro Chemicals the value of the lost shares and Ferro
Chemicals was constrained to cede its fights over its chrome plant in Misamis Oriental in favor of Chemphil as a
result. An action for damages was then initiated by Ferro Chemicals over Chemical Industries, Antonio Garcia, and
its Corporate Secretary (Navarro). Ferro Chemicals imputes liability upon Navarro for tortious interference when he
facilitated in the execution of the sale by showing the Stock and Transfer Book to assure petitioner that there was
no lien other than those in the books to conceal the consortium banks’ lien.

ISSUES:
1. WON the Corporate Secretary is liable for failing to record the lien in favor of the Consortium Banks
2. WON Chemical Industries can be held liable for the supposed fraud and breach of contract perpetrated by
Antonio Garcia

HELD:
1. No. As the Corporate Secretary of Chemical Industries, he is under no obligation to record the attachment of the
Consortium Banks, not being a transfer of ownership but merely a burden on the title of the owner. Only absolute
transfers of shares of stock are required to be recorded in the corporation's stock and transfer book in order to
have "force and effect as a against third persons." In Chemphil Export and Import Corporation v. Court of Appeals,
et al., the Court enunciated the rule that attachments of shares are not considered "transfer" and need not be
recorded in the corporations' stock and transfer book. "A 'transfer' is the act by which the owner of a thing delivers
it to another with the intent of passing the rights which he has in it to the latter, and a chattel mortgage is not
within the meaning of such term.

Shares of stock being personal property, may be the subject matter of pledge and chattel mortgage. Such
collateral transfers are however not covered by the registration requirement of Section 63, since our Supreme
Court has held that such provision applies only to absolute transfers thus, the registration in the corporate books of
pledges and chattel mortgages of share cannot have any legal effect.

2. No. The sale contract was entered by Antonio Garcia in his personal capacity and not as the President of
Chemical Industries. Considering the nature of the transaction involved, whatever obligation [Antonio Garcia]
incurred, it was incurred in his personal capacity.

Even if Antonio Garcia was selling his shares of stocks in the Chemical Industries, the corporation was neither
made a party to the contract nor did the sale redound to its benefit. As a matter of fact, the subject of the
purchase agreement was not limited to Antonio Garcia's shares in Chemical Industries, but likewise included his
shares in Vision Insurance Consultants, Inc., Alabang Country Club, Inc. and Manila Polo Club, Inc. His shares of
capital stocks with Chemical Industries became the subject of controversy because of the allegation that he
intentionally withheld the information from Ferro Chemicals that these shares were subject of the Consortium
Banks' claim. Notably, the purported misrepresentation was: not alleged to have been authorized or abetted by the
corporation. It was a purely personal act of the seller desirous to dispose conveniently his shares in the
corporation. It bears underscoring that a corporation has a personality separate and distinct from that of each
stockholder. It has the right, of continuity or perpetual succession, that is, its existence is not extinguished by the
transfer of ownership of its shares of capital stock from one shareholder to another.
CORPORATION LAW CASE DIGESTS | 92
FEU JD4401 | 2nd SEMESTER, S.Y. 2017-2018

(56) Joseph Omar O. Andaya vs. Rural Bank of Cabadbaran, Inc.


GR No. 188769 August 3, 2016
By: Valencia, Emmanuelle Nicole L.

Topic: Transfer of shares of stock

Doctrines: The registration of a transfer of shares of stock is a ministerial duty on the part of the
corporation.

Andaya bought 2200 shares of stock in the Rural Bank of Cabadbaran from Chute for a total price of PhP
220,000.00. The transaction was evidenced by a notarized document denominated as Sale of Shares of Stock.
Chute duly endorsed and delivered the certificates of stock to Andaya, and requested that the bank register the
transfer, and issue new stock certificates in Andaya’s favor. Andaya also separately communicated with the bank’s
corporate secretary, Oraiz, reiterating Chute’s request for the issuance of new stock certificates in Andaya’s favor.

A few days later, the bank’s corporate secretary wrote to Chute to inform her that he could not register the
transfer. His explanation was that under a previous stockholder’s resolution, existing stockholders were given
priority to buy the shares of others in the event that they were offered for sale (right of first refusal). The corporate
secretary then asked Chute if she wished to have her shares offered to existing stockholders instead, stating that if
no other stockholder would buy them, she could then proceed to sell the shares to outsiders.

The bank’s legal counsel, Gonzalez, informed Andaya that his request had been referred to the bank’s board of
directors for evaluation. Gonzalez also furnished him with a copy of the bank’s letter to Chute concerning her
request. Andaya responded by reiterating his earlier request for the registration of the transfer and issuance of new
stock certificates in his favor. He cited Section 98 of the Corporation Code, claiming that the purported restriction
could not deprive him of his right as a transferee, especially since such restriction did not appear in the bank’s
articles of incorporation, bylaws or certificates of stock.

The bank denied Andaya’s request. They claimed that there was a conflict of interest, and that the purchase of
shares could be the beginning of a hostile bid to take over control of the bank. It claimed that it may refuse to
accept a competitor as one of its stockholders, citing Gokongwei, Jr. vs. Securities and Exchange Commission.

Andaya instituted an action for mandamus to compel the bank to record the transfer of shares, and to issue new
stock certificates in his name.

May Andaya initiate an action for mandamus, compelling Rural Bank of Cabadbaran to record the
transfer and issue new stock certificates in his favor?

YES.

It is already settled jurisprudence that the registration of a transfer of shares of stock is a ministerial duty on the
part of the corporation. Aggrieved parties may resort to the remedy of mandamus to compel corporations that
wrongfully or unjustifiably refuse to record the transfer or to issue new certificates of stock.

In Price vs. Martin, the Supreme Court held that “a person who has purchased stock, and who desires to be
recognized as a stockholder, for the purpose of voting, must secure a standing by having the transfer recorded
upon the books. If the transfer is not duly made upon request, he has, as his remedy, to compel it to be made.”

In Pacific Basin Securities Co., Inc. vs. Oriental Petroleum and Minerals Corp, the Supreme Court held that “clearly,
the right of a transferee/assignee to have stocks transferred to his name is an inherent right flowing from his
ownership of the stocks.”

The Court had ruled in Rural Bank of Salinas, Inc. v. Court of Appeals that the “corporation’s obligation to register
is ministerial” and “the only limitation imposed by Section 63 of the Corporation Code is when the corporation holds
any unpaid claim against the shares intended to be transferred.”

Therefore, transferees of shares of stock are real parties-in- interest having a cause of action for mandamus to
compel the registration of the transfer and the corresponding issuance of stock certificates.
CORPORATION LAW CASE DIGESTS | 93
FEU JD4401 | 2nd SEMESTER, S.Y. 2017-2018

6. RIGHTS OF SHAREHOLDERS AND CORPORATE BOOKS AND RECORDS

(57) SANTIAGO CUA, JR., SOLOMON S. CUA and EXEQUIEL D. ROBLES, in their capacity as Directors of
PHILIPPINE RACING CLUB, INC. v. MIGUEL OCAMPO TAN, JEMIE U. TAN and ATTY. BRIGIDO J. DULAY
G.R. No. 181455-56 4 December 2009
By: Valencia, Mary Clydeen L.

DOCTRINE: A derivative suit must be differentiated from individual and representative or class suits. Suits by
stockholders or members of a corporation based on wrongful or fraudulent acts of directors or other persons may
be classified into individual suits, class suits, and derivative suits. Where a stockholder or member is denied the
right of inspection, his suit would be individual, because the wrong is done to him personally and not to the other
stockholders or the corporation. Where the wrong is done to a group of stockholders, as where preferred
stockholders’ rights are violated, a class or representative suit will be proper for the protection of all stockholders
belonging to the same group. But where the acts complained of constitute a wrong to the corporation itself, the
cause of action belongs to the corporation and not to the individual stockholder or member. / The right to
information, which includes the right to inspect corporate books and records, is a right personal to each
stockholder.

PRCI holds a franchise granted under Republic Act No. 6632, as amended by Republic Act No. 7953, to operate a
horse racetrack and manage betting stations. Under its franchise, PRCI may operate only one racetrack. It owns
only two real properties – one is known as the Sta. Ana Racetrack (Makati property) and the other is located in the
towns of Naic and Tanza in the province of Cavite (Cavite property).

Following the trend in the development of properties in the same area, PRCI wished to convert its Makati property
from a racetrack to urban residential and commercial use. Given the location and size of its Makati property, PRCI
believed that said property was severely under-utilized. Hence, PRCI management decided to transfer its racetrack
from Makati to Cavite. PRCI began developing its Cavite property as a racetrack, scheduled to be completed by
April 2008. As to its Makati property, PRCI management decided that it was best to spin off the management and
development of the same to a wholly owned subsidiary, so that PRCI could continue to focus its efforts on pursuing
its core business competence of horse racing. Instead of organizing and establishing a new corporation for the said
purpose, PRCI management opted to acquire another domestic corporation, JTH Davies Holdings, Inc. (JTH).

On 26 September 2006, the PRCI Board of Directors held a meeting. Among the directors present were petitioners
Santiago Sr., Santiago Jr., and Solomon as well as respondent Dulay. After discussing and deliberating on the
matter of the acquisition of JTH by PRCI, all the directors present, except respondent Dulay, voted affirmatively to
pass and approve, among others, a resolution declaring the intention to acquire and purchase shares of stock of
another company. The next day, 27 September 2006, PRCI entered into a Sale and Purchase Agreement for the
acquisition from JME of 41,928,290 common shares or 95.55% of the outstanding capital stock of JTH. In the
Special Stockholders’ Meeting held on 7 November 2006, attended by stockholders with 481,045,887 shares or
84.42% of the outstanding capital stock of PRCI, the acquisition by PRCI of JTH was presented for approval.
According to the President, PRCI is intending to acquire up to 100% of the shares of JTH Davies Holdings, Inc.
another listed company in the PSE. By 22 November 2006, PRCI was able to additionally acquire 1,160,137
common shares of JTH from the minority stockholders of the latter, giving PRCI ownership of 98.19% of the
outstanding capital stock of JTH. Thereafter, PRCI, in executing its intended spin-off to JTH of the management and
development of PRCI’s Makati property, determined that the Makati property, with a total zonal value of
₱3,817,242,000.00, could be transferred to JTH in exchange for the unissued portion of the latter’s recently
increase authorized capital stock, amounting to ₱397,908,894.50, divided into 795,817,789 shares with a par value
of ₱0.50 per share.

The matter of the proposed exchange was taken up and approved by the PRCI Board of Directors in its meeting
held on 11 May 2007, again with the lone dissent of respondent Dulay. The 11 May 2007 Resolution of the PRCI
Board of Directors on the property-for-shares exchange between PRCI and JTH was supposed to be presented for
approval by the stockholders under the afore-quoted Items No. VII and No. VIII of the Agenda. However, on 10
July 2007, respondents Miguel, et al., as minority stockholders of PRCI, filed before the RTC a Complaint,
denominated as a Derivative Suit with prayer for Issuance of TRO/Preliminary Injunction, against the rest of the
directors of PRCI and/or JTH.

The Complaint, docketed as Civil Case No. 07-61, was based on three causes of action: (1) the approval by the
majority directors of PRCI of the Board Resolutions dated 26 September 2006 and 11 May 2007 -- with undue
haste and deliberate speed, despite the absence of any disclosure and information -- was not only anomalous and
fraudulent, but also extremely prejudicial and inimical to interest of PRCI, committed in violation of their fiduciary
duty as directors of the said corporation; (2) respondent Solomon, as PRCI President, with the acquiescence of the
majority directors of PRCI, maliciously refused and resisted the request of respondents Miguel, et al., for complete
and adequate information relative to the disputed Board Resolutions, brazenly and unlawfully violating the rights of
the minority stockholders to information and to inspect corporate books and records; and (3) without being
officially and formally nominated, the majority directors of PRCI illegally and unlawfully constituted themselves as
CORPORATION LAW CASE DIGESTS | 94
FEU JD4401 | 2nd SEMESTER, S.Y. 2017-2018

members of the Board of Directors and/or Executive Officers of JTH, rendering all the actions they have taken as
such null and void ab initio.

After conducting hearings on the prayer for the issuance of a TRO, RTC Judge Untalan issued a Resolution on 16
July 2007 partially granting the prayer of PRCI for the issuance of Temporary Restraining Order upon the herein
defendants subject to the posting of Php100,000.00 bond. Thus, in order that these subject matters and items of
the Agenda of the aforesaid Stockholders’ Meeting shall not be taken up, the herein Defendants, their agents,
proxies and representatives, jointly and severally, are hereby ordered to delete and remove from the Agenda said
three (3) above stated items of the Agenda before the start and conduct of the said stockholders’ meeting.

On 19 July 2007, petitioners Santiago Jr., et al., as PRCI directors, filed a Petition for Certiorari with the Court of
Appeals, docketed as CA-G.R. SP No. 99769. On 20 July 2007, Santiago Sr., also as PRCI director, filed his own
Petition for Certiorari and Prohibition, docketed as CA-G.R. SP No. 99780. Both Petitions assailed the RTC
Resolution dated 16 July 2007, granting the issuance of a TRO, for being rendered with grave abuse of discretion
amounting to lack or excess of jurisdiction. CA-G.R. SP No. 99769 and No. 99780 were subsequently consolidated.

On 6 September 2007, the Court of Appeals promulgated its Decision dismissing the Petitions in CA-G.R. SP No.
99769 and No. 99780 for lack of merit, mootness and prematurity. Petitioners in CA-G.R. SP No. 99769 and No.
99780 filed their respective Motions for Reconsideration of the foregoing Decision of the Court of Appeals.

In the meantime, upon the expiration of the TRO issued by RTC Judge Untalan in Civil Case No. 07-610, the Annual
Stockholders’ Meeting of PRCI was again scheduled on 10 October 2007. However, Judge Untalan issued on 8
October 2007 a Resolution granting the issuance of permanent injunction against the defendants until the instant
case is finally resolved.

In its Resolution dated 22 January 2008, the Court of Appeals denied the Motions for Reconsideration of
petitioners. Failing to obtain any relief from the Court of Appeals, petitioners turned to the Supreme Court which
issued a TRO directed against the respondents of G.R. No. 182008, namely, respondents Miguel, et al., and Judge
Untalan.

Pending action on the foregoing incidents, petitioners Santiago Jr., et al., filed before the Court a Manifestation and
Motion to Set Case for Oral Arguments. In their Manifestation, petitioners Santiago Jr., et al., admitted that the
PRCI Board of Directors had already called and set the Annual Stockholders’ Meeting on 18 June 2008, and among
the items on the Agenda for confirmation and approval by the stockholders was the property-for-shares exchange
between PRCI and JTH. Petitioners Santiago Jr., et al., brought to the attention of the Court the fact that on 5 June
2008, another set of minority stockholders of PRCI, namely, Jalane Christie U. Tan, Marilou U. Pua, Aristeo G.
Puyat, and Ricardo S. Parreno (Jalane, et al.) filed with the RTC of Makati a Complaint against petitioners and the
other directors of PRCI and/or JTH, docketed as Civil Case No. 08-458.

Acting on the Complaint of Jalane, et al. in Civil Case No. 08-458, Executive Judge Winlove Dumayas (Executive
Judge Dumayas) of the Makati City RTC issued a 72-hour TRO, enjoining PRCI directors from presenting,
discussing, and ratifying the items in the Agenda for the Annual Stockholders’ Meeting set on 18 June 2008 related
to the property-for-shares exchange between PRCI and JTH. However, upon being apprised of the TRO issued by
this Court on 9 April 2008 in G.R. No. 182008, in relation to Civil Case No. 07-610 pending before the Makati City
RTC, Branch 149, Executive Judge Dumayas gave verbal advice that the Annual Stockholders’ Meeting of PRCI
should proceed on 18 June 2008 as if the 72-hour TRO had not been issued. Consequently, the Annual
Stockholders’ Meeting of PRCI proceeded on 18 June 2008.

The Annual Stockholders’ Meeting of PRCI, held on 18 June 2008, was attended by stockholders with a total of
493,017,509 shares or 86.52% of the outstanding capital stock of PRCI, more than the necessary 2/3 to constitute
a quorum. Discussed in the meeting were the same items, whose presentation to the stockholders was sought to
be enjoined by respondents Miguel, et al., in Civil Case No. 07-610 and by Jalane, et al., in Civil Case No. 08-458.

With 75.23% of the outstanding capital stock of PRCI voting in favor of the exchange of its Makati property for
shares of stock of JTH Davies, the Chairman then declared said motion as carried and approved. Hence, at their
annual meeting on 18 June 2008, the PRCI stockholders had already confirmed and approved the actions and
resolutions of the PRCI Board of Directors, which were to subject matters of Civil Cases No. 07-610 and No. 08-
458. Resultantly, on 7 July 2008, PRCI and JTH duly signed and executed a Deed of Transfer with Subscription
Agreement, covering the exchange of the Makati property of PRCI for shares of stock of JTH.

A. Does the Complaint in Civil Case No. 07-610 filed by Respondents Miguel, et al. constitute a
valid derivative suit? May the case be a derivative suit and at the same time an intracoporate action
arising from devices or schemes employed by the PRCI board of directors amounting to fraud or
misrepresentation?

NO. A derivative suit must be differentiated from individual and representative or class suits, thus:
CORPORATION LAW CASE DIGESTS | 95
FEU JD4401 | 2nd SEMESTER, S.Y. 2017-2018

Suits by stockholders or members of a corporation based on wrongful or fraudulent acts of directors or other
persons may be classified into individual suits, class suits, and derivative suits. Where a stockholder or member is
denied the right of inspection, his suit would be individual, because the wrong is done to him personally and not to
the other stockholders or the corporation. Where the wrong is done to a group of stockholders, as where preferred
stockholders’ rights are violated, a class or representative suit will be proper for the protection of all stockholders
belonging to the same group. But where the acts complained of constitute a wrong to the corporation itself, the
cause of action belongs to the corporation and not to the individual stockholder or member. Although in most every
case of wrong to the corporation, each stockholder is necessarily affected because the value of his interest therein
would be impaired, this fact of itself is not sufficient to give him an individual cause of action since the corporation
is a person distinct and separate from him, and can and should itself sue the wrongdoer. Otherwise, not only would
the theory of separate entity be violated, but there would be multiplicity of suits as well as a violation of the
priority rights of creditors. Furthermore, there is the difficulty of determining the amount of damages that should
be paid to each individual stockholder.

However, in case the majority of the board of directors wastes or dissipates the funds of the corporation or
fraudulently disposes of its properties, or performs ultra vires acts, the court, in the exercise of its equity
jurisdiction, and upon showing that intracorporate remedy is unavailing, will entertain a suit filed by the minority
members of the board of directors, for and in behalf of the corporation, to prevent waste and dissipation and the
commission of illegal acts and otherwise redress the injuries of the minority stockholders against the wrongdoing of
the majority. The action in such a case is said to be brought derivatively in behalf of the corporation to protect the
rights of the minority stockholders thereof. Otherwise stated, where corporate directors are guilty of a breach of
trust — not of mere error of judgment or abuse of discretion — and intracorporate remedy is futile or useless, a
stockholder may institute a suit in behalf of himself and other stockholders and for the benefit of the corporation,
to bring about a redress of the wrong inflicted directly upon the corporation and indirectly upon the stockholders.
In such suits, the suing stockholder is regarded as the nominal party, with the corporation as the party in interest.

A thorough study of the subject Complaint reveals that the distinction is deceptive. The supposed devices and
schemes employed by the PRCI Board of Directors amounting to fraud or misrepresentation are the very same
bases for the derivative suit. They are the very same acts of the PRCI Board of Directors that have supposedly
caused injury to the corporation. In their Complaint, respondents alleged that they are filing the same "as
shareholders, for and in behalf of the Corporation, in order to redress the wrongs committed against the
Corporation and to protect or vindicate corporate rights and to prevent wastage and dissipation of corporate funds
and assets and the further commission of illegal acts by the Board of Directors." Although respondents Miguel, et
al., also aver that they are seeking "redress for the injuries of the minority stockholders against the wrongdoings of
the majority," the rest of the Complaint does not bear this out and is utterly lacking any allegation of injury
personal to them or a certain class of stockholders to which they belong.

To reiterate, a derivative suit, on one hand, and individual and class suits, on the other, are mutually exclusive. A
shareholder's derivative suit seeks to recover for the benefit of the corporation and its whole body of shareholders
when injury is caused to the corporation that may not otherwise be redressed because of failure of the corporation
to act. Thus, ‘the action is derivative, i.e., in the corporate right, if the gravamen of the complaint is injury to the
corporation, or to the whole body of its stock and property without any severance or distribution among individual
holders, or it seeks to recover assets for the corporation or to prevent the dissipation of its assets.’ [Ci tations.]"
(Jones, supra, 1 Cal.3d 93, 106, 81 Cal.Rptr. 592, 460 P.2d 464.) In contrast, "a direct action [is one] filed by the
shareholder individually (or on behalf of a class of shareholders to which he or she belongs) for injury to his or her
interest as a shareholder. ... [¶] ... The two actions are mutually exclusive: i.e., the right of action and recovery
belongs to either the shareholders (direct action) *651 or the corporation (derivative action)." (Friedman, Cal.
Practice Guide: Corporations, supra, ¶ 6:598, p. 6-127.)

Applied in the instant case, considering the allegations in the Complaint of Miguel, et al. in Civil Case No. 07-610,
there is only a derivative suit based on the devices and schemes employed by the PRCI Board of Directors that
amounts to mismanagement, misrepresentation, fraud, and bad faith.

At the crux of the Complaint of respondents Miguel, et al., in Civil Case No. 07-610 is their dissent from the
passage by the majority of the PRCI Board of Directors of the "disputed resolutions," particularly: (1) the
Resolution dated 26 September 2006, authorizing the acquisition by PRCI of up to 100% of the common shares of
JTH; and (2) the Resolution dated 11 May 2007, approving the property-for-shares exchange between PRCI and
JTH.

B. Will the derivative suit, insofar as it concerns the Resolution dated 26 September 2006 of the PRCI
Board of Directors regarding the acquisition of JTH, prosper?

NO. For one, it is dismissible for being moot and academic. The Resolution dated 26 September 2006 of the PRCI
Board of Directors was approved and ratified by the stockholders, holding 74% of the outstanding capital stock in
PRCI, during the Special Stockholders’ Meeting held on 7 November 2006; while Respondents Miguel, et al.
CORPORATION LAW CASE DIGESTS | 96
FEU JD4401 | 2nd SEMESTER, S.Y. 2017-2018

instituted Civil Case No. 07-610 only on 10 July 2007, against herein petitioners Santiago Sr., Santiago Jr.,
Solomon, and Robles, together with Renato de Villa, Lim Teong Leong, Lawrence Lim Swee Lin, Tham Ka Hon, and
Dato Surin Upatkoon, in their capacity as directors of PRCI and/or JTH. Clearly, the acquisition by PRCI of JTH and
the constitution of the JTH Board of Directors are no longer just the acts of the majority of the PRCI Board of
Directors, but also of the majority of the PRCI stockholders. By ratification, even an unauthorized act of an agent
becomes the authorized act of the principal. To declare the Resolution dated 26 September 2006 of the PRCI Board
of Directors null and void will serve no practical use or value, or affect any of the rights of the parties, because the
Resolution dated 7 November 2006 of the PRCI stockholders -- approving and ratifying said acquisition and the
manner in which PRCI shall constitute the JTH Board of Directors -- will still remain valid and binding.

For another, it is dismissible for failure to implead indispensable parties, namely, the majority of the PRCI
stockholders. Rule 3, Section 7 of the Rules of Court defines an indispensable party as a party-in-interest, without
whom there can be no final determination of an action. The interests of such indispensable party in the subject
matter of the suit and the relief are so bound with those of the other parties that his legal presence as a party to
the proceeding is an absolute necessity. As a rule, an indispensable party’s interest in the subject matter is such
that a complete and efficient determination of the equities and rights of the parties is not possible if he is not
joined.

The majority of the stockholders of PRCI are indispensable parties to Civil Case No. 07-610, for they have approved
and ratified during the Special Stockholders’ Meeting on 7 November 2006 the Resolution dated 26 September
2006 of the PRCI Board of Directors. Obviously, no final determination of the validity of the acquisition by PRCI of
JTH or of the constitution of the JTH Board of Directors can be had without consideration of the effect of the
approval and ratification thereof by the majority stockholders.

Respondents Miguel, et al. cannot simply assert that the majority of the PRCI Board of Directors named as
defendants in Civil Case No. 07-610 are also the PRCI majority stockholders, because respondents Miguel, et al.,
explicitly impleaded said defendants in their capacity as directors of PRCI and/or JTH, not as stockholders.

C. Will the derivative suit, insofar as it concerns the Resolution dated 11 May 2007 of the PRCI Board of
Directors regarding the property-for-shares-exchange, prosper?

NO. It is dismissible for lack of cause of action. A stockholder’s right to institute a derivative suit is not based on
any express provision of the Corporation Code or even the Securities Regulation Code, but is impliedly recognized
when the said laws make corporate directors or officers liable for damages suffered by the corporation and its
stockholders for violation of their fiduciary duties. In effect, the suit is an action for specific performance of an
obligation, owed by the corporation to the stockholders, to assist its rights of action when the corporation has been
put in default by the wrongful refusal of the directors or management to adopt suitable measures for its protection.
The basis of a stockholder’s suit is always one of equity. Thus, it cannot prosper without first complying with the
legal requisites for its institution.

Rule 8, Section 1 of the Interim Rules of Procedure for Intra-Corporate Controversies (IRPICC) lays down the
following requirements which a stockholder must comply with in filing a derivative suit:

Sec. 1. Derivative action. – A stockholder or member may bring an action in the name of a corporation or
association, as the case may be, provided, that:

(1) He was a stockholder or member at the time the acts or transactions subject of the action
occurred and at the time the action was filed;
(2) He exerted all reasonable efforts, and alleges the same with particularity in the complaint, to
exhaust all remedies available under the articles of incorporation, by-laws, laws or rules
governing the corporation or partnership to obtain the relief he desires;
(3) No appraisal rights are available for the act or acts complained of; and
(4) The suit is not a nuisance or harassment suit. (Emphasis ours.)

In their Complaint before the RTC in Civil Case No. 07-610, respondents Miguel, et al. made no mention at all of
appraisal rights which could or could not have been available to them. In their Comment on the Petitions at bar,
respondents Miguel, et al. contend that there are no appraisal rights available for the acts complained of, since (1)
the PRCI directors are being charged with mismanagement, misrepresentation, fraud, and breach of fiduciary
duties which are not subject to appraisal rights; (2) appraisal rights will only obtain for acts of the Board of
Directors in good faith; and (3) appraisal rights may be exercised by a stockholder who shall have voted against
the proposed corporate action, and no corporate action has yet been taken herein by PRCI stockholders who still
have not voted on the intended property-for-shares exchange between PRCI and JTH.

The Supreme Court disagreed.


CORPORATION LAW CASE DIGESTS | 97
FEU JD4401 | 2nd SEMESTER, S.Y. 2017-2018

Every derivative suit is necessarily grounded on an alleged violation by the board of directors of its fiduciary duties,
committed by mismanagement, misrepresentation, or fraud, with the latter two situations already implying bad
faith. The import of establishing the availability or unavailability of appraisal rights to the minority stockholder is
further highlighted by the fact that it is one of the factors in determining whether or not a complaint involving an
intra-corporate controversy is a nuisance and harassment suit. The availability or unavailability of appraisal rights
should be objectively based on the subject matter of the complaint, i.e. the specific act or acts performed by the
board of directors, without regard to the subjective conclusion of the minority stockholder instituting the derivative
suit that such act constituted mismanagement, misrepresentation, fraud, or bad faith.

The raison d’etre for the grant of appraisal rights to minority stockholders has been explained thus:

“x x x [Appraisal right] means that a stockholder who dissented and voted against the proposed corporate
action, may choose to get out of the corporation by demanding payment of the fair market value of his
shares. When a person invests in the stocks of a corporation, he subjects his investment to all the risks of
the business and cannot just pull out such investment should the business not come out as he expected.
He will have to wait until the corporation is finally dissolved before he can get back his investment, and
even then, only if sufficient assets are left after paying all corporate creditors. His only way out before
dissolution is to sell his shares should he find a willing buyer. If there is no buyer, then he has no recourse
but to stay with the corporation. However, in certain specified instances, the Code grants the stockholder
the right to get out of the corporation even before its dissolution because there has been a major change
in his contract of investment with which he does not agree and which the law presumes he did not foresee
when he bought his shares. Since the will of two-thirds of the stocks will have to prevail over his
objections, the law considers it only fair to allow him to get back his investment and withdraw from the
corporation. x x x.”

The Corporation Code expressly made appraisal rights available to the dissenting stockholder in the following
instances:

Sec. 42. Power to invest corporate funds in another corporation or business or for any other purpose. –
Subject to the provisions of this Code, a private corporation may invest its funds in any other corporation
or business or for any purpose other than the primary purpose for which it was organized when approved
by a majority of the board of directors or trustees and ratified by the stockholders representing at least
two-thirds (2/3) of the outstanding capital stock, or by at least two-thirds (2/3) of the members in case of
non-stock corporations, at a stockholders’ or members’ meeting duly called for the purpose. Written notice
of the proposed investment and the time and place of the meeting shall be addressed to each stockholder
or member at his place of residence as shown on the books of the corporation and deposited to the
addressee in the post office with postage prepaid, or served personally; Provided, That any dissenting
stockholder shall have appraisal right as provided in this Code: Provided, however, That where the
investment by the corporation is reasonably necessary to accomplish its primary purpose as stated in the
articles of incorporation, the approval of the stockholders or members shall not be necessary.

Sec. 81. Instances of appraisal right. – Any stockholder of a corporation shall have the right to dissent and
demand payment of the fair value of his shares in the following instances:

1. In case any amendment to the articles of incorporation has the effect of changing or restricting
the rights of any stockholders or class of shares, or of authorizing preferences in any respect
superior to those of outstanding shares of any class, or of extending or shortening the term of
corporate existence;
2. In case of sale, lease, exchange, transfer, mortgage, pledge or other disposition of all or
substantially all of the corporate property and assets as provided in this Code; and
3. In case of merger or consolidation.

Respondents Miguel, et al. themselves admitted that the property-for-shares exchange between PRCI and JTH,
approved by majority of the PRCI Board of Directors in the Resolution dated 11 May 2007, involved all or
substantially all of the properties and assets of PRCI. Irrefragably, the property-for-shares exchange between PRCI
and JTH, involving as it did substantially all of the properties and assets of PRCI, qualified as one of the instances
when dissenting stockholders, such as respondents Miguel, et al., could have exercised their appraisal rights. And
contrary to their contentions, respondents Miguel, et al. themselves caused the unavailability of appraisal rights by
filing the Complaint in Civil Case No. 07-610 in which they prayed that the 11 May 2007 Resolution of the Board of
Directors approving the property-for-shares exchange between PRCI and JTH be declared null and void even before
the said Resolution could be presented to the PRCI stockholders for approval or rejection. More than anything, the
argument of respondents Miguel, et al., raises questions of whether their derivative suit was prematurely filed for
they had failed to exert all reasonable efforts to exhaust all other remedies available under the articles of
incorporation, by-laws, laws, or rules governing the corporation or partnership, as required by Rule 8, Section 1(2)
of the IRPICC. The obvious intent behind the rule is to make the derivative suit the final recourse of the
stockholder, after all other remedies to obtain the relief sought have failed.
CORPORATION LAW CASE DIGESTS | 98
FEU JD4401 | 2nd SEMESTER, S.Y. 2017-2018

D. Will the alleged violation of the stockholders’ right to inspect corporate books and records prosper?

NO. The right to information, which includes the right to inspect corporate books and records, is a right personal to
each stockholder. In the instant case only respondent Dulay actually made a demand for a copy of "all the records,
documents, contracts, and agreements, emails, letters, correspondences, relative to the acquisition of JTH x x x."
There is no allegation that his co-respondents (who are his co-plaintiffs in Civil Case No. 07-610) made similar
demands for the inspection or copying of corporate books and records. Only respondent Dulay complied then with
the requirement under Rule 7, Section 2(2) of IRPICC which applies to disputes exclusively involving the rights of
stockholders or members to inspect the books and records and/or to be furnished with the financial statements of a
corporation under Sections 74 and 75 of the Corporation Code. Even so, respondent Dulay’s Complaint should be
dismissed for lack of cause of action, for his demand for copies of pertinent documents relative to the acquisition of
JTH shares was not denied by any of the defendants named in the Complaint in Civil Case No. 07-610, but by Atty.
Jesulito A. Manalo (Manalo), the Corporate Secretary of PRCI, in a letter dated 17 January 2006. Section 74 of the
Corporation Code, the substantive law on which respondent Dulay’s Complaint for inspection and copying of
corporate books and records is based, states that:

Sec. 74. Books to be kept; stock transfer agent. –


xxxx
Any officer or agent of the corporation who shall refuse to allow any director, trustees, stockholder or
member of the corporation to examine and copy excerpts from its records or minutes, in accordance with
the provisions of this Code, shall be liable to such director, trustee, stockholder or member for damages,
and in addition, shall be guilty of an offense which shall be punishable under Section 144 of this Code:
Provided, That if such refusal is pursuant to a resolution or order of the Board of Directors or Trustees, the
liability under this section for such action shall be imposed upon the directors or trustees who voted for
such refusal: x x x.

Based on the foregoing, it is Corporate Secretary Manalo who should be held liable for the supposedly wrongful and
unreasonable denial of respondent Dulay’s demand for inspection and copying of corporate books and records; but,
as previously mentioned, Corporate Secretary Manalo is not among the defendants named in the Complaint in Civil
Case No. 07-610. There is also utter lack of any allegation in the Complaint that Corporate Secretary Manalo
denied respondent Dulay’s demand pursuant to a resolution or order of the PRCI Directors, so that the latter (who
are actually named defendants in the Complaint) could also be held liable for the denial.

E. Will Civil Case No. 08-458 prosper?

NO. The very nature of Civil Case No. 07-610 as a derivative suit bars Civil Case No. 08-458 and warrants the
latter’s dismissal. In Chua v. Court of Appeals, the Supreme Court stressed that the corporation is the real party in
interest in a derivative suit, and the suing stockholder is only a nominal party:

x x x For a derivative suit to prosper, it is required that the minority stockholder suing for and on behalf of the
corporation must allege in his complaint that he is suing on a derivative cause of action on behalf of the corporation
and all other stockholders similarly situated who may wish to join him in the suit. It is a condition sine qua non that
the corporation be impleaded as a party because not only is the corporation an indispensable party, but it is also
the present rule that it must be served with process. The judgment must be made binding upon the corporation in
order that the corporation may get the benefit of the suit and may not bring subsequent suit against the same
defendants for the same cause of action. In other words, the corporation must be joined as party because it is its
cause of action that is being litigated and because judgment must be a res judicata against it.

The reasons given for not allowing direct individual suit are:

(1) x x x "the universally recognized doctrine that a stockholder in a corporation has no title legal or
equitable to the corporate property; that both of these are in the corporation itself for the benefit of the
stockholders." In other words, to allow shareholders to sue separately would conflict with the separate
corporate entity principle;
(2) x x x that the prior rights of the creditors may be prejudiced. Thus, our Supreme Court held in the
case of Evangelista v. Santos, that "the stockholders may not directly claim those damages for themselves
for that would result in the appropriation by, and the distribution among them of part of the corporate
assets before the dissolution of the corporation and the liquidation of its debts and liabilities, something
which cannot be legally done in view of Section 16 of the Corporation Law xxx;"
(3) the filing of such suits would conflict with the duty of the management to sue for the protection of all
concerned;
(4) it would produce wasteful multiplicity of suits; and
(5) it would involve confusion in ascertaining the effect of partial recovery by an individual on the
damages recoverable by the corporation for the same act.
CORPORATION LAW CASE DIGESTS | 99
FEU JD4401 | 2nd SEMESTER, S.Y. 2017-2018

It is for these reasons that the derivative suit, Civil Case No. 08-458, although filed by a different set of minority
stockholders from those in Civil Case No. 07-610, should still not be allowed to proceed.
CORPORATION LAW CASE DIGESTS | 100
FEU JD4401 | 2nd SEMESTER, S.Y. 2017-2018

(58) Legaspi Towers 300, Inc. vs. Muer


G.R. No. 170783 June 18, 2012

Doctrine: Since the real party-in-interest in a derivative suit is the corporation, the reliefs prayed for must be for
the benefit or interest of the corporation. When the reliefs prayed for do not pertain to the corporation, then it is an
improper derivative suit.

The requisites for a derivative suit are as follows:


a) the party bringing suit should be a shareholder as of the time of the act or transaction complained of, the
number of his shares not being material;
b) he has tried to exhaust intra-corporate remedies, i.e., has made a demand on the board of directors for the
appropriate relief but the latter has failed or refused to heed his plea; and
c) the cause of action actually devolves on the corporation, the wrongdoing or harm having been, or being caused
to the corporation and not to the particular stockholder bringing the suit.

Facts: Pursuant to the by-laws of Legaspi Towers 300, Inc., petitioners Lilia Marquinez Palanca, Rosanna D. Imai,
Gloria Domingo and Ray Vincent, the incumbent Board of Directors, set the annual meeting of the members of the
condominium corporation and the election of the new Board of Directors for the years 2004-2005 on April 2, 2004
at 5:00 p.m. at the lobby of Legaspi Towers 300, Inc. The Committee on Elections of Legaspi Towers 300, Inc.,
however, found most of the proxy votes, at its face value, irregular, thus, questionable; and for lack of time to
authenticate the same, petitioners adjourned the meeting for lack of quorum.

However, respondents challenged the adjournment of the meeting. Despite petitioners’ insistence that no quorum
was obtained during the annual meeting held on April 2, 2004, respondents pushed through with the scheduled
election and were elected as the new Board of Directors and officers of Legaspi Towers 300, Inc. Subsequently,
they submitted a General Information Sheet to the SEC with the a new set of officers.

Petitioners filed a Complaint for the Declaration of Nullity of Elections against respondents with the RTC. Before
respondents could file an Answer, petitioners filed an Amended Complaint, which was admitted by the RTC. Again,
before respondents could submit an Answer to the Amended Complaint, petitioners filed an Urgent Ex- Parte
Motion to Admit Second Amended Complaint.

The RTC denied the plaintiffs’ motion to admit amended complaint to include Legaspi Towers 300, Inc. as plaintiff.
Thus, petitioners then filed a petition for certiorari with the Court of Appeals, which was dismissed. The CA held
that the RTC judge did not commit grave abuse of discretion in denying petitioners’ Motion To Admit Second
Amended Complaint. The Court stated that petitioners’ complaint sought to nullify the election of the Board of
Directors held on April 2, 2004, and to protect and enforce their individual right to vote. The appellate court held
that as the right to vote is a personal right of a stockholder of a corporation, such right can only be enforced
through a direct action; hence, Legaspi Towers 300, Inc. cannot be impleaded as plaintiff in this case.

Issue: Whether a derivative suit filed by petitioners in behalf of Legaspi Towers 300 is proper.

Held: No, a derivative suit filed by petitioners in behalf of legaspi Towers 300 is not proper because what the
petitioners seek to protect and enforce is their individual right.

In the Amended Complaint, petitioners stated that they are the incumbent reconstituted Board of Directors of
Legaspi Towers 300, Inc., and that respondents are the newly-elected members of the Board of Directors; while in
the Second Amended Complaint, the plaintiff is Legaspi Towers 300, Inc., represented by petitioners as the
allegedly incumbent reconstituted Board of Directors of Legaspi Towers 300, Inc.

The Court of Appeals is correct in ruling that the Second Amended Complaint is meant to be a derivative suit filed
by petitioners in behalf of the corporation. The CA stated in its Decision that petitioners justified the inclusion of
Legaspi Towers 300, Inc. as plaintiff in Civil Case No. 0410655 by invoking the doctrine of derivative suit, as
petitioners specifically argued.

Since it is the corporation that is the real party-in-interest in a derivative suit, then the reliefs prayed for must be
for the benefit or interest of the corporation. When the reliefs prayed for do not pertain to the corporation, then it
is an improper derivative suit.

The requisites for a derivative suit are as follows:


a) the party bringing suit should be a shareholder as of the time of the act or transaction complained of, the
number of his shares not being material;
b) he has tried to exhaust intra-corporate remedies, i.e., has made a demand on the board of directors for the
appropriate relief but the latter has failed or refused to heed his plea; and
c) the cause of action actually devolves on the corporation, the wrongdoing or harm having been, or being caused
to the corporation and not to the particular stockholder bringing the suit.
CORPORATION LAW CASE DIGESTS | 101
FEU JD4401 | 2nd SEMESTER, S.Y. 2017-2018

In this case, petitioners, as members of the Board of Directors of the condominium corporation before the election
in question, filed a complaint against the newly-elected members of the Board of Directors for the years 2004-
2005, questioning the validity of the election held on April 2, 2004, as it was allegedly marred by lack of quorum,
and praying for the nullification of the said election.

As stated by the Court of Appeals, petitioners’ complaint seek to nullify the said election, and to protect and
enforce their individual right to vote. Petitioners seek the nullification of the election of the Board of Directors for
the years 2004-2005, composed of herein respondents, who pushed through with the election even if petitioners
had adjourned the meeting allegedly due to lack of quorum. Petitioners are the injured party, whose rights to vote
and to be voted upon were directly affected by the election of the new set of board of directors. The party-in-
interest are the petitioners as stockholders, who wield such right to vote. The cause of action devolves on
petitioners, not the condominium corporation, which did not have the right to vote. Hence, the complaint for
nullification of the election is a direct action by petitioners, who were the members of the Board of Directors of the
corporation before the election, against respondents, who are the newly-elected Board of Directors. Under the
circumstances, the derivative suit filed by petitioners in behalf of the condominium corporation in the Second
Amended Complaint is improper.
CORPORATION LAW CASE DIGESTS | 102
FEU JD4401 | 2nd SEMESTER, S.Y. 2017-2018

(59) Ching v. Subic Bay Golf and Country Club, Inc.


G.R. No. 174353, September 10, 2014
By; Arid, Hannah Mhae G.

Dotrine: In filing a derivative suit, stockholder filing it should have exerted all reasonable efforts to exhaust all
remedies available under the articles of incorporation, by-laws, laws or rules governing the corporation or
partnership to obtain the relief he desires; and to allege such fact with particularity in the complaint. The obvious
intent behind the rule is to make the derivative suit the final recourse of the stockholder, after all other remedies to
obtain the relief sought had failed.

Facts: Petitioners Nestor Ching and Andrew Wellington own stocks of the Subic Bay Golf and Country Club,
Inc.(SBGCCI). On June 27, 1996, Securities and Exchange Commission (SEC) approved amendments to SBGCCI
Articles of Incorporation which the petitioners alleged such amendments made their shares non-proprietary.
Petitioners further alleged that this change was made without the appropriate disclosure of SBGCCI to its
shareholders. Furthermore, petitioners allege several instances of fraud committed by SBGCCI’s board of directors
in its February 26, 2003 complaint. Respondent’s answered the complaint by refuting allegations made by
petitioners. As a way of defense, respondents underscored petitioners’ failure to: show that it was authorized by
SBGSI to file complaint on behalf of the company and that they failed to comply with the requisites for filing a
derivative suit and an action for receivership to justify their prayer for injunctive relief since the complaint may be
considered a nuisance or harassment suit Thus, respondents prayed for dismissal of the complaint.

RTC: held that the action is a derivative suit and that the complaint was intended not only for the benefit of the
two petitioners. This is apparent from the caption of the case which reads Nestor Ching, Andrew Wellington and the
Subic Bay Golfers and Shareholders, Inc., for and in behalf of all its members as petitioners. However, the lower
court held that petitioners failed to exhaust their remedies within the respondent corporation itself hence, it issued
an order dismissing the complaint.

CA: affirmed the RTC.

Issue: (a) Is the suit one of a derivative suit?


(b) Should the complaint be dismissed for non-exhaustion of intra-corporate remedies?

Held: (a)YES. A shareholder’s derivative suit seeks to recover for the benefit of the corporation and its whole body
of shareholders when injury is caused to the corporation that may not otherwise be redressed because of failure of
the corporation to act. Thus, ‘the action is derivative, i.e., in the corporate right, if the gravamen of the complaint
is injury to the corporation, or to the whole body of its stock and property without any severance or distribution
among individual holders, or it seeks to recover assets for the corporation or to prevent the dissipation of its
assets.

The reliefs sought in the Complaint, namely that of enjoining defendants from acting as officers and Board of
Directors of the corporation, the appointment of a receiver, and the prayer for damages in the amount of the
decrease in the value of the shares of stock, clearly show that the Complaint was filed to curb the alleged
mismanagement of SBGCCI. The causes of action pleaded by petitioners do not accrue to a single shareholder or a
class of shareholders but to the corporation itself.

However, as minority stockholders, petitioners do not have any statutory right to override the business judgments
of SBGCCI’s officers and Board of Directors on the ground of the latter’s alleged lack of qualification to manage a
golf course. Contrary to the arguments of petitioners, Presidential Decree No. 902-A, which is entitled
REORGANIZATION OF THE SECURITIES AND EXCHANGE COMMISSION WITH ADDITIONAL POWERS AND PLACING
THE SAID AGENCY UNDER THE ADMINISTRATIVE SUPERVISION OF THE OFFICE OF THE PRESIDENT, does not
grant minority stockholders a cause of action against waste and diversion by the Board of Directors, but merely
identifies the jurisdiction of the SEC over actions already authorized by law or jurisprudence. It is settled that a
stockholder’s right to institute a derivative suit is not based on any express provision of the Corporation Code, or
even the Securities Regulation Code, but is impliedly recognized when the said laws make corporate directors or
officers liable for damages suffered by the corporation and its stockholders for violation of their fiduciary duties.

(b) YES, the complaint should be dismissed for non-exhaustion of intra-corporate remedies.

Section 1, Rule 8 of the Interim Rules of Procedure Governing Intra-Corporate Controversies imposes the following
requirements for derivative suits:
(1) He was a stockholder or member at the time the acts or transactions subject of the action occurred
and at the time the action was filed;
(2) He exerted all reasonable efforts, and alleges the same with particularity in the complaint, to exhaust
all remedies available under the articles of incorporation, by-laws, laws or rules governing the corporation
or partnership to obtain the relief he desires;
(3) No appraisal rights are available for the act or acts complained of; and
CORPORATION LAW CASE DIGESTS | 103
FEU JD4401 | 2nd SEMESTER, S.Y. 2017-2018

(4) The suit is not a nuisance or harassment suit.

While the RTC dismissed the Complaint for failure to comply with the second and fourth requisites above, the
Supreme Court held that upon a careful examination of the Complaint, the same should not have been dismissed
on the ground that it is a nuisance or harassment suit. Although the shareholdings of petitioners are indeed only
two out of the 409 alleged outstanding shares or 0.24%, the Court has held that it is enough that a member or a
minority of stockholders file a derivative suit for and in behalf of a corporation. As regards the second requisite,
the Supreme Court held that that petitioners failed to state with particularity in the Complaint that
they had exerted all reasonable efforts to exhaust all remedies available under the articles of
incorporation, by-laws, and laws or rules governing the corporation to obtain the relief they desire. The
Complaint contained no allegation whatsoever of any effort to avail of intra-corporate remedies.
Indeed, even if petitioners thought it was futile to exhaust intra-corporate remedies, they should have
stated the same in the Complaint and specified the reasons for such opinion. Failure to do so allows the
RTC to dismiss the Complaint, even motu proprio, in accordance with the Interim Rules. The
requirement of this allegation in the Complaint is not a useless formality which may be disregarded at
will.

The wordings of Section 1, Rule 8 of the Interim Rules of Procedure Governing Intra-Corporate Controversies are
simple and do not leave room for statutory construction. The second paragraph thereof requires that the
stockholder filing a derivative suit should have exerted all reasonable efforts to exhaust all remedies
available under the articles of incorporation, by-laws, laws or rules governing the corporation or
partnership to obtain the relief he desires; and to allege such fact with particularity in the complaint.
The obvious intent behind the rule is to make the derivative suit the final recourse of the stockholder,
after all other remedies to obtain the relief sought had failed.
CORPORATION LAW CASE DIGESTS | 104
FEU JD4401 | 2nd SEMESTER, S.Y. 2017-2018

(60) Villamor, Jr. v. Umale


G.R. Nos. 172843, September 24, 2014
By: Bernardo, Michael Gerard T.

Doctrine: Refer to relevant discussions in this case section.

Facts: Pasig Printing Corporation (PPC) obtained an option to lease Mid-Pasig’s property. PPC’s board of directors
issued a resolution waiving all its rights, interests and participation in the option to lease Mid-Pasig’s property in
favor of Villamor. PPC received no consideration for this waiver in favor of Villamor. PPC represented by Villamor
entered into a Memorandum of Agreement with MC Home Depot under which it will continue to occupy the area as
PPC’s sub-leasee for 4 years at monthly rental of P4.5 million plus goodwill of P18 million. MC Home Depot issued
postdated checks for the rentals and goodwill and gave them to Villamor who did not turn over to PPC the amount
of the checks upon encashment. Balmores, a stockholder and director of PPC wrote a letter to the directors of PPC
informing them that Villamor should be made deliver to PPC the value of the checks issued by MC Home Depot.
Due to inaction of the directors, Balmores filed an intra-corporate controversy complaint with the Regional Trial
Court against the directors and Villamor.

Issue: Whether or not the action is a derivative suit.

Held: No. The action filed by Balmores was not a derivative suit. In derivative suits, the real party-in-interest is
the corporation, and the suing stockholder is a mere nominal party. Balmores failed to show that he exhausted all
administrative remedies. Though he tried to communicate with PPC’s directors about the checks in Villamor’s
possession before he filed an action, Balmores was not able to show that this comprised all the remedies available
under the articles of incorporation, by-laws, laws or rules governing PPC. Balmores also did not implead PPC as a
party in the case nor did he allege that he was filing on behalf of the corporation.

Relevant discussions in this case:


What is a derivative suit? A derivative suit is an action filed by stockholders to enforce a corporate action. It is an
exception to the general rule that the corporation’s power to sue is exercised only by the board of directors or
trustees. Individual stockholders may be allowed to sue on behalf of the corporation whenever the directors or
officers of the corporation refuse to sue to vindicate the rights of the corporation. It is allowed when the “directors
(or officers) are guilty of breach of trust, not of mere error of judgment. In derivative suits, the real party-in-
interest is the corporation, and the suing stockholder is a mere nominal party.

What are the elements of derivative suit? A stockholder or member may bring an action in the name of a
corporation or association, as the case may be, provided that: (1) He was a stockholder or member at the time the
acts or transactions subject of the action occurred and at the time of the action was filed; (2) He exerted all
reasonable efforts, and alleges the same with particularity in the complaint, to exhaust all remedies available under
the articles of incorporation, by-laws of the corporation to obtain the relief he desires; (3) No appraisal rights are
available for the acts complained of and (4) The suit is not a nuisance or harassment suit, and (5) The action
brought by the stockholder or member must be in the name of the corporation or association.

What are the reasons for disallowing individual suits to enforce remedies for the corporation? The reasons for
disallowing direct individual suit are: (1) A stockholder in a corporation has no title legal or equitable to the
corporate property; to allow shareholders to sue separately would conflict with the separate corporate entity
principle; (2) Prior rights of the creditors may be prejudiced; (3) Filing of such suit would conflict with the duty of
the management to sue; (3) would cause multiplicity of suits, and (5) would cause confusion in ascertaining the
effect of partial recovery by an individual on the damages recoverable by the corporation for the same act.
CORPORATION LAW CASE DIGESTS | 105
FEU JD4401 | 2nd SEMESTER, S.Y. 2017-2018

(61) Lorell Lim Po vs. DOJ


G.R. No. 195198, February 11, 2013
By: Donna Bigornia

Topic: Refusal to allow inspection and copying of the corporate books.

Doctrine: The requisites in order for the penal provision under Section 144 of the Corporation Code to
apply in a case of violation of a stockholder or members right to inspect the corporate books/records
as provided for under Section 74 of the Corporation Code, are the following: a) A director, trustee,
stockholder or member has made a prior demand in writing for a copy of excerpts from the
corporations records or minutes; and b) Any officer or agent of the concerned corporation shall refuse
to allow the said director, trustee, stockholder or member of the corporation to examine and copy said
excerpts.

Facts: Herein private respondent, Tan, is a stockholder of Coastal Highpoint Ventures, Inc. (CHVI), a real estate
development company. Antonio Ng Chiu is its President, while petitioner Po is Chiu’s personal accountant.
Respondent Tan filed a complaint against Chiu and petitioner Po for violation of Section 74(2), in relation to section
144 of the Corporation Code for the latters’ refusal to allow the former to inspect the corporate books of CHVI
despite respondent’s Tan repeated written requests for copies of financial statements and inspection of corporate
books. On the other hand, the petitioners argued that the complainant through the inspection team was allowed to
view/inspect, which was however limited only to the following, books of accounts for 2006 to 2007 with carry
forward balances and not detailed scheduled of accounts except bank reconciliation.

Issue: Whether or not the petitioner violated Section 74(2) in relation to section 144 of the Corporation
Code.

Ruling: Yes.

The requisites in order for the penal provision under Section 144 of the Corporation Code to apply in a
case of violation of a stockholder or members right to inspect the corporate books/records as provided for under
Section 74 of the Corporation Code, are enumerated in the recent case of Sy Tiong Shiou, et al. vs. Sy Chim, et al.,
citing Ang-Baya, et al. vs. Ang, to wit:

a. A director, trustee, stockholder or member has made a prior demand in writing for a copy of excerpts from
the corporation’s records or minutes; and

b. Any officer or agent of the concerned corporation shall refuse to allow the said director, trustee, stockholder
or member of the corporation to examine and copy said excerpts.
In this case, the requisites mentioned above are present. It is noted that private respondent had repeatedly
expressed in writing his request to inspect CHVIs corporate books and records but his written requests were turned
down on the pretext that the petitioner. The initial written demand was made on October 10, 2007 but it was only
on April 24, 2008 that the audit team sent by the private respondent was able to inspect some of the documents of
CHVI. However, it appears that the inspection was ineffective since the petitioner and Loreli Lim Po refused to
present the other documents demanded by the inspection team. PO even prevented the team from copying the
corporate books and records.

There is ample evidence on record to support the said decision. Those who were part of the Inspection Team sent
by Tan to CHVI, executed a Joint Affidavit stating that the documents made available to them for inspection were
limited. Further, they claimed that on the day of the inspection, they brought a portable photocopying machine to
CHVIs premises but they were not allowed to use the same. The offense punishable under Section 74, in relation to
Section 144 of the Corporation Code, for which Chiu was indicted, requires the unjustified disallowance or refusal
by a suspect, of a stockholders written request to examine or copy excerpts of a corporations books or minutes.
The absence of any ascribed ill motives on the part of the aforementioned accountants to make statements adverse
or unfavorable to Chiu lends credibility to their declarations.
CORPORATION LAW CASE DIGESTS | 106
FEU JD4401 | 2nd SEMESTER, S.Y. 2017-2018

(62) Yujuico v. Quiambao


G.R. No. 180416, June 2, 2014
By: Corona, Jose Enrico V.

Dotrine: It is clear then that a criminal action based on the violation of the second or fourth paragraphs of Section
74 can only be maintained against corporate officers or such other persons that are acting on behalf of the
corporation. Violations of the second and fourth paragraphs of Section 74 contemplates a situation wherein a
corporation, acting thru one of its officers or agents, denies the right of any of its stockholders to inspect the
records, minutes and the stock and transfer book of such corporation.

Facts: Yujuico was elected as President and Chairman of STRADEC replacing the respondent Quiambao. Yujuico
appointed Sumbilla as treasurer and Blando as corporate secretary. Petitioners filed a criminal complaint against
respondent Casanova.

Petitioners theorize that the refusal by the respondents and Casanova to turnover STRADEC's corporate records
and stock and transfer book violates their right, as stockholders, directors and officers of the corporation, to
inspect such records and book under Section 7 4 of the Corporation Code. For such violation, petitioners conclude,
respondents may be held criminally liable pursuant to Section 144 of the Corporation Code.

It was alleged that respondents refused to turnover corporate records such as ledgers, journals, accounting files,
and other records. It was also alleged that the respondent caused the removal of the corporate records from the
company’s office. This alleged acts were said to be done after the election of Yujuico.

Respondents were charged for (1) for removing the stock and transfer book of STRADEC from its principal office,
and (2) for refusing access to, and examination of, the corporate records and the stock and transfer book of
STRADEC at its principal office.

RTC issued an order dismissing the Criminal Case regarding the refusing access to corporate records. RTC opined
that refusal to allow inspection of the stock and transfer book of a corporation is not a punishable offense under the
Corporation Code.

Issue: Is the act of refusing to allow inspection of the stock and transfer book of a corporation a punishable
offense under the Corporation Code.

Held: Yes. However, the dismissal of the Criminal Case must be sustained.

A criminal action based on the violation of a stockholder's right to examine or inspect the corporate records and the
stock and transfer book of a corporation under the second and fourth paragraphs of Section 74 of the Corporation
Code-such as Criminal Case No. 89724--can only be maintained against corporate officers or any other persons
acting on behalf of such corporation. The submissions of the petitioners during the preliminary investigation,
however, clearly suggest that respondents are neither in relation to STRADEC.

Section 74. Books to be kept; stock transfer agent. - Every corporation shall keep and carefully preserve at its
principal office a record of all business transactions and minutes of all meetings of stockholders or members, or of
the board of directors or trustees, in which shall be set forth in detail the time and place of holding the meeting,
how authorized, the notice given, whether the meeting was regular or special, if special its object, those present
and absent, and every act done or ordered done at the meeting. Upon the demand of any director, trustee,
stockholder or member, the time when any director, trustee, stockholder or member entered or left the meeting
must be noted in the minutes; and on a similar demand, the yeas and nays must be taken on any motion or
proposition, and a record thereof carefully made. The protest of any director, trustee, stockholder or member on
any action or proposed action must be recorded in full on his demand.

The records of all business transactions of the corporation and the minutes of any meetings shall be open to
inspection by any director, trustee, stockholder or member of the corporation at reasonable hours on business days
and he may demand, in writing, for a copy of excerpts from said records or minutes, at his expense.

Any officer or agent of the corporation who shall refuse to allow any director, trustees, stockholder or member of
the corporation to examine and copy excerpts from its records or minutes, in accordance with the provisions of this
Code, shall be liable to such director, trustee, stockholder or member for damages, and in addition, shall be guilty
of an offense which shall be punishable under Section 144 of this Code: Provided, That if such refusal is made
pursuant to a resolution or order of the board of directors or trustees, the liability under this section for such action
shall be imposed upon the directors or trustees who voted for such refusal: and Provided, further, That it shall be a
defense to any action under this section that the person demanding to examine and copy excerpts from the
corporation's records and minutes has improperly used any information secured through any prior examination of
the records or minutes of such corporation or of any other corporation, or was not acting in good faith or for a
legitimate purpose in making his demand.
CORPORATION LAW CASE DIGESTS | 107
FEU JD4401 | 2nd SEMESTER, S.Y. 2017-2018

Section 144. Violations of the Code. - Violations of any of the provisions of this Code or its amendments not
otherwise specifically penalized therein shall be punished by a fine of not less than one thousand (₱1,000.00) pesos
but not more than ten thousand (₱10,000.00) pesos or by imprisonment for not less than thirty (30) days but not
more than five (5) years, or both, in the discretion of the court. If the violation is committed by a corporation, the
same may, after notice and hearing, be dissolved in appropriate proceedings before the Securities and Exchange
Commission: Provided, That such dissolution shall not preclude the institution of appropriate action against the
director, trustee or officer of the corporation responsible for said violation: Provided, further, That nothing in this
section shall be construed to repeal the other causes for dissolution of a corporation provided in this Code.

It is clear then that a criminal action based on the violation of the second or fourth paragraphs of Section 74 can
only be maintained against corporate officers or such other persons that are acting on behalf of the corporation.
Violations of the second and fourth paragraphs of Section 74 contemplates a situation wherein a corporation, acting
thru one of its officers or agents, denies the right of any of its stockholders to inspect the records, minutes and the
stock and transfer book of such corporation.

The problem with the petitioners' complaint and the evidence that they submitted during preliminary investigation
is that they do not establish that respondents were acting on behalf of STRADEC. Quite the contrary, the scenario
painted by the complaint is that the respondents are merely outgoing officers of STRADEC.
CORPORATION LAW CASE DIGESTS | 108
FEU JD4401 | 2nd SEMESTER, S.Y. 2017-2018

(63) Terelay Investment and Development Corp vs Yulo


G.R. No. 160924. August 5, 2015
By: Kathrina De Castro

DOCTRINE: The Corporation Code has granted to all stockholders the right to inspect the corporate books and
records, and in so doing has not required any specific amount of interest for the exercise of the right to inspect.

Cecilia Yulo wrote a letter addressed to Terelay requesting that she be allowed to examine its books and records on
September 17, 1999 at 1:30 o’clock in the afternoon at the latter’s office on the 25th floor, Citibank Tower, Makati
City. TERELAY denied the request for inspection and instead demanded that she show proof that she was a bona
fide stockholder. Cecilia Yulo again sent another letter clarifying that her request for examination of the corporate
records was for the purpose of inquiring into the financial condition of TERELAY and the conduct of its affairs by the
principal officers. The following day, Cecilia Yulo received a faxed letter from TERELAY’s counsel advising her not to
continue with the inspection in order to avoid trouble.

Cecilia Yulo filed with the Securities and Exchange Commission (SEC), a Petition for Issuance of a Writ of
Mandamus with prayer for Damages against TERELAY. In her petition, she prayed that judgment be rendered
ordering TERELAY to allow her to inspect its corporate records, books of account and other financial records.
Following the enactment of Republic Act No. 8799 (The Securities Regulation Code), the case was transferred from
the Securities and Exchange Commission to the RTC. Accordingly, petitioner’s application for inspection of
corporate records is granted by the RTC and was affirmed by the CA.

A. Whether respondent is a stockholder entitled to inspect Terelay books and records, and should be
allowed to inspect its corporate records despite her shareholding being a measly .001% interest

YES. Terelay argued that Yulo was registered in the Articles of Incorporation is separate and distinct from being so
in the stock and transfer book. TERELAY's argument cannot be sustained. A careful review of the records would
show that in the Preliminary Conference both parties represented by their respective counsels, agreed on the fact
that petitioner-appellee was "registered as a stockholder in respondent-appellant's stock and transfer book subject
to the qualifications in the Answer." The records failed to disclose any objection by TERELAY. Neither did TERELAY
raise this matter in the SEC hearing held on August 7, 2000 as one of the issues to be determined and resolved.

It further argued that, Yulo’s name as incorporator, stockholder and director in the Articles of Incorporation and
Amendments were unsigned; that she did not pay for the five (5) shares appearing in the Amended Articles of
Incorporation and General Information Sheet of TERELAY, that she did not subscribe to the shares; that she has
neither been in possession of nor seen the certificate of stock covering the five (5) shares of stock and the donation
of the five (5) shares claimed by her was null and void for failure to comply with the requisites of a donation under
Art. 748 of the Civil Code; and that there was no acceptance of the donation by her as donee. The court did not
agree. The records disclose that the corporate documents submitted, which include the Articles of Incorporation
and the Amended Articles of Incorporation, as well as the General Information Sheets and the Quarterly Reports all
bear the signatures of the proper parties and their authorized custodians. The signature of appellee under the
name Cecilia J. Yulo appears in the Articles of Incorporation of TERELAY. Likewise, her signatures under the name
Cecilia Y. Blancaflor appear in the Amended Articles of Incorporation where she signed as Director and Corporate
Secretary of TERELAY. The General Information Sheets all exhibited that she was recognized as director and
corporate secretary, and that she had subscribed to five (5) shares of stock. The quarterly reports do not show
otherwise. Yulo has thus presented enough evidence that she is a stockholder of TERELAY. The corporate
documents presented support her claim that she is a registered stockholder in TERELAY's stock and transfer book
thus giving her the right, under Section 74 par. 2 and Section 75 of the Philippine Corporation Law, to inspect
TERELAY's books, records, and financial statements. Secondly, TERELAY's submission that the respondent's
"insignificant holding" of only .001% of the petitioner's stockholding did not justify the granting of her application
for inspection of the corporate books and records is unwarranted. The Corporation Code has granted to all
stockholders the right to inspect the corporate books and records, and in so doing has not required any specific
amount of interest for the exercise of the right to inspect. Ubi lex non distinguit nee nos distinguere debemos.
When the law has made no distinction, we ought not to recognize any distinction. Neither could the petitioner
arbitrarily deny the respondent's right to inspect the corporate books and records on the basis that her inspection
would be used for a doubtful or dubious reason. Under Section 74, third paragraph, of the Corporation Code, the
only time when the demand to examine and copy the corporation's records and minutes could be refused is when
the corporation puts up as a defense to any action that "the person demanding" had "improperly used any
information secured through any prior examination of the records or minutes of such corporation or of any other
corporation, or was not acting in good faith or for a legitimate purpose in making his demand."

The right of the shareholder to inspect the books and records of the petitioner should not be made subject to the
condition of a showing of any particular dispute or of proving any mismanagement or other occasion rendering an
examination proper, but if the right is to be denied, the burden of proof is upon the corporation to show that the
purpose of the shareholder is improper, by way of defense.
CORPORATION LAW CASE DIGESTS | 109
FEU JD4401 | 2nd SEMESTER, S.Y. 2017-2018

7. MERGER AND CONSOLIDATION

(64) Bank of Philippine Islands v. BPI Employees Union


G.R. No. 164301, October 19, 2011
By: Gaite, Rhio Angeline

Doctrine: Section 80 of the Corporation Code commands, as the surviving corporation has the legal obligation to
assume all the obligations and liabilities of the merged constituent corporation.

The legal fiction in the law on mergers (that the surviving corporation continues the corporate existence of the non-
surviving corporation) is mainly a tool to adjudicate the rights and obligations between and among the merged
corporations and the persons that deal with them. Such a legal fiction cannot be unduly extended to an
interpretation of a Union Shop Clause so as to defeat its purpose under labor law.

Facts: BPI moves for reconsideration of our Decision dated August 10, 2010, holding that former employees of the
Far East Bank and Trust Company (FEBTC) "absorbed" by BPI pursuant to the two banks’ merger in 2000 were
covered by the Union Shop Clause in the then existing collective bargaining agreement (CBA) of BPI with
respondent BPI Employees Union-Davao Chapter-Federation of Unions in BPI Unibank (the Union).

To recall, the Union Shop Clause involved in this long standing controversy provided, thus: “Section 2. Union Shop
- New employees falling within the bargaining unit as defined in Article I of this Agreement, who may hereafter be
regularly employed by the Bank shall, within thirty (30) days after they become regular employees, join the Union
as a condition of their continued employment. It is understood that membership in good standing in the Union is a
condition of their continued employment with the Bank.”

In opposition to petitioner’s arguments, the Union, in turn, adverts to our discussion in the August 10, 2010
Decision regarding the voluntary nature of the merger between BPI and FEBTC, the lack of an express stipulation in
the Articles of Merger regarding the transfer of employment contracts to the surviving corporation, and the
consensual nature of employment contracts as valid bases for the conclusion that former FEBTC employees should
be deemed new employees.10 The Union argues that the creation of employment relations between former FEBTC
employees and BPI (i.e., BPI’s selection and engagement of former FEBTC employees, its payment of their wages,
power of dismissal and of control over the employees’ conduct) occurred after the merger, or to be more precise,
after the Securities and Exchange Commission’s (SEC) approval of the merger.

Issue: Whether the employment contracts are automatically assumed by the surviving corporation in a merger,
even in the absence of an express stipulation in the articles of merger or the merger plan

Held: Yes. As Justice Brion stated in his dissenting opinion that due consideration of Section 80 of the Corporation
Code, the constitutionally declared policies on work, labor and employment, and the specific FEBTC-BPI situation —
i.e., a merger with complete "body and soul" transfer of all that FEBTC embodied and possessed and where both
participating banks were willing (albeit by deed, not by their written agreement) to provide for the affected human
resources by recognizing continuity of employment — should point this Court to a declaration that in a complete
merger situation where there is total takeover by one corporation over another and there is silence in the merger
agreement on what the fate of the human resource complement shall be, the latter should not be left in legal limbo
and should be properly provided for, by compelling the surviving entity to absorb these employees. This is what
Section 80 of the Corporation Code commands, as the surviving corporation has the legal obligation to assume all
the obligations and liabilities of the merged constituent corporation.

Not to be forgotten is that the affected employees managed, operated and worked on the transferred assets and
properties as their means of livelihood; they constituted a basic component of their corporation during its
existence. In a merger and consolidation situation, they cannot be treated without consideration of the applicable
constitutional declarations and directives, or, worse, be simply disregarded. If they are so treated, it is up to this
Court to read and interpret the law so that they are treated in accordance with the legal requirements of mergers
and consolidation, read in light of the social justice, economic and social provisions of our Constitution. Hence,
there is a need for the surviving corporation to take responsibility for the affected employees and to absorb them
into its workforce where no appropriate provision for the merged corporation's human resources component is
made in the Merger Plan.

By upholding the automatic assumption of the non-surviving corporation’s existing employment contracts by the
surviving corporation in a merger, the Court strengthens judicial protection of the right to security of tenure of
employees affected by a merger and avoids confusion regarding the status of their various benefits which were
among the chief objections of our dissenting colleagues. However, nothing in this Resolution shall impair the right
of an employer to terminate the employment of the absorbed employees for a lawful or authorized cause or the
right of such an employee to resign, retire or otherwise sever his employment, whether before or after the merger,
subject to existing contractual obligations. x x x
CORPORATION LAW CASE DIGESTS | 110
FEU JD4401 | 2nd SEMESTER, S.Y. 2017-2018

Notwithstanding this concession, we find no reason to reverse our previous pronouncement that the absorbed
FEBTC employees are covered by the Union Shop Clause.

Even in our August 10, 2010 Decision, we already observed that the legal fiction in the law on mergers (that the
surviving corporation continues the corporate existence of the non-surviving corporation) is mainly a tool to
adjudicate the rights and obligations between and among the merged corporations and the persons that deal with
them. Such a legal fiction cannot be unduly extended to an interpretation of a Union Shop Clause so as to defeat its
purpose under labor law. Hence, we stated in the Decision that:

In any event, it is of no moment that the former FEBTC employees retained the regular status that they possessed
while working for their former employer upon their absorption by petitioner. This fact would not remove them from
the scope of the phrase "new employees" as contemplated in the Union Shop Clause of the CBA, contrary to
petitioner's insistence that the term "new employees" only refers to those who are initially hired as non-regular
employees for possible regular employment.

Although by virtue of the merger BPI steps into the shoes of FEBTC as a successor employer as if the former had
been the employer of the latter’s employees from the beginning it must be emphasized that, in reality, the legal
consequences of the merger only occur at a specific date, i.e., upon its effectivity which is the date of approval of
the merger by the SEC. Thus, we observed in the Decision that BPI and FEBTC stipulated in the Articles of Merger
that they will both continue their respective business operations until the SEC issues the certificate of merger and
in the event no such certificate is issued, they shall hold each other blameless for the non-consummation of the
merger. We likewise previously noted that BPI made its assignments of the former FEBTC employees effective on
April 10, 2000, or after the SEC approved the merger. In other words, the obligation of BPI to pay the salaries and
benefits of the former FEBTC employees and its right of discipline and control over them only arose with the
effectivity of the merger. Concomitantly, the obligation of former FEBTC employees to render service to BPI and
their right to receive benefits from the latter also arose upon the effectivity of the merger. What is material is that
all of these legal consequences of the merger took place during the life of an existing and valid CBA between BPI
and the Union wherein they have mutually consented to include a Union Shop Clause.
CORPORATION LAW CASE DIGESTS | 111
FEU JD4401 | 2nd SEMESTER, S.Y. 2017-2018

(65) BPI vs. LEE


G.R. No. 190144 August 1, 2012
By: Grande, Jonicocel

Doctrine: It should be emphasized that a merger of two corporations produces, among others, the following
effects: 1. The constituent corporations shall become a single corporation which, in case of merger, shall be the
surviving corporation designated in the plan of merger; and in case of consolidation, shall be the consolidated
corporation designated in the plan of consolidation; 2. The surviving or consolidated corporation shall be
responsible and liable for all the liabilities and obligations of each of the constituent corporations in the same
manner as if such surviving or consolidated corporation had itself incurred such liabilities or obligations; and any
pending claim, action or proceeding brought by or against any of such constituent corporations may be prosecuted
by or against the surviving or consolidated corporation. The rights of creditors or liens upon the property of any of
such constituent corporations shall not be impaired by such merger or consolidation.

Facts:
Respondent Carlito Lee (Lee) filed a complaint for sum of money with damages and application for the issuance of
a writ of attachment against Trendline. RTC issued a writ of preliminary attachment whereby the Savings Accounts
of Trendline with Citytrust Bank.
Citytrust and BPI merged, with the latter as the surviving corporation. Lee filed a Motion for Execution and/or
Enforcement of Garnishment seeking to enforce against BPI the garnishment of Trendline’s deposit.
BPI insists that it cannot be considered as a party to the case by virtue of its merger with Citytrust, the garnishee
of trendline’s deposits.

Issue: WON BPI became a party-in-interest in the case upon the approval of the SEC of its merger with Citytrust
Bank.

Held: YES.

Section 5, Rule 65 of the Revised Rules of Court requires that persons interested in sustaining the proceedings in
court must be impleaded as private respondents. Upon the merger of Citytrust and BPI, with the latter as the
surviving corporation, and with all the liabilities and obligations of Citytrust transferred to BPI as if it had incurred
the same, BPI undoubtedly became a party interested in sustaining the proceedings, as it stands to be prejudiced
by the outcome of the case.

It is a settled rule that upon service of the writ of garnishment, the garnishee becomes a "virtual party" or "forced
intervenor" to the case and the trial court thereby acquires jurisdiction to bind the garnishee to comply with its
orders and processes.
Citytrust, therefore, upon service of the notice of garnishment and its acknowledgment that it was in possession of
trenline’s deposit accounts in its letter-reply, became a "virtual party" to or a "forced intervenor" in the civil case.
As such, it became bound by the orders and processes issued by the trial court despite not having been properly
impleaded therein. Consequently, by virtue of its merger with BPI as the surviving corporation, effectively became
the garnishee, thus the "virtual party" to the civil case.

Corollarily, it should be emphasized that a merger of two corporations produces, among others, the following
effects:
1. The constituent corporations shall become a single corporation which, in case of merger, shall be the
surviving corporation designated in the plan of merger; and in case of consolidation, shall be the consolidated
corporation designated in the plan of consolidation;
2. The surviving or consolidated corporation shall be responsible and liable for all the liabilities and
obligations of each of the constituent corporations in the same manner as if such surviving or consolidated
corporation had itself incurred such liabilities or obligations; and any pending claim, action or proceeding brought
by or against any of such constituent corporations may be prosecuted by or against the surviving or consolidated
corporation. The rights of creditors or liens upon the property of any of such constituent corporations shall not be
impaired by such merger or consolidation.

In sum, although Citytrust was dissolved, no winding up of its affairs or liquidation of its assets, privileges, powers
and liabilities took place. As the surviving corporation, BPI simply continued the combined businesses of the two
banks and absorbed all the rights, privileges, assets, liabilities and obligations of Citytrust, including the latter’s
obligation over the garnished deposits of the defendants.
CORPORATION LAW CASE DIGESTS | 112
FEU JD4401 | 2nd SEMESTER, S.Y. 2017-2018

(66) Bank of Commerce v. Radio Philippines


G.R. No. 157479, April 21, 2014
By: Jovero, John Tristram V.

Topic: Merger and Consolidation; De Facto Merger

Doctrine: No merger or consolidation will take place if the records do not show any plan or articles of merger or
consolidation, or that SEC did not issue any certificate of merger or consolidation. De facto merger can be pursued
by one corporation acquiring all or substantially all of the properties of another corporation in exchange of shares
of stock of the acquiring corporation.

Facts: In late 2001 the Traders Royal Bank (TRB) proposed to sell to petitioner Bank of Commerce (Bancommerce)
for ₱10.4 billion its banking business consisting of specified assets and liabilities. Bancommerce agreed subject to
prior Bangko Sentral ng Pilipinas' (BSP's) approval of their Purchase and Assumption (P & A) Agreement. On
November 8, 2001 the BSP approved that agreement subject to the condition that Bancommerce and TRB would
set up an escrow fund of PSO million with another bank to cover TRB liabilities for contingent claims that may
subsequently be adjudged against it, which liabilities were excluded from the purchase.

To comply with the BSP mandate, on December 6, 2001 TRB placed ₱50 million in escrow with Metropolitan Bank
and Trust Co. (Metrobank) to answer for those claims and liabilities that were excluded from the P & A Agreement
and remained with TRB.

Shortly after or on October 10, 2002, acting in G.R. 138510, Traders Royal Bank v. Radio Philippines Network
(RPN), Inc., this Court ordered TRB to pay respondents RPN, Intercontinental Broadcasting Corporation, and
Banahaw Broadcasting Corporation (collectively, RPN, et al.). But rather than pursue a levy in execution of the
corresponding amounts on escrow with Metrobank, RPN, et al. filed a Supplemental Motion for Execution where
they described TRB as "now Bank of Commerce" based on the assumption that TRB had been merged into
Bancommerce. On February 20, 2004, having learned of the supplemental application for execution, Bancommerce
filed its Special Appearance with Opposition to the same questioning the jurisdiction of the RTC over Bancommerce
and denying that there was a merger between TRB and Bancommerce.

Issue: Was there a merger or at least a de facto merger between TRB and Bancommerce?

Held: None. The transaction between TRB and Bancommerce was neither a merger nor a de facto merger but a
mere "sale of assets with assumption of liabilities."

Indubitably, it is clear that no merger took place between Bancommerce and TRB as the requirements and
procedures for a merger were absent. A merger does not become effective upon the mere agreement of the
constituent corporations. All the requirements specified in the law must be complied with in order for merger to
take effect. Section 79 of the Corporation Code further provides that the merger shall be effective only upon the
issuance by the Securities and Exchange Commission (SEC) of a certificate of merger.

Here, Bancommerce and TRB remained separate corporations with distinct corporate personalities. What happened
is that TRB sold and Bancommerce purchased identified recorded assets of TRB in consideration of Bancommerce’s
assumption of identified recorded liabilities of TRB including booked contingent accounts. There is no law that
prohibits this kind of transaction especially when it is done openly and with appropriate government approval. In
strict sense, no merger or consolidation took place as the records do not show any plan or articles of merger or
consolidation. More importantly, the SEC did not issue any certificate of merger or consolidation.

In his book, Philippine Corporate Law, Dean Cesar Villanueva explained that under the Corporation Code, "a de
facto merger can be pursued by one corporation acquiring all or substantially all of the properties of another
corporation in exchange of shares of stock of the acquiring corporation.

No de facto merger took place in the present case simply because the TRB owners did not get in exchange for the
bank’s assets and liabilities an equivalent value in Bancommerce shares of stock. Bancommerce and TRB agreed
with BSP approval to exclude from the sale the TRB’s contingent judicial liabilities, including those owing to RPN, et
al.
CORPORATION LAW CASE DIGESTS | 113
FEU JD4401 | 2nd SEMESTER, S.Y. 2017-2018

(67) Commission of Internal Revenue vs. Pilipinas Shell


G.R. No. 192398. September 29, 2014
by. Lapuz, Jesus Jr. Ros

Doctrine: In a merger of two (2) existing corporations, one of the corporations survives and continues the
business, while the other is dissolved, and all its rights, properties, and liabilities are acquired by the surviving
corporation. The transfer of properties therein is not subject to DST under tax code as there exists no sale of real
property in merger of corporations.

Facts: On April 27, 1999, Pilipinas Shell entered into a Plan of Merger with its affiliate, Shell Philippine Petroleum
Corporation (SPPC), a corporation organized and existing under the laws of the Philippines. In the Plan of Merger, it
was provided that the entire assets and liabilities of SPPC will be transferred to, and absorbed by, respondent as
the surviving entity. The Securities and Exchange Commission approved the merger on July 1, 1999. Upon
payment of the necessary documentary stamp taxes, the BIR, on the other hand, ruled that said merger that no
gain or loss shall be recognised, if, in pursuance to a plan of merger or consolidation, a shareholder exchanges
stock in a corporation which is a party to the merger or consolidation solely for the stock of another corporation
which is also a party to the merger or consolidation. The BIR ruled, among others, that no gain or loss shall be
recognized by the stockholders of SPPC on the exchange of their shares of stock of SPPC solely for shares of stock
of respondent pursuant to the Plan of Merger. Respondent believing that it had erroneously paid taxes for the
merger applied for tax refund. As it was not heeded, respondent elevated the case to CTA. CTA ruled against the
petitioner which was later affirmed by the CA both ruling that the transfer of the properties of SPPC to respondent
was not in exchange for the latter’s shares of stock but is a legal consequence of the merger. The CA ruled that the
actual transfer of SPPC’s real properties to respondent was not effected by or dependent upon any voluntary deed,
conveyance or assignment but occurred by operation of law. The CA held that since the basis of the BIR in
imposing the documentary stamp tax is not applicable to a transfer of real property by operation of law, PSPC
erroneously paid the documentary stamp tax and is therefore, entitled to a tax refund or tax credit.

Issue: Is the transfer of real properties of SPPC to respondent in exchange for the latter’s shares stock subject to
documentary stamp tax under tax code?

Held: NO.

Conveyance of property taxable under the code covers only all transfers and conveyances for valuable
consideration. In other words, only sales of real property are contemplated therein. It should be emphasized that in
the instant case, the transfer of SPPC’s real property to respondent was pursuant to their approved plan of merger.
In a merger of two existing corporations, one of the corporations survives and continues the business, while the
other is dissolved, and all its rights, properties, and liabilities are acquired by the surviving corporation. Although
there is a dissolution of the absorbed or merged corporations, there is no winding up of their affairs or liquidation
of their assets because the surviving corporation automatically acquires all their rights, privileges, and powers, as
well as their liabilities. Here, SPPC ceased to have any legal personality and respondent PSPC stepped into
everything that was SPPC’s, pursuant to the law and the terms of their Plan of Merger. Pertinently, a merger of two
corporations produces the following effects, among others: Sec. 80. Effects of merger or consolidation.—
x x x x x x x 4. The surviving or the consolidated corporation shall thereupon and thereafter possess all the rights,
privileges, immunities and franchises of each of the constituent corporations; and all property, real or personal,
and all receivables due on whatever account, including subscriptions to shares and other choses in action, and all
and every other interest of, or belonging to, or due to each constituent corporations, shall be taken and deemed to
be transferred to and vested in such surviving or consolidated corporation without further act or deed.
CORPORATION LAW CASE DIGESTS | 114
FEU JD4401 | 2nd SEMESTER, S.Y. 2017-2018

(68) Commissioner of Internal Revenue v. La Tondena Distillers, Inc.


G.R. No. 175188, July 15, 1988
By: Mano, Razna I.

Doctrine: The transfer of real property to a surviving corporation pursuant to a merger is not subject to
Documentary Stamp Tax (DST).

Respondent La Tondeña Distillers, Inc. entered into a Plan of Merger with three corporations. As a result of the
merger, the assets and liabilities of the absorbed corporations were transferred to respondent, the surviving
corporation. Pursuant to a BIR Ruling stating that the transfer of assets, such as real properties, shall be subject to
DST under Section 196 of the NIRC, respondent paid DST. Claiming that it is exempt from paying DST, respondent
filed with petitioner Commissioner of Internal Revenue (CIR) an administrative claim for tax refund or tax credit
representing the DST it allegedly erroneously paid on the occasion of the merger. Petitioner posits that DST is
levied on the exercise of the privilege to convey real property regardless of the manner of conveyance. Thus, it is
imposed on all conveyances of realty, including realty transfer during a corporate merger. As to the subsequent
enactment of RA 9243, petitioner claims that respondent cannot benefit from it as laws apply prospectively.
Respondent, on the other hand, contends that DST is imposed only on conveyances, deeds, instruments, or
writing, where realty sold shall be conveyed to a purchaser or buyer. In this case, there is no purchaser or buyer as
a merger is neither a sale nor a liquidation of corporate property but a consolidation of properties, powers, and
facilities of the constituent companies.

Is the respondent exempt from payment of DST?

Yes, respondent is exempt from payment of DST.

A perusal of the Section 196 of the NIRC would clearly show it pertains only to sale transactions where real
property is conveyed to a purchaser for a consideration. The phrase "granted, assigned, transferred or otherwise
conveyed" is qualified by the word "sold" which means that documentary stamp tax under Section 196 is imposed
on the transfer of realty by way of sale and does not apply to all conveyances of real property. Indeed, as correctly
noted by the respondent, the fact that Section 196 refers to words "sold", "purchaser" and "consideration"
undoubtedly leads to the conclusion that only sales of real property are contemplated therein.

Section 80 of the Corporation Code provides for the effects of a merger, among others, that all property, real or
personal, and all receivables due on whatever account, including subscriptions to shares and other choses in action,
and all and every other interest of, or belonging to, or due to each constituent corporations, shall be taken and
deemed to be transferred to and vested in such surviving or consolidated corporation without further act or deed.

In the instant case, the transfer of the real properties to respondent was pursuant to their approved plan of
merger.1âwphi1In a merger of existing corporations, one of the corporations survives and continues the business,
while the other is dissolved, and all its rights, properties, and liabilities are acquired by the surviving corporation.
Although there is a dissolution of the absorbed or merged corporations, there is no winding up of their affairs or
liquidation of their assets because the surviving corporation automatically acquires all their rights, privileges, and
powers, as well as their liabilities. Here, the three corporations ceased to have any legal personality and
respondent stepped into everything that was the three corporations’, pursuant to the law and the terms of their
Plan of Merger.

In a merger, the real properties are not deemed "sold" to the surviving corporation and the latter could not be
considered as "purchaser" of realty since the real properties subject of the merger were merely absorbed by the
surviving corporation by operation of law and these properties are deemed automatically transferred to and vested
in the surviving corporation without further act or deed. Therefore, the transfer of real properties to the surviving
corporation in pursuance of a merger is not subject to documentary stamp tax. As stated at the outset,
documentary stamp tax is imposed only on all conveyances, deeds, instruments or writing where realty sold shall
be conveyed to a purchaser or purchasers. The transfer of the three corporations’ real properties to respondent
was neither a sale nor was it a conveyance of real property for a consideration contracted to be paid as
contemplated under Section 196 of the Tax Code. Hence, Section 196 of the Tax Code is inapplicable and
respondent is not liable for documentary stamp tax.

Therefore, respondent is not liable for DST as the transfer of real properties from the absorbed corporations to
respondent was pursuant to a merger.
CORPORATION LAW CASE DIGESTS | 115
FEU JD4401 | 2nd SEMESTER, S.Y. 2017-2018

8. NON-STOCK CORPORATIONS

(69) Cebu Country Club, Inc. v. Elizagaque


G.R. No. 160273, January 18, 2008
By: Marasigan, Mariz Angelle R.

Doctrine: Section 31 of the Corporation Code provides:

SEC. 31. Liability of directors, trustees or officers. — Directors or trustees who willfully and knowingly vote for or
assent to patently unlawful acts of the corporation or who are guilty of gross negligence or bad faith in directing
the affairs of the corporation or acquire any personal or pecuniary interest in conflict with their duty as such
directors, or trustees shall be liable jointly and severally for all damages resulting therefrom suffered by the
corporation, its stockholders or members and other persons.

Cebu Country Club, Inc. (CCCI), petitioner, is a domestic corporation operating as a non-profit and non-stock
private membership club, having its principal place of business in Banilad, Cebu City.

Sometime in 1987, San Miguel Corporation, a special company proprietary member of CCCI, designated
respondent Ricardo F. Elizagaque, its Senior Vice President and Operations Manager for the Visayas and Mindanao,
as a special non-proprietary member. The designation was thereafter approved by the CCCI’s Board of Directors. In
1996, respondent filed with CCCI an application for proprietary membership. The application was indorsed by
CCCI’s two (2) proprietary members, namely: Edmundo T. Misa and Silvano Ludo. As the price of a proprietary
share was around the P5 million range, Benito Unchuan, then president of CCCI, offered to sell respondent a share
for only P3.5 million. Respondent, however, purchased the share of a certain Dr. Butalid for only P3 million.
Consequently, on September 6, 1996, CCCI issued Proprietary Ownership Certificate No. 1446 to respondent. On
August 1, 1997, respondent received a letter from Julius Z. Neri, CCCI’s corporate secretary, informing him that
the Board disapproved his application for proprietary membership. Edmundo T. Misa, on behalf of respondent,
wrote CCCI several letters of reconsideration but CCCI did not reply.

Consequently, on December 23, 1998, respondent filed with the Regional Trial Court (RTC), Branch 71, Pasig City a
complaint for damages against petitioners. The RTC rendered its Decision in favor of respondent. On appeal by
petitioners, the Court of Appeals, in its Decision, affirmed the trial court’s Decision.

Whether in disapproving respondent’s application for proprietary membership with CCCI, petitioners
are liable to respondent for damages, and if so, whether their liability is joint and several

Yes, respondents are solidarily liable.

Section 3, Article 1 of CCCI’s Amended By-Laws provides:

SECTION 3. HOW MEMBERS ARE ELECTED – The procedure for the admission of new members of the Club shall be
as follows:
(a) Any proprietary member, seconded by another voting proprietary member, shall submit to the Secretary a
written proposal for the admission of a candidate to the "Eligible-for-Membership List";
(b) Such proposal shall be posted by the Secretary for a period of thirty (30) days on the Club bulletin board during
which time any member may interpose objections to the admission of the applicant by communicating the same to
the Board of Directors;
(c) After the expiration of the aforesaid thirty (30) days, if no objections have been filed or if there are, the Board
considers the objections unmeritorious, the candidate shall be qualified for inclusion in the "Eligible-for-Membership
List";
(d) Once included in the "Eligible-for-Membership List" and after the candidate shall have acquired in his name a
valid POC duly recorded in the books of the corporation as his own, he shall become a Proprietary Member, upon a
non-refundable admission fee of P1,000.00, provided that admission fees will only be collected once from any
person.

On March 1, 1978, Section 3(c) was amended to read as follows:


(c) After the expiration of the aforesaid thirty (30) days, the Board may, by unanimous vote of all directors present
at a regular or special meeting, approve the inclusion of the candidate in the "Eligible-for-Membership List".

Obviously, the CCCI Board of Directors, under its Articles of Incorporation, has the right to approve or disapprove
an application for proprietary membership. But such right should not be exercised arbitrarily. Articles 19 and 21 of
the Civil Code on the Chapter on Human Relations provide restrictions, thus:
CORPORATION LAW CASE DIGESTS | 116
FEU JD4401 | 2nd SEMESTER, S.Y. 2017-2018

Article 19. Every person must, in the exercise of his rights and in the performance of his duties, act with justice,
give everyone his due, and observe honesty and good faith.

Article 21. Any person who willfully causes loss or injury to another in a manner that is contrary to morals, good
customs or public policy shall compensate the latter for the damage.

In GF Equity, Inc. v. Valenzona, we expounded Article 19 and correlated it with Article 21, thus:

This article, known to contain what is commonly referred to as the principle of abuse of rights, sets certain
standards which must be observed not only in the exercise of one's rights but also in the performance of one's
duties. These standards are the following: to act with justice; to give everyone his due; and to observe honesty
and good faith. The law, therefore, recognizes a primordial limitation on all rights; that in their exercise, the norms
of human conduct set forth in Article 19 must be observed. A right, though by itself legal because recognized or
granted by law as such, may nevertheless become the source of some illegality. When a right is exercised in a
manner which does not conform with the norms enshrined in Article 19 and results in damage to
another, a legal wrong is thereby committed for which the wrongdoer must be held responsible. But
while Article 19 lays down a rule of conduct for the government of human relations and for the maintenance of
social order, it does not provide a remedy for its violation. Generally, an action for damages under either Article 20
or Article 21 would be proper. (Emphasis in the original)

In rejecting respondent’s application for proprietary membership, we find that petitioners violated the rules
governing human relations, the basic principles to be observed for the rightful relationship between human beings
and for the stability of social order. The trial court and the Court of Appeals aptly held that petitioners committed
fraud and evident bad faith in disapproving respondent’s applications. This is contrary to morals, good custom or
public policy. Hence, petitioners are liable for damages pursuant to Article 19 in relation to Article 21 of the same
Code.

It bears stressing that the amendment to Section 3(c) of CCCI’s Amended By-Laws requiring the unanimous vote
of the directors present at a special or regular meeting was not printed on the application form respondent filled
and submitted to CCCI. What was printed thereon was the original provision of Section 3(c) which was silent on the
required number of votes needed for admission of an applicant as a proprietary member.

Lastly, petitioners’ argument that they could not be held jointly and severally liable for damages because only one
(1) voted for the disapproval of respondent’s application lacks merit.

Section 31 of the Corporation Code provides:

SEC. 31. Liability of directors, trustees or officers. — Directors or trustees who willfully and knowingly vote for or
assent to patently unlawful acts of the corporation or who are guilty of gross negligence or bad faith in directing
the affairs of the corporation or acquire any personal or pecuniary interest in conflict with their duty as such
directors, or trustees shall be liable jointly and severally for all damages resulting therefrom suffered by the
corporation, its stockholders or members and other persons. (Emphasis ours)
CORPORATION LAW CASE DIGESTS | 117
FEU JD4401 | 2nd SEMESTER, S.Y. 2017-2018

(70) Co, Sr. vs. Philippine Canine Club, Inc.


G.R. No. 190112. April 22, 2015.
By: Pangilinan, Gene Alexis

Doctrine: The Court cannot rule on the issue of the validity of the Amended By-Laws to prevent a prejudgment on
the merits of the main case that is pending before a lower court.

The respondent, Philippine Canine Club, Inc. (PCCI), is a nonstock, nonprofit organization established in 1963 for
the principal purpose of promoting the breeding of purebred dogs. The petitioners were members of PCCI.

The Asian Kennel Club Union of the Philippines, Inc. (AKCUPI) was established as a corporate entity, and made
known its intention to hold and to sponsor dog shows and events similar to those being held and conducted by
other kennel clubs in the Philippines, including the PCCI. Believing that there was no conflict in the goals and the
objectives of PCCI and AKCUPI, and that there was no prohibition on members of PCCI whether express or implied
from joining and affiliating themselves with other kennel clubs, the petitioners registered their dogs with AKCUPI.

PCCI amended its bylaws, allegedly without the participation of its nonvoting members, including the petitioners.
Among the amendments was inclusion in the term prejudicial conduct the act of “Membership in or direct or
indirect participation in the formation, organization, operation and activities of an incorporated or unincorporated
organization whose purposes and activities have been determined by the Board of Directors to be prejudicial to the
best interest of PCCI, its members and the purebred dog sport.”

Subsequently, PCCI’s Board of Directors ordered the immediate suspension of petitioners Co, Cruz, Alegado and
Jester, due to their registration of their dogs with AKCUPI. PCCI sent Co, Cruz and Jester identical letters dated
December 15, 2008, informing them of their expulsion from the organization due to their alleged “conduct
prejudicial to the best interest” of PCCI.

The petitioners filed a case for Annulment of the Amended By-Laws, Injunction and Damages with application for
the issuance of a Temporary Restraining Order (TRO) and/or Writ of Preliminary Injunction. They claimed that the
adoption of the questioned Amended By-Laws, particularly Article VI on suspension, expulsion and termination of
membership, without the participation of PCCI’s nonvoting members, constitutes a violation of the Corporation
Code.

Issue:
1. Whether or not the court Court may rule on the issue of the validity of the Amended By-Laws.
2. Whether or not the RTC may enjoin the enforcement of the PCCI Amended By-Laws, which has already been in
effect, and the enforcement of the penalty of expulsion against the petitioners, which has already been
implemented.

Held:
1. No. The Court cannot rule on the issue of the validity of the Amended By-Laws to prevent a prejudgment on the
merits of the main case that is pending before the RTC. The Court resolved only the issue on the propriety of the
issuance of the writ of preliminary injunction against the implementation of the Amended By-Laws considering that
these were approved by the SEC and enforced prior to the filing of the case.

2. Yes. Consummated acts can no longer be restrained by injunction. In this case however, not all of the petitioners
were expelled or suspended at the time the RTC issued the writ of preliminary injunction. Thus, it appears that the
trial court can still enjoin the enforcement of the Amended By-Laws with respect to Joseph and Cham, as to whom
the sanctions were not yet implemented. However, as regards the suspended and expelled members namely, Co,
Cruz, Alegado and Jester, the trial court can no longer enjoin the enforcement of the Amended By-Laws as the
latter has already been consummated.
CORPORATION LAW CASE DIGESTS | 118
FEU JD4401 | 2nd SEMESTER, S.Y. 2017-2018

(71) The Orchard Golf and Country Club vs Yu


G.R. No. 190112. April 22, 2015
By: Radovan

The events leading to these consolidated cases began with a game of golf.
Yu and Yuhico were set to play golf at the Orchard Golf and Country Club with one more member of the club.
Unfortunately, this other member cancelled at the last minute. Because of the Club’s policy, which prohibited
“twosomes” from teeing off on weekends and public holidays before 1pm, petitioners requested management to
look for another player to join them. When a third player could not be found, petitioners requested that they be
allowed to play. The Club refused, but they played anyway, in violation of the Club’s rules. As a result, an incident
report was filed with the Club’s Board of Directors. The Board resolved to suspend both members for 3 months.

Petitioners filed complaints with the Securities Investigation and Clearing Department of the Securities and
Exchange Commission, at that time the tribunal vested by law with jurisdiction to hear and decide intra-corporate
controversies. The SICD-SEC issued a TRO effective for 20 days, restraining the Club from implementing the
suspension. 2 days before the TRO would lapse, however, the SEC issued guidelines wherein parties would be
allowed to file their cases before August 8, 2000 but any provisional remedies the SEC granted them were to be
effective only until that date. On August 7, 2000, the SIDC-SEC issued a writ of preliminary injunction enjoining
respondents from implementing the suspensions.
Two months later, the Board resolved it was going to implement the suspension as the August 8 cut-off had
already lapsed. Petitioners filed a petition for indirect contempt with the RTC of Dasmariñas, Cavite. The RTC
restored the writ of preliminary injunction. Respondents appealed with the CA, which reversed the RTC’s decision
and the petitioner’s suspension was finally implemented.

Meanwhile petitioners filed a motion ad cautelam in the RTC of Imus, Cavite, praying for the issuance of a TRO
and/or writ of injunction to enjoin respondents from implementing the suspension orders. The RTC issued the TRO.
It was after this issuance that the petitioners filed a motion for reconsideration with the CA. The CA denied the
motion and thus the first case was elevated to the Supreme Court.

The respondents questioned the order of the Imus RTC with a motion for reconsideration, which the RTC denied.
The respondents turned to the CA, which issued a TRO that enjoined the RTC from implementing the writ of
preliminary injunction. The petitioners then filed the second case with the Supreme Court, questioning the CA’s
TRO.

Did the SEC guidelines shorten the life span of the writs of preliminary injunction issued by the SEC–
SICD, thereby making them effective only until August 8, 2000?

YES. Petitioners contend that the guidelines could not have possibly limited the effectivity of their writs for two
reasons: (1) the intention of the guidelines was to cover applications for such writs and provisional remedies made
on or after August 1, 2000 and (2) in any event, the guidelines were void for lack of publication. The petitioners’
contentions have no merit. The guidelines were clear and categorical, such that there simply was no need for
petitioners’ extended interpretation. Under the guidelines, the parties were allowed to file their cases before August
8, 2000 but any provisional remedies the SEC granted them were to be effective only until that date. On the 2nd
contention, the court ruled that interpretative regulation and those merely internal in nature regulating only the
personnel of the administrative agency and not the public, need not be published. The guidelines were clear that
they were meant for the information of the officers of the SEC only. No doubt, the guidelines were meant to serve
as an advisory to all SEC officers to refrain from accepting new cases because of the impending transfer of
jurisdiction to the regular courts. The issuance or recall of a preliminary writ of injunction is an interlocutory matter
that remains at all times within the control of the court or quasi-judicial body that issued it. Thus, petitioners could
not rightfully claim a vested right to an injunctive writ.
CORPORATION LAW CASE DIGESTS | 119
FEU JD4401 | 2nd SEMESTER, S.Y. 2017-2018

(72) Ching vs. Quezon City Sports Club, Inc.


G.R. No. 200150, November 07, 2016
By: Rosario, Patricia Kaye T.

Doctrine: The prevailing rule is that the provisions of the articles of incorporation and the by-laws must be strictly
complied with and applied to the letter.

Respondent Club is a duly registered domestic corporation providing recreational activities, sports facilities, and
exclusive privileges and services to its members. Petitioner Catherine became a member and regular patron of
respondent Club in 1989. Per policy of respondent, petitioner's membership privileges were extended to immediate
family members.

The NLRC rendered a Decision ordering respondent to pay backwages, 13th and 14th month pay, and allowances to
six illegally dismissed employees. Because respondent was not in a financial position to pay the monetary awards,
the BOD approved a Board Resolution to "seek the assistance of its members by assessing each member the
amount of P2,500.00, payable in 5 equal monthly payments. The amount of P500.00 was debited and/or charged
to Catherine's account each month from September 2001 to January 2002.

Petitioner Catherine believed that the imposition of the special assessment was unjust, however, she took no action
against the same. Petitioner Catherine simply avoided paying the special assessment. The BOD then passed
another Board Resolution which suspended the privileges of the members of respondent Club who had not yet paid
the special assessment.

Petitioner Catherine continued availing herself of the services of respondent Club and regularly paid the amounts
due in her Statements of Account from February 2002 to May 2003, but always leaving behind a balance of more
or less P2,500.00. Petitioner Catherine was not personally informed of the second Board Resolution nor advised
that she was already deemed delinquent in the payment of any other Statements of Account. She only learned of
the suspension when respondent refused to accommodate her son.

Are the Club and its Board of Directors liable to pay moral and exemplary damages, attorney's fees, and
costs of suit for suspending petitioner Catherine's membership privileges without prior notice as
required by the By-Laws of respondent Club?

NO, only the Club is liable to pay nominal damages.

The Court had previously recognized in Forest Hills Golf and Country Club, Inc. v. Gardpro, Inc., that articles of
incorporation and by-laws of a country club are the fundamental documents governing the conduct of the corporate
affairs of said club; they establish the norms of procedure for exercising rights, and reflected the purposes and
intentions of the incorporators. The by-laws are the self-imposed rules resulting from the agreement between the
country club and its members to conduct the corporate business in a particular way. In that sense, the by-laws are
the private "statutes" by which the country club is regulated, and will function. Until repealed, the by-laws are the
continuing rules for the government of the country club and its officers, the proper function being to regulate the
transaction of the incidental business of the country club. The by-laws constitute a binding contract as between the
country club and its members, and as among the members themselves. The by-laws are self-imposed private laws
binding on all members, directors, and officers of the country club. The prevailing rule is that the provisions of the
articles of incorporation and the by-laws must be strictly complied with and applied to the letter.

Section 35(a) of the By-Laws requires notice and hearing prior to a member's suspension. Definitely, petitioner
Catherine did not receive notice specifically advising her that she could be suspended for nonpayment of the
special assessment. Respondents merely relied on the general notice printed in petitioner Catherine's Statements
of Account from September 2001 to April 2002 warning of automatic suspension. Petitioner Catherine's right to due
process was clearly violated. Nevertheless, petitioner Catherine herself admitted violating Board Resolution No. 7-
2001 by not paying the P2,500.00 special assessment. Thus, the Court finds no bad faith on the part of
respondents in implementing petitioner Catherine's suspension. In addition, the posting of the list of suspended
members in conspicuous places in respondent Club did not necessarily connote bad faith on the part of
respondents.

Considering that there was justifiable ground for the suspension of petitioner Catherine's privileges in respondent
Club, but her right to due process was violated as she was not afforded notice and hearing prior to the suspension,
the Court deems it proper to award nominal damages to petitioners. The Court clarifies that only respondent Club
shall be liable for the nominal damages because in the absence of malice and bad faith, officers of a corporation
cannot be made personally liable for the liabilities of the corporation which, by legal fiction, has a personality
separate and distinct from its officers, stockholders, and members.
CORPORATION LAW CASE DIGESTS | 120
FEU JD4401 | 2nd SEMESTER, S.Y. 2017-2018

9. DISSOLUTION AND LIQUIDATION

(73) Alabang Development Corporation v. Alabang Hills Village Association


G.R. No. 187456, June 2, 2014
By: Samson, Maria Johanna Ilyssa

Topic: Distinction between Lack of Legal Capacity to Sue and Lack of Capacity to Sue

Doctrine: Lack of legal capacity to sue means that the plaintiff is not in the exercise of his civil rights, or does not
have the necessary qualification to appear in the case, or does not have the character or representation he claims.
On the other hand, lack of capacity to sue refers to a plaintiff’s general disability to sue, such as on account of
minority, insanity, incompetence, lack of juridical personality or any other general disqualifications of a party.

First paragraph of Section 122 of the Corporation Code states that “every corporation whose charter expires by its
own limitation or is annulled by forfeiture or otherwise, or whose corporate existence for other purposes is
terminated in any other manner, shall nevertheless be continued as a body corporate for three (3) years after the
time when it would have been so dissolved, for the purpose of prosecuting and defending suits by or against it and
enabling it to settle and close its affairs, to dispose of and convey its property and to distribute its assets, but not
for the purpose of continuing the business for which it was established.”

The case traces its roots to the Complaint for Injunction and Damages filed with the RTC of Muntinlupa City by
herein petitioner, Alabang Development Corporation (ADC) against herein respondents, Alabang Hills Village
Association, Inc. (AHVAI) and Rafael Tinio (Tinio), President of AHVAI. The Complaint alleged that petitioner is the
developer of Alabang Hills Village and still owns certain parcels of land therein that are yet to be sold, as well as
those considered open spaces that have not yet been donated to the local government of Muntinlupa City or the
Homeowner’s Association. ADC learned that AHVAI started the construction of a multi-purpose hall and a swimming
pool on one of the parcels of land still owned by ADC without the latter’s consent and approval, and that despite
demand, AHVAI failed to desist from constructing the said improvements. ADC thus prayed that an injunction be
issued enjoining defendants from constructing the multi-purpose hall and the swimming pool at the Alabang Hills
Village. AHVAI denied ADC’s asseverations and claimed that the latter has no legal capacity to sue since its
existence as a registered corporate entity was revoked by the Securities and Exchange Commission (SEC) on May
26, 2003; that ADC has no cause of action because by law it is no longer the absolute owner but is merely holding
the property in question in trust for the benefit of AHVAI as beneficial owner thereof. RTC rendered judgment
dismissing herein petitioner’s complaint on the grounds that the latter has no personality to file the same, that the
subject property “is a reserved area for the beneficial use of the homeowners, as mandated by law” and, that the
Housing and Land Use Regulatory Board (HLURB), not the RTC, has exclusive jurisdiction over the dispute between
petitioner and respondents. CA affirmed the decision of the RTC. The CA ruled that the RTC correctly dismissed
petitioner’s complaint as the same was filed when petitioner was already defunct and, as such, it no longer had
capacity to file the said complaint.

Did the Court of Appeal gravely erred in relying on the case of Columbia Pictures Inc. v. Court of
Appeals in resolving Petitioner’s lack of capacity?

No. The Court does not agree that the CA erred in relying on the case of Columbia Pictures, Inc. v. Court of
Appeals. The CA cited the case for the purpose of restating and distinguishing the jurisprudential definition of the
terms “lack of capacity to sue” and “lack of personality to sue” and of applying these definitions to the present
case. Thus, the fact that, unlike in the instant case, the corporations involved in the Columbia case were foreign
corporations is of no moment. The definition of the term “lack of capacity to sue” enunciated in the said case still
applies to the case at bar. Indeed, as held by this Court and as correctly cited by the CA in the case of Columbia:
“lack of legal capacity to sue” means that the plaintiff is not in the exercise of his civil rights, or does not have the
necessary qualification to appear in the case, or does not have the character or representation he claims; “lack of
capacity to sue” refers to a plaintiff’s general disability to sue, such as on account of minority, insanity,
incompetence, lack of juridical personality or any other general disqualifications of a party.” In the instant case,
petitioner lacks capacity to sue because it no longer possesses juridical personality by reason of its dissolution and
lapse of the three-year grace period provided under Section 122 of the Corporation Code.
CORPORATION LAW CASE DIGESTS | 121
FEU JD4401 | 2nd SEMESTER, S.Y. 2017-2018

First paragraph of Section 122 of the Corporation Code states that “every corporation whose charter expires by its
own limitation or is annulled by forfeiture or otherwise, or whose corporate existence for other purposes is
terminated in any other manner, shall nevertheless be continued as a body corporate for three (3) years after the
time when it would have been so dissolved, for the purpose of prosecuting and defending suits by or against it and
enabling it to settle and close its affairs, to dispose of and convey its property and to distribute its assets, but not
for the purpose of continuing the business for which it was established.”

In the instant case, there is no dispute that petitioner’s corporate registration was revoked on May 26, 2003. Based
on the above-quoted provision of law, it had three years, or until May 26, 2006, to prosecute or defend any suit by
or against it. The subject complaint, however, was filed only on October 19, 2006, more than three years after
such revocation. It is likewise not disputed that the subject complaint was filed by petitioner corporation and not by
its directors or trustees. In fact, it is even averred, albeit wrongly, in the first paragraph of the Complaint that
“plaintiff is a duly organized and existing corporation under the laws of the Philippines, with capacity to sue and be
sued.

In the present case, petitioner filed its complaint not only after its corporate existence was terminated but also
beyond the three-year period allowed by Section 122 of the Corporation Code. Thus, it is clear that at the time of
the filing of the subject complaint petitioner lacks the capacity to sue as a corporation. To allow petitioner to
initiate the subject complaint and pursue it until final judgment, on the ground that such complaint was filed for the
sole purpose of liquidating its assets, would be to circumvent the provisions of Section 122 of the Corporation
Code.
CORPORATION LAW CASE DIGESTS | 122
FEU JD4401 | 2nd SEMESTER, S.Y. 2017-2018

(74) Viguilla, et. al. v. Philippine College of Criminology, Inc.,


G.R.No. 200094, June 10, 2013
By: Torres, Ma. Roma
TOPIC: Dissolution and Liquidation

DOCTRINE: A dissolved corporation can enter into an agreement such as releases, waivers and quit-claims beyond
the 3-year winding up period under Sec 122 of the Corporation Code.

FACTS: Respondent is a non-stock educational institution while petitioners were janitors, janitresses and
supervisor in the Maintenance department of respondent. The petitioners, however, were made to understand,
upon application with respondent school, that they were under MBMSI, a corporation engaged in providing
janitorial services to clients. Respondent discovered that the Certificate of Incorporation of MBMSI had been
revoked and later, respondent terminated the school’s relationship with MBMSI, resulting in the dismissal of the
employees or maintenance personnel under MBMSI. Petitioners who were dismissed filed a case of illegal dismissal
against MBMSI and respondent. Respondent denied that it is the employer of petitioners and presented the
releases, waivers and quitclaims in favor of MBMSI executed by the complainants to prove that they were
employees of MBMSI and not respondent. Petitioners argue that MBMSI had no legal personality to incur civil
liabilities as it did not exist as a corporation on account of the fact that its Certificate of Incorporation had been
revoked. Petitioners ask this Court to exempt MBMSI from its liabilities because it is no longer existing as a
corporation.

ISSUE: WON a dissolved corporation can enter into an agreement such as releases, waivers and quitclaims beyond
the 3-year winding up period under Section 122 of the Corporation Code

HELD: Yes. The executed releases, waivers and quitclaims are valid and binding notwithstanding the revocation of
MBMSI’s Certificate of Incorporation. The revocation does not result in the termination of its liabilities. Section 122
of the Corporation Code provides for a three-year winding up period for a corporation whose charter is annulled by
forfeiture or otherwise to continue as a body corporate for the purpose, among others, of settling and closing its
affairs.

Even if said documents were executed in 2009, six (6) years after MBMSI’s dissolution in 2003, the same are still
valid and binding upon the parties and the dissolution will not terminate the liabilities incurred by the dissolved
corporation pursuant to Sections 122 and 145 of the Corporation Code. In the case of Premiere Development Bank
v. Flores, the Court held that a corporation is allowed to settle and close its affairs even after the winding up period
of three (3) years. The Court wrote: Although the time during which the corporation, through its own officers, may
conduct the liquidation of its assets and sue and be sued as a corporation is limited to three years from the time
the period of dissolution commences, there is no time limit within which the trustees must complete a liquidation
placed in their hands. What is provided in Section 122 of the Corporation Code is that the conveyance to the
trustees must be made within the three-year period. But it may be found impossible to complete the work of
liquidation within the three-year period or to reduce disputed claims to judgment. The trustees to whom the
corporate assets have been conveyed pursuant to the authority of Section 122 may sue and be sued as such in all
matters connected with the liquidation.

Furthermore, Section 145 of the Corporation Code clearly provides that "no right or remedy in favor of or against
any corporation, its stockholders, members, directors, trustees, or officers, nor any liability incurred by any such
corporation, stockholders, members, directors, trustees, or officers, shall be removed or impaired either by the
subsequent dissolution of said corporation." Even if no trustee is appointed or designated during the three-year
period of the liquidation of the corporation, the Court has held that the board of directors may be permitted to
complete the corporate liquidation by continuing as "trustees" by legal implication.
CORPORATION LAW CASE DIGESTS | 123
FEU JD4401 | 2nd SEMESTER, S.Y. 2017-2018

(75) Ramon E. Reyes and Clara R. Pastor vs. Bancom Development Corp.
GR No. 190286 January 11, 2018
By: Valencia, Emmanuelle Nicole L.

Topic: Liability under continuing guaranty of loans executed by a corporation; Abatement of


suit following revocation of corporation’s certificate of registration by the SEC

Doctrines: No right or remedy in favor of or against any corporation, its stockholders, members, directors,
trustees, or officers, nor any liability incurred by any such corporation, stockholders, members, directors, trustees,
or officers, shall be removed or impaired either by the subsequent dissolution of said corporation or by any
subsequent amendment or repeal of this Code or of any part thereof.

A continuing guaranty was executed in favour of Bancom by the Reyes Group. In the instrument, the group agreed
to guarantee the full and due payment of obligations incurred by Marbella under an Underwriting Agreement with
Bancom. These obligations included certain promissory notes issued by Marbella in favor of Bancom with an
aggregate amount of PhP 2,828,140.32.

Marbella was unable to make payment at the time of the maturity of the notes. Consequently, it issued a set of
replacement promissory notes, this time for the increased amount of PhP 2,901,466.48. Again, it defaulted on the
notes, leading to the issuance of a third set of promissory notes amounting to PhP 3,002,333.84, and eventually a
fourth set for the same amount.

Due to Marbella’s continued failure to pay back the loan, despite repeated demands, Bancom filed a complaint for
sum of money. The case sought payment of a total sum of PhP 4,300,247.35, and named Marbella as the principal
debtor, and imploded the Reyes Group as guarantors.

The RTC held that Marbella and the Reyes Group were solidarity liable to Bancom. During the appeal to the CA,
the lawyers for Bancom withdrew their appearance as counsel, citing that it had “totally lost contact” with the
client, and had “received reports that the client has undergone a merger with another entity.” Nevertheless, the CA
ruled in favor of Bancom.

On motion for reconsideration to the CA, Reyes and Pastor contended that the action must be considered abated,
pursuant to Section 122 of the Corporation Code, since the certificate of registration of Bancom, issued by the SEC,
had been revoked, and that no trustee or receiver had been appointed to continue the suit. The CA nevertheless
denied the motion.

Should the CA have declared the suit abated, in light of the fact that Bancom no longer exists?

NO.

Sec. 122 of the Corporation Code provides that a corporation whose charter is annulled, or whose corporate
existence is otherwise terminated, may continue as a body corporate for a limited period of three years, but only
for certain specific purposes enumerated by law. These include the prosecution and defense of suits by or against
the corporation, and other objectives relating to the settlement and closure of corporate affairs.

Based on the provision, a defunct corporation loses the right to sue and be sued in its name upon the expiration of
the three-year period provided by law. Jurisprudence has carved out an exception to the rule in cases where the
Court has appointed a receiver, assignee or trustee, such appointee may institute suits or continue pending actions
on behalf of the corporation, even after the winding up period.

In subsequent cases, the Court further clarified that a receiver or assignee need not even be appointed for the
purpose of bringing suits or continuing those that are pending. In Gelano vs. Court of Appeals, the Court held that
in the absence of a receiver or an assignee, suits may be instituted or continued by a trustee specifically
designated for a particular matter, such as a lawyer representing the corporation in a certain case.

Here, it appears that the SEC revoked the Certificate of Registration issued to Bancom on 26 May 2003. Bancom
does not seem to have conveyed its assets to trustees or to its stockholders and creditors. The corporation has also
failed to appoint a new counsel. The mere revocation of the charter of a corporation does not result in the
abatement of proceedings. Since its directors are considered trustees by legal the fact that Bancom did not convey
its assets to a receiver or implication, assignee was of no consequence. It must also be emphasized that the
dissolution of a creditor-corporation does not extinguish any right or remedy in its favor.

Sec. 145 of the Corporation Code states that no right or remedy in favor of or against any corporation, its
stockholders, members, directors, trustees, or officers, nor any liability incurred by any such corporation,
stockholders, members, directors, trustees, or officers, shall be removed or impaired either by the subsequent
dissolution of said corporation or by any subsequent amendment or repeal of this Code or of any part thereof. The
CORPORATION LAW CASE DIGESTS | 124
FEU JD4401 | 2nd SEMESTER, S.Y. 2017-2018

corresponding liability of the debtors of a dissolved corporation must also be deemed subsisting. To rule otherwise
would be to sanction the unjust enrichment of the debtor at the expense of the corporation.

Are the guarantors of the loans of Marbella liable to Bancom?

YES.

Having executed a continuing guaranty in favour of Bancom, Reyes and Pastor are solidarily liable with Marbella for
the payment of the amounts indicated on the promissory notes.

Reyes and Pastor did not challenge the genuineness and due execution of the promissory notes. Neither did they
deny their nonpayment of Marbella’s loans, or the fact that these obligations were covered by their guaranty. Their
sole defense was that the promissory notes were not binding, because the funds released were not loans, but
merely additional financing.

The obligations of Marbella and the Reyes Group under the Promissory Notes and the Continuing Guaranty,
respectively, are plain and unqualified. Under the notes, Marbella promised to pay Bancom the amounts stated on
the maturity dates indicated. The Reyes Group, on the other hand, agreed to become liable if any of Marbella's
guaranteed obligations were not duly paid on the due date. There is absolutely no support for the assertion that
these agreements were not meant to be binding.

It is evident from the provisions of the contract that Bancom extended additional financing to Marbella on the
condition that the loan would be paid upon maturity. It is equally clear that the latter obligated itself to pay the
stated amount to Bancom without any condition. The unconditional tenor of the obligation of Marbella to pay
Bancom for the loan amount, plus interest and penalties, is likewise reflected in the Promissory Notes issued in
favor of the latter.

As to petitioners, the Continuing Guaranty evidently binds them to pay Bancom the amounts indicated on the
original set of Promissory Notes, as well as any and all instruments issued upon the renewal, extension,
amendment or novation thereof.
CORPORATION LAW CASE DIGESTS | 125
FEU JD4401 | 2nd SEMESTER, S.Y. 2017-2018

9. FOREIGN CORPORATIONS

(76) TUNA PROCESSING, INC. v. PHILIPPINE KINGFORD, INC.


G.R. No. 185582 29 February 2012
By: Valencia, Mary Clydeen L.

DOCTRINE: The foreign corporation’s capacity to sue in the Philippines is not material insofar as the recognition
and enforcement of a foreign arbitral award is concerned.

In a Petition for Review on Certiorari via Rule 45, petitioner Tuna Processing, Inc. (TPI), a foreign corporation not
licensed to do business in the Philippines, prays that the Resolution dated 21 November 2008 of the RTC of Makati
City be declared void and the case be remanded to the RTC for further proceedings. In the assailed Resolution, the
RTC dismissed petitioner’s Petition for Confirmation, Recognition, and Enforcement of Foreign Arbitral Award
against respondent Philippine Kingford, Inc. (Kingford), a corporation duly organized and existing under the laws of
the Philippines, on the ground that petitioner lacked legal capacity to sue.

Petitioner TPI argues, however, that it is entitled to seek for the recognition and enforcement of the subject foreign
arbitral award in accordance with Republic Act No. 9285 (Alternative Dispute Resolution Act of 2004), the
Convention on the Recognition and Enforcement of Foreign Arbitral Awards drafted during the United Nations
Conference on International Commercial Arbitration in 1958 (New York Convention), and the UNCITRAL Model Law
on International Commercial Arbitration (Model Law), as none of these specifically requires that the party seeking
for the enforcement should have legal capacity to sue.

A. How do we reconcile the provisions of the Corporation Code of the Philippines on one hand, and
the Alternative Dispute Resolution Act of 2004, the New York Convention and the Model Law on the
other?

In several cases, the Supreme Court had the occasion to discuss the nature and applicability of the Corporation
Code of the Philippines, a general law, viz-a-viz other special laws. Thus, in Koruga v. Arcenas, Jr., this Court
rejected the application of the Corporation Code and applied the New Central Bank Act. It ratiocinated:

Koruga’s invocation of the provisions of the Corporation Code is misplaced. In an earlier case with similar
antecedents, we ruled that the Corporation Code, however, is a general law applying to all types of
corporations, while the New Central Bank Act regulates specifically banks and other financial institutions,
including the dissolution and liquidation thereof. As between a general and special law, the latter shall
prevail – generalia specialibus non derogant."

Further, in the recent case of Hacienda Luisita, Incorporated v. Presidential Agrarian Reform Council, the Supreme
Court held without doubt that the Corporation Code is the general law providing for the formation, organization and
regulation of private corporations. On the other hand, RA 6657 is the special law on agrarian reform. As between a
general and special law, the latter shall prevail—generalia specialibus non derogant.

Following the same principle, the Alternative Dispute Resolution Act of 2004 shall apply in this case as the Act, as
its title - An Act to Institutionalize the Use of an Alternative Dispute Resolution System in the Philippines and to
Establish the Office for Alternative Dispute Resolution, and for Other Purposes - would suggest, is a law especially
enacted "to actively promote party autonomy in the resolution of disputes or the freedom of the party to make
their own arrangements to resolve their disputes." It specifically provides exclusive grounds available to the party
opposing an application for recognition and enforcement of the arbitral award. Inasmuch as the Alternative Dispute
Resolution Act of 2004, a municipal law, applies in the instant petition, we do not see the need to discuss
compliance with international obligations under the New York Convention and the Model Law. After all, both already
form part of the law.

B. Now, does a foreign corporation not licensed to do business in the Philippines have legal capacity to
sue under the provisions of the Alternative Dispute Resolution Act of 2004?

YES. Sec. 45 of the Alternative Dispute Resolution Act of 2004 provides that the opposing party in an application
for recognition and enforcement of the arbitral award may raise only those grounds that were enumerated under
Article V of the New York Convention. However, not one of these exclusive grounds touched on the capacity to sue
of the party seeking the recognition and enforcement of the award. Pertinent provisions of the Special Rules of
Court on Alternative Dispute Resolution, which was promulgated by the Supreme Court, likewise support this
position.

Rule 13.1 of the Special Rules provides that "[a]ny party to a foreign arbitration may petition the court to recognize
and enforce a foreign arbitral award." The contents of such petition are enumerated in Rule 13.5. Capacity to sue is
CORPORATION LAW CASE DIGESTS | 126
FEU JD4401 | 2nd SEMESTER, S.Y. 2017-2018

not included. Oppositely, in the Rule on local arbitral awards or arbitrations in instances where "the place of
arbitration is in the Philippines," it is specifically required that a petition "to determine any question concerning the
existence, validity and enforceability of such arbitration agreement" available to the parties before the
commencement of arbitration and/or a petition for "judicial relief from the ruling of the arbitral tribunal on a
preliminary question upholding or declining its jurisdiction" after arbitration has already commenced should state
"[t]he facts showing that the persons named as petitioner or respondent have legal capacity to sue or be sued."

Indeed, it is in the best interest of justice that in the enforcement of a foreign arbitral award, we deny availment by
the losing party of the rule that bars foreign corporations not licensed to do business in the Philippines from
maintaining a suit in our courts. When a party enters into a contract containing a foreign arbitration clause and, as
in this case, in fact submits itself to arbitration, it becomes bound by the contract, by the arbitration and by the
result of arbitration, conceding thereby the capacity of the other party to enter into the contract, participate in the
arbitration and cause the implementation of the result. Although not on all fours with the instant case, also worthy
to consider is the wisdom of then Associate Justice Flerida Ruth P. Romero in her Dissenting Opinion in Asset
Privatization Trust v. Court of Appeals, to wit:

xxx Arbitration, as an alternative mode of settlement, is gaining adherents in legal and judicial circles here
and abroad. If its tested mechanism can simply be ignored by an aggrieved party, one who, it must be
stressed, voluntarily and actively participated in the arbitration proceedings from the very beginning, it
will destroy the very essence of mutuality inherent in consensual contracts.

Clearly, on the matter of capacity to sue, a foreign arbitral award should be respected not because it is favored
over domestic laws and procedures, but because Republic Act No. 9285 has certainly erased any conflict of law
question.
Finally, even assuming, only for the sake of argument, that the court a quo correctly observed that the Model Law,
not the New York Convention, governs the subject arbitral award, petitioner may still seek recognition and
enforcement of the award in Philippine court, since the Model Law prescribes substantially identical exclusive
grounds for refusing recognition or enforcement.

Premises considered, petitioner TPI, although not licensed to do business in the Philippines, may seek recognition
and enforcement of the foreign arbitral award in accordance with the provisions of the Alternative Dispute
Resolution Act of 2004.
CORPORATION LAW CASE DIGESTS | 127
FEU JD4401 | 2nd SEMESTER, S.Y. 2017-2018

(77) Philippine Deposit Insurance Corp. vs. Citibank, N.A.


G.R. No. 170290 April 11, 2012
By: Alba, Ma. Angela

Doctrine: While individual bank branches may be treated as independent of one another, each branch, unless
separately incorporated, must be viewed as a part of the parent bank rather than as an independent entity.

Facts: Respondent Citibank, N.A. (Citibank) is a banking corporation while respondent Bank of America, S.T. &
N.A. (BA) is a national banking association, both of which are duly organized and existing under the laws of the
United States of America and duly licensed to do business in the Philippines, with offices in Makati City.

In 1977, PDIC discovered that Citibank received from its head office and other foreign branches a total of
P11,923,163,908.00 in dollars, covered by Certificates of Dollar Time Deposit that were interest-bearing with
corresponding maturity dates. These funds, which were lodged in the books of Citibank under the account “Their
Account-Head Office/Branches-Foreign Currency,” were not reported to PDIC as deposit liabilities that were subject
to assessment for insurance. As such, PDIC assessed Citibank for deficiency in the sum of P1,595,081.96.

Similarly, sometime in 1979, PDIC examined the books of accounts of BA which revealed that BA received from its
head office and its other foreign branches a total of P629,311,869.10 in dollars, covered by Certificates of Dollar
Time Deposit that were interest-bearing with corresponding maturity dates and lodged in their books under the
account “Due to Head Office/Branches.” Because BA also excluded these from its deposit liabilities, PDIC wrote to
BA seeking the remittance of P109,264.83 representing deficiency premium assessments for dollar deposits.

Citibank and BA each filed a petition for declaratory relief before the Court of First Instance. In their petitions,
Citibank and BA sought a declaratory judgment stating that the money placements they received from their head
office and other foreign branches were not deposits and did not give rise to insurable deposit liabilities under
Sections 3 and 4 of R.A. No. 3591 (the PDIC Charter) and, as a consequence, the deficiency assessments made by
PDIC were improper and erroneous.

The RTC ruled in favor of Citibank and and BA, holding that the subject money placements were not deposits and
did not give rise to insurable deposit liabilities, and that the deficiency assessments issued by PDIC were improper
and erroneous. Therefore, Citibank and BA were not liable to pay the same.

Aggrieved, PDIC appealed to the CA which affirmed the ruling of the RTC the CA found that the money placements
were received as part of the bank’s internal dealings by Citibank and BA as agents of their respective head offices.
This showed that the head office and the Philippine branch were considered as the same entity. Thus, no bank
deposit could have arisen from the transactions between the Philippine branch and the head office because there
did not exist two separate contracting parties to act as depositor and depositary.

Issue: Whether a branch of a bank has a separate legal personality.

Held: No, a branch has no separate legal personality.

There are two ways by which a foreign corporation can establish its presence in the Philippines. It may choose to
incorporate its own subsidiary as a domestic corporation, in which case such subsidiary would have its own
separate and independent legal personality to conduct business in the country. In the alternative, it may create a
branch in the Philippines, which would not be a legally independent unit, and simply obtain a license to do business
in the Philippines.

In the case of Citibank and BA, it is apparent that they both did not incorporate a separate domestic corporation to
represent its business interests in the Philippines. Their Philippine branches are, as the name implies, merely
branches, without a separate legal personality from their parent company, Citibank and BA. Thus, being one and
the same entity, the funds placed by the respondents in their respective branches in the Philippines should not be
treated as deposits made by third parties subject to deposit insurance under the PDIC Charter.
CORPORATION LAW CASE DIGESTS | 128
FEU JD4401 | 2nd SEMESTER, S.Y. 2017-2018

In the leading case of Sokoloff v. The National City Bank of New York, the Supreme Court of New York held that:

Where a bank maintains branches, each branch becomes a separate business entity with separate books of
account. A depositor in one branch cannot issue checks or drafts upon another branch or demand payment from
such other branch, and in many other respects the branches are considered separate corporate entities and as
distinct from one another as any other bank. Nevertheless, when considered with relation to the parent bank they
are not independent agencies; they are, what their name imports, merely branches, and are subject to the
supervision and control of the parent bank, and are instrumentalities whereby the parent bank carries on its
business, and are established for its own particular purposes, and their business conduct and policies are controlled
by the parent bank and their property and assets belong to the parent bank, although nominally held in the names
of the particular branches. Ultimate liability for a debt of a branch would rest upon the parent bank.

This ruling was later reiterated in the more recent case of United States v. BCCI Holdings Luxembourg, where
United States Court of Appeals, District of Columbia Circuit, emphasized that “while individual bank branches may
be treated as independent of one another, each branch, unless separately incorporated, must be viewed as a part
of the parent bank rather than as an independent entity.”

In addition, Philippine banking laws also support the conclusion that the head office of a foreign bank and its
branches are considered as one legal entity. Section 75 of The General Banking Law and Section 5 of R.A. No. 7221
(An Act Liberalizing the Entry of Foreign Banks) both require the head office of a foreign bank to guarantee the
prompt payment of all the liabilities of its Philippine branch.
CORPORATION LAW CASE DIGESTS | 129
FEU JD4401 | 2nd SEMESTER, S.Y. 2017-2018

(78) Steelcase, Inc. v. Design International Selections, Inc.,


G.R. No. 171995, April 18, 2012
By; Arid, Hannah Mhae G.

Doctrine: Philippine Supreme Court declared that a foreign corporation doing business in the Philippines without
the requisite license may sue in Philippine Courts against a Philippine citizen or entity who had contracted with and
benefited by said corporation. In other words, a party is estopped to challenge the personality of a corporation
after having acknowledged the same by entering into a contract with it.

Facts: Petitioner Steelcase, Inc. ("Steelcase") is a foreign corporation existing under the laws of Michigan, United
States of America (U.S.A.), and engaged in the manufacture of office furniture with dealers worldwide. Respondent
Design International Selections, Inc. ("DISI") is a corporation existing under Philippine Laws and engaged in the
furniture business, including the distribution of furniture.

Sometime in 1986 or 1987, Steelcase and DISI orally entered into a dealership agreement whereby Steelcase
granted DISI the right to market, sell, distribute, install, and service its products to end-user customers within the
Philippines. The business relationship continued smoothly until it was terminated sometime in January 1999 after
the agreement was breached with neither party admitting any fault. Steelcase filed a complaint for sum of money
against DISI alleging, among others, that DISI had an unpaid account of US$600,000.00. Steelcase prayed that
DISI be ordered to pay actual or compensatory damages, exemplary damages, attorney’s fees, and costs of suit.
Among the counter-arguments raised, DISI alleged that the complaint failed to state a cause of action and to
contain the required allegations on Steelcase’s capacity to sue in the Philippines despite the fact that Steelcase was
doing business in the Philippines without the required license to do so. Consequently, it posited that the complaint
should be dismissed because of Steelcase’s lack of legal capacity to sue in Philippine courts.

The Regional Trial Court (RTC) dismissed the complaint and granted the temporary restraining order prayed for by
DISI. The RTC stated that in requiring DISI to meet the Dealer Performance Expectation and in terminating the
dealership agreement with DISI based on its failure to improve its performance in the areas of business planning,
organizational structure, operational effectiveness, and efficiency, Steelcase unwittingly revealed that it
participated in the operations of DISI. Despite a showing that DISI transacted with the local customers in its own
name and for its own account, the RTC stated that any doubt in the factual environment should be resolved in
favor of a pronouncement that a foreign corporation was doing business in the Philippines, considering the twelve-
year period that DISI had been distributing Steelcase products in the Philippines. The RTC concluded that Steelcase
was "doing business" in the Philippines, as contemplated by the Foreign Investments Act of 1991, and since it did
not have the license to do business in the country, it was barred from seeking redress from our courts until it
obtained the requisite license to do so. Steelcase moved for the reconsideration of the dismissal but the same was
denied.

Aggrieved, Steelcase appealed the case to the Court of Appeals. The Court of Appeals rendered its Decision
affirming the RTC orders, ruling that Steelcase was a foreign corporation doing or transacting business in the
Philippines without a license. Steelcase filed a motion for reconsideration but it was denied by the Court of Appeals.

Steelcase filed a Petition for Review with the Supreme Court.

Issue: (a) Is Steelcase doing business in the Philippines without a license?


(b) Is DISI estopped from challenging the Steelcase’s legal capacity to sue?

Held: (a) The Supreme Court ruled in favor of Steelcase.

Steelcase is an unlicensed foreign corporation not doing business in the Philippines


According to the Supreme Court, the following acts shall not be deemed "doing business" in the Philippines: (a)
mere investment as a shareholder by a foreign entity in domestic corporations duly registered to do business,
and/or the exercise of rights as such investor; (b) having a nominee director or officer to represent its interest in
such corporation; (c) appointing a representative or distributor domiciled in the Philippines which transacts
business in the representative's or distributor's own name and account; (d) the publication of a general
advertisement through any print or broadcast media; (e) maintaining a stock of goods in the Philippines solely for
the purpose of having the same processed by another entity in the Philippines; (f) consignment by a foreign entity
of equipment with a local company to be used in the processing of products for export; (g) collecting information in
the Philippines; and (h) performing services auxiliary to an existing isolated contract of sale which are not on a
continuing basis, such as installing in the Philippines machinery it has manufactured or exported to the Philippines,
servicing the same, training domestic workers to operate it, and similar incidental services.
Based on this list, the Supreme Court said that the appointment of a distributor in the Philippines is not sufficient to
constitute "doing business" unless it is under the full control of the foreign corporation. If the distributor is an
independent entity which buys and distributes products, other than those of the foreign corporation, for its own
name and its own account, the latter cannot be considered to be doing business in the Philippines.
CORPORATION LAW CASE DIGESTS | 130
FEU JD4401 | 2nd SEMESTER, S.Y. 2017-2018

Applying these rules, the Supreme Court said that DISI was founded in 1979 and is independently owned and
managed. In addition to Steelcase products, DISI also distributed products of other companies including carpet
tiles, relocatable walls and theater settings. The dealership agreement between Steelcase and DISI had been
described by the owner himself as a buy and sell arrangement. This clearly belies DISI’s assertion that it was a
mere conduit through which Steelcase conducted its business in the country. From the preceding facts, the only
reasonable conclusion that can be reached is that DISI was an independent contractor, distributing various
products of Steelcase and of other companies, acting in its own name and for its own account. As a result,
Steelcase cannot be considered to be doing business in the Philippines by its act of appointing a distributor as it
falls under one of the exceptions under R.A. No. 7042.

(b) DISI is estopped from challenging Steelcase's capacity to sue

On this point, the Supreme Court declared that “if indeed Steelcase had been doing business in the Philippines
without a license, DISI would nonetheless be estopped from challenging the former’s legal capacity to sue xxx A
foreign corporation doing business in the Philippines may sue in Philippine Courts although not authorized to do
business here against a Philippine citizen or entity who had contracted with and benefited by said corporation. To
put it in another way, a party is estopped to challenge the personality of a corporation after having acknowledged
the same by entering into a contract with it. And the doctrine of estoppel to deny corporate existence applies to a
foreign as well as to domestic corporations. One who has dealt with a corporation of foreign origin as a corporate
entity is estopped to deny its corporate existence and capacity.”

Relevance: Although the foreign corporation in this case was declared to be not doing business in the Philippines,
this case, nonetheless, explicitly declares another exception to the rule provided in Section 133 of the Corporation
Code of the Philippines that “[n]o foreign corporation transacting business in the Philippines without a license, or its
successors or assigns, shall be permitted to maintain or intervene in any action, suit or proceeding in any court or
administrative agency of the Philippines…” Following the ruling in this case, a foreign corporation doing business in
the Philippines without a license may maintain suit in the Philippines against a domestic corporation or person who
is party to a contract as the domestic corporation or person is deemed estopped from challenging the personality of
the foreign corporation.
CORPORATION LAW CASE DIGESTS | 131
FEU JD4401 | 2nd SEMESTER, S.Y. 2017-2018

10. PENAL PROVISIONS


(79) Yuico v. Quiambao
G.R. No. 180416, June 2, 2014
By: Bernardo, Gerard T.

Doctrine: Refusal to allow inspection of the stock and transfer book of a corporation when done in violation of
Section 74(4) of the Corporation Code, properly falls within the purview of Section 144 of the same code and thus
may be penalized as an offense. However, a criminal action based on the violation of a stockholder’s right to
examine or inspect the corporate records and the stock and transfer book of a corporation under the second and
fourth paragraphs of Section 74 of the Corporation Code can only be maintained against corporate officers or any
other persons acting on behalf of such corporation.

Facts: Yujuico was elected as president and chairman of Strategic Alliance Development Corporation (STRADEC).
Yujuico replaced respondent Quiambao. STRADEC appointed Blando as corporate secretary. Blando replaced
respondent Pilapil as corporate secretary of STRADEC.

Petitioners filed a criminal complaint against respondents and Casanova. The complaint accuses respondents and
Casanova of violating Section 74 in relation to Section 144 of the Corporation Code. Petitioner alleged that during
the stockholders’ meeting on 1 March 2004, Yujuico — as newly elected president and chairman of STRADEC —
demanded Quiambao for the turnover of the corporate records of the company, particularly the accounting files,
ledgers, journals and other records of the corporation’s business. Quiambao refused. As it turns out, the corporate
records of STRADEC were in the possession of Casanova. Quiambao and Casanova caused the removal of the
corporate records of STRADEC from the company’s offices in Pasig City.

Petitioners theorize that the refusal by the respondents and Casanova to turnover STRADEC’s corporate records
and stock and transfer book violates their right, as stockholders, directors and officers of the corporation, to
inspect such records and book under Section 74 of the Corporation Code. For such violation, petitioners conclude,
respondents may be held criminally liable pursuant to Section 144 of the Corporation Code.

Petitioner filed two information, (1) for removing the stock and transfer book of STRADEC from its principal office;
and (2) for refusing access to, and examination of, the corporate records and the stock and transfer book of
STRADEC at its principal office.

RTC issued an order dismissing the Criminal Case regarding the refusing access to corporate records. RTC, opined
that refusing to allow inspection of the stock and transfer book, as opposed to refusing examination of other
corporate records, is not punishable as an offense under the Corporation Code.

Issue: Whether or not refusing to allow inspection of the stock and transfer book is a violation of section 74 of the
Corporation Code, for which Section 144 of the same code prescribes a penalty.

Held: Yes. The RTC indeed made an inaccurate pronouncement when it held that the act of refusing to allow
inspection of the stock and transfer book of a corporation is not a punishable offense under the Corporation Code.
Such refusal, when done in violation of Section 74(4) of the Corporation Code, properly falls within the purview of
Section 144 of the same code and thus may be penalized as an offense.

However, a criminal action based on the violation of a stockholder’s right to examine or inspect the corporate
records and the stock and transfer book of a corporation under the second and fourth paragraphs of Section 74 of
the Corporation Code can only be maintained against corporate officers or any other persons acting on behalf of
such corporation. The submissions of the petitioners during the preliminary investigation, however, clearly suggest
that respondents are neither in relation to STRADEC.
CORPORATION LAW CASE DIGESTS | 132
FEU JD4401 | 2nd SEMESTER, S.Y. 2017-2018

(80) James Lent vs. Tullett Philippines, Inc., G.R. No. 189158, January 11, 2017
By: Donna Bigornia

Topic: Penalty for violation of Section 31 and/or 34 of the Corporation Code

Doctrine: The penalty as used in Section 144 of the Corporation Code encompasses administrative
penalties, not only criminal sanctions.

Facts: Tullett, through one of its directors filed a Complaint-Affidavit with the City Prosecution Office of Makati City
against the officers/employees of the Tradition Group for violation Sections 31 and 34 of the Corporation Code
which made them criminally liable under Section 144. Impleaded as respondents in the Complaint-Affidavit were
Villalon (who was formerly President and Managing Director of Tullett), Chuidian (who was formerly a member of
Tullett's Board of Directors), petitioners lent and Schulze, for conspiring with Villalo and Chuidan, and other John
and Jane Does. Villalon and Chuidian were charged with using their former positions in Tullett to sabotage said
company by orchestrating the mass resignation of its entire brokering staff in order for them to join Tradition
Philippines.

The petitioners argued that Section 144 of the Corporation Code applies only to violations of provisions of the
Corporation code which do not provide for a penalty for its violation. Therefore, Section 144 cannot be applied to
Sections 31 and 34 of the same Code considering that such provisions already contains the penalties or remedies
for their violation. On the other hand, respondent have taken the position that the term “penalized” under Section
144 should be interpreted as referring to criminal penalty, such as fine or imprisonment, and that it could not
possibly contemplate “civil” penalties such as damages, accounting or restitution. It added that such provision
should be strictly construed as contemplating only penal penalties.

Issue: Whether section 144 of the Corporation Code is applicable for violation of section 31 and 34 of
the same code considering that the latter already provides penalties for its violation.

Ruling: No.

SECTION 31. Liability of Directors, Trustees or Officers. - Directors or trustees who wilfully and knowingly vote for
or assent to patently unlawful acts of the corporation or who are guilty of gross negligence or bad faith in directing
the affairs of the corporation or acquire any personal or pecuniary interest in conflict with their duty as such
directors or trustees shall be liable jointly and severally for all damages resulting therefrom suffered by the
corporation, its stockholders or members and other persons.

When a director, trustee or officer attempts to acquire or acquires, in violation of his duty, any interest adverse to
the corporation in respect of any matter which has been reposed in him in confidence, as to which equity imposes a
disability upon him to deal in his own behalf, he shall be liable as a trustee for the corporation and must account
for the profits which otherwise would have accrued to the corporation.

SECTION 34. Disloyalty of a Director. - Where a director, by virtue of his office, acquires for himself a business
opportunity which should belong to the corporation, thereby obtaining profits to the prejudice of such corporation,
he must account to the latter for all such profits by refunding the same, unless his act has been ratified by a vote
of the stockholders owning or representing at least two-thirds (2/3) of the outstanding capital stock. This provision
shall be applicable, notwithstanding the fact that the director risked his own funds in the venture.

SECTION 144. Violations of the Code. - Violations of any of the provisions of this Code or its amendments not
otherwise specifically penalized therein shall be punished by a fine of not less than one thousand (₱1,000.00) pesos
but not more than ten thousand (₱10,000.00) pesos or by imprisonment for not Jess than thirty (30) days but not
more than five (5) years, or both, in the discretion of the court. If the violation is committed by a corporation, the
same may, after notice and hearing, be dissolved in appropriate proceedings before the Securities and Exchange
Commission: Provided, That such dissolution shall not preclude the institution of appropriate action against the
director, trustee or officer of the corporation responsible for said violation: Provided, further, That nothing in this
section shall be construed to repeal the other causes for dissolution of a corporation provided in this Code.

Section 31, 34 and all provisions of the Corporation Code including section 74 of the same Code provide for civil or
pecuniary liabilities for the acts covered therein but what is significant is the fact that, all of those provisions that
provide for consequences other that penal, only section 74 expressly states that a violation thereof is likewise
considered an offense under section 144. If respondent and the Court of Appeals are correct, that Section 144
automatically imposes penal sanctions on violations of provisions for which no criminal penalty was imposed, then
such language in Section 74 defining a violation thereof as an offense would have been superfluous. There would
be no need for legislators to clarify that, aside from civil liability, violators of Section 74 are exposed to criminal
liability as well. We agree with petitioners that the lack of specific language imposing criminal liability in Sections
31 and 34 shows legislative intent to limit the consequences of their violation to the civil liabilities mentioned
CORPORATION LAW CASE DIGESTS | 133
FEU JD4401 | 2nd SEMESTER, S.Y. 2017-2018

therein. Had it been the intention of the drafters of the law to define Sections 31 and 34 as offenses, they could
have easily included similar language as that found in Section 74.
CORPORATION LAW CASE DIGESTS | 134
FEU JD4401 | 2nd SEMESTER, S.Y. 2017-2018

11. CASES UNDER THE SECURITIES REGULATIONS CODE

(81) Power Homes Unlimited Corp. v. SEC


G.R. No. 164182, February 26, 2008
By: Corona, Jose Enrico V.

Doctrine:

Facts: Petitioner is a domestic corporation registered with SEC. Private Respondent requested SEC to investigate
the petitioner’s business due to sale of inexistent properties and without any broker’s license. Another inquiry was
made by Munsayac with the SEC inquiring whether petitioner’s business involves legitimate network marketing.

SEC held a conference. Petitioner’s incorporators attended the conference. After investigation conducted, SEC
found that the petitioner was engaged in the sale or offer for sale or distribution of investment contracts, which are
considered securities under Sec. 3.1 (b) of Republic Act (R.A.) No. 8799 (The Securities Regulation Code), but
failed to register them in violation of Sec. 8.1 of the same Act.

SEC then issued a cease and desist order against Petitioner. Petitioner then went to the Court of Appeals imputing
grave abuse of discretion amounting to lack or excess of jurisdiction on respondent SEC for issuing the Order.

Court of Appeals denied the petition and affirmed the Cease and Desist Order.

Issue: (1) whether public respondent SEC followed due process in the issuance of the assailed CDO; and
(2) whether petitioner’s business constitutes an investment contract which should be registered with public
respondent SEC before its sale or offer for sale or distribution to the public.

Held: (1). Yes. There was no violation of due process.

Sec. 64. Cease and Desist Order. 64.1. The Commission, after proper investigation or verification, motu proprio or
upon verified complaint by any aggrieved party, may issue a cease and desist order without the necessity of a prior
hearing if in its judgment the act or practice, unless restrained, will operate as a fraud on investors or is otherwise
likely to cause grave or irreparable injury or prejudice to the investing public.

We hold that petitioner was not denied due process. The records reveal that public respondent SEC properly
examined petitioners business operations when it (1) called into conference three of petitioners incorporators, (2)
requested information from the incorporators regarding the nature of petitioners business operations, (3) asked
them to submit documents pertinent thereto, and (4) visited petitioners business premises and gathered
information thereat. All these were done before the CDO was issued by the public respondent SEC. Trite to state, a
formal trial or hearing is not necessary to comply with the requirements of due process. Its essence is simply the
opportunity to explain one’s position. Public respondent SEC abundantly allowed petitioner to prove its side.

(2). Yes. petitioner’s business constitutes an investment contract.

To be a security subject to regulation by the SEC, an investment contract in our jurisdiction must be proved to be:
(1) an investment of money, (2) in a common enterprise, (3) with expectation of profits, (4) primarily from efforts
of others.

In the petitioner’s business, an investor enrolls in petitioners program by paying US$234. This entitles him to
recruit two (2) investors who pay US$234 each and out of which amount he receives US$92. A minimum
recruitment of four (4) investors by these two (2) recruits, who then recruit at least two (2) each, entitles the
principal investor to US$184 and the pyramid goes on.

An investor enrolls under the scheme of petitioner to be entitled to recruit other investors and to receive
commissions from the investments of those directly recruited by him. Under the scheme, the accumulated amount
received by the investor comes primarily from the efforts of his recruits.
CORPORATION LAW CASE DIGESTS | 135
FEU JD4401 | 2nd SEMESTER, S.Y. 2017-2018

(82) SEC vs Prosperity.com, Inc


G.R. No164197. January 25, 2012
By: Kathrina De Castro

DOCTRINE: The Securities Regulation Code treats investment contracts as “securities” that have to be registered
with the SEC before they can be distributed and sold. An investment contract is a contract, transaction, or scheme
where a person invests his money in a common enterprise and is led to expect profits primarily from the efforts of
others.

Conditions for Investment Contracts to Arise:


Commission v. W.J. Howey Co., that, for an investment contract to exist, the following elements, referred to as the
Howey test must concur: (1) a contract, transaction, or scheme; (2) an investment of money; (3) investment is
made in a common enterprise; (4) expectation of profits; and (5) profits arising primarily from the efforts of
others.

PCI sold computer software and hosted websites without providing internet service. To make a profit it devised a
scheme in which, for the price of US$234.00, a buyer could acquire from it an internet website of a 15MB capacity.
At the same time, by referring to PCI his own down-line buyers, a first-time buyer could earn commissions, interest
in real estate in the Philippines and in the United States, and insurance coverage worth P50,000.00. To benefit
from this scheme, a PCI buyer must enlist and sponsor at least two other buyers as his own down-lines. These
second tier of buyers could in turn build up their own down-lines. For each pair of down-lines, the buyer-sponsor
received a US$92.00 commission. But referrals in a day by the buyer-sponsor should not exceed 16 since the
commissions due from excess referrals inure to PCI, not to the buyer sponsor. PCI patterned its scheme from that
of GVI, which the SEC issued a cease and desist order against it to stop its operation. It turned out that same
people behind GVI directed PCI’s actual operations. Aggrieved elements of GVI filed a complaint with the SEC
against PCI, alleging that latter had taken over GVI’s operations.

SEC – issued cease and desist order against PCI. It ruled that PCI’s scheme constitutes Investment contract and,
following the Securities Regulation Code, it should have first registered such contract or securities with SEC

CA – granted PCI’s petition and setting aside the cease and desist order issued by SEC. It ruled that, following the
Howey test, PCI’s scheme did not constitute an investment contract that needs registration pursuant to R.A. 8799.

A. Does the PCI’s scheme constitutes an investment contract that requires registration under R.A. 8799

No. The Securities Regulation Code treats investment contracts as “securities” that have to be registered with the
SEC before they can be distributed and sold. An investment contract is a contract, transaction, or scheme where a
person invests his money in a common enterprise and is led to expect profits primarily from the efforts of others.

The United States Supreme Court held in Securities and Exchange Commission v. W.J. Howey Co., that, for an
investment contract to exist, the following elements, referred to as the Howey test must concur:
(1) a contract, transaction, or scheme;
(2) an investment of money;
(3) investment is made in a common enterprise;
(4) expectation of profits; and
(5) profits arising primarily from the efforts of others.

Thus, to sustain the SEC position in this case, PCI’s scheme or contract with its buyers must have all these
elements.

Here, PCI’s clients do not make such investments. They buy a product of some value to them: an Internet website
of a 15MB capacity. The client can use this website to enable people to have internet access to what he has to offer
to them, say, some skin cream. The buyers of the website do not invest money in PCI that it could use for running
some business that would generate profits for the investors. The price of US$234.00 is what the buyer pays for the
use of the website, a tangible asset that PCI creates, using its computer facilities and technical skills. Actually, PCI
appears to be engaged in network marketing, a scheme adopted by companies for getting people to buy their
products outside the usual retail system where products are bought from the store’s shelf. Under this scheme,
adopted by most health product distributors, the buyer can become a down-line seller. The latter earns
commissions from purchases made by new buyers whom he refers to the person who sold the product to him. The
network goes down the line where the orders to buy come.

The commissions, interest in real estate, and insurance coverage worth P50,000.00 are incentives to down-line
sellers to bring in other customers. These can hardly be regarded as profits from investment of money under the
Howey test.
CORPORATION LAW CASE DIGESTS | 136
FEU JD4401 | 2nd SEMESTER, S.Y. 2017-2018

The CA is right in ruling that the last requisite in the Howey test is lacking in the marketing scheme that PCI has
adopted. Evidently, it is PCI that expects profit from the network marketing of its products. PCI is correct in saying
that the US$234 it gets from its clients is merely a consideration for the sale of the websites that it provides.
CORPORATION LAW CASE DIGESTS | 137
FEU JD4401 | 2nd SEMESTER, S.Y. 2017-2018

(83) Securities and Exchange Commission v. Santos


G.R. No. 195542, March 19, 2014
By: Gaite, Rhio Angeline

Doctrine: An investment contract or participation in a profit sharing agreement that falls within the definition of
the law. When the investor is relatively uninformed and turns over his money to others, essentially depending upon
their representations and their honesty and skill in managing it, the transaction generally is considered to be an
investment contract. The touchstone is the presence of an investment in a common venture premised on a
reasonable expectation of profits to be derived from the entrepreneurial or managerial efforts of others.

Facts: Sometime in 2007, yet another investment scam was exposed with the disappearance of its primary
perpetrator, Michael H.K. Liew (Liew), a self- styled financial guru and Chairman of the Board of Directors of
Performance Investment Products Corporation (PIPC-BVI), a foreign corporation registered in the British Virgin
Islands.

To do business in the Philippines, PIPC-BVI incorporated herein as Philippine International Planning Center
Corporation (PIPC Corporation).

Because the head of PIPC Corporation had gone missing and with it the monies and investment of a significant
number of investors, the SEC was flooded with complaints from 31 individuals against PIPC Corporation, its
directors, officers, employees, agents and brokers for alleged violation of certain provisions of the Securities
Regulation Code, including Section 28 thereof. Santos was charged in the complaints in her capacity as investment
consultant of PIPC Corporation, who supposedly induced private complainants Luisa Mercedes P. Lorenzo (Lorenzo)
and Ricky Albino P. Sy (Sy), to invest their monies in PIPC Corporation.

The common recital in the 31 complaints is that: x x x Due to the inducements and solicitations of the PIPC
corporation’s directors, officers and employees/agents/brokers, the former were enticed to invest their hard-earned
money, the minimum amount of which must be US$40,000.00, with PIPC-BVI, with a promise of higher income
potential of an interest of 12 to 18 % per annum at relatively low-risk investment program. The private
complainants also claimed that they were made to believe that PIPC Corporation refers to Performance Investment
Product Corporation, the Philippine office or branch of PIPC-BVI, which is an entity engaged in foreign currency
trading, and not Philippine International Planning Center Corporation.

On the whole, Lorenzo and Sy charge Santos in her capacity as investment consultant of PIPC Corporation who
actively engaged in the solicitation and recruitment of investors. Private complainants maintain that Santos, apart
from being PIPC Corporation’s employee, acted as PIPC Corporation’s agent and made representations regarding its
investment products and that of the supposed global corporation PIPC-BVI. Facilitating Lorenzo’s and Sy’s
investment with PIPC Corporation, Santos represented to the two that investing with PIPC Corporation, an affiliate
of PIPC-BVI, would be safe and full-proof.

Issues: Whether or not respondent Santos acted as agent of PIPC Corp. or had enticed Luisa Mercedes P. Lorenzo
or Ricky Albino P. Sy to buy PIPC Corp. or PIPC-BVI’s investment products

Held: Yes. There is no quarrel that Santos was in the employ of PIPC Corporation and/or PIPC-BVI, a corporation
which sold or offered for sale unregistered securities in the Philippines. To escape probable culpability, Santos
claims that she was a mere clerical employee of PIPC Corporation and/or PIPC-BVI and was never an agent or
salesman who actually solicited the sale of or sold unregistered securities issued by PIPC Corporation and/or PIPC-
BVI.

Solicitation is the act of seeking or asking for business or information; it is not a commitment to an agreement.
Santos, by the very nature of her function as what she now unaffectedly calls an information provider, brought
about the sale of securities made by PIPC Corporation and/or PIPC-BVI to certain individuals, specifically private
complainants Sy and Lorenzo by providing information on the investment products of PIPC Corporation and/or
PIPC- BVI with the end in view of PIPC Corporation closing a sale. While Santos was not a signatory to the
contracts on Sy’s or Lorenzo’s investments, Santos procured the sale of these unregistered securities to the two (2)
complainants by providing information on the investment products being offered for sale by PIPC Corporation
and/or PIPC-BVI and convincing them to invest therein. No matter Santos’ strenuous objections, it is apparent that
she connected the probable investors, Sy and Lorenzo, to PIPC Corporation and/or PIPC-BVI, acting as an
ostensible agent of the latter on the viability of PIPC Corporation as an investment company. At each point of Sy’s
and Lorenzo’s investment, Santos’ participation thereon, even if not shown strictly on paper, was prima facie
established. In all of the documents presented by Santos, she never alleged or pointed out that she did not receive
extra consideration for her simply providing information to Sy and Lorenzo about PIPC Corporation and/or PIPC-
BVI. Santos only claims that the monies invested by Sy and Lorenzo did not pass through her hands. In short,
Santos did not present in evidence her salaries as a supposed "mere clerical employee or information provider" of
PIPC-BVI. Such presentation would have foreclosed all questions on her status within PIPC Corporation and/or
PIPC-BVI at the lowest rung of the ladder who only provided information and who did not use her discretion in any
CORPORATION LAW CASE DIGESTS | 138
FEU JD4401 | 2nd SEMESTER, S.Y. 2017-2018

capacity. The SC cannot overemphasize that the very information provided by Santos locked the deal on
unregistered securities with Sy and Lorenzo. What is palpable from the foregoing is that Sy and Lorenzo did not go
directly to Liew or any of PIPC Corporation’s and/or PIPC-BVI’s principal officers before making their investment or
renewing their prior investment. However, undeniably, Santos actively recruited and referred possible investors to
PIPC Corporation and/or PIPC-BVI and acted as the go-between on behalf of PIPC Corporation and/or PIPC-BVI.

Individual complainants and the SEC have categorically alleged that Liew and PIPC Corporation and/or PIPC-BVI is
not a legitimate investment company but a company which perpetrated a scam on 31 individuals where the
president, a foreign national, Liew, ran away with their money. Liew’s absconding with the monies of 31 individuals
and that PIPC Corporation and/or PIPC-BVI were not licensed by the SEC to sell securities are uncontroverted facts.

The transaction initiated by Santos with Sy and Lorenzo, respectively, is an investment contract or participation in
a profit sharing agreement that falls within the definition of the law. When the investor is relatively uninformed and
turns over his money to others, essentially depending upon their representations and their honesty and skill in
managing it, the transaction generally is considered to be an investment contract.23 The touchstone is the
presence of an investment in a common venture premised on a reasonable expectation of profits to be derived
from the entrepreneurial or managerial efforts of others.24

At bottom, the exculpation of Santos cannot be preliminarily established simply by asserting that she did not sign
the investment contracts, as the facts alleged in this case constitute fraud perpetrated on the public. Specially so
because the absence of Santos’ signature in the contract is, likewise, indicative of a scheme to circumvent and
evade liability should the pyramid fall apart.
CORPORATION LAW CASE DIGESTS | 139
FEU JD4401 | 2nd SEMESTER, S.Y. 2017-2018

(84) SEC vs. CJH Development


G.R. No. 210316, November 28, 2016
By: Grande, Jonicocel

Doctrine: The non-appealability of a CDO issued by the SEC is provided for under the 2006 Rules of Procedure of
the Commission.

Under the doctrine of primary administrative jurisdiction, courts will not determine a controversy where the issues
for resolution demand the exercise of sound administrative discretion requiring the special knowledge, experience,
and services of the administrative tribunal to determine technical and intricate matters of fact, which under a
regulatory scheme have been placed within the special competence of such tribunal or agency.

Facts:
CJHDC entered into a Lease Agreement (Agreement) with the Bases Conversion and Development Authority
(BCDA) for the development into a public tourism complex.
Subsequently, CJHDC came up with a development plan and put it into effect. Part of such development plan was
the construction of two (2) condotels which it named as "The Manor" and "The Suites". Subject to CJHDC's
leasehold rights under the Agreement, the residential units in these condotels were then offered for sale to the
general public by means of two schemes. The first is a straight purchase and sale contract. The second scheme
involved the sale of the unit with an added option to avail of a "leaseback" or a "money-back" arrangement. Under
this added option, the buyer pays for the unit bought and, subsequently, surrenders its possession to the
management of CJHDC or CJHSC. These corporations would then create a pool of these units and, in tum, will offer
them for billeting under the management of the hotel operated by the Camp John Hay Leisure, Inc. (CJHLI).

BCDA acquired information regarding CJHDC and CJHSC's scheme of selling "The Manor" and "The Suites" units.
Hence, in a letter, the BCDA requested the SEC to conduct an investigation into the operations of CJHDC and
CJHSC on the belief that the "leaseback" or "money-back" arrangements they are offering to the public is, in
essence, investment contracts which are considered as securities.

SEC investigated the matter and issued a CEASE and DESIST from further engaging in the business of selling
securities until they comply with the requirements provided by law.

ISSUE: (1) WON CDO issued by SEC appealable.

NO.
The non-appealability of a CDO issued by the SEC is provided for under the 2006 Rules of Procedure of the
Commission. Thus, Section 10-8 of the Rules provides:c
SEC. 10-8. Prohibitions. - No pleading, motion or submission in any form that may prevent the resolution of an
application for a CDO by the Commission shall be entertained except under Rule XII herein. A CDO when issued,
shall not be the subject of an appeal and no appeal from it will be entertained; Provided, however, that an
order by the Director of the Operating Department denying the motion to lift a CDO may be appealed to the
Commission En Banc through the office of the General Counsel. Hence, the CDO issued by SEC is non-appealable.

(2) WON the court may determine whether the sale of “The Manor” or “The Suites” units to the general public
under the “leaseback” or “money-back” scheme is a form of investment contract or sale of securities.

NO.
The main issue, as to whether or not the sale of "The Manor" or "The Suites" units to the general public under the
"leaseback" or "money-back" scheme is a form of investment contract or sale of securities, is not a pure question
of law. On the contrary, it involves a question of fact that falls under the primary jurisdiction of the SEC. Under the
doctrine of primary administrative jurisdiction, courts will not determine a controversy where the issues for
resolution demand the exercise of sound administrative discretion requiring the special knowledge, experience, and
services of the administrative tribunal to determine technical and intricate matters of fact, which under a regulatory
scheme have been placed within the special competence of such tribunal or agency.
CORPORATION LAW CASE DIGESTS | 140
FEU JD4401 | 2nd SEMESTER, S.Y. 2017-2018

In other words, if a case is such that its determination requires the expertise, specialized training, and knowledge
of an administrative body, relief must first be obtained in an administrative proceeding before resort to the court is
had even if the matter may well be within the latter's proper jurisdiction. The objective of the doctrine of primary
jurisdiction is to guide the court in determining whether it should refrain from exercising its jurisdiction until after
an administrative agency has determined some question or some aspect of some question arising in the proceeding
before the court.

In the instant case, the resolution of the issue as to whether respondents' scheme of selling the subject condotel
units is tantamount to an investment contract and/or sale of securities, as defined under the SRC, requires the
expertise and technical knowledge of the SEC being the government agency which is tasked to enforce and
implement the provisions of the said Code as well as its implementing rules and regulations. In fact, after the
issuance of the CDO, the SEC is yet to hear from respondents and receive evidence from them regarding this issue.
Nonetheless, respondents prematurely filed an appeal with the CA, which erroneously gave due course to it in
disregard of the doctrine primary jurisdiction.
CORPORATION LAW CASE DIGESTS | 141
FEU JD4401 | 2nd SEMESTER, S.Y. 2017-2018

(85) Abacus Securities Corp. v. Ruben Ampil


G.R. No. 160016, Feb. 27, 2006
By: Jovero, John Tristram V.

Doctrine:

Facts: In April 1997, respondent opened a cash or regular account with petitioner for buying and selling securities
as evidenced by the Account Application Form. The parties’ business relationship was governed by the terms and
conditions stated therein.

Since April 10, 1997, respondent actively traded his account, and as a result of such trading activities, he
accumulated an outstanding obligation in favor of petitioner in the sum of P6,617,036.22 as of April 30, 1997.
Respondent failed to pay petitioner his liabilities. Petitioner sold respondent’s securities to set off against his
unsettled obligations.

After the sale of respondent’s securities and application of the proceeds thereof against his account, respondent’s
remaining unsettled obligation to petitioner was P3,364,313.56.

Petitioner demanded that respondent settle his obligation plus the agreed penalty charges accruing thereon
equivalent to the average 90-day Treasury Bill rate plus 2% per annum. Despite said demand and the lapse of said
requested extension, respondent failed and/or refused to pay his accountabilities to petitioner. Respondent claims
that he was induced to trade in a stock security with petitioner because the latter allowed offset settlements
wherein he is not obliged to pay the purchase price. Rather, it waits for the customer to sell. And if there is a loss,
petitioner only requires the payment of the deficiency (i.e., the difference between the higher buying price and the
lower selling price). In addition, it charges a commission for brokering the sale. However, if the customer sells and
there is a profit, petitioner deducts the purchase price and delivers only the surplus – after charging its
commission.

RTC- Makati City held that petitioner and respondent were in pari delicto and therefore without recourse against
each other. CA upheld the lower court’s finding that the parties were in pari delicto. It castigated petitioner for
allowing respondent to keep on trading despite the latter’s failure to pay his outstanding obligations. It explained
that “the reason behind petitioner’s act is because whether respondent’s trading transaction would result in a
surplus or deficit, he would still be liable to pay petitioner its commission. Hence, this Petition.

Issue: What is margin requirement?

Held: The main purpose of the statute on margin requirements is to regulate the volume of credit flow, by way of
speculative transactions, into the securities market and redirect resources into more productive uses. It is also to
give a government credit agency an effective method of reducing the aggregate amount of the nation’s credit
resources which can be directed by speculation into the stock market and out of other more desirable uses of
commerce and industry.

It is for the stabilization of the economy. Restrictions on margin percentages are imposed “in order to achieve the
objectives of the government with due regard for the promotion of the economy and prevention of the use of
excessive credit.” Otherwise stated, the margin requirements set out in the RSA are primarily intended to achieve a
macroeconomic purpose — the protection of the overall economy from excessive speculation in securities. Their
recognized secondary purpose is to protect small investors.

The law places the burden of compliance with margin requirements primarily upon the brokers and dealers.
Sections 23 and 25 and Rule 25-1, otherwise known as the “mandatory close-out rule,” clearly vest upon petitioner
the obligation, not just the right, to cancel or otherwise liquidate a customer’s order, if payment is not received
within three days from the date of purchase. For transactions subsequent to an unpaid order, the broker should
require its customer to deposit funds into the account sufficient to cover each purchase transaction prior to its
execution. These duties are imposed upon the broker to ensure faithful compliance with the margin requirements
of the law, which forbids a broker from extending undue credit to a customer.

It will be noted that trading on credit (or “margin trading”) allows investors to buy more securities than their cash
position would normally allow. Investors pay only a portion of the purchase price of the securities; their broker
advances for them the balance of the purchase price and keeps the securities as collateral for the advance or loan.
Brokers take these securities/stocks to their bank and borrow the “balance” on it, since they have to pay in full for
the traded stock. Hence, increasing margins i.e., decreasing the amounts which brokers may lend for the
speculative purchase and carrying of stocks is the most direct and effective method of discouraging an abnormal
attraction of funds into the stock market and achieving a more balanced use of such resources.
CORPORATION LAW CASE DIGESTS | 142
FEU JD4401 | 2nd SEMESTER, S.Y. 2017-2018

The nature of the stock brokerage business enables brokers, not the clients, to verify, at any time, the status of
the client’s account. Brokers are in the superior position to prevent the unlawful extension of credit. Because of this
awareness, the law imposes upon them the primary obligation to enforce the margin requirements.

Nonetheless, these margin requirements are applicable only to transactions entered into by the present parties
subsequent to the initial trades of April 10 and 11, 1997. Thus, we hold that petitioner can still collect from
respondent to the extent of the difference between the latter’s outstanding obligation as of April 11, 1997 less the
proceeds from the mandatory sell out of the shares pursuant to the RSA Rules. Petitioner’s right to collect is
justified under the general law on obligations and contracts.

The right to collect cannot be denied to petitioner as the initial transactions were entered pursuant to the
instructions of respondent. The obligation of respondent for stock transactions made and entered into on April 10
and 11, 1997 remains outstanding. These transactions were valid and the obligations incurred by respondent
concerning his stock purchases on these dates subsist. At that time, there was no violation of the RSA yet.
Petitioner’s fault arose only when it failed to: 1) liquidate the transactions on the fourth day following the stock
purchases, or on April 14 and 15, 1997; and 2) complete its liquidation no later than ten days thereafter, applying
the proceeds thereof as payment for respondent’s outstanding obligation.

Since the buyer was not able to pay for the transactions that took place on April 10 and 11, the broker was duty-
bound to advance the payment to the settlement banks without prejudice to the right of the broker to collect later
from the client.

In securities trading, the brokers are essentially the counterparties to the stock transactions at the Exchange. Since
the principals of the broker are generally undisclosed, the broker is personally liable for the contracts thus made.
Hence, petitioner had to advance the payments for respondent’s trades. Brokers have a right to be reimbursed for
sums advanced by them with the express or implied authorization of the principal, in this case, respondent. Not to
require respondent to pay for his April 10 and 11 trades would put a premium on his circumvention of the laws and
would enable him to enrich himself unjustly at the expense of petitioner.

In the present case, petitioner failed to enforce the terms and conditions of its Agreement with respondent,
specifically paragraph 8 thereof, purportedly acting on the plea of respondent to give him time to raise funds
therefor. By failing to ensure respondent’s payment of his first purchase transaction within the period prescribed by
law, thereby allowing him to make subsequent purchases, petitioner effectively converted respondent’s cash
account into a credit account. However, extension or maintenance of credits on non-margin transactions, are
specifically prohibited under Section 23(b). Thus, petitioner was remiss in its duty and cannot be said to have come
to court with “clean hands” insofar as it intended to collect on transactions subsequent to the initial trades of April
10 and 11, 1997.

On the other hand, respondent is equally guilty in entering into the transactions in violation of the RSA and RSA
Rules. Respondent is an experienced and knowledgeable trader who is well versed in the securities market and who
made his own investment decisions. In fact, in the Account Opening Form, he indicated that he had excellent
knowledge of stock investments; had experience in stocks trading, considering that he had similar accounts with
other firms. He knowingly speculated on the market, by taking advantage of the “no-cash-out” arrangement
extended to him by petitioner.

Both parties acted in violation of the law and did not come to court with clean hands with regard to transactions
subsequent to the initial trades made on April 10 and 11, 1997.

Since the initial trades (April 10 and 11) are valid and subsisting obligations, respondent is liable for them. Justice
and good conscience require all persons to satisfy their debts. Ours are courts of both law and equity; they compel
fair dealing; they do not abet clever attempts to escape just obligations.

Pursuant to RSA Rule 25-1, petitioner should have liquidated the transaction (sold the stocks) on the fourth day
following the transaction (T+4) and completed its liquidation not later than ten days following the last day for the
customer to pay (effectively T+14). Respondent’s outstanding obligation is therefore to be determined by using the
closing prices of the stocks purchased at T+14 as basis.

We consider the foregoing formula to be just and fair under the circumstances. When petitioner tolerated the
subsequent purchases of respondent without performing its obligation to liquidate the first failed transaction, and
without requiring respondent to deposit cash before embarking on trading stocks any further, petitioner, as the
broker, violated the law at its own peril.

WHEREFORE, the assailed Decision and Resolution of the Court of Appeals are hereby MODIFIED.
CORPORATION LAW CASE DIGESTS | 143
FEU JD4401 | 2nd SEMESTER, S.Y. 2017-2018

(86) Securities and Exchange Commission vs. Interport


G.R. No. 135808. October 6, 2008.
by: Lapuz, Jesus Jr. Ros

Doctrine: INSIDER TRADING - the insider’s misuse of nonpublic and undisclosed information is the gravamen of
illegal conduct. The intent of the law is the protection of investors against fraud, committed when an insider, using
secret information, takes advantage of an uninformed investor. Insiders are obligated to disclose material
information to the other party or abstain from trading the shares of his corporation. This duty to disclose or abstain
is based on two factors: first, the existence of a relationship giving access, directly or indirectly, to information
intended to be available only for a corporate purpose and not for the personal benefit of anyone; and second, the
inherent unfairness involved when a party takes advantage of such information knowing it is unavailable to those
with whom he is dealing.

Facts: The Board of Directors of IRC approved a Memorandum of Agreement with GHB (Ganda Holdings Berhad).
Under said memorandum of agreement, IRC acquired 100% of the entire capital stock of GEHI (Ganda Energy
Holdings Inc.) which would own and operate a 102 megawatt gas turbine power generating barge. In exchange,
IRC will issue to GHB 55% of the expanded capital stock of IRC. On the side, IRC would acquire 67% of the entire
capital of PRCI (Philippine Racing Club).

It is alleged herein that a press release announcing the approval of the agreement was sent to the Philippine Stock
Exchange through facsimile and the SEC, but the facsimile machine of the SEC could not receive it. However, the
SEC received reports that the IRC failed to make timely public disclosures of its negotiations with GHB and that
some of its directors, heavily traded IRC shares utilizing this material insider information. For this reason, the SEC
required the directors to appear before the SEC to explain the alleged failure to disclose material information as
required by the Rules on Disclosure of Material Facts. Unsatisfied with the explanation, the SEC issued an order
finding that the IRC violated the Rules in connection with the then Old Securities Act when it failed to make timely
disclosures of its negotiations with GHB. In addition, the SEC found that the directors of IRC entered into
transactions involving IRC shares in violation of the Revised Securities Act.

The case reached the CA and said court ruled in favor of the respondents and effectively enjoined the SEC from
filing any criminal, civil or administrative cases against respondents. In its resolution, the CA stated that since
there are no rules and regulations implementing the rules regarding DISCLOSURE, INSIDER TRADING OR ANY OF
THE PROVISIONS OF THE REVISED SECURITIES ACT, the SEC has no statutory authority to file any suit against
respondents. The CA, therefore, prohibited the SEC from taking cognizance or initiating any action against the
respondents for the alleged violations of the Revised Securities Act.

Issue: Is insider trading punishable under the SEC Rules despite the lack of implementing rules providing for it?

Held: YES.

The Revised Securities Act does not require the enactment of implementing rules to make it binding and effective.
The provisions of the RSA are sufficiently clear and complete by themselves. The requirements are specifically set
out and the acts which are enjoined are determinable. To rule that absence of implementing rules can render
ineffective an act of Congress would empower administrative bodies to defeat the legislative will by delaying the
implementing rules. Where the statute contains sufficient standards and an unmistakable intent (as in this case,
the RSA) there should be no impediment as to its implementation

This obligation to disclose is imposed upon "insiders" which are particularly officers, directors or controlling
stockholders but that definition has already been expanded and not includes those persons whose relationship of
former relationship to the issuer or the security that is not generally available and the one who learns such a fact
from an insider knowing that the person from whom he learns such fact is an insider. In some case, however, there
may be valid corporate reasons for the nondisclosure of material information but it should not be used for non-
corporate purposes.

Respondent contends that the terms "material fact", "reasonable person", "nature and reliability" and "generally
available" are vaguely used in the RSA because under the provision of the said law what is required to be disclosed
is a fact of special significance, meaning:
1. a material fact which would be likely to affect the market price of a security or
2. one which a reasonable person would consider especially important in determining his course of action
with regard to the shares of stock.

But the court dismissed said contention and stated that material fact is already defined and explained as one which
induces or tends to induce or otherwise affect the sale or purchase of securities. On the other hand, "reasonable
person" has already been used many times in jurisprudence and in law since it is a standard on which most of legal
doctrines stand (even the doctrine on negligence uses such standard) and it has been held to mean "a man who
relies on the calculus of common sense of which all reasonable men have in abundance"
CORPORATION LAW CASE DIGESTS | 144
FEU JD4401 | 2nd SEMESTER, S.Y. 2017-2018

(87) La Bugal-B’Laan Tribal Association, Inc. v. Ramos


G.R. No. 127882, December 1, 2004
By: Mano, Razna I.

Doctrine: It is not necessary for government to attempt to limit or restrict the freedom of the shareholders in the
contractor to freely transfer, dispose of or encumber their shareholdings, consonant with the unfettered exercise of
their business judgment and discretion.

This case dealt with questions around exploration, development and utilization of mineral resources in the
Philippines with the help of foreign companies. A Petition before the Court was filed challenging the constitutionality
of corresponding parts of the Philippine Mining Act, related rules and regulations, and the Financial Technical
Assistance Agreement (FTAA) executed by the government with Western Mining Corporation (Philippines), Inc.
(WMCP).

A sale of WMC shares in WMCP to Sagittarius (60 percent of Sagittarius equity is owned by Filipinos and/or Filipino-
owned corporations while 40 percent is owned by Indophil Resources NL, an Australian company) and subsequent
transfer and registration of the FTAA from WMCP to Sagittarius were had.

a. Is FTAA allowed to be entered only by a foreign corporation?

Argument: Petitioners claim that the alleged invalidity of the transfer of the WMCP shares to Sagittarius violates
the fourth paragraph of Section 2 of Article XII of the Constitution. In short, petitioners claim that a Filipino
corporation is not allowed by the Constitution to enter into an FTAA with the government.

No. A textual analysis of the first paragraph of Section 2 of Article XII does not support petitioners’ argument.

The Court finds completely outlandish petitioners contention that an FTAA could be entered into by the government
only with a foreign corporation, never with a Filipino enterprise. Indeed, the nationalistic provisions of the
Constitution are all anchored on the protection of Filipino interests. How petitioners can now argue that foreigners
have the exclusive right to FTAAs totally overturns the entire basis of the Petition -- preference for the Filipino in
the exploration, development and utilization of our natural resources. It does not take deep knowledge of law and
logic to understand that what the Constitution grants to foreigners should be equally available to Filipinos.

b. What is the proper interpretation of the phrase Agreements Involving Either Technical or Financial
Assistance contained in paragraph 4 of Section 2 of Article XII of the Constitution?

The phrase agreements involving either technical or financial assistance, referred to in paragraph 4, are in fact
service contracts. But unlike those of the 1973 variety, the new ones are between foreign corporations acting as
contractors on the one hand; and on the other, the government as principal or owner of the works. In the new
service contracts, the foreign contractors provide capital, technology and technical know-how, and managerial
expertise in the creation and operation of large-scale mining/extractive enterprises; and the government, through
its agencies (DENR, MGB), actively exercises control and supervision over the entire operation.

Such service contracts may be entered into only with respect to minerals, petroleum and other mineral oils. The
grant thereof is subject to several safeguards, among which are these requirements: (1) that the service contract
be crafted in accordance with a general law setting standard or uniform terms, conditions and requirements; (2)
the President be the signatory for the government; and (3) the President report the executed agreement to
Congress within thirty days.

c. Was the contractor’s right to mortgage objectionable?

Argument: Petitioners also question the absolute right of the contractor to mortgage and encumber not only its
rights and interests in the FTAA and the infrastructure and improvements introduced, but also the mineral products
extracted.

No, the contractor’s right to mortgage is not objectionable.

Ordinarily, banks lend not only on the security of mortgages on fixed assets, but also on encumbrances of goods
produced that can easily be sold and converted into cash that can be applied to the repayment of loans. Banks
even lend on the security of accounts receivable that are collectible within 90 days. It is not uncommon to find that
a debtor corporation has executed deeds of assignment by way of security over the production for the next twelve
months and/or the proceeds of the sale thereof -- or the corresponding accounts receivable, if sold on terms -- in
favor of its creditor-banks. Such deeds may include authorizing the creditors to sell the products themselves and to
collect the sales proceeds and/or the accounts receivable.
CORPORATION LAW CASE DIGESTS | 145
FEU JD4401 | 2nd SEMESTER, S.Y. 2017-2018

In any case, as will be explained below, even if it is allowed to mortgage or encumber the mineral end-products
themselves, the contractor is not freed of its obligation to pay the government its basic and additional shares in the
net mining revenue, which is the essential thing to consider.

d. Are the shareholders free to sell their stocks?

Argument: Petitioners criticize the authority given to the contractor to change its equity structure at any time.

Yes, the shareholders are free to sell their stocks.

It is not necessary for government to attempt to limit or restrict the freedom of the shareholders in the contractor
to freely transfer, dispose of or encumber their shareholdings, consonant with the unfettered exercise of their
business judgment and discretion. Rather, what is critical is that, regardless of the identity, nationality and
percentage ownership of the various shareholders of the contractor -- and regardless of whether these
shareholders decide to take the company public, float bonds and other fixed-income instruments, or allow the
creditor-banks to take an equity position in the company -- the foreign-owned contractor is always in a position to
render the services required under the FTAA, under the direction and control of the government.

e. Were the financial benefits surrendered to the contractor?

Argument: One of the main reasons certain provisions of RA 7942 (Philippine Mining Act) were struck down was the
finding mentioned in the January 2004 Decision that beneficial ownership of the mineral resources had been
conveyed to the contractor. This finding was based on the underlying assumption, common to the said provisions,
that the foreign contractor manages the mineral resources in the same way that foreign contractors in service
contracts used to. By allowing foreign contractors to manage or operate all the aspects of the mining operation,
certain provisions of R.A. No. 7942 have in effect conveyed beneficial ownership over the nations mineral
resources to these contractors, leaving the State with nothing but bare title thereto. As the WMCP FTAA contained
similar provisions deemed by the ponente to be abhorrent to the Constitution, the January 2004 Decision struck
down the Contract as well.

No. The financial benefits are not surrendered to the contractor.

Beneficial ownership has been defined as ownership recognized by law and capable of being enforced in the courts
at the suit of the beneficial owner. Black’s Law Dictionary indicates that the term is used in two senses: first, to
indicate the interest of a beneficiary in trust property (also called equitable ownership); and second, to refer to the
power of a corporate shareholder to buy or sell the shares, though the shareholder is not registered in the
corporations books as the owner. Usually, beneficial ownership is distinguished from naked ownership, which is the
enjoyment of all the benefits and privileges of ownership, as against possession of the bare title to property.
Any interest the contractor may have in the proceeds of the mining operation is merely the equivalent of the
consideration the government has undertaken to pay for its services. All lawful contracts require such mutual
prestations, and the WMCP FTAA is no different. The contractor commits to perform certain services for the
government in respect of the mining operation, and in turn it is to be compensated out of the net mining revenues
generated from the sale of mineral products. What would be objectionable is a contractual provision that unduly
benefits the contractor far in excess of the service rendered or value delivered, if any, in exchange therefor.

A careful perusal of the statute itself and its implementing rules reveals that neither RA 7942 nor DAO 99-56 can
be said to convey beneficial ownership of any mineral resource or product to any foreign FTAA contractor.
CORPORATION LAW CASE DIGESTS | 146
FEU JD4401 | 2nd SEMESTER, S.Y. 2017-2018

(88) CEMCO Holdings, Inc. v. National Life


G.R. No. 171815, August 7, 2007
By: Marasigan, Mariz Angelle R.

Doctrine: The coverage of the mandatory tender offer rule covers not only direct acquisition but also indirect
acquisition or any type of acquisition.

Union Cement Corporation (UCC), a publicly-listed company, has two principal stockholders UCHC, a non-listed
company, with shares amounting to 60.51%, and petitioner Cemco with 17.03%. Majority of UCHCs stocks were
owned by BCI with 21.31% and ACC with 29.69%. Cemco, on the other hand, owned 9% of UCHC stocks.

In a disclosure letter dated 5 July 2004, BCI informed the Philippine Stock Exchange (PSE) that it and its subsidiary
ACC had passed resolutions to sell to Cemco BCIs stocks in UCHC equivalent to 21.31% and ACCs stocks in UCHC
equivalent to 29.69%.

In the PSE Circular for Brokers No. 3146-2004 dated 8 July 2004, it was stated that as a result of petitioner
Cemco’s acquisition of BCI and ACCs shares in UCHC, petitioners total beneficial ownership, direct and indirect, in
UCC has increased by 36% and amounted to at least 53% of the shares of UCC. As a consequence of this
disclosure, the PSE, in a letter to the SEC dated 15 July 2004, inquired as to whether the Tender Offer Rule under
Rule 19 of the Implementing Rules of the Securities Regulation Code is not applicable to the purchase by petitioner
of the majority of shares of UCC. Director Callangan confirmed that the SEC en banc had resolved that the Cemco
transaction was not covered by the tender offer rule.

Feeling aggrieved by the transaction, respondent National Life Insurance Company of the Philippines, Inc., a
minority stockholder of UCC, sent a letter to Cemco demanding the latter to comply with the rule on mandatory
tender offer. Cemco, however, refused. Respondent National Life Insurance Company of the Philippines, Inc. filed a
complaint with the SEC asking it to reverse its 27 July 2004 Resolution and to declare the purchase agreement of
Cemco void and praying that the mandatory tender offer rule be applied to its UCC shares. Impleaded in the
complaint were Cemco, UCC, UCHC, BCI and ACC, which were then required by the SEC to file their respective
comment on the complaint. In their comments, they were uniform in arguing that the tender offer rule applied only
to a direct acquisition of the shares of the listed company and did not extend to an indirect acquisition arising from
the purchase of the shares of a holding company of the listed firm.

Whether or not the rule on mandatory tender offer applies to the indirect acquisition of shares in a
listed company, in this case, the indirect acquisition by Cemco of 36% of UCC, a publicly-listed
company, through its purchase of the shares in UCHC, a non-listed company

Yes.

Tender offer is a publicly announced intention by a person acting alone or in concert with other persons to acquire
equity securities of a public company. A public company is defined as a corporation which is listed on an exchange,
or a corporation with assets exceeding P50,000,000.00 and with 200 or more stockholders, at least 200 of them
holding not less than 100 shares of such company. Stated differently, a tender offer is an offer by the acquiring
person to stockholders of a public company for them to tender their shares therein on the terms specified in the
offer. Tender offer is in place to protect minority shareholders against any scheme that dilutes the share value of
their investments. It gives the minority shareholders the chance to exit the company under reasonable terms,
giving them the opportunity to sell their shares at the same price as those of the majority shareholders.

Under Section 19 of Republic Act No. 8799, it is stated:

Tender Offers. 19.1. (a) Any person or group of persons acting in concert who intends to acquire at least fifteen
percent (15%) of any class of any equity security of a listed corporation or of any class of any equity security of a
corporation with assets of at least Fifty million pesos (P50,000,000.00) and having two hundred (200) or more
stockholders with at least one hundred (100) shares each or who intends to acquire at least thirty percent (30%) of
such equity over a period of twelve (12) months shall make a tender offer to stockholders by filing with the
Commission a declaration to that effect; and furnish the issuer, a statement containing such of the information
required in Section 17 of this Code as the Commission may prescribe. Such person or group of persons shall
publish all requests or invitations for tender, or materials making a tender offer or requesting or inviting letters of
such a security. Copies of any additional material soliciting or requesting such tender offers subsequent to the
initial solicitation or request shall contain such information as the Commission may prescribe, and shall be filed
with the Commission and sent to the issuer not later than the time copies of such materials are first published or
sent or given to security holders.

Under existing SEC Rules, the 15% and 30% threshold acquisition of shares under the foregoing provision was
increased to thirty-five percent (35%). It is further provided therein that mandatory tender offer is still applicable
CORPORATION LAW CASE DIGESTS | 147
FEU JD4401 | 2nd SEMESTER, S.Y. 2017-2018

even if the acquisition is less than 35% when the purchase would result in ownership of over 51% of the total
outstanding equity securities of the public company.

The SEC and the Court of Appeals ruled that the indirect acquisition by petitioner of 36% of UCC shares through
the acquisition of the non-listed UCHC shares is covered by the mandatory tender offer rule. This interpretation
given by the SEC and the Court of Appeals must be sustained.

The SEC and the Court of Appeals accurately pointed out that the coverage of the mandatory tender offer rule
covers not only direct acquisition but also indirect acquisition or any type of acquisition. This is clear from the
discussions of the Bicameral Conference Committee on the Securities Act of 2000, on 17 July 2000.

SEN. S. OSMENA. Eto ang mangyayari diyan, eh. Somebody controls 67% of the Company. Of course, he will pay a
premium for the first 67%. Control yan, eh. Eh, kawawa yung mga maiiwan, ang 33% because the value of the
stock market could go down, could go down after that, because there will (p. 41) be no more market. Wala nang
gustong bumenta. Wala nang Imean maraming gustong bumenta, walang gustong bumili kung hindi yung majority
owner. And they will not buy. They already have 67%. They already have control. And this protects the minority.
And we have had a case in Cebu wherein Ayala A who already owned 40% of Ayala B made an offer for another
40% of Ayala B without offering the 20%. Kawawa naman yung nakahawak ngayon ng 20%. Ang baba ngshare sa
market. But we did not have a law protecting them at that time.

CHAIRMAN ROCO. So what is it that you want to achieve?


SEN. S. OSMENA. That if a certain group achieves a certain amount of ownership in a corporation, yeah, he is
obligated to buy anybody who wants to sell.

CHAIRMAN ROCO. Pro-rata lang. (p. 42).

xxxx

REP. TEODORO. As long as it reaches 30, ayan na. Any type of acquisition just as long as it will result in 30
(p.50) reaches 30, ayan na. Any type of acquisition just as long as it will result in 30, general tender, pro-
rata. (Emphasis supplied)
CORPORATION LAW CASE DIGESTS | 148
FEU JD4401 | 2nd SEMESTER, S.Y. 2017-2018

(89) Abacus Securities Corporation vs. Ampil


G.R. No. 160016. February 27, 2006
By: Pangilinan, Gene Alexis

Doctrine: An ordinary civil case seeking to enforce rights arising from an Account Opening Form (AOF) between an
investor and a stock broker is within the jurisdiction of the Regional Trial Courts, and determination whether the
parties fulfilled their obligations in no way deprives the Securities and Exchange Commission of its authority to
determine willful violations of the RSA and impose appropriate sanctions therefor.

Petitioner is engaged in business as a broker and dealer of securities of listed companies at the Philippine Stock
Exchange Center. Respondent opened a cash or regular account with petitioner for the purpose of buying and
selling securities. Respondent actively traded his account, and as a result of such trading activities, he accumulated
an obligation in favor of petitioner amounting to P6,617,036.22. Petitioner sold respondent’s securities to set off
against his unsettled obligations when the latter failed to pay. After the sale, the remaining unsettled obligation to
petitioner was P3,364,313.56.

The respondent filed a case before the RTC. Petitioner alleged that the trial court had no jurisdiction over violations
of the Revised Securities Act.

Issue: Whether or not the trial court has jurisdiction over the case.

Held: Yes. The instant controversy is an ordinary civil case seeking to enforce rights arising from the agreement
between petitioner and respondent. It relates to acts committed by the parties in the course of their business
relationship. The purpose of the suit is to collect respondent’s alleged outstanding debt to petitioner for stock
purchases.

Nevertheless, the Revised Securities Act (RSA) and its Rules are to be read into the Agreement entered into
between petitioner and respondent. Compliance with the terms of the agreement necessarily means compliance
with the laws. Thus, to determine whether the parties fulfilled their obligations in the agreement, the Court had to
pass upon their compliance with the RSA and its Rules. This, in no way, deprived the SEC of its authority to
determine willful violations of the RSA and impose appropriate sanctions therefor, as provided under Sections 45
and 46 of the Act.

You might also like